Вы находитесь на странице: 1из 1006
ONE FUNCTIONS, LIMITS, AND CONTINUITY 1.1 FUNCTIONS AND THEIR GRAPHS Real Number Inequalities Intervals Absolute Value Sign, Step Funetion ‘Triangle Inequality Graphing 1.1.1 Definition 1.1.2 Definition Greatest Integer can be expressed as a decimal. A rational number is a repeating or terminating decimal. @<6 (is less than 6) if b—a is positive; a> b (ais greater than b) if a—b is positive @ bifa>bora=b. (a8) ={zlaa} (~00,8] (20, +00) = tal= (2, fez ple bl= {gre —lifz<0 sne=fo ite=0-v@)={? 1 ife>0 la+d| D: (—00, +00) Ds (—00, +00) > D: (—00, +00) R: (00, +00) R: [0,+00) R: (—00,5] Y y 8 5 | Seep tet “4 «| = 12. 14. G(z) = 27 42 1p > D: : (—00, +20) [2,+20) » » 3 5 4 5 “ ee CCAS err 2 4 “| “4 4 “6 4 4 1. 19. ea 21. g(x) = V9-27 > D: 2-120, > 20, > D:9-2730 [1, +00) ( ]U [2, +00) [-3,3] R: [0, +00) R: [0,+00) R: [0,3] y ty y 3 3 3 6 6 6 4 4 2 e Z z z Saaz pe 4 6 SET ES an 4 “| zg + a 4 4 FUNCTIONS, LIMITS, AND CONTINUITY 18. F(z) = /9—z 20. o(z) = V4 - 2" 22. f(x) = V2? =1 > D:9-2>0 > D:4-27>0 > Di2?-130 (-20,9] (-2,2] (ee, -1]U[1, +00) R: [0, +00) R: (0,2] R: [0, +00) wy 8 2 6 ‘ ——_4 2] SSS a PT Tes 364 68 “4 “6 4 25. F(z) =|32-+2| > D: (—00,+00) >. D: (00, +00) R: [0, +00) R: [0, +00) 3 sf 5 ‘ 4 2 z : SORES CE Sapa a ee a “4 «i 6 3 4 24. H(z) =|5—2! > D: (co, +00) R: (0,+00) SST Tes Ll FUNCTIONS AND THEIR GRAPHS 5 ifz<-2 3 z if2<3r<2 mw po={ee? ect ifo ee > D: (~00,+00) > f(2)=3242 2. Diz f#-2,3 R: {-4,-1,3} D: (-00, +00), Rey #5 - | & 4 4 BPh 4 2f. z £ SRE Pere — “4 “ 24 itz #3 its 7 itzs-2 siraa(y) tt on oe ig? rues > D: (~00, 400) > D: (00,400) R: (20,1) U1, +00) R: (09, 3] U (00,6) = (20, +20) =00,8) : | q ; 4 al z E z SER TS SOS s | W E * = fe <0 9-2? ifzZ-3 P-4 if2<3 36. a@={2 ifz=-3 * FE) {e-4 if3 D: (~00,-+00), Ri (~00,9] > Di (~00, +00) ) Note: G(3)=0 R: [-4, +90) U[5, +00) (00, -2]U (1, +00) =[-4,+00) , by s d s 6 4 +f 4| 4 2 2 x 3 Z SSSR ET ECT SSG2 PI es AAARER EES 2 4 4 ~ 4 a 6 FUNCTIONS, LIMITS, AND CONTINUITY mth ife<-5 A. Wo= (ew if -z ifi D: (20,408) R: (integers) D: £2, R: [0,400) Note: F(-2) =4 | ol Fi 7 Seas ee “ \ “i 2 ifr<-4 a Visa if 4c 2<4 M4. G(2) = 238 zoo = if4se e+ > D: (—00,-+00) = Sy <2 Rz (~00,-2]U (0,4) U(—00,-2] Di 2 4-3, R: [0, +00) = (00, -2]U (0,4) Note: G(3) = 9 sal ee satbelns 4 | 4 4. Me) wwe (oe)-1 (@ue)—ve-1) ifz<0 > aft fs ={t | =f! =f tess 1 ifz>0 1 if2>1 0 ife>0 i) eet 2 iE Z| x Zi = a z= 4 J 4 3 Zi | 2 3 1.1 FUNCTIONS AND THEIR GRAPHS 7 48. (2) 2Ufz) (b) (2 +1)U(e+1) (6) @+DU(e+ 1)—2U(z) fe<-t eof) ita<0 (0 ifz<-1 {i Pee | ‘ sf 6 4 ri 4 4 4 5 _ 2 . op ae. eat eee ereprr et “4 4 + * “| “| “ 4 4 49. (a) sgn 2 (b) zsgn 2 (c) 2-2 sgn z (4) 2—2sgn 2 iE ifz<0 (—z ife<0 if (z+2 ifz<0 > =40 ifr=0 =\0 ifz=0 =tt ee -{ itz—0 1 250 itz >0 = ifz>0 =lz! 4 2 PeCe SALE? SCT 4 “| “ 50. (a) sgn(2+1) (>) sgn(z~1) (6) sgn(z+1)—sen(z- 1) 1 ife<-1 1 ifz<1 ife<-lorz>1 wef eta =f ee fT Hecalersct 1 ifz>-1 i ifz>1 2 if-1 Using the point-slope formula for each segment give ‘ 2-242) f-2<2S-1 22-2 if-2<2 Katee) Helces0 yet oles so 91-12 ifo Using the pointslope formula for each segment give J 042(242) -2s2 M44 if -2 fy =e, fg=-tor fH lel fy =e! 54. Define the graph of the letter Z as the union of f,, fa, fy > Use relational operators to get segments, rather than lines. In Exercises 55-58, define the function piecewise and sketch the graph. m1 ifz<-1 -l-fiz if-l1 3] 3 6 2 z z z S64Db 2468 S64Tb 24 ee S442 P IEE “4 4] -4| | “ +6 =| + 5 Exercise 56 Exercise 57 Exercise 58 =43z-27 ifo3 In Exercises 59 and 60, sketch the graph of the function and determine its domain and range. Check by plotting. kL 59. h{z) =2—[s]. D: (—c0,-+00} R: [01) Exercise 59 Exercise 60 60. F(z) =2+ [Zz]. D: (—00, +00); R: [2k, 2k + 1), where & is any f: —8e ifr<0 Jeger. 61, Define two other functions whose graphs resemble two different letters of the alphabet. > U: f(2)=1-V1-24, V: fe) Sle! 1.2 OPERATIONS ON FUNCTIONS AND TYPES OF FUNCTIONS 1.2.1 Definition Given the two functions f and g: (j) their sum, denoted by f +g, is the function defined by (F4+.0(2) = (2) + of2) their difference, denoted by f — 9, is the function defined by (f -9)(2) = F(z) - (2) (iii) their product, denoted by fg, is the function defined by (F-9\(z) = F(2)- 92) (iv) their quotient, denoted by f/g, is the function defined by (F/ az) = f(2)/a(2) In each case the domain of the resulting function consists of those values of z common to the domains of f and 9, with the additional requirement in case (iv) that the values of 2 for which g(z) <0 are excluded, 1.2 OPERATIONS ON FUNCTIONS AND TYPES OF FUNCTIONS 9 1.2.2 Definition Given two functions f and g, the composite function, denoted by f 99 and read f of gy is defined by (fog)(2) = f(gl2)) and the domain of fog is the set of all nurnbers + in the domain of g each that g(z} is in the domain of f- ‘The composition of functions is associative, that ia (fog) 24 = So(goh). Definition (i) A function is said to be an even function if for every = in the domain of #, F(—=)= f(z), (ii) A funetion is said to be an odd funetion if for every + in the domain of f, f(—*) = —f(x). das both parts (i) and (i) it is understood that —z is it the domain of f whenever 2 is: “Even powers of x are evem functions; odd powers and odd roots of rare odd functions ‘Combinations ¢ven + even = even, odd + odd = odd, even x even = even, odd x odd = even, even x odd = ‘Geenfeven = even, odd/odd = even, even odd = odd, odd even = odd, See Exercise 32. odd ives 1.2 Brercises 1-10, define the following functions and determine their domain D: fa) f+9 (b) F—a fe) 9 (A) S/o Ce) oft ‘> We adopt the common practice of omitting *(2]". Values not in the implicit damain are bolded. f=r-5, 71 (a) ft gaa? +26, D: (—o9,e0) (b) fg = 2? +2—4, D: (—o000) be) f-g= Yet 1) = Fe 2 +5, DF (8) fla = aH, Dew #31) off = Faye Des 2 fa Je pat 41@) foga Vere +1 Di [0,+08) (b) 9 = 22? =1, D: (0,400) ke) fog = ale? +1), Ds [0 408) (€) ffs “4 Ds [0 +00) (2) ot =, 1D: (0, +26) 42e=1 Seay eo biega1@fe=ttpd= tb Dee 01 220,1(09 sith ebecha got Deeg a101 faeg=4-2? (a) ftg= vette D: [0, +22) (b) f—g = E427, D: [0, +09) {e) F- = /@(4—22), D: [0, +20) (d) t= De [0,2)U (2.406) (@) a/f “ Ds (0,400) fave 21 (a) f eg = e422 — 1, Di (0)-+00) (b) fag = WEF? +1, D: [Os +00) fe) f-7 = V2(2" —1), B: (0, +99) (a) Wn B: [0,1) U(L, +09) (6) a6 = 253. b: (0,420) (00,00) fle) g=le—31 (2) fg =l2l+l2—3l Di (20,400) (b) fog =le|—lz— 3), D: (-08, +90) (2 Feamlele— 8h D: (ote) te) fa = p25, FS Le oT = Tae Dao fa st41, p= 32-2 (a) fg = 2743-1, Ds (—00, 400) (b) J—g = 2) — de +8, Dt (00, +00) fe) fg = (22+ Ge 2) = 328-22? + 82-2, De (cert) fin= Eth De 93 oat f= D: (20,460) b fa vera p= 22-4 (a) ft a= yeti te 4, Di [4 +e) (b) f-g= Verd-w 44, = 1: [4,21 (=2,2) U2, toe) 10 FUNCTIONS, LIMITS, AND CONTINUITY depo? Leet ol e248 etd 2 ® fagpp sete) ftesgeproate-a esl FH so Diz¢-L2 ala phi =¥ O-e=shp beet bette} orn “erties Dist “415 yes za 2 Det -1,0,2 @oll=siateem eee . 10. f= 2. 9 =n (a) fee (ot oe In Brerives L1-L4y compute (fe g}() by two methods: (0) Pind fe) Set (}) Find (0a) Ke i. fla) = 308 — Az, gle) = 2-5 = 4 & (a) (a) = 26-9 =3, JO) =15 ‘pe 5)? — {22 3) = Alte” — 2x + Bh) — Be +20 = 122" —08r + 95 De (0, 420) (b) f= D:(0,-400) V2, (0, +50) (a) F a 2/2, Di: (0) +00) ‘f= dee 12 f(e) = Ve —6, g(x) = 2" - 32, = b (a) ofS) =5?—-5 = 10, (20) = Vi0" = 36 = 5 () fale) VET, Juen= (errs ies > (ay aQ)= i= a) wa h@=5 8) fale) =z erst aye ua. fiz)= 1 sz) = w f a plea a BETS: Hol-2)) = ap pg HBF HAD) + = SVE = In Baercises 15-24, find (a) Fog (0) gef (©) Fos (A) 909 and give their domains D. Wi fee) ne gle) #47 fb) o( F(z) = (2-247 = (9) a(g(z)) = 16 fiz) = 3-2, oz! (a) fg{2)) = 3— 26-32) = Be | ne, 400) (b} al f(z)) = 6 = 3(3 - 22) = 62 ~ 3, Ds (~29, 420), te) JU (= d— 28 - 22) ae Pee) atgle)) = 6-H6 —2) = Bx — 12, D: (oy -r00) 1. fle) =r—5, g(2)= 27-1 & (@) faz) = (7-5 =? (20, +20) fe) este 5)? 1 = a — 102 4.24, Dr (~00, 420) © He =10, Ds (20) 448) (@) (a(z))=(2?-1) - 2x7, D: (00, boo) 18, f(a) = oF » (e) hie EFI, Ds (28,420) (b) af(2)) = (v2) +2 = 2+ 1, Di [0,+e0) fe) srl) = foe = Y2, Ds (0.400) (a) poly) = (2? +1) + 1 at + 2a? +2, D: (2,490) 19. f(z)=¥F—2%, oleae? -2 P(e) f(a(2)) = ylz*—2)-2 = va" —4, D: (0, #00) (b) of f(z) = (2-27 -2 = 2-2-2 = 2-4, [2,408) (© fle) = fen 2H 2 Di a—224, (6400) @) pale =@P=2 = B= +42? +2, D: (2a 428) 1.2 OPERATIONS ON FUNCTIONS AND TYPES OF FUNCTIONS Lt 2, fle) 22-1, oz) =F aiken a) ae=(-1= 2 (6) (Ue) = F-18254, De (00,400) Ud) afl) = =F pe i aq bs eee (h) 9(s(a)) = apap Dee # St L Dine an fley=ha(z)= v2 : © (a) Fale) = de Ds (0,490) tb) ata) = Y= +p Ds (0,425) idea rp=s Db ae0 (0) alot) = Ve = YE 2:0, 420) m2. fe)= VE olt)= & (a) fate) = yf & De: (-00.0) (b) af (=) = (e) Fle) = Va = YE: D: [0.4208 2) ofa(e) = BB. f(z) siah g(z)=l2+2| & (a) figiz)) =||e+2!]=le +2) De (-29, #20) (b) gf f(2)) = [le 2| = 12142, De (—20, +00) (6) AUCe)) =Well= 124 Ds (98.400) (4) a(ote)) [24 2/42) =]=4 214 2, Ds (20,400) 2, f(r) = VET, af G1 & (0) fate) = Venta = VenTHT = VEE, B40) (by afc) = vet 1-1, V1 A, (28, VM UL V9, 400} © 1G) = feat ave = V8 (@) ofgtz)) = Ve= 1-1, Ds x= 1 > 1 [2 +98) a5. f(a) =e la) He = VE lek D: Hotes) 0) LP = [Zt = 2, Ds [0 +00) SIU v2.40) (9 sla) = V2 = VD: 1,4) (2) HE(-2)) = Y=, De (00,0) 2. fel=z hy) fe) = ph eee At (b) Le) = yop Be A @sve)=— Lezeesl mie yor (@) s(-2) =) pd T-T(t==) e In Erereises 27-98, express the function as f(g(#)) in two ways. p> Bxpress the function using of twice; break at the first or the serandd. mf square coot of the difference of feign st—dor faVa-4§, 8. (9-27)? =2 power of the sum of --- fax g= 94s or f= (0+ vs. (5) bP pomec of the reciprocal ofa faa, phy or f=(E) 9= 22 30. 0 BeOS et de cual 41. ={° Lite cn 2 edd (even Fedd} 42, alel= tteen (odd ) 0 either a(-2) —(e-)+(e42)=4 tc 2 4 evalcietate [leo eoa) ~2r if -2 << 2 is odd (2-2)—(242)=-4 ifa> 2fe ife<-1 =(e4+1)4 (2-1 4, Been fie" ee ite [-1,0)U(0)1] ixeven (2+ 1) —(—1)p. ite>1 le 45, fog gg f. See Exercises 16-24, (a) versus (b). In Hrercises 46-50, show that f and g are inverse functions by showing that /(g(z)) = 2 and g(f(x)) = 2. 4B. f= 22-3, 9 = Mr +3). fog = Kr +3) -3 = (x 4+8)-3=2, of =H(22-3) +2) = }22) 1 ied z - =W(e+l) r+ teeta Hfeg pen es fee a eat Sloe =e ee W. for cz0ig= Vz fog=(ya)=2, gfe 49. fore c0,9=—y% foo=(-\SP =, 90 f = Ve =-(-2)=2 80. f= (2-1), 9=14 9%. fog = [+ Y2)— IP = (8e =s, go fais el) = 140-1 =2 SL. Find formulas for sgn(U(z)) and U(sgn(z}) and sketch their graph. 0 fed if Ulz}< 5 never > vencuy={o ifUG)= = treo betes 1 if Ulr) =1 SI ifz>0 vomtn={ PR TEfa{} 220) ‘ $2. If f and g are odd functions, prove that f+ and f —g are add and f-g and fq are even. aI > f(-r) = =F), o(~2) = a(2). q d(-a) +9{-#, {-2) - of-2) =—S(e) (F 9N-2) = S-as(-2) = [-Fe)-af Gfa)mz) = F(-2)fol—s) = -402)/-a12) 5%. If fio any function and y ix eves, F(G{—H)) = Skala) s0 0 9 is even, WJ and g are both odd, F(al=2)) = J(—(ala)) =—fa(z)) wo fog is odd. If f is even and g is odd, f(a(—=)) = f{—a(2)) = f(a(2)) 20 fog is even SA. Pind formulas for (f o.9)(2). Sketch the graphs of f, 7, and f og. Uf +92) 9-2) a iO rate) SKS fe ase ’ : : asap tt 3 1.2 OPERATIONS ON FUNCTIONS AND TYPES OF FUNCTIONS 13 155. Hind formulas for (go f){-2) and sketch its graph. 0 if fi2)=0 0 ifel > 9ff(2)) {ine it0< fla)sl -f iozr0} = (4,420) but dom G = (—20,—1]U (4, +00), 1.3 FUNCTIONS AS MATHEMATICAL MODELS: Variables should be described asing units: 2 feet is the length. vis Proportional to z Directly: y = kx. Inversely: y = kz. Jointly proportional to x and ‘Extremum of ar? + be + occurs when * = —b/2e; this can be obtained by completing the square. Maximum Product of a set of positive numbers of constant. sam is when factors are equal. See Ex, 28 ‘Minimum Sum ofa set of positive numbers of eoustant product is when terma are equal. See Ex. 22. Exercises 1.3 In each exercise, obtain a function as a mathematical model of the situation. Be sure to write a conclusion, ‘L.A payroll of p dollars is directly proportional to the number w of workers, and a crew of 12 workers enms $810. (a) Find p(w). (b} What is the payroll for a crew of 15 workers? b p= kw. 810 =K(12), £ = 67.5 (a) pw) = 67.54 (b) p15} = GT-5- 15 = 1012.50 2. A person's brain weight 6 Ib is directly proportional to his body weight w Ib, and a person weighing 160 Ib has a 4 Ib broin (a) b(w). (b} Pind tke brain. weight of » 176 Ib person be b= kus d= A{L50), b= oe (a) (te) = Sw (b) 6176) = e176 = 4.69 3. The period p se of a pendulum i directly proportional to the square root of the mumber 2 of feet in ils length, and au 8 ft pendulum has a 2 see period. (a) Find p(s). (b) Find the period of a ft pendulum. p pakye 2=kV8, V2 (8) plz} = /2F2 (b) nl2) = V1 = 1. The period is 1 sec, 4. The frequency {per see of @ vibration is directly proportional to the square root of the tension ¢ hig, and is 64 /sce. when the tension is 24 kg. (a) Find {(1). (b) Find the frequency under e Lensian of 6 ky, p fa kyi, 864 = by 24 = ine, = Tab (a) f(t) = 12/61 (b) F(6) = 72-6 = 182 te Oe £50 Ir £50 <2 < 200. (50) =2.2%50 052 if > 200 ($1) = 2.1 x51 = 107.10, C52) = 2.1 w 52 = 109.20, C(68) = 21 x53 = 111.90, C(200) = 2.1 x 200 = 420 (202) = 2.05% 202 = 414.10), C204) = 2.05 x 204 = 418.20, C(208) = 2.05 x 206 = 422.90 5. C(2) dollars ie the cost of shipping 2 tb. Cla 110, y(z) cents is the cost of mailing ounces. y = 9— 23-2]. y(1.6) 9 —23(—2) = 55 yl) =9 9 23(—3) = 78 yao = 28(-11) = 262 1030 30 40 50 60 10 3 100 1 tO TT aa 4a es poll Exercise 5 Exercise 6 Exercise 7 Exercise 8 7. x(a) cents is the cost of an = minute call. vz) = 10—30[—z]. w(0.5) = 10 — 30[—0.5] = 10 —30(—1) = 40 1 30[-2] = 10 — 30-2) wW25) = 10 — 30[-2.5] = 10 — $0(—3) = 100 10— 30[-3] = 10 ~ 30-2) (35) = 10 — 80f-8.5] = 10 —80(—4) = 10— 30-5] = 10 — 30-3) 8, The adult admission price is $7, while the price for cildeen under 12 years is $4 and the price for seniore at least 60 yers ie $5. (a) Find « model of price as a function of age. (b) Sketch the graph of your function in (a). 4 if0 0, S=60/r 4 60/r + xr? (product = 3600n) is minimum when 60/r = x72, r? =60/x, 7 22.67 in 23. A. page with margins of 1.5 in. at the top asd Dottons sod 1 in. a the sides is to contain 24 in? of print. (a) Find the total area of the page, A(z} in?, when the width of the printed region is = in. (b) What is the domain D af A? (e) Approximaue to thy pesrest hundredth of gr inch the size ofthe smallest page. > (a) The length of the printed region is £8 in, A(z) =(z +2y(24+ 3) =30 + (32 +25) (preduct = 144) (b) De 5-0 (C) Aji = 54 when Bz = 48, 2 a4, 2 +25 442=5, 249024) 30, The smallest page is 6 in, wide and 9 in. Tong. AL A lot with walkways 22 fe ide at the frowt and hack and 15 ft nt the sides is to contain a 13,200 7 building. {2) Find the total area of the lot, A(z) 2, when the width of the front és = ft. (b) What is the domain D of ‘Af (c) Approximate to the nearest hundredth of a foot Uhe size ofthe smallest lot » do) The length of the building is “2° n, A(z) = (2 + 90y(49200 + 44) — 14520 + (442 + 996000/) (0) Di = >0 (e) Because the product ofthe variable term Ie 742,00, te smalls ot has area 2,868.4 62 sehen 442 = 396,000/2, 2 = /9000 = 94.57. The field is 124.87 ft by 189.24 fL 25, A box of tength 2 Gn. with square croc wetion has 100 in. as the um of its length and girth, (a) Find the ‘Yolume ¥ in® as a function of 2. (b) What is the domain D of V7 (c) Approximate to the nearest inch the dimensions of the largest box. (a) The width of the box is (100 — 2) in. V= (b) D: 20 < 2 < 100 (length > width) (¢) V, "The largest box is about 33 by 17 by 17 in. YA. The growth cae J bctri/in of m colony in jointly proportions! 40 the narber = af bacteria ged the number 1,000,000 —2 of capacity. f(z) = (1,000,000 — 2), f(1000) = 60 = 10008 999,000, & = za Saas satan: J= #(1.000,000—2)/16,650,000, 0-< x < 1,000,000. (£00,000) = 108,000 -90,000/16,658,000 5405 bacteria/min. f(z)= 540,000, 21. The growth rate f infected/day of an epidemic number 5,000—z of capacity. f(z) = ha(,000 ~ 2), $(100) ‘92(5,000 — 2)/490,000, 0 < 2 < 5,000. f(200) = 9 200 »4800/190,000 r 17.6 = 18 people/day. (m2? +5,0002) is maximum when 50 /—2 = 2500. 26. ‘The base of a pyramidal tent is 22 m square and a triangular side has height 2.5—z m. The height h of the tent satisfies A? +2? = (2.5—2)%, & via 25— 55. The volume is ¥ = 3 6.25 —S2, 0< 2 < 1.35. Vv(0.8) = 12-08) (2a BOB =} 16/208 = 1.28 m®, v8 = Bell — de) = Je yhe (elle yl — Je) Because the eum of the variable factors is 1, V is maximum whe nf, 2=1 100 — 2)? = -210100 ~ 2}(100 ~ 2) (sum = 200) GP)? 0250 when 100—2 = 22, 2 = MO. (2? +1,000,000x) is maximum when x = —1,000,000/— jelntly proportional to the number z of infected and the ‘INoke- 4900, # = 9/190,000. 16 FUNCTIONS, LIMITS, AND CONTINUITY 14 GRAPHICAL INTRODUCTION TO LIMITS OF FUNCTIONS “Phe formal definition of limit is given in the next sect 1.4.1 Definition Let f be a function that is defined at every number in some open interval containing a, ‘exept possibly at the number a itself. The dimit of ffs} ex x approaches o is L, written as fin He) = if the following statement is trxe: Given any ¢ > 0, however small, there exists a > 0 such that iO From the figure, 6 =.1 ; 2. f(z) =2=82, 05-1, L=5, = 08. > From the figure, 5 TS oh 3 and 25. f(z) i Bercise 1 e 2, -1=3-0.03, 2, =4 = 0.03. &,—1= 8 40.03, 22-4 = 0.03, Choose § = 0.03, |P(2)—1|=|(2—1) —3|=12— 41-26 when [2a] <1 = 0.03 = 5. 4 and 26. f(z) =242,0=3,L=5,¢=0.02 Bo 2y$2=5-002, 2-35 0.02. r.+2=5 +002, x; — $= 0.02. Choose 6 = 0.02. [f@)-1 pepe tele-ise 9.02 =¢ 5. fe)= & Intde Tand 27. f(z) = 52-3, 0 Sz, )~1= 001 5a = |(2—8)—2]=lde—S]=ae— 1) ce whenle-1/5 ke = Band 28. f(x) = 42 ~5, 0 =2, b= 3, ¢= 0.001 sé > da 5 = 3.001, 42 001, 2 —2 = =00028. 42,5 = 34.001, dz = 8=.001, 2) =2 = 0007: |@=—5)-3|=]4a—8]= 4x21 << when |z-21< = 0.00025 =F 9% f(z) =9—4,a=-1, L=7, += 0.02. f is decreasing, Bade = 74.02, 4 42, = 02, 2, 41 = 005. 9-4, = T= 02, —4 Ae, = —09, 2541 =.005. 6 = 005 10. s(2)=2452, a= 2, Lb =-8, > 2482,=-8— 00252, +10= 002.2; +2 = —0004. 2+ S2y=—8 +.002,52y +10 = 002.2, +2=.00t =é 14 GRAPHICAL INTRODUCTION TO LIMITS OF FUNCTIONS 17 2, b=—4e= 001, & fe)=2-2ife #2 =Olony P= 84.01, 2-2 = 44.01, 2) +42 = 01 nL. He)= 24 a= & 2,-2=-4-01, 2,-2= 4-01, 4,42 12 soya 42} a=} L=2,e=0m > f(e)= S241 ite x} 32,41 = 2-01, 32, — 01) zy p= gpg Bg +1 2401, 3zy—-1= 01, yeah 6 1B and 29. f(z) =A MES gL a4 20080 fe) =2e—3ife et 22-3 = 94.03, 2x, 41 = =.08, 2p t= O15. 2s, 3S = A+ 03, Ae, 41 = 08, 2 $= 05. 6 = 15 +g | From the figure, # = min(2— /7/2, 972-2) = min(0.126,0.121) = 0.121 Bote Fok noalineat J, the Values of-d obtained Geom tnetualilies are tually’ sinaller ‘than those obtained directly. The factoring is really done first to determine what ‘bounds we need, Aand 31. f(z) =s7, o=3,L=9.¢=0.5 & OF 0 hay, ty > 0 27 = 9-05 = 8.5, a = 85 = 2915, 2-2, = 085, 2,° = 9-405 = 0.5, 25 = y/0 = 1.082, 2, 3 = 082. 8 = 082 Gl. Choose 6< 180 -1¢4-3-<¢1, S243 <7. Then le? —9l=[e43lle-3l 2 a) 44a, $4 = 1-08, (2) + To4.08, (2, +2)? = 1.08, 0) 42 = 108. |*44r4aiale4aliatiicaie+il Fits Ch fle decreasing 3z,2Ta, 2,5 (7— VP 4-3-3776 = 809, 1-2, = 191. Because Fa) 2 #Q)> (74+ W748 8.7)/6 = 1.524, a1 5.5%. 8 ‘36. Choose 6 <1 s0 -1<2-1<1 —8<32-3<3,-4<3r-4<2, [(a=?-72 +2) - 23] |e? = 7r44|=|32—4l2—il@. Substitution Li ‘Theorems 1-10 imply that the limit as z approaches a of any formula (that is, excluding functions defined in pieces) can be found by substitution, if substitution is ‘meaningful Fractions Note that we cannot use Limit Theorem 9 to find the limit of a fraction if the limit of the denomiaxtor is xer0. However, when the limit of both the numerator and denominator i aero at =a we may be able to apply one of the following methods: (i) If the oumerator and deneminater are both polynomials, we apply the factor theorem which states that if x =a is a zero of « polynomial, then #—a is & faster. ‘The commen factor z—a may then be canceled, This is illustrated im Exercise 32. (ii) If the denominator is polynomial and a term of the uuimerator is a radical, we may multiply by its conjugate, which is noi zero al a, and apply method {i} to the product, This is shown in Exercises 20 and 32. Ino doing, we use the following theorem, a consequence of Definition 1.6.1. ‘Thoorem A if im yz) =E and f(x) = g(x) for all = #0, then lim f(z) ‘We cannot use Limit Theorems 1-10 to find the limit, of.a fraction for which the limit of the denominator is zero and for which the limit of the numerator is not zeta, In Section 1.7 we have-a theorem for finding the limit of such a fraction. Buorcises 1.5, 3 ‘We wich to determine a § > 0 such that ifd 0, any choice of 6 > 0 will suffice; in particular take ¢ ‘Wee wish to determine a 6 > 0 such that i€0 0, any ekoiee of 6 > 0 will suffice; in particular take & = ¢. We wish to determine a 6 > 0 such. that O<|e—4| de—4l< athe) = |2r41)-9] <0 ‘We wish te deterr aé>0 such that iO<|x—L|<& then |(42+3}-7|<¢ e@0<|z-1l<6 then de-il lz—11< 4(de) = |(42 +3) -7[ <6. 20. FUNCTIONS, LIMIES, AND CONTINUITY b We wish to determine a 6> 0 such that if 0 Fr44|c2(p) > |(2r+7)-1] cn T. We wish to determing 4 4 > 0 such thi Oe |e 42/8 then [(1+32)- e@itdcle+2/<6 then drt2icc @itO<|e+2|<é then |rt2}che Heuce, take $= fe; then O-<|2+2|< 6+ Slz-+2| < ae) = |(1+32)—(-8)|<6 Ve wish vo determine a4 > 0 such that ifo 242] < 80 >| +82)-(-3)| 0 such that jet—1 ce ito<|etifes then ES P—-2) Si 0<]e41/<8 then |e t1l0<[rtll 10. We wish to determine a é> 0 such that 2 W0 there is a 8 > 0 such that | f(z) —L| 0 there is a > 0 such that |[f(z) —L]—O|<¢ when 0 0 there isa 6 > 0 such that | f(z) —Ti|0 there is a 6> 0 such that | f(t-+a)—L]and m is a positive integer, or a< Qand m is an odd positive integer, lim YE = lima 9 15.16 Theorem If lim f(z) =, and kim f(z) = by then Ly = bye =f ein oo Supplementory Beercises £5 & ‘The key to ashort solution is to factor in the second line before completing the first fine. 1. lime" Jim & Choose é<¢lso-l Choose << leo-lery3el —Te2-ge-t Then |2?-9|=|2-3| 243) <]2+3) Choose 65 1x0 <1 cr=2cl > ~6< 62-12 <6 5 < 62-117. ‘Then [6z?~ 132+ 5~3|=[6r—1 [2-2] < 1'x—2[ <¢ when |2—2| < he. #= min(, he), & lim(42*— 1324 12) =3 b Choose §< Leo -l¢r-l eb —tede—4 ed 9 cdr) ‘Phen | 42? —192+12— 4] |4e— 9-1) <2 1] ce whew [2-1] < fe. F = min( Life). We wish to prove by mathematical induction that if im fylez) = Ly) lim fy(2) =Ly...and then Jin [f,(2) + Ja(2) £0004 Jq(2)] = by £1g teeth ® Proof: By LT.4, lim [f,(2)+42(2)] =14 Lp Therefore, Equation (1) holds when n= 2. Assume Equation (1) holds when m= ky that is Jim [f() # L@) 2-0 fal] = by th Salt) = ely 2 26 FUNCTIONS, LIMITS, AND COMEMSUTEY, 12. 13, Ma 15. We wish to prove that Equation (1) tlds when n=l, From. be Jim {fa (#) Fafa) +--+ Su Fegal bt From Equation (2) and because by hypothesis lim /,.4,(2) = ly, the right-hand side of the above equation is (Ly Bly +e 1) £1 yg. Therefore, Equation (1) holds fora =k + |, and henee for overy integer n. + We are given lim f(s) = Land Jimo(s) = 0. Tn onder to prove thar lim [/(z}-9(2)] =0 we must show that, for any © 0 there exists a 6 > 0 such that if 0 <|z—al<é then |f(z)-9(z)|<° (a) Because Jim f(z) = L it follows trom Definition 2.1.1 that there is 6, > such that if0 0 such that if 0 <1 —al <4 then | f(z) |-<1 +1] Because Jim g(x) =0 it follows from Definition 2.1.1 that there is a 4, > 0 such that 0 Jim{ f(z) -L}= Oand dim [atz) ~M]= 0. Hence iF) =e) = Hn (Le) ~ Dla) fae) — MY + a “Elim (L7t2)— Late) + fim {ate} — MI} + 5 We are given Ii MPS o20+LM We wish to prove by mathematical induction that if Him f(x) = Ly, lim f(x) = bey When irr, £4462) - fal) eg E)) = llega a Proof: By £.T.6, lim [f,(2)- #{2]] = Ly» Ly- Therefore, Bquation (1) holds when n Assume Equation (1) holds when ‘bat is Him (fy (2)- fale)---> Fete) = by bao ty @ ‘We wish to prove that Equation (1) holds when u=k-+1. Fe ‘Mi (UF Ce) Fyfe dev Lg) Sygate) = Bem Ute) falz) ): diem, Fega (2)P From Equation (2) and because by hypothesis lim f,.,9(2) = bg.) the right-hand side of the above equation is (Ly “Lop Ly) “iggy. Therefore, Equation (1) holds for n= +1, and hence for every integer o. ga = for any ¢ > O there is a 6 > 0 such that :, We get the equivalent conclusion Suppose a <0 and so —a>0, By what was proved, lg Suppose @< 0 so ~a > 0, By what was proved, lin ye = Ye => for any ¢ > O there is ad > O such that | $= Wa] () and oneysided limits, discussed in the next. seeti ewan Oc fy—(-a) [ea Replacing g by ce 1] 0, however small, there exists a é > @ such that if0 0, however small, there exists a é > @ such that if0 (@) lim, fie)= im, (-3) =-3 (6) tim (2) ® (©) lim F{e) does not exist because Tim ffx) # tim, fz). = a a0 ase={z? BTS) > (a) lim fle)= lim 2=2 (b) im (2) = ie (6) lig fe) dows not exist because Tim f(e) # lim, £12). 2 t+4 tts 3 no={itt if—4 —2 then li (b) Because gfs) = 643 if ¢<=2, then (e) Because lim, s(s) # Him ale), by Theorem 1.6.9 lim, 2(8) does not exis eae ee ee i 8—2r fe <2 PEST { tee © (a) im, P(e) = tim, (8-2) =4 () lm Fle} = lim a? =4 (2) Him, P(e) = 4 by Theveem 168. Qe 4d =z 6. we=[ ist > (a) lim, A(z) = lim, (10-2)=10-3=7 aa eat (b) ling. A(z) = Ning (28 + 1)—2(8) +1 = 7 (6) fim Fla) = 7 by Theorem 1.6.3. arte ifr 0 1 ite S(z] wensi=[o ifz=0 1 ite>0 (6) Because agn x =1 if x >0, then Jaga 2|= 1 if'2>0, and lim 82) =e, jogo |= Him aot root (oy Boe sgn aS (e) Because lim, S{x) = lim S(z 1 se) , > O) | () — © Ji (a) dose nat ta becouse lim _ fr) # tim , #(2) af Nee (2) i Je) = tim VE =e =o te) 1 lim, 2) = “i, 2) a © i 1 J (=) docs nok exist because is Ree ie Fiz). ~ etl ife<-l We fe)={2?— if-1ce<1 <2 ifz>1 y * (a) (e+ I=-1+1=0 * oo nats (-iPa2 (©) “tim " £(2) does not exit because tim _ f(a) | lim, fle). = S(2)= lime? = 1 = 1 (e) iim, He) = clit Set) =2- i (0 Lim fhe) = 1 by Tiki a 1. F(t) {3 Hee vt ifose > fa) is, 10) = ji, vi=o (6) Him J(2)= 0 by Theotem 1.6.3 4 Yor itso male voce : » (ite =lim Ye=0 4 2 (b) tim o(2) = tim {/—e =0 Sea teas —S tim_g(2) =, them by Theorem 1.6.3 ln, oe) =O. a (e) Because fm, 92 16 ONE-SIDED LIMITS 31 © (8) lim Ple)= tim ) “im Pes “im, vo (©) tim, ie) = 009 Teocem 1.63 = VFI weg G)=\Vi—F t-ia ifez0 “1-O=-1 ifr0 (@) Ma) = 0 > 0, ana lim, Ma) = 0. (6) Me) = 1 if 2< 0, and lim Ae) te) peaee xi n dz) # lim" Ke), ‘then ling: h(a) dows not exist, fim, [=1=2 () tim, [T= gia does not est cause Lim ble! la, [el lim fa} = [14] =1 (®) Jim [2-3] =[1-]=0 Ie fa ss tsi ata tims fe] # tien, [eh mt =f aaah lnk ife4—(n s0 G(e) =n +(3—n) =3. (2) Jip, Glo) = tims (ch Because tn, (2) = fim Gla) = 8, by Theorem 1.6.5 lim, (=) Sr+k ifise Jim f(x) exists E{z) exists 10) va ftt fred od only free) lim, eh im Ne) = Nim (82-42) = ~ ‘Therefore im (exist if and only if 20-4 = 14; bent Kaj= wens ifze-t Pek ifr>-1 lim, £2) exists Wand only if lim fle)= lim, #(=). li fe) = tim _ fe-9Sc3 Therefore lim Fe) existe Hf and only if #3 = 14 oe) = Jim, (62 +H) = 20+ tim , (@? +h) = 14k at itr<-2 He)shor+b W-2 such that |f(2)-L| << whea 0 <12—al<é 4 For every ¢ 20 there ir a 6, > 0 such the |He)—L) ce when Oc r—a<5 and there isn 6:50 such that | f(z) —L| 200 > (8) lim fe) = tim 2.27 = 2.2-21 (b) ae 21-50 = 108 fe), Bin fla) =, lim 2-1 (“iim s | 208= = 205.200 = 10 ret) 924-2). 40. F(z} cents is the cost of mailing + ounces. 1 32 sad so lim £(2) = lim,32 = 92 and im P(x) = 92 at sot © {aandb) If 0 <2 <1 then P(e) = 9-23 {cand d) If (6) tio gfe) = 10 - s0f-(1- fe) 7 42. G(z) dollats is the admission for age x years. G(x) = tH © (0) lim Gla) = Tig 4 =4 (e) “hig 34 FUNCTIONS, LIMITS, AND CONTINUITY fle)= a243 ite! at ite et 48. fhe) = 5° and o(2)= 5 Oe Gres OT ee © (a) tm fle) = Yip (2449) = 1849 <4 ond ti, f=) = him, (e-+ 1) Ltt =? salt (b) Jim ote) = ficeae? west tc) F(z) +942) = (24292) #50 (@) Jim | [(e)-ofe)] = (1 + 3)09) =4 and tim [f(2}-o(2) = + 12) = 4 ‘Therefor, Lim n (12) gC] exits and in 4 rhl ife 0 there exists a 6 > 0 such that if0N L172 Definition Let f be o function that is defined at every number in some open interval containing; except possibly at the number a itself. As z approuches a, fe) decreases without bound, which is weitten lim, fe) = if for any number N < 0 there exists a 6 > 0 such that if0 0 and Sf Je) 0 theough positive values of f(z). lim 2 = +c0 (i) ite > and if /(e) +0 tueounh negative values of F(2), lim oe ole) (il) he <0 and if {(+) +0 theough positive values of f(z), fing i iv) if'e <0 and if f(z) +0 through negative values of ff, lim 22! = joe Tie} ‘The theocert is also valid if “r + a” is cepleced by “2— at” or “r+ a7” We can now usually find the limit of a fraction if ether the numerator or the denominator has limit zero. If the numerator has limit zero and the denominator has a limit thet is not cero, then by Limit Theorem 9 the limit of the fraction ix zera. If the denominator hhas limit zero and the numerator has a limit that is not aero, then one of Une cases of Limit ‘Theorem 12 is uoually satisfied and the limit of the fraction is -toc or —o0, depending on which case. If both the numerator and denominator have limit zere, then one af the methods of Section 1.5 often marks: (i) Fector the numerator and denominator and cancel the common factors (i) Rationalize either the numerator or denominator and proceed as it (i). We will learn additional methods as we proceed through the book, but it is important to realize that some limits simply don’t exist 12.8 Definition The line z= a is said to be a vertical eeymptote of the graph of the function f if at least one of the following statements is true, Eack statement is illustrated by the graph below it, (iim, Hz) = 00 (Ui, fle) =e iim F(@)= +00 tin) lim fa) = - T/\ Ba fons +0 im fos)= 7 Exercises 1.7 ‘is Exercises: 1-12, do the following: (a) Use a calculator to tabulate values of f{2) for the specified valuet of 2, ‘and from thece values make a statement regarding the apparent behavior of f(). (b] Support. your answer in part G2) by plotting the graph of f. {c) Confirm your anawer in part (a) analytically by computing the indicated limit. > Let. 0+ and 0" denote quantities that approach @ through positive and negative walues, respectively, Wf p and ¢ are positive and negative numbers, then Limit Theo-ers 12 can be restated ax below. Sec Solutions 9 and 10. @ p/o* = (iil) 4/04 = (a) The able “Gives the values of f(z) for the specified values of =, From the table, f(x) appears to be decreasing without bound as approaches 3 from the right. = 4 Ba a1 $01 3.001 3.0001 7 fte)= 45, 914-151 63.0 603 6003 — 60003 o-# ) sie 1, (4?) = 86 aad ia, (9—2") = 0, Purthermore, since 2+ 3*, shears Sant = 2? <0. Thus, 9-2? Ferran 0 through ‘eehltee vale. By Lint beeen 126), Sin A aato—* | Exercises 13-82, find the limit and support your answer by plotting the graph af the function, 42 £42 Sere Omens Mater ie -t-)_ x ang a Shay i. 242 es et stan Meeeg— ere ‘tin VE vet ot 3 lim, /F-Pe? = VF and lim, x = 0. Moreover, since x= 0*, x approaches 0 through positive values, Thus by sot ant = to =? jn Vitae! = V3 and lin =O through negative values, Therefore vie—3 = 0 through Positive values, ‘Therefore, lim, z tim ¥16= = Because both the numerator and denominator have limit 0, we cannot use Limit Theorem 12. We must factor. Because 2 + 47, then + <4 or, equivalently, 4-2 > 0. Thus, r=4=-(1-z)= -Va-2F ‘Therefor Jawa) cae == tim WEEE "tl fiae Ji-e Htin V a) aaa Because lim 142 = vB and | lim —y4 z= 0 and —y4—= approaches 0 through negative values, by a= ‘Limit Theorem 12{Hi) and Eq, (1) we have lim - 38 FUNCTIONS, LIMITS, AND CONTINUITY 2 a el -)= im, 2 0 itive walues. 2 lim A)= tn, 2h lim, (2-1) = and lim, #'= 6 shroagh positive valu zal Therefore, tim, ae 22, im, 4 = im in Sgae aye 28. lim (249) = 2 and len (524-4324) =0 through poiive values. Hence, tim =o sail 24. Jim (by) > iG = te0. BAL ee2 3 siege oie Toe Moreover, ln (¢~1) = 1 and lim (a~2)(6-+2) = 0. Heeasne #27, then 6 2-< 05 thus (6 —2}(6 +2) approaches 0 through negative S Taluea; Therefore; by Limit Thearecn 2248), ten m (aby -gy)=- _at=1)_} 5 3t Js Gea aaa) oe += 1) _(¢+4)(= 1) = 0 through positive values. Therefore, (32) =0 through positive valnes, lim_(2)—3=—1 and Ii = cay gat aT b Sinee 2 17, then <1. 1f0-< 2 1, then [27] lim [e7]-1=—1 Because 2 Because the numerator and denominator both have limit }, Limit ‘Theorem 12 does not. apply. We rationalize ‘the denosninator zm ete BYES =a Ohya mes “abe tim 2224 ve =) er ae Gee tim 2a 2+ Vea) tin A Vt =P (r= 3 =tim 2 + ve =F o Now tm (2+ y/4=—73) = 4 ond lrg (2-2) 0. Furthermore, 2~2 appronches 0 through negative vals ‘Thus, by Limit Theorem 12Gi) and Eg. (1) we eonelude that tk 5 _attr—6 _ (e-2We+3) _ S48 i Be) a er eae he ae—4) aa aim Ae) = him 1, fle) = =f. lim, fz) = ge = -20. Jim fel = yea 40 si c+ Sr — 2 wis 3. f(2)= ee ae Tim _f(z)= tim feat a lima fle)= j= 20. lim f(z) = arte ad eat at In Exercises 35-44, find the vertical asymptote(s) of the graph of the function, and sktetch the graph, 35. (2) Because tim. 2 ce or became fim, b= too, = Oe veel anymptte (b) Because lim Jy= +00, ¢= is a vertical asymptove (©) Because im y= 20 8 because ies, a pate is a vertical asymptote. ee shoo, = 0 is a vertical asymptote aaa pes 6 a “| “| Exercise 36(2) Exercise 85(¢) Exercise 85(d) (4) Because fim, 4 Exercise 85(b) 49 FUNCTIONS, LIMITS, AND CONTINUITY 36. (a) Because lim —l= +00 by Limit Theorem 12{i¥) or because lim, —L= oo by Limit ‘Theorem 12(ii7), a0" eat = is a vertical asymptote. (00) Because lim, 4) = 20 hy Limit Theorem 12(iii), = 0 isa vertical asymptote, fe) Because tim = too by Limit ‘Theorem 12{iv) or because im Woh= oo by Limit Theorem 12(iii), #20 is w:vertbeal eopraptote, Bxercise 36/3) Exercise 36(b) ‘Bxercine 36(c) Exercise 86(d) 37. Because tim 25 =—oo or because lim, 27 = poo, x = 4 is n vertical asymptote, eat ee : ear a3 deg 2-1 38, Beraure lim 35 se or because tim $7 = 8 = 400, 2 = —1 is 9 vertical asymptote. 89. Beceuse lim =A ao. (=) oi 5, then 2-5 Ties, fle) = ave (Ot because « —5 < Vif r <5, by Limit Theorem 12{iv) ied a Syste | ‘Thus, 2 = Exercise 37 Exercise 38 Exercise 39 . i = is a vertical asymptote, 4, Bocouse im, 579 Sis yp = Sis a vertical asyinplote. _ fa) = 420 oF becanse lim, f(e) = ~c0, == 5 i vertical asymptote ccacieeaipe lor, St) trace m3 3 is a vertical asymptote: L7 INFINITE LIMITS 41 Bt -aih > ie)= ee lm (2+ 6) =O and lim (2-1) = 7, Because 2 — 6°, then +6 < 0s thus (x 4 6)(2—1) approaches 0 rough aeyative values, Therafoe, by Limit ‘Theorem 1260) lim yt vertical asymptote, Nin (@-+6)= 7 andl lim (2—1)=0, Becanse 2 — 1%, then 2—1 > 0; thus (=-+6)'2—1) approaches @ shew sare Jottive valuse Hanes! BP Limit Theorem 126) lin, = too, Thus # = 1 isa vertical asymptote, 0. Thus 2 = —6is a Exercise 4, Brereise 4 Exercive 43 Exercige 4 “Hs Bherciocs 45 and 46, evaluate the limits from the graph of funetion f sketched in the ccocpssying Bese, (a) him fle) =0(b) lim f(z) =~ (@) ten, fle) = +00 (dling F(x) = 0 (0) Him f(2) = 4450 (lim fle) = 20 (8) ita =10 Bm, f(e) =—o0.G) ap Ae) tim too tb) ty) = 2 (6) tim , fea) = +25 (8) lim 2) 0 @) tm fla) = (0) Fim, (2) = 0 (a) tim_F(2) = —00 (h) im, 8) 20 (tin, Fle) = 0 Be ccied a7 aid a8) aheazh toe graph of « Atilon,jttetvlng jho-piven pope “AT. The domain of f is [—5,5]. f(-5) = 0: f(-3)=% f(-1 =O; f= lim f=) = Fou set! Tin, init = =O; Top f(s) = [24 Pile4 Sie + 1) PS czcmt > te oe er ffi =) W-ice The figure shows the graph af 49. lis fc, oe = shoo because r and n are positive. dati ia) = + i 8. fle) = ty and a= 95 (a) i gas = =e s0 lim f(z) does not exist. tim 2 20 Umo{=) does not exist. (b) f(z) + g{z) = 0 if x #2. {e) i yl fx) + 9(z)h = 0 42 PUNCTIONS, LIMITS, AND CONTINUITY (4) Because lim f(z) and lima(z) do not exist, the hypotheses of Limit Theorem 4 are not satisfied. 51. tim a. P= +00. lin, 72 does not exist because 1 (w/e)? <0 if 5. The limit as wae ‘afey? or ett (ole)? v= @ does not exist because neither I-sided limit exists, ‘17 SUPPLEMENT “Theorem B lim f(s)= +6 ond only if lim —/(2) Proof Let M lim fey = +50 44 for every N 0 there is a > 0 such that f(z) > N when 0 0 such that —f(z) 0 and N>o Wo 0 such that HO 0 and N O such that if-8<2<0then ben if0 —n + 0 -N (because rie an ode integer) 0 <2 <6 then (-2)"<-}, (because —2 > and —N > 0) @ if 0<-2< 6 then -2 <(a" (because x is positive) ‘The last statement holds if § = ay. fz i an even positive ipteger we must show that for any N'> 0 there is a > 0 ouch Uhad Ssaien oa ie eit < —2-<6 then Ly > N (becouse ris an even ingen) HO 5 2< 8 then (—2y" Op 9 if0<—2<6 then -=<(h)"" {because r is positive} ‘The last statemens holds if 6 = (L)* & Prove Theorem 1.7.4 ® We want to prove that if im fz) = 0 with f(z} — 1.7 SUPPLEMENT 43 0 throngh negative values of f(2), and Jim (2) = with =>, then lim i, 2. Now lim-—j(z)= 0 with =f(x) — 0 through postive values, and so, by ease () fnt2) oo. Hence by Theorem lim 06, OF livatently, lim=— = —co, Peeper Hove by There 8 as covalent ng} & Prove Theorem 1.7.4(iii). “ Wewant to prove that i lim, f(z) = 0 with fz) —0 throwgh positive walues of f=), mnd Jim a(x) =e with <0, then tim, a. 00, Now lim —a(e)=—e with ~2 0, end 20, by ease (), in 5 = 400. ‘Hence by Theorem B, in $5) & Prove Theorem 1.7.4(iv). y We want to prove that iflim f(z) =0-with f(z) — © 04 thea Bag $2) = ce, Now Lge f(2) = ith Because lin 9( if 0 0 there is a 6 > 0 such that H0N Because lim (} = +00, then for N+2je+1>0 there is a & > 0 such that if0-<1z—al <6 then fla) > N+2Bel+1 15 = min(S,,4,), by statement (1) we get if0N-+Rej+1—2e/—1 er if0 6 Hence part (1) is proved. then for any © > 0 there ix. d fin o(2) =, where e is any GQ) ife> 0, lam f(a) -9(z) = +00 By Theorem By fist f(2) =—0e, By Theotesn 17.7 ife> 0, Jim=f(c)-9(2) =—o0 Applying Theorem B again to the limit of the produ —29, or equivalently, lim ate) BEA) through negative values of f(z), end lim g(x) =< with =f(2] 0 through positive values, and tis 92) =e with -2> 0, and ao, by case (), “int. = 40, or equivalently, fim 82 = 2 Prove Theorem 1.7.6: If cis any constant and f) if lim f(x) = +00 and lim f(x) = « then lim [f(z) + g(#)] = 430 (9) Jim, (2) = 20 and Jim g(a) =e then lim [f(2) +af=)] = =o > 0 such that, eis ad, such that (Dy ‘To prove part (ii) we must show Gal for any N <0 there is a8 >0 such that i O<|2—a/< 5 then f(2)+9(z)<2N Because lim, f(z) = —o0, then for N—®el-1 <0 there is.a 6, 30 such that if 0, lim f(z) p(2) = —o0 (ii) ife< 0, = fiz) -s(z) = +00 » To prove part (f) we must show that for any N 0 such that iO 0 then for €=Je> 0 there ie a 6, > 0 such that HO if 0 — fe PTO cie—al be > 0 (2) Because lim f(x) =—o0 then for Neg there is a 4, > 0 such that #0 —2% > 043) Let 4 = min(é,,4,). It fellows from statements (2) and (3) that if 0.<12—a1<4 then —fl2)-9(z) > —28. ‘This is statemont (1) proving part (i) 10, Use Definition 1.7-1 10 prove that eke > Choose é< Leo that |z+3|<6= -1<248 <1 —Sd> N whenever 6 < ff Choose 6 = min(1,7/}. 1.8 CONTINUFLY OF A FUNCTION AT A NUMBER. ‘There are many theorems having a hypothesis that inchudes the condition that « funetion continuous at & number 4. Hence we must be able to determine if a function is continuous is a break in the graph of f at the paint where 2 =a, then F a discontinuous at a. This situation is included in the following analytic defiuition, discontinuous at a. If ther 1.8.1 Definition The function f is said to be condinuous at the number if and only if the following thy conditions are satisfied. () Fla) exists (2) Jim. f() exists (iti) Jim 42) = 41a) If one or more of these three conditions fails to hold at a, the function f is said to discomtinuows at a. We note that we may have to comider onesided limits and use Theorem 1.6.3 te determine whether or not condition (ii) in Definition 1.8.1 is satisfied. ‘This is iMlustrated Se Exereise 7. If f is a funetion that. is discontinuous at the number a, but for which lie, fle fexivts and is the real number L, then elther f(a) #L or else f(a) does not exigt. Such 2 discon tinuity is called a removaile discontinuity because if we define f(a) = the new function i eontinuaus at a A removable discontinuity will not skow as a break when the graph is plotted on @ graphics caleulator unless the 4 ‘To prove part {ii) we must show that for any N > there is a § > 0 such that iO N (4) Because jim g(z) =< and ¢ <0 then for = te > 0 there is ad > 0 Sueh that i D = Je> 08) aN. Because lim /(z)=—e then for 2° <9 there is a & > 0 such that 0 2X0 (6) Let in(bs84). Tt follows from statements (5) and (6) that if 0<|2—s1< 6 then f(x)-g{z) > — ek N ‘This is statement (4) proving part (i) age is carefully chosen. If lim, _ f(z) ie nets real number, then f has an essential discontinuity at a, and the discontinuity cannot. be rexnowed. The following theorems concerning the continuity of a function follaw from Definities 1.8.1. Often they can be used to determine if e function is continuous at a number. 1.8 CONTINUITY OF A FUNCTION AT A NUMBER. 45. 13:2 Theorem If f and g are two funetions that are continuous at the number a, then Gi) £+97 continuous at a (ii) f—g is continuous at a (iii) f-9 is-continuons at a Gv) ff is continuous at a, provided that g(a) #0 183 Theorem A polynomial function i contintions at every number 18.4 Theorem A rational function is continuous at every number in its domain. 1.8.5 Theorem If n is a positive imteger and f(x) =", thea (@) if nis odd, f is continuous at every number Gi) ifn iseven, J is continuous at every positive number ‘Theorem ‘The absolute-value funetion is continuous at every number. eB Exercises 1-14, sketch the graph of the function. By observing where there is a bron jn the graph, determine fsumber at which the function is diteantinuous; and show why Defsition 1.8.1 is Hot aatisfed al this mumber, eette—6_ (e+ 5)e—2)_ Hea) = oo = Ea -2ite ge 3. ‘There is a break in the graph at ~3, F(=3) does not exist. Hence condition (i) of Definition 18.1 fails ax —3. fey = tegen d OEY rapa ‘There is a break in the graph at 4 F(4) does nat exist, Hence condition (i) of Definition 1.8.1 fails at 4. Pte , F Steet tet-1 foie ep a =) ee af sx : toot ife=-3 “> There is a break in the graph at = 6 (8) = 35) tim, gz) = Gli) tien, gfx) 4 a —3). "Thus condition (iii) of Definition 1.8.1 fails at ~ 3, Hence, gis dlocontinuous at 3. Pose 4 Moi. f e-2 | feet 2 fend > Because z aye +8) _ i sits SA eer) deg then ana! tots ‘There ix a “hole” in the line at. the point (4,5) because G(x) # 2-411 if thus G is discontinuous at 4. We show how Definition 1.8.1 is not satisfied. Beesuse G(2)=lim (2 +1)=8 and G{4) =2 zea Aim, G(2) # Gl) and thus condition (iii) of Definition £.8.1 is not satisfied then 10. Ma} = 8 soy (fee 1 =fF FUNCTION ) LIMITS, AND Ct a ‘There ig a break in the graph at 4 (A) does not exist. Thus, condition (i) of Definition 1.8.1 fails at 4. ‘Therefore, f is discontinuous at 4. H@ej=-1y ‘There is a break in the graph at ~: ‘A(—2) does not exist, ‘Thus, condition (3) of Definition 1.8.1 fails at 2. Therefore, f is discontinuous at ~ Wena ‘There is break in the ‘Therefore, Ff is discontinuons at 4. way= o =-2 ‘These is a hieak in the graph at s0 We use Definition 1.8.1 at a =—2 to show there is a discontinuity. Because o(—2) = 0, condition (i) is satisfied. However lim ,e(z) = #00 and_lim_g{z) = —o0 fe tex -2 s coudition (i) is not satisfied. Therefore, gis discontinuous at —2 1 fed fa) = fe itz=0 VE fcr ‘There is a break in the graph at 0. liza 2) mip —Dand tim, fe) = tm, Ys iterator tim Fe) does not exist. THIS conction GS) Definition 1.8.1 fails at 0. Hence f is diseontinuous at 0. rol ifsel f(z)=51 ife=1 <2 iferd ‘There is a break in the graph at 1 () $0) = 45 Gi) lim. fle) = Tim (2 =1) = 0 and Jim, 1 (a) = Sa 0. Thus Lim (=) =0. (ii) lim f(2) # £2) ee ‘condition {iii falls at 1, Henee f is discontinuous ut 1. 18 CONTINUITY OF A FUNCTION AT A NUMBER 47 P44 fe? = sth=14 t=? b-? itace trea in the graph at 2 ie (P-4)=Oand | Thes condition (iii) of Definition 1.8.1 faits at 2. Beace ¢ is discontinuous at 2. +2 ifes-2 aj2—e Wf —2 There is a breok In the graph at 0. (0) does not exist. Henee condition (i) of Definition 1.8.1 fails at 0. “Therefore, f 38 discontinueus at 0. voi Et ‘> ‘There isa break in the graph at 0. (i) (0) = G5) lim o(s) = Nim —zJ2 ==! and Tim, 92) ti x/x = 1. Thus Jimg(z) does not exist. ie Definition 1.8.1 is not satistied at 0, and Tn Exercises 15-28, f is diseontiquous at a (2) Plot the graph of f and look for a break oh z= a. Does the discontinuity appear to he removable? If so, how should f be redefined to remove it? (b) Confirm analytically. NEF _ pine 2 Define f(2)= 2424 Ws. f(a) ae (2 +3)(r41) ar 3 >» fies To make f continuous, we should define /(-3) = -341 = 8 aso Srtlited a. = VE +9 if 2 #9. Define f(9) = 0-45 1-2 > f(z) 19. J(2)= cc ee en ° Ie) era0 77 VET ASM eres) Ses tga | TAS Patino 1(5)- 2, je =0 _VvEth-Vi_ Jenb- V5 2 IL { 7a OS GEO=8 “ETE EAD a a if #0. To make f eontinuais, we should define | “a5 = 1 pas O° Ee BE a ya Mi-vert_ vi- an ee OS GEE=E “EE- iyas te” ifs £0, Define f(0) = Ped Ly? MET GF=4" (YEP I_a\(Vestay) ” verte tte > fe)= 2. 1) EH, pa» : Te its 46, Define £0) = Sota mM. fee HI os =p a > f(a) = ete (et1PFo1 ia “ery ARF (+1? ie +P see = 1 m tos BaP peep AU To male fecenaons, 1 1 wwe should define f(y) = O-GrPP sora 8 %. fle)= PES aa E49 S-4lz|_(e+9G+1el)_34ie) + Nels SS THel™ Gone eae = wee [re BH, it: #8. Define f(—3) = $43 1.8 CONTINUITY OF A FUNCTION AT A NUMBER 49 e+tl+ 4 dh Jetiles — Ga4sMertlid) sella [erilea (e+)-Meth+a] 2-3 > He) Beg -5. Define /(-5) = EELS PoSpe=s > tim Fl) = tim 543 3-5 6 = 450; Fasontial diseontinuity. 245 “e1-4 2 e-in Sepp tor mat crit Beeauve the denominator is “spproaching 0 and the numerator is nol, Hence the discontinuity is essentialy 5 29-40, determine the numbers at which the function is continuous gl “and etate the roazon. = b fiz) = 27(2 +3} sa polynomial. Hence, f is continuous for all eal number fi) az) A {z—5)?(x? 44) is a polynomial. Hence, f is continuous for all real numbers. $j ie a ational forction. Tence f is continuods on its domain: al real numbers except 3. aes B)= a5 ‘Because fis a rational function, by Theorem LS, fis continuous at every number in its demain. Therefore, Bis continuous at every number for which 2z +570 or equivalently, « #—j. Fs) 2) ap = 82 is rational function, Hence G Is continuoos of its domain: 6@)=523 = ee tional function. Henec G is continneds on ite domain: all real ‘Butabers except 2 and —4, The discontinuity at 2 is removable. He)= (3 oe tx #2 #2) is @ polynomial. Hence f is continuous at numbers other than 2. ell lg a rational fonction. Hence F is continuous on its domain: all real numbers except 2 and = a2? its = Tim Be-1 fim f(z) = fim. ‘Therefore, f is discontinuous at 2. {z+2) if'z0, by Theotem 1.8.5, f is continuous at every «#0. Furthermore, Sand lim, flz)= lim, (4-25) 0 thus Jina 7(=) dose met exist, e}= lim f(2)= Nim (2487 = in, f{z) = Tima (a*+2) = a) ot ae ‘Therefore, lim f=) dose not exist, so J is discontinuous at 0 He (HS DIETS fell # by Pe) ational nein, tenes ime) = im t/(e-+ 1) =f = 70) and tim, f(2) = ea, 1/(3—2) =}. Hence f In continuous at 1 m iinfiy feSbteas fap 12) = lig (1/2) = Sand Sin, fe) = continuous exept at —L and 3. Itz, fle) is a rational function, Hence J is continuous if = #0 oF 9. 249-2) x (8). Mence F is continuous at 3. = ae) (0 HESS Ife# 0, /iscontinuows by Theorem 1.85. lim_ Ae) = lim (2 + 92) = 0 and lim, A(z) = lim, (= = i umbers. ig Me) = liga + YS) = 8 andi, hfs) +) =. Hence fis continuous for all real m 50 FUNCTIONS, LIMITS, AND CONTINUITY d2-Yeites aye ite ® The radicals are continuous by Theorem 1.8.5. 22 and 2 are polynomial and so are continuous. Hence if = # 1g is continuous by Theorem 1.8.2 (i) and ce Furthermore, limn_o(z) (az-Jaj)s2- =o(l) and a 1,9(2}= lim, eyE= ls i=1 Therefore g is continuous for all real numbers, — In Exercises 41-4: (a) Find ¢ and & that make f continuous everywhere. (b) Sketch the graph of f we pont? Bet Tim_ f(z) = lim_(e+7) = 19 = f(a); * = fim, f(2) = "lim, (ke—1) =4k—1 ett saat Hence J in enatinuous at 4 if and only if 40. ofz) = ‘Therefore’ is continuous at 1 iteiaealy ted lim_f(z) = fez k) =de+k; fm, Ae aot Solving c+k = 1 and de ft + 2c fre -2 a. re)= fier if9<2<1 jek le > For all values of ¢ and & the function f is continuous at all =, except possibly at x =—2 and 2 If {is continvons at —2, then If f is continuons at 1, then alin fe) = lim, fle) a8 Soba we get ¢= 3 and & lim (Gee + &) = lim, (82 — 28) pelt 3o+k =3—2k Solving these equations sinvultancously, we get e=3 and &= 4. Substituting these values for ¢ and & into Eo. (1), we have Ft ife<—2 +h Fla)=yor$ 2c <1 or, equivalently, fle) = EF so-$ fice Now, f(1)=1+§=§. Forthermore, jefe) = lim (+8) = Fendi, He) im, (82 ~$)=3 Thos lim f(2)= $= f(a). 7 tert, f is continuous at 1. Hence, { is continuous on {—20, +90). 1.8 CONTINUITY OF A FUNCTION AT A NUMBER 51 Eseries 45-40, i (09-46) sow that ft the gre is discontinuous by showing how Definition 1.8.1 foils, (a) = =O ti, fle) # tim J(eh cental ) 2 1: Bile) = 8 # HOO) removable: define f(1) = ©: ig ste) docs nol exist; essential SB e=0: ‘im fe) = 8 4 f(O};remounble: define (0) =8 (b) += 2 lg Mls) # lim, Fz) event (c) 2=4: lim, f(x) does not esiat; essential Bxercisel S7EGT, sketch » graph of some function f satisfying the given properties. = J is continuous in (4,2), (~22) and (2-4): f ie discontinuous at —2 and 25 f(—2) = Ean (2) = +00, Tim f(a) = 0; sin fe) =% life) = {is continvows on (~00,~1), (1,1), and (1y-+0k tinuous ab —1 and ty F{—1)=0 and f(1) = 05 ‘ey “f(e) and Tina) both exist but neither i 0; neither Tip f(@) noe tm, (2) exis Exercises 49-52, state where the function is discontinuous by showing kow Definition 1.8.1 fi, ite NE S50 ‘Hkz) dollars is the cost of shipping + lb. f(e)=42lz ifs0<=< 200, 052 ite > 200 lig ffs) = 1004 105 = lin f(z). 2= 200: Nig, (2) = 420 #410 = lms f=) 1100" F(z) cents is the cost of mailing = se Fa) =6~ ~23[-2b ® Hn iany positive integer, Tim (2) thn, F(2) condition (i) not slid. (2) cents is the cost ofan & minute cal (= 0-s0l-2) > Ifnis any positive integer, lim o(=) + lit, o(2}; condition (ii) is not satisfied. and f(2) = > = 00: WO<2<12 6) ata nmin a age seu. te) =f WW im v(t) = liu dt? 20) = 4(2)2 + 20 = 30.= (2) and Tis, r(8) = Tim (ABH +4) = 80 (ai? 420)? WOCt? n,m(I6L+ 4 = =(36F HA. At ¢ minutes, A(t) m? is the area, A(t) = ar? = > Him r(t)= im x(a? +20)? = (88)? = (2) and tim, (t) = “SE. [fn isa positive integer, lity f(e) = Jim ==! tennant se) = lime” Ag oP pet 1) = 1414-41 =n. The discontinuity is removed by defining, (1) = 9. $2 FUNCTIONS, LIMITS, AND CONTINUITY 2nr 36. Thefanction f i defined by f(2) = lim, — Sketch the graph of f. At what waluss af 2 i ? discontinuous? & Wecdivide the numerator and denominator by 1 to- obtain the li : F 2 feo 4 | t Furthermore; by seplichag:¢ with O'fa the Wefinition of f; sepa $0) =m S=0 —aH == and J(0)=0, J is diecomtinuous at 2 = 0. 5 Because lin (3) = fi srei={it Esp meti={l EsShuene=(" ESE Vim_ f(z) = tim_{-2)=Oand lim, f(2)= lim, 1 = 1; hence tim f(x) does not exist so fel Me Je ae fim a(2)= im 1 = Land lim a(2)-= lim, = 0; thus limg(2) does not exist. zo" a sot ‘Therefore, g i discontinuoas Ji 0. (f+ 9)(2} =|#| 00 fg is continuous everyswhere; 5A. Let f(2)=2 and o[2)= sgn. Then f is continuous at 0, 9 iv discontinuous at O and f(x)gts)=Vel bs continuous at Uh. 59, Let sai={t fe <4 aad ofa} particular, f-g is continuous at 0. 1 ife 0. Br Theorem 1.8.3, g is continuous for all x and g(z) > 0 for 3c 2c 3. pe by Theorem 1.9.2, f o@ is continuows for all x in (-: eh Because Ee, (Fogi(z) =0= (Foah-8) and lim (fo4)l2)=0 = (Fo 9\(8).F9 i continuous on [-3, 4. = (feats) = ve =16 , fog is continuous for 9(z) > 0. Bp Theorem 1.8.3, g is continuous for all and g(z) > 0 for x¢—4or e >A. ce by Theoret 1.9.2, f og is continuous for all 2 in (—s0,—4) U (4, +90}. lim (Fe9)(2) =0=(F 29)(-4) and lim, (Fea)(H) =0=(F oA, fp is continuous on (—20,—4]U[t. 00). ts) f(z) = & (fegi(z)= vib = Bebcorers 1.85(1), 729s continuous for e(2)'> 0. Sy Theorem 1.8.3, gis continuous for all z and g(z) >) for 4c <4. ig ‘Thepeem, 1.9.2, f'o.9 is continuous forall = ia (4,4), Becatc (foall2) =0= (Fog ~d) and im (f9)(z) =0= (Fog) 4}, Fog is continuous on [4,8 ) fle) = JF ox) = 27 +4 & (fags) = Vera Be Theorem 1.8.5(ii), fey is continuous for a2) > 0. Sy Theoret 1.8.3, gis continuous for all z and g(z) > 0 all 2. feece by Theorem 1.9.2, f 0g is continuous for all 2, that is, on HS. (=) f(e)= Y# ale) = 1 > (rea\e)= vig orem 1.8.5(ii), f og is continuaus for 9{z) > 0. orem 1.8.4, g is continuous except at 2 and giz) >0 for z > 2. ge by Theorem 1.9.2, fog is continuous in (2, +00). im, (fog)(z) does not exist, f og is continuous only on (2,400). 90, 00), 54 FUNCTIONS, LIMITS, AND CONTINUITY (b) Fle) = bp ale) = YE & (Fenix) = By Theorem 1.8.4, J 2 is continuous for g(z) #2. By Theorem 1.8,5ffi), g is continuow foc 2:>0 and g{z) =2 for x = 4. Hence by Theorem 1.9.2, fey is continuous for all positive numbers except 4. Because im, Ue g)(z) =} =(f 2 9)(0), fog is continuons on (0,4) 1 (4, +00) Aand 20. (a) f(r) = YF g(t) = EFT > (fools) = Yet ‘By Theorem 1.8.5(i), fog is contionons for all g(x). By Theorem 1.4.5(ii), g is continuous for 2 > Hence by Theorem 1.8.2, fo g is continuons in (—1, +50). Because tim | (f9)(2) =0= (/os\=1), fog x continuous on [=1, +00), (0) fle) = VerI; ofe)= YE > (Fog\eh=fYret By Theorem 1.8,0(ii), fog is continuous fur gf) > —1 By Theorem 1.8.5(i), ¢ is continuous for all x and g(z) > =1 for z > =I. Hence by Theorem. 1.9.2, f og js eantinuous on (—1,-+00). Beemume lim, (fes}(s)=0=(Fo9N(-1) f° 9s continuous on [-0, +00), Sand at. 10) = VEE ofe)='1 > (fogzb= se V4 —2? is continuous for z in (-2,2); fz] — is continuous for x in (—e0,—1) U(1, +00). By Theorem 1.8.2(i¥), fog is continuous for = =1)U (1,3). Because a 4 (Fe aie) =0= (f ogM-2) and ie (2) =0= (Fog) gay Groote eset =u. Band 2. fle) = VES; of) =e » (eae) =e et =1 is continuous for 2 in (06,1) U (1, +00); y4—1=] is continuous for z in (—4,4). By Theorem 1.8.2(iv), f 9 g is continuous for x in (—4,~1) (1,4), Because (f 6 9)(—4) and (fo gif) do not exist, fog is continuous only on [—4,—1) U (1.4). In Exercises 7-16, find the domain of the funetion. Deterrmis % Ala) = Ey; the domain of fis all real numbers except —5. f is continuous on(8,7),(-8, +20}, [=10, =! B alz) = pEy (—00,0), [0,+20}, (0,2), (0,2), (2, +00), (2, 4-00) > The domain is the set of all real numbers, except 2. Because a rational funetion is continuous om ite and g is a rational function, g is continuous om any interval that does not inelude 2. Therefore, g is conti on (6:0 (0,2), a (2-0) And 9 enti on (tc) (0,2 and (2-0) % FQ =ah ifs santas oon (0,1),(=1,1),(—1,0), (2,400); f is discontinuous on (0, 1], (ec,—1]. 10. f(r) = be the domain of f is all real numbers except 2 and —2. F is continuous on (~2,2), (2,00) and discontinuons on (0,4), (co, ~2, [~4,4], (2,2) 1. 92} = VFF=S; the domain of g is {x | 2?—9 > 0} = (20,3) U[3, 400). aig continuous on (—o6,—3), (3,00); g le discontinuous an (—3,3). Recaise Hi se) =, lis WaF=9=0 = g{-2) and tim gle)= li vi=8 =0=4(3), wat a ib hoe contin cu (~00;--3) and (8-400). for each interval if it is-continuous om that int ): f is discontinuous on{—6, 4], (00,0), [=5, #00). real numbers except I and <1 41.9 CONTINUITY OF A COMPOSITE PUNCTION AND CONTINUITY ON AN INTERVAL 55, fk (-3), Gp. 0.2), 0.2), G21 » The greatest integer function is defined for every real number, so the domain ef f is the set of all real Seber. If n is any integer, then Fe]=a-1 if n-1S2 0} =(-2,2). 2 See faba Sonffrnous on (25), (2.7), and [-2,2) on oa Seago ( 13), [-1,3} [-1.8), (=1,3] gered Ssy-~ Gv) ty) domain of F is the sct of all real numbers except 3 and —1. Beeause F is a rational function, F is continu: en its domsin. Thus, F is continuous on (—1,3), and F is diseotitinwous on (—1,2], [—1,3), and (—1,3}. See Ex.1 18. See Ex.2 19. See Ex.3 20. See Bx4 21. See Ex.d 22 See Ex ifz<-2 {ie if -2<2<2 Continuous on (—o0,—2)U[-2,2]U(2, +20), See Ex.1.6.17 for fg. and limits: Bi uaas ‘the largest interval (or union of intervals) on which the funetion is eontinuous: (z+5. ifec=3 flej=,Vo-2? if-3 Because f ie continuous at 4, then f(-4) |, wo the graph contains the point (4,0), 3, atid the raph contains (0,3). Because fis contisuous at 4 and lim f(x) Because Tim_/() # lim_ f(x), there is a break 0 oe Because lim /(z)= 3, from the left the graph approsehes the point (0,3), but this point és not on the graph. the graph at the point where z= 0, Exercise 25 Exercise 26 __ Bxetcise 27 Exercise 28 In Exercises 29-34, prove thet the function obiained in Exerciser 1.5 is continuous om its domain 29. (a) Ex. 13. afr) = (220—2)2 = 1208 —z!,0<¢5< 120, is a polynomial. (b) Bx. 15. ofa) = 1240 - z)z = 1202 —Le?, 0 < z < 240, ie a polynomial, 80. (a) Bx. 14. af) = (0—a)e it —27, 0.5 S50, is a polynomial. (b} Bx. 16. afz) = 100 oe — 44, 0 <2 < 100, is a polynomial, SH. (a) Bx. 17. V(z) = (8—2r)(15—25)2 = 1202-462? +42", 0 <2 < 4, is'a polynomial, (bb) Bx. 19. V(z) = (12 = 2215 —22)r = 42° — 5427+ 1802, 0 <2 <6, isa polynominl. 32. (a) Exercise 18. V(x) = (12-22)Pz = 42*—4827 4 ddr, OS 2 <6. (b) Exercise 20, V(x) = (40 ~ 22)(50 —2r)s = 42°~ 1802? +2002, 0 <2 < 29, Both of these are polynomials, continuous at every sumber by Theorem 18:3, je 33. (a) Ex. 21. fr) = (20/4 4er), r > 0. 16.4 rational function, continuous on its domain by Th, 1.8.4, (b} Bx. 28, Aa) = (e+ 224 43) = 32 EME ABs 5 9, ig atso a retional function, ML (a) Ex. 22. fr) = 2120/2 + 2x74), +> 0, is a tational function, continuous on its domain by Th: 1.8.4 fb) Bx. 24, Az) = (2 + 30)(22200.4 aa), >, ig also g rational function. In Exercises 35-42, dues the intermnediate-valuc theorem hold for the function j, interval (a, ] and constant K? if, solve f(e) =k graphically and analytically to 4 decimals and sketeh the graph shoving the point (c,). 35. f(z) =2+2~ 24 [a,b] = [0,9]: b= & Tis between f(0) =2 and (3) = -3 and f is continuo on (0,4). Therefore, the intermediste-value theorem holds and there exists a number ¢ between 0 and such that fe) Oye = M1 V5); and {1 + 5) = 1.6180 te in (0,2), 2ye-Pa te? 19 CONTINUITY OF A COMPOSITE FUNCTIOL 36. f(z) = —Vi100 =, [a,b] = [0,8] & Because fla) = f(0) = 10 and f(b} = f(8) = ~6, then f(a) & F(0), and bis between f(a) and f(b}. Furthermore, f is continuons on [0,8]. Thus, the hypothesis of the intermediate-value theorem is satisfied, and we eaa find a number c such that ND CONTINUITY ON AN INTERVAL $7 Because —6 is not beiwcen 0 and 8, ihe only suitable choice of ¢ is:6. The figure shows the graph of f and the line 8. 37. f(z) = ¥265— 2"; [a,b] & Bis between f(—4.5) =}/ 19% 2.18 and (3) = and f is continuous on [—4.5,3]. Therefore, ‘the intermediate-value theorem holds and there exists a number ¢ heroes 4.5 and 3 such that f{c) = a sand — is in (45,3). 38. fe) = + 52-6 [8 > Ais between f(-1) = —10 and f(2) =8 and f ie continuous on (—L,2]. Therefore, the intermediate-value theorem holds and there exists a number ¢ between —I and 2 auch that fc) Fbe-6 = 48+ be—10 = 0; ¢= 4-5 and }(—5+ 5/65) 1.5811 is in (—1,2). 38. fle) = hy [ab] = [3,1 & fis between f(—% and f(\)=$ but ¥ is discontinuous at —2, and —2 is in [—3, I] 73 and there may not be a number ¢ between ~3 and 1 3 such that fo-3 \ Z| peeH RBS eH c= 6. Wut Bin not in (-8.1) 3 4 f(2) = 5553 [ond] = [0.1} & > Hecause f is discontinuoue at 1 then f is not continuous on the elosed interval {0,1} ‘Thus the hypothesis of the intermediste-valuc theorem is not satisfied and the theorem dacs not hold. The figure shows the graph of the function on [0,1] and the | __ Tine y.= 2. Because the line does not intersect the curve, there is no number ¢ that satisfies, bie conclusion of the intermediate-value theorem, 5 —4if-2ee0<1 . fle + [ab] =[-2,4); &= Ae He) a ticecs SUH af 2 A & —Lis not beteren f(-2) =0 and f(9) = 8 and f is not ; * ‘continuous on [-2,3], Hence the intermediate-value thearem does not hold and there may not be a number ¢ 4 2 and 3 such that f{c) 7 3; and —/3 isin (-2,3). f-1205-2 . ye = teaeser HS=(41 » fiacontinuous at —2, and —2 is in [—4, 1). ‘Therefore the intermediate-value theorem does wot hold and there may not be a number c between ~# and 1 such that f(c)=4. The graph shows there is no such number. 58 FUNCTIONS, LIMITS, AND CONTINUITY Tn Exercises 43-48, (a) apply the intermediate zero theorem to show that f has the indicated number of zerox between a and 6. (b) Approximate thein to 2 decimal places. 43. f(z)= a 6243. f(-3) = 6, f(-2) = 7; J(0)= 3, FA) #(2) £13) = 12. Thus there is a zero In each of the intervals: (—8,—2}, (0,1), (1,2). The teros are —2.609 = ~2.67, 6.524 ~ 0-52, 2.1451 2.13, 44 fe) = 284 Tx? 42-5; two sera = 10 & S(-8) = 486, f{—7) ==15; f(0) = -8, (1) =1 By the imtermediate-zero theorern, there is a zero in each of the intervals (-8,-7), (0,1). To two decimals, the seros ate ~7.04 and 0.96. 40. f(z) = 424 Se" +2" —5. f(-1) =: f(1) = 3, #2) = There is a sero in the interval (1,2). The zero is 1.168% 1.17 46. f(a) = 324— 2127 +362" 4 22-8 F(—1) = 50, M0) = 8, FEL) = 1 $2) SSS vero. There ie one in each interval: (—1,0}, (0,1), (2,3). They are 0.440 -0.4, 0.518 = 0.52 AT. f(a) = a8 As? +243, J(1)=1, £(2)=-9. To 3 decimals the root i 1.239. 48, Show thet the intermediate-value theorem guarantees thal s*4243=0 has a root between —2 and 2 and use your graphics calculator to approximate the toot to two devimad plnecs. & Let f(z) =ser4. A and f(2}= 13. Because 0 is between —7 and 13, there is a number ¢ between 2 and? with f(c) =O, Two 2 decimal places the root is — 1-21 49, m(u) = ie continuous an [Oe fi vfeyt \ 50, Let 2 Tis continuous at ¢ = lim f(x) = f(a) > for every ¢> 0 there isa é>O such that | f(x) —f(a)|ce when Ocjz—aj<8 = for every ¢ > 0 there is a 6 > O auch that |f(a—t)— f(a)]ce when O-< (4/28 > lim fle 0 = fa) a time) 2 i Lyte? 2” [imi 7TF. viewed as « one-sided limit it Jim fz) = 0. 52. Prove that if lim f(z) = L, then tim) f(s)|=IL © By the triangle inequality, | f(z} — 12 || flx)|-1L|] Therefore Jms@y= = for every <> O there is a> 0 such that | f(z)—L|0 there is a 6.> 0 such that || f(2)|—|LI|<¢ when 6 0 and f()-< 1. Let g be the function defined by s(2)=H(a)—2 Qy ‘Thea 9(0) = f(0) > @ and 9(1)= (1) 150. Because 9 is continuous on [0.1], and 0 ie between 9(0) and | (2), by the intermediate-value theoccm there exists a number « between) and 1 such that limagn # dees not existe Koso By Eg. (1), we have K(e)-c=0 feyee 19 CONTINUITY OF A COMPOSITE FUNCTION AND CONTINUITY ON AN INTERVAL 59 BA, f(z) = [2-2], We seek the largest value of & for which J is continuous on the interval [3,3 +k). 103) 7]. Wr > 3, then f(r) <7 ifand only ifr? +< 8, that is < 10. ‘Thus the largest value of E satisfies 34+4= 10; &= Vi0-3. 55. Not equivalent: f continuous on the closed interval implies the existence of I-vided limits at the endpoints; f ‘eantinuous al every number in the closed interval implies the existence of 2-sided limita at the endpoints, 1.10 CONTINUITY OF THE TRIGONOMETRIC FUNCTIONS AND THE SQUEEZE THEOREM 1.10.1 Squeeze Theorem Suppose thot the functions J, 9, and hare defined on some opea interval | eontalaing 3, except possibly ab a liself, and that f(z) <9(e} $ A(x) for all # tn I for which = # a. Also suppose that lim, _, f(z) and lim, Az) both exist and ate equal to L. Then Virtyog O{) also exists and is equal te I, ‘The squeeze theorem is used to prove the following results, jim sit Corallany: fig Bo = 1.10.2 Theorem fim = 1 si a eg =e 1.10.5 Thoore tim, =o = 8 ‘To verify Theorem 1.10.2 on a caleulator, you must switeh to radian made, Note alee that, angle measure ig baseel on ate length, which is nol defined until Chapter 6. The above theorems are used to prave: 1.10.6 Theorem The sine and cosine Functions are continuous al every seal number. 1.10.7 Theorem The tangent, cotangent, secant, and cosecant functions are continuous on their domains. ‘Therefore, the tangent and secant functions are continuous at every real number, except Jet kx, where & is any integer. The cotangent and cosccant functions are continuous at Every real number except ix, where & ia any integer Ueful Identities 1—eos?s = sine cos 2 Brercises 1.10 oe % Jt) nde ~ 2 alo Gn oe : 13 4. fim The limit appears to be ; oan ‘To apply Theoroxs 1.10.2 we divide the numerator aud denominator by G2. asin 32 jim Se ae a [er ad tiny a Be 2) 2 se iw ak diny Sin or ~ fim “sin 6 EU ade 3 a tin 33 Furthermore, because both Sz and 62 approach 0 when # approaches 0, then by Theorem 1.10.2 jin 30 de Pc tim 2035 —1 and tim 20 82— fim nfs = 1 roa 8 geno Substituting these into Ea. (1) we have lim Made ny HE 8 tim =5-=f—_1_ =}. 4-3 =0 sean Sz 5° sin Sz 5 ae ga 1 7 tng Se (ith tree ae ante = (be) = 60 FUNCTIONS, LIMIT (D CONTINUITY 8 tim sin? The limit to bes. Jim, si he limit appears to be &. pt ‘We apply Theorem 1.10.2. Because 22 0 when 20, we have Lees vine = fy el 20 fv : Dict Ses) =, 7 =i-050 & The limit appears to be 6. 5] ae Baa ae “as 18. lim —3°)__= i = =14(1)"=12 im l= eo sin’r _ I/j,8i0. lava Ms bag ge Seg = Kutt gen? = Hatta = fin —4_foin 22" oe Ba So) oe ‘sin 22" pita oh is, Cass) Because the cosine function is continuous at 0, tn teat sO ens" 22 cost By Theorem 1.10, " in 23 im, (85.22) 3) Substituting from Equations (2) and (3) inte Eq, (1), obtain tim dan®2 rod dt lim 1= 905 22 _ —cos dz | ‘ile do¢os 22.1 de hye Me ag an i ise in, 20 p so 2sin? in ds 2 we plea sat E pag TEP =I go TP PE eiies (e+3)e _ = wm radeon ae wee) gp = =3 20. ty ane > The limit appears t0 be 3 hp ‘Theorem 1.10.2, we have Spe = tim 82 — en 02. ty ty a Foye Saree a ae ete zl aint. 1 ain 1.10 CONTINUITY OF THE TRIGONOMETRIC FUNCTIONS AND THE SQUEEZE THEOREM 61 ton, tind in a sin 4U/at St = tim, = = tim SRAM = co by Limit ‘Theorem 12 weet Deiat He eat 80 Gain Sefaqpt = 7° PF Limit Theorem 8 mn 280/44 I =o and lim — 75 = =1 aes ae eae Gin WE OE Tasin 23. Leti=fa—z2. Thene=}r-t. lim a - ser Fae m4. lim, (Hint: Let x= 49-1.) » ‘The limit appears to be 1. Ie aye 1. Thus im ft : ; Fae a a oo es a 5. Let t2—m, Then arte. im Stes im Sint + 8) sin. lim, 222 = om, lim, Sat. eae eee 8. Lot (=a—a. Then e tim. lim BEE = an stim =o aisle. 27. Because sin r is continuous for all 2 and 29 is continuous for all @, the composition sin 29 ia continuons for all 9, and so is the product (Vg?/g}sin 22. BB. Let P(0) = pa BY 5 where 0 << 1, Prove that F is continuous on [0, fr). & Because sin 820 and cor # > U1 on [0,5] and both are not seco for the same #, then #sin 9 + e080 > 0/00 [0,34]. Because sin #, and hence kin #, and cos 0 arr continuous for all 9, then by Theorem 1.8.2 (i) sin 8 + cos 0 is continuous for all 8. Heace by ‘Theorem 1.8.X{iv) the quotient F(9) is continuous on [Oi], In Exercisas 20-52, uve the squeeze theorem to find the limit 29. If 2 £0 then -1 0 we must have |sin{1/2)—L]<1 whenever —8<2 <4. Let & be a positive integer > 1/(276) and let 4 = Y/R +H)u and 2, = —1/(2k+ 3}. Then ~8 <2, <8 and dcr, <4, By the Lriangle inequality we get the following contradiction LL >|sintt/2y) =] +stn(a/e,) —1[> |[sin()/,)—b]—Esin(i/29) — b]] a(l/a) ~sin( 1/04) = [sin( 2b +4) x —sinl (2k +4)x]|=|1 (13) =2 ‘Thus there is no sueh number L amd the limit does not exist. Tingcon(1/z) dees uot exist. See Bx td. 46. Yimtan(1/z) doos not exist. Soy Rett, Miscellaneous Erercises far Chapter | 2 g@a)=Vi—e > (a) t)=VI-T= J HV=4—2& (a) Mi)=4at?=3 (6) f(-2)=4-(-2)7 (@) f(e- 1) =4-(2-1)' =-2? +2243 (eo) fa?) = 4-7 Heth) J) _I-(c+ hI : coun, Hexh) 0-2)=yi= i Aid vinz vize-he vine __(l-e-h)-(1-2) ‘Wcsskavice Mviss=ie oles th= I= GFT) = yor gle +h) ~ a4 (eta go, EF se) z xh = “hiV/l-a—-ha Jia) fl-e—kt/l—e 2 Tn Bxeretses 9-6, define the following functions and determine their domain D: a (a) Fea tb) S96) S-3 Lia (oh as (8 Soa te) oF fa Verh g=22-4 (a) tg = ftsd+2"—4, &: [-2, 400} VEHE~ 2244, Ds [-2400) (0) fogs YEP ast 4), De [-2, +20) CF yD (BBY (2 +00} (0) aff = sabe (-2:t00) feet 4142 = v2? —2, D: (00, -VB]U[%, +00) (s) 9(ffe)) = (FF) ~4 = (2 4+2)-4 2, Di [-24e0) fasta 9, 9 = 255 (a) fey = 2-94 VFF5, Di [5 400) {b) F- 2-9 —fetb, “5, +00) (c) f-4 = (2? ~Sjx/e Fo, D: [-8. 400} D: (=5, +00) (e) of F= ech D: [=5,=3)U{-3,3)4 (3,450) ( Flol2)) = (VE F5)-9 =fe4-8)-9 = 24, Di f-8, 400) (a) at ftx)) = ye? 9) 45 = vaF—4, Di (00, —2]U (2, 400) 64 FUNCTIONS, LIMITS, AND CO} 5 fado= vi le) F+9 fe) f9= + ear, iy +00) d= iran Bal sole =a “ =z"? D: (0,400) (0 fete) = wth (00) (a) Fel) = yfp= ph, Di a AO 22 _ate3e-1 “(e=e ty Dee#—Bl Ofe=sipah vets t 2 sie EN 2tU Deg Ot Iie +2) te) i LS e=)-H (0 Kote) = eqs (e) 012) = ee type 2a et) In Erercises 7 and &, plot the graph and determine if the function is even, odd, or neither, 7. (a) 22° — 32 is odd (odd — odd) (b} Se*4 227— | ip even (gum of even) (e) —254+2?— 2 is neither (odd +even) (d) (="+1)/(2? —2) is ede (even + ode) 3 (yee 23 is odd (odd + even) is odd (odd + even) = 0 while P{-0.5) = 9. (a) fle) = 4-28 (b) ofa) 228-4 PD: (-oat00), R: (-2e, 400) Dr (20,400), Rr [-4,+00) [0,+e0} @ Fiz} = VI6 (e) fz) =15—-2! ( ofz)=5 -1al Ds [-4,4], ‘D: (28,408), R: [0, +50) B: (-00, +90), Re (20,5) a ey o)SG@)= () Wz) = Vina? Ds (—00, +00), R: (-20, +20) Dr (20,-b00), Be (-00,9) D; [1,1], R: [0,1] (a) Ge) = WF=1 {e) fz) = [244] © fey=lelt4 Ds (-00,—1]U(1,400}, Bs [O,-t00) Dz {=90,-boe}, R: (0, +00} Bs: (~00,-400), Rt [44-400) MISCELLANBOUS EXERCISES FOR CHAPTER 1 65 fin Kxercises 11-14, determine the domain D and range R of the function and sketeh its graph. (en aie +4) _ Ee Q-2 ife D: (00,429) B: (1,422) UL, $00) =[L, +20) tn Exercises 15-20, determine » & > 0 such that] f(x)— 1] <¢ when ) <|z— |< 4 by finding the smallest 2, and largest 235 and by using inequalities. 1B. f(z) = 22-5, a=3, L=1,¢=.05 > 2x, —5 = 1-05, 6-22, Bax, =-025; 22, |(@e=5)=1]=12r=61= Fx—3) <5 when |e 3] <. 16. f(z) =324+2,e=1,b=5,c=02 » Wecause J is increasing, 2, coerespoudls to the smaller value of f. 82, +2=5-02, 8—3r, =0.2, 1-2, 32, 42=5402, 32,-8=02, 2)— |(e42)-5|=182—3] = ale—11< 0.2 when |e —11< 3 +05, 2m, 66 FUNCTIONS, LIMITS, AND CONTINUITY 1. zy 45510404, tS =O. 6=01 O<|z-5l< 01.5 = 0.1 » fey OLEH D 2eg+h=1 +08, 2p 44 delede el | | (or-+5)—1[el2e+4|= de 2| <.09 when [2-+2|-<.015. 6 = 010 19. ffa}= 24 4,0=2.L=8¢=03 » Because a> 0, then xy, 2, >0. +03, 274.3, 13, 289.1, 2, = EF = 19M, 9-2, = 076. 284 s 2074, cy —2= AA. F= ATA, Choose £< 1s0 -1ee-2e1dee42<5, [tet +4) -8| ale? —4|=]2+2||2-2/< 5]2—21< 0.9 when [2-2] < 06 = 6 Be 20. f(z) =r? =32,0=3,L=0,¢=.08 Because f is increasing at 3, r, corresponds to the smatler value of f. Because a> 0, 2, and 2, > 0) 3a b08 = 0, 2 Yat y= 4( 08] = 2.973. 4a + (574 4(.08)] = 3.026 22-08 = 0, 2» 2,502, 2,3 = 26. 6 = 026 Choose 6c 1s -1ermdelter |@r+5)—5[ [3243] =e + ll0 such that if <|2—B)-<8 then [(42—11) 9) (a) See the figure, f(x) appeats to be approaching 2 as z approaches 0. Oy rota 1 eee ne) an 7 3 i (a) See the figure. #(2) appears to be approaching 4 as z approaches 0. 3 ' 0) Nagra By ge dtaeg lig toe ig, gem tomo 3 iam sin 32 Puy Cob rhb cosa/sin e or In Exercises 61-68, find the vertical asymptotes and use then to sketch the graph of the fumetion, > Note that if f()—L is arbitrarily close to 0 for all large values of x, then the graph af y = f(r) appronches the line y —L-at the extzeme left and sight, o Aorizontel asymptote (sce Seetion 3.7). STL fe) =—coar Tim, fle} = +20. pode retinal anveartte Hine F(ey—1 = 2481 = 212s small when 2 is lange, y= Lisa horizontal sere MISCELLANEOUS EX! ERCISES FOR CHAPTER 171 To find the vertical asymptote, we factor the denominator. i f= —ypERH Because (2 +2)(2—3)= 0 ifs =S.or 2 = —2, we Bind the following Tita Wer > 3, then (2422-3) > 0, Thus, by Limit Theorem 12(i), we have lim, f(z) =--oo, andthe line x= 9 ina vertinl asymptote ofthe graph of J if72 <2-€ 3, then (2 +2)(2~3) <0, 90 by Limit ‘Theorem 12(i slim fle) = too. Therefor, the curve approaches the axympinte = isthe figure. Also, by Limit Theorem 22) im, fie) = 420,90 the line 2 = 2 8 as shown a is a vertical asymptote for the graph of f, Bocanse f(x) is small when # is lange, he line y =0 is a horizontal asymptote. 6. o(z)=1- 5 & Because ie §(8) = 00.07 lim, g(x) =o. ro x ae ical asymptote, Because g(t) is close to 1 when = is larue, y= 18.4 horizontal asymptote, 6. fix) = =P > Beeause tim f(z) = s0.0r lim, Jz) = +00, 22's a vertical asymptote Because f(z)—3=S2=2 3 = 45 is small whea sie henna epagiite or. = > Because im f(s) = coor lim, fle) = 420, 2 =2iis a vertical asymptote Beesute thm fa) = hoo or zs 2 is vertical aaymptots Because J(x)— 5 $25 = PO smal when 5 is a horizontal asymptote. > Beaune tim. fz) = sor fia, Ste) = 495 ing vertical asjmptode. Beeson Tim _f{@) =ta0 or bm, fa) 1 i's vertical arymptote, Because 1(2)—2= 32-2 = 32 is anal when 2 is lange, y= 2 e's horizontal asymptote 72 FUNCTIONS, LIMITS, AND CONTINUITY Ih Brercises 69-74, sketch the graph of the functions then by noting beni in the graph, determine the values of ft which the function is discontinuous, and show why Definition 1.8.1 ie not satisfied at each diseontinuity. ee: fayette? 42 ee 1) 2F 3 *atea) ete ‘There are breaks in the graph at —2 and 1. S(-2) end (1) do not exist. Uenee condition (@) of Definition 2.6.1 fails st —2 and 1, Therefore, fis discontinuous st —2 and 1 fe Ie M1) _ i=+ Me = a £1. There are breaks in the graph at —] and 1. ‘J(-1) and #(1) do net exit. Henee condition (3) of Definition 1.8.1 fails at —1 and 1. Therefore, J is discontinuous at =1 and 1 etl itze—2 feat ites <2 fees ® There is a break in the graph at —! Him ate) = lim Ge 1) = Tim , a(e) =" im | (2-2) = 4: y anaat therefore tim, g[=) tom not exist. Thus condition () of eX Definition 1.8.1 fails at —2, so g is discontinuous al ~2 if + 4 is eontinvous at 2 because aims 960) = tim, o(2) = 0= (2) (sya [ldczl ited mere (St et ; & There is a “hole” in the graph at the point (4,0) because Fla) £14 —a]if's = 4, and thus F is discontinuous at 4. We show how Definition 1.8.1 is not satisfied. Because the absolate-valuc function is continnous, lin (2) Him = (and P(A) = ~2 ‘Thus, lim F (2) # F(4} and s0 condition (ji) of Definition 1.8.1 is uot satiated. A riot Palit Th. ole Az ifes) 13. Me) = pK ee ‘There are breaks in the graph at 0 and 1. f(0) docs not existe Hence condition (i) of Definition 2.6.1 fails at 0 and fis discontinuous at 0. fen A(z) = lim, a? —. =f=h fienee condition (ii) of Definition 1.8.1 falls at 1 vend b is discontinuous af 1. (2-9 ife. Theve is a brenk in the graph at 3. Beg f{z}= lim (2*=9)=0 but J(8)=5 ‘Thus condition (if) or condition (ii) of Deflation 1.8.1 is not satisfied MISCELLANEQUS EXERCISES FOR CHAPTER 173 In Exercises 16-78, prove that fis dicontinuoun a 0. Ite donna a remoraie, redefine (eto remove it c Faas (25 (2—2) f=— -GaG-) +3e F(A) doe! not exist. Therefore f is discontinuous at A Jin f(@) = tin i SPH 8. tthe discontinuity is removable by cedefining f(—) = § » et Week Vay wige! . lim, {(@) =lim_ (4-3) =3 lim, F(2) = him (2r +3) =5 a“ zat ae ent Because lim_f(2) ¢ lim, f(2), then Him f(z) does not exist. Thus, there is an essential diseontinuity at 1. ar! aa ri m1. 12) ={y/"~ e im. F(z) = lim —oo. Thus, there is am essential discontinuity ak 2, 1s. 7(@) = 1228 F(8) docs nat exit, Therefore is diseontinaoas at 3 lim fle) = tim, 2) tim (1) =-1 acs = = Because fim f(2)4 im, f(a), theo lin f{2) doesnot exist. Therefore the discon F: ee 3 In Exercises 79-82, f ix diseontinnovs at a. (a) Plot the graph of f and look for a break at z =a, Doss the diseontinuity appear to be removable? If so, how should / be redefined to remove it? (b) Confirm analytically. @=2-3 0. © lm fle) ig E=# » g(-1) = -1. Denne £0) 0. We nationalize the numeratar. eee ey eee ce = aa4 Ward) nated) “| Te ie erring nae a ae -— 10)= =}. 25 avant B1. fls)=ae a0 & fla) a Jae o=3 Jeeta Jaxtes eee AVEEESS) Jape 43s 0, Define ((0)= VE+8S6. 2 Keagets 1 re Spiess PORN aa Aaa ite Dene Fm v4 Te Ia Bis 3 and (0) toe Jo) emt i fan a ae he eo 83. (6) fle) = Jas ole) = 35 > (feat = By Theorem 1.5.5(i), f'0g is continuous for gf} > 0. BY Theorem 1.83, tis continuows forall al 9(2) > 0 fo Hence by Theotens 1.92, fo 3 contiovoue forall xin (3,3), 0, By Theorem 1.8.8, gis continuous for all z and g(2) > 0 for (—22,~5}U(5, +00) = Hence by Theorem 1.8.2, fog is eontinuows for all in ly (6) £2) = VeFT ale = hy * Vente) = (ats = By Theorern 1.8.5fii), fe is continuous for gfz)+1>0 ‘By Theorem 1.8.4, g 16 continuous for all z # § and o{z)+1 > 0 when p> (e} fla) Sogn 2; g(r) = 2-2 (Fo g)(z)=ogn(s?— 2) Je9% continuous for gz) #0, gfx) = x{z—1) so g(2) #0 in (29,0) U (0,1) U(1,+90) = 1 Hence by Theorem 1.9.2, fog is continuous for all x in 1 4a Exercioes 85 and 86, find @ and} that make J continuous at every susmber, Then sketch the graph of f. rel ifzs3 +l fz<3 suo-fel eres» soy=fite as 325 3 e438 6 # is continuous on (—se,~3), (3,8) and (9. Fac). For J to be continuous at -3, For f to be continuons at 3. Jim _ fe) =_ lim | fle) tim f(2) = tim, s(2) poet a a3 aa i Tim (Bar—78) lin (ar lim, ( —128) wat = rast 90-Th= 31% 90436 =3 Solving simultancously, we get a= 2 and b= -3, 1 if ris an integer 0 Fis not an integer 40 integer, then for all x in the open intervals =0-Thus fim f{2) =0and lim, fe) 0. fo) Ia fa-1,), His not an integer, a isin the open interval (kk + 1) for eome integer k Then for a= in the open intervals (hua) aad (a,k + 1), f(x Thus ima fle) =O and lim, f(s) 0. Hence, fim, fe) ‘Therefore, linn /(z) exists forall velucs of a (© Fie ncomtinuoun at every intogee because tiny Fle) = 0, ba 70) MISCELLANEOUS EXERCISES FOR CHAPTER 1 75 THowever for any nuuaber that isnot an intoger, fim f(z) = Oand J(e) ‘Therefore f is continuous at all non-integers, . ch fn f(z) exits but im f(=) dors nos exist aii Hes ie) L=—Land tim,sga2 = lim 85. Give an example of = function for Any f for which tim f(x) = A simple example is f=) tn Eyercaes 89-92, fd the largest interval or unioa of intervals op which the finetiog is continous. 89. (0) f2)= V2 —=* p the domain of f is {z|25—z2? > 0} = [-5,5). {is continuous on (-5,5). Because tin, f=) = Hm M25 2 f(-5) and Bim f(e)= ti, J95— 2.0 = FO). S is also continuous on [-5, (y) ste) = VE 8 b. The domain of fix {| 2-25 20) = (-#0,-5]ULS, 400) =1. _ fla) _ Ve aH = 0= f-5) and J(3) ie also continuous on I. J is continuous on (90,3) U (5,400). Because lim, {() = tim, soit mast 20. (a) Jie) = EY & The domain of f is {2 [i2i— 1 #0} = (-00,—1)U (1,1) U (1s +00) 5 phe {{ is continuous on L. Since 7 is continuous aaly on 1. ee =e = lim Eth = 2 = 400, Spee et = > The domain of g ix {x| 9-2? > 0} ~ {2} =[-3.2) (2,3) gis continuous on (—3,2) U (2,8). Because Hea al?) (=3), Tim,ote) = lin EP = we toe, and lim g(2) = “BEE = f= 0 = o(8) gis continuous on I. aL. (ay f(e) = =F > The domain of f is {212-2 # 0} = (—s0,2) U42, +00) Beesuse lin (2) = ig = Pls ig (-1) ==1 and Be (0) = Jima} tim1=1, J is continuous only on TL. mae=ay & The domain of 9 is [2] 2? —4 g£ 0) =(=00,-2)U(—2,2) U2.400) = = pha +00 and tims gfx) = lim, wq7a = +00, g is continuous. ra aa only on a4 feed oo. r(e)= {Bw it 1a £4» The domain of Fall = By Theorems 1.8.3 and L.5(i), P is <2 ifz>4 continuous on (—2¢,—4) U{-4,4) UA, #00). Because fim_(2)= lim _(e-+4)=0 and fim F(2)= lim Vi6—2" =0 oa > me ae then F is continuous at ~4. Because ion F(z) = lie V6 — = PU) and tim, (2) = Ti lig P(2) = im V16 a? = 0 = PCA) aod Pe) = then F is continuous only from the left at 4. Therefore F -2 Q=3) {inwous on (—28, 4] (4,20). 78 FUNCTIONS, LIMITS, AND CONTINUITY. Js Brersiees 93-96, does the intermediate-value theorem hold for the function f, interval [ab] and constant £7 28, solve fle}= k graphically and anal 10 4 decimals and sketch the graph showing the point (e,). 98. fe) = 2745 + 15 [ast] = (10,0), b= 10 & The interval from f{-10) = 141 to f(0}= 1 contains 10 and J is continuous on [—10,0]. Therefore, the intetmediate-walue theorem holds and the: amumber ¢ between —10 and @ such that /(e Pndet 1 = 10; 8 de-$= 0; c= 24 and 2— 13 2 —1.6058 is in (10,0). M4. f(z) e*— 4241; [a,8] =[0, The interval fom Al) 166 Fi is wortoes old hoa the intermediate-value theorem holds and there exists Aaumber c between 0 and 10 such that ozs 1 ee tai tendete He and Has ith 5.0086 in (00) 9%. f(z) — wi6= 27; [4,6] > ne neina ri and f is continuous on [0 therefore, hs caine ak Ua kee ad er can Shamberc berm Dan fmach ee eh e- VBI = cp 2 eV det 4a 0-2 He | and =14 7 ~ 1.6658 is in (0,4). ~ Vi6—=4, fo,5] = [4,0]; b= -2 terval from #(—4) = —4 ts /(0) = 4 does not contain ~2, ‘Thus the hypothesis ofthe intermedinis-value theorem isnot satisfied and the theorem does aot hold. The figure shows the graph of the function on [4] and the line y = ~2. Because the line does not intersect the curve, there ia no numibet © that satisfies the conclusion of the intermediate-valve theorems ‘Ty to solve fic) = eV F-20429 VO dod tm ch bP tte 12 = 0 <1 V7 and —1 ~ yz ~3.0458 ie in (—1,0). However, ¢ +2 cannot be ‘pegative, and 00 the solution is extraneous, Treas Ge=4yT Tn Exercises 97 and 98, answer the questions from the graph of f in the figure. 1. (a) io, f(2) =0 (0) lim, J(2) = oo (€) dim (2) —3 (2)lim_ fle) = 00 (2) Min, fle) = +000) tim fix) = 1) lim fta)=4 (8) Fis discontinuous at 3 (removable, aise 4(9)-= 0 (removable, define f{0) = 2}; 2 (esseatial); 3 (eential) 2 (essentially 98 (a) | lim _f(z)= 1 (0) Jim A=) = 0 (e) li i te) (0) ip fz) = ~0 (0 “i, fe) = 0(g) ty fle) = —1 (8) J is discontinuous af “2 (essential); —1 (essential): @ (removable, define £(0) = 0); 1 (essential); 2 (removable, define 7(2) = —1) +20 (d) lim f(z) = 0 MISCELLANEOUS EXERCISES FOR CHAPTER 1 77 Jin Exercises 99-102, sketch the graph of a function satistying the given conditions. 99. 5, -3, -1, and 2 are the only zeros of f: lin, f(z)=4; lim f(z) = +905 ‘im fe) = +00; f is continuous on (—20,—3), (3-1), (1,0), (0,400) 100. fis continuous om (-20, ~2), [-2,4), [14], and (8-400); Tim Z2) Fin Fe) = 8) fie, f(e) = 00; lim, fe) = 2 lim f=) = 4 im, sel att at Bef Si Hn J Oh AY =, dn ih aa a a FL) Becouse lim _f{z) = +50) the line = 2 iss vertical agsmptote and she curve approactes the asymptote from the leh in the upward direction. Because f is continuous from the right at —2 and tim, f(s) =0, then (2) 0 and he geepb eotaias the paint {-2)8), The rapt conta the paint [Sy becouie fie Gontinyous at © and Tiny f(2)==3. Peeause ime fz) = -00) the line 2 = 1 is. wetieal asymptote, and the curve approaches the asymptote from the left in the dawnward direction. Because f is continuous from the fight at I, and Him, f(z) =2, Ue graph costaiae the point (1,2). Because J is continuous from the left at 3, S24) then the graph contains the point (5,4). We are glven that lim f(a)==1, Because i than f i. discookinuous-at 3, aod there is a brea in the graph sf" the potol. where 2 = 3 Because im, f(s) =—1y the point (3.1) i. Hmit point of the graphy but the point isnot part ofthe raph this ia ole bythe open cite al the point (8,1), Fiaaly, because li Je 5, the graph contains the point (5,0), ne 8 and is continuous at Exercise 99 Exercise 100 TOL. F(A) = 2; -2, 0,2, 4, and 5 are the only zeros of fi im f(z) =05 lim. f(2)=—o; lim , fz) = tex; ip AE) = 05 lima f@) ih ob) = Pinca a conn aaa a sot = 100f ie continuoud of (co, ~A), (4,4), and [4, 400}; imgfle)= 05 lim Shey=% lim, #2) = lisa fle) = 0; Jim f(x) = 3; lim (2) = 04 Tm Ste) ie a Sa) = 2 lin foo = ine squares are eit fhom the comets of 3 ‘Tim in ses and the sidan Curmed up, (0) Find the volume (a) in, (b) Find dama(V}. (6) Prove ¥ is continuaus on its domain. (4) Maximize the volume graphically b Va twh = (14-22 )(18— 2e)2, 0 <2 <7. (¢) V is @ polynomial, (d) When 2 ~ 2.6049, Vag = 29286 in 108. = 104. An open box having a square base is to have a volume of 4000 in®. fa) Find a mathematical model expressing the (otal surface area of the bax as a function of the length of a side of the square base. (b) What is the domain of your function? (e) Prove that the function is continuous on its domain, (d) Om your graphics calculator determine, to the nearest iach, the dimensions of the box that ean be constructed with the least amount of material. & (a) The base has side x in,, total eurface § in®, Volume = fwh= 27h = 4000, A= ao. S = area of bottom +4 -ntea ofa side = x? 44h = 2? +42, M00 a? ST (y) Dom 8 ix (0,00) (6) A retional function is continuous on its doraain. (d) Because § = 24 +000 8.00 A constant product ‘78 FUNCTIONS. LIMITS, AND CONTINUITY 8,000, the sum § is least when the terms are equal: x? SQM 2? = 8,000, 2 = 20. The dimensions at 20 in x 20 fn x 10 in 105. A sign with margins of 4 m at the top and bottom and 2im at the sides is to contain 50 im? of print. (a} Fin ‘he total area of the sign, A(z) m7, when the width of the printed region is = in, (bb) Find the domain D of A (©) Prove that A Is continuous om its domain (2) Determine to the nearest meter the sizeof the smallest gn, & fa) The length af the printed region is 30m. A(s (b) Ds = >0 (6) A is a rational function (4) A, 50 yO ag Prs=Biga 2+ 42 +8) = 82 + (82+ 20) (product = 1600) 54 when 82 = 200, g? = te 5, 244 = 5 44—9, 8. ‘The smallest sign ie © m wide and 18 m long 106. The growth rate J fish/week is jointly proportional to the number % of fh and the number 10,000-—2@ capacity. f(2}= k2(10,000—2), f(2000) = 9 = 1000-9000, & = 1/ 100,000. 2(10,000 — £)/100,000, 0 < x < 10,000. A polynomial is continuous everywhere. flz)=H—2*+ 10,0002) is maxiraum when 2 = —10,000/2 — 5000. 107. Fz) = sgn + fet) hy fr past {2 fe+1<0_J-1-1=-1if-10-Wr=0 ite-0 eee L-I=1 itz>0 F ie dicontinuons at -1 and 0 because the left-and sight hand limits disagcee there, 0 ife<-1 In Bxereises 108 and 100, use the squeeze theorem to find the litt 108. tim 9(2), if |o(z)+5|< fd — 2)? for all x, & Because 0 <| g(x) +5|< 3(4—c}*, then —3(4— 2)? < 9(x) +8 Let n be a negative integer. [1 — 2] =n when ASl- gent, nnl eee, (neat oon 2E [-/=(9= TV =n)U =n, fe Bg Je) = limo =0 4 f(0) = 1 e0 Fig not continuous at 0, > 4 LAL, Sketel the graph g(2) = (2~ 1)f=) for © (e- Ile] = n{r 1) when n <2 endl. ing (2) = 0 and y is consinuows at 1. [-82 ~ 4 oat Exercise 110 Exercise 111 12 Suppose {() = a(e) forall valuea of except a (a) Prove that tim {Ce} = Tim o(z) if the limits exist, {b) Prove that if Iir9(=) dows not exist shen lita f(x) does not exit. & (0) Suppose lis f(2) =1., Then For every ¢ > 0 there is a 6>0 ouch that | f(2)—L] 0 there is a 8 >0 such that [a(z)—L| Because f is continuous at 0, £(0) = lim /(0 +2) = lim f(O}/(=) = f(0)/(0). Therefore, 4(0) = 0 or 4(0) = le ‘Suppose f(0) = 0. Then f(r) = f(z +0) = f{z)f(@) = fl=)0=0. Thus f is a constant function and f is wontinunds Suppose f{0) = 1. Let « be any number, Then lin f(a-+)— final fa) f(=)] = sla}(0) = Fla)-1 = Aa) ‘Thus f is continuous at a. it follows from Theorem 1.9.1 that. tim fe ~4)] = £(0) 116, Suppose the function f is defined on the open interval (0,1) and f(z) = 325, Défine fat 0 and 1 20 that f is continuous on the closed interval [0,} & We must define (0) and f(1) so that Jim fC) = 100) and Tim fe) = #0) Now =r =F lim, sin zy) sor {By Theorem 1.10.2) Thus, we define 4(0 Him £0) = ln x, Next, we lel t= r—1. Then sin(rt+ ) te =i SR! im jim sinat Te ee ==s lim ci =n ‘Thus we define f(1) = 21 THE TANGENT LINE AND THE DERIVATIVE 24.1 Definition 2.1.2 Definition 2.1.3 Definition WO THE DERIVATIVE AND DIFFERENTIATION Suppose the function f is contiuous at 2. The tanjent line tothe graph of fat che po PG f(z) 8 (@ the line throxgh P having slope m(e,), even by snde,) =tin £0 82)= Sle) X21) =fin Ss it hin nite (i) the line r=, if tin Sete) FE) 5, aay Os I neither () nor (i) of Definition 2.1.1 holds, thn thee i:no tangent line io the grap of f the pote Fey (2) Tr we want to fnd the slope of the tangert line to the cuve at more than one point, fist find ihe limit and then make the indiated placements for =y- Howerer, we Interested in finding the tangent line at oaly ote point ard ifthe ccordinates of the point tangency ace known, seer to fis! make Whe ndicated replacement ad the Bad the it fHey+ Oa) ~ Key) foo or -asand ling MEDEA MED 0 or 6 The rormel line to & graph at given point i the Hine perpendicalar to the tangent lise att point Formla (I) forthe slope ofthe tangeat lin is special cae of the formula for the derivative ‘function. Following is one of the most important defaitions in the ealeuls The derivative ofthe function fis that funtion, denoted by f/ such that its valve ab mum 2 in the domaia of given bythe equivalent formulas 1 = tim LEASE= HE) = yy HeV= LE) 2) = tim LE +82) HE) ym a and (a! 1'2)=im FERED tm AES ) ona iC thin fmt exis, where = 3 + Ae. (f! i road “f plies” and (eis ead “f prime of 22) We also use the symbols 42 (e) and D,fiz) to represent (2) If y= f(2), then the sym Daan fds ae vont Wid to prent w H Aye dened by Ay = fie+ Ae) —fle) shen Dv =i, 22 1 ay isa pastiealar somber in the dornair of f, then to End J'(z,) wo may use either oft equivalent formeios, Geremicl He)aim, LE) ey “To we formulas (8) and (4), note that Gea)? a 2 + BeAr (Sa)? and (2-4 As) a2? +See + Bef Ae} + (2) ‘To we formulas (3") and (7), note that ere yan ‘The slope of the tangent line to the graph of 9= fe) at the point (2p, fle)) * preibely derivative of F evaluated a! zy. Thus foemlas (8), (3, (4, and |7) ane interchangable “The equation of a (tangent) line with lope m and passing through (2,8) is y= lz) +8 A nortnal me bas slope —1/f'(2) in He= Hon) a or fa) me) (eae bony 2) aster sey! 21 THE TANGENT LINB AND THE DERIVATIVE 81 Beonsins 8.1 Te Deercies 1-6 fod an equation ofthe ingot ina pie, Seth he pap aad gent of he ange hyste Go) winjess-o sn(2)= in, 224) tin, 222) pte Aw equation of the tangent i 2 ya2+4(-15) tim SE) ‘An equation ofthe tangent li 3 yatet +43; (-2,0) Me) 1-9 _ mt gS eee areca e+ (-2)) +414) +4 =A, The Cangent line iy m(—2) = Ji Exercise 4 y= 2-62 +9:(3,0) erine 3 = 68-49, Use formula (4) with ny = 2 9462+ (Az)? 18-62-49 a in SO A2)=J0) py (+8)? a, ae ao (a? = gin GOP = sm axe =0, An equson of te tangent Bne oy = (2-2) +0 9 =0 B y= 48 (1) Pe, & Let f(2)= ae (2-0) mma) = Bal = li@t+sen=3 ‘Am equation ofthe (sient lina ix y = 3(2— 1) +44 6 y=t-z4(2,-7) £2)- $0) _ ala Enatioe §2 THE DERIVATIVE AND DIFFERENTIATION Ta Beers 710, (2) ind the lope ofthe tangent at (zy. f(2))-(B) Find where the tangent is horizontal. Sketch, 7 fay 8d Te fley+ 82) —fley) ee > (0) m(ay)= fin em, Betta =e + Ba) +8 Ga = 1848) in, ese Mae BAS einen 2 (0) fey) = O when ry =2 and $2) =~ {0 Ue gba harm otzontal tangent os (2). a fe)=1-02-2 > (a) Appling formula (2) we have * ayant aioe he) toy ett, pg Tabet ent de~ (ae 74a? pat a nae —22,Ae~(Ae} =i, pa EE “Tin (-6~25,~z) 4 ores (&) may) = O when 2, = -8 and Y(-3) (C218)! Other pins on the graph me 8 f= 2-62 +92 6 so the graph has horizontal tangent at 1). (8,12), (8,18), (-1032) and (1,0). 7 (Pao ve-2)—Ueh igre) PP aap = ‘ “ 5 = Jin GH +08) = fim (a + eye bey?) —8le 2) +8] J a Bey? 124 +9 =e, —42, +3)= 364, 11-8) (me) =e when 3, =1, {0} =2and 3, 3.16 GReGaSa he boizbnud tages at (Land (2) ban? 12 iy Le). py OF =8)=08-20)9 set) 62; = 62y(%,—1) (alee trbn st JO) == Saige etna at a es banc ngn nt a te Pow me im [2a 4x2 +22) —H2 +29) ty cies 11-1 fd UL y= Verh 8,2) pb m= linLORLE) - yy VF 1=3 = hig BAS? tiny = = SS = nya aes a eat “The tangent ne hie equation y = Ye—1)+2= 42+ a(e-3)42= e414 ‘The normal line hat tlope —4 and equation 2. THE TANGENT LINE AND TIE DERIVATIVE 83 VirR (53) To find the slope ofthe tangent line fo the curve at (-B,3}, ws formala (7) wit 4(2)= VATE and 3, =—>. = seated i> I it fiz ‘ m(-8) = Jin 8) Via 4 Sin =a any Pes Use the pelatslope form with = Eto Sind an equation of the tangent line to the curve atthe point (3,8) =-He-(C0)+3 Use the pcint-slope form with m = 6 to fird an equation of the aormal lin to the curve al the point (5,3). pict adatene = in (ES-ES tm [o- (26440) ‘The normal line has dpe Jy and eauation y= fle +2) 44 = fe +B. '— 42; (0,0) > Let f(z)=2'—4z, 42)-100),. yp, (20)'A82 194 82)-10) yy (Oe =A82 28, The tangent line has equation y = —10(2 +2)+4 = —102 +10 (0) = lim, ‘The tangent line has equation (2,1) 44 HEI $0) «yg 3t 9, mO= yaaa = MEH e Sete ‘The tangent line has equation ia, dim, [(a2)?-4)= ~4 ij 4e-+ y= 0. The normal line bas equation y andy =0. ‘The normal line has slope | and equation y= 1(2~2)+ Fett) Toad the slope ofthe tagent line tothe curve at (4), ase formal (2) with f(2) m(e,) tim L)=H) 5 Be yen eg GAM Hal 2) vet? m= By aI aoa ES aye Yo eee OME =n oe ae oe pita with =He- 444) Usps mh S4 THE DERIVATIVE AND DIFFERENTIATION 1 Beles 17-20, (a) Tabulae [/(2-} Ae) ~ /(]/Se when Ae = 010 to 0.01 otap —0.01 and -0.10 to -0.01 step 0.01 and guess the limit. () fo 201 step 001 ard 1.90 to 1.99 1d FQ) wing forma (4). () Tabula [f(2) ~ f@2)\e ~2) when £ = 2.10 ‘step 0.01 and uss the lit. (4) Find (2) wing forma (7) > The symmetric cifferesce quotien: of 2.8 is included for comparsen. It exact for quadratic fanctions 17, f(z) =e! Te ae {0+42)- 02) ee s0)= 102) (N24 Bz) — f(2—Ax)|/2dz Ae 2s 1-42) 102) _ J)- 12) ‘a = (2+ As)— f(2—Ary|/2dz (0) fn BO ant 12Ae 48s? Tae 1 fee a ea 1242) —7(2)_ fl6)~ J12) ee f(2+82)—J@- 2s) dha 10402) -90)_ H0)= 412) (2+ A) (3.87.2) Bet 4 404 Ant) — 7004 An) —(2.22 Tables for (3) aed (c) The limit appears tobe 5. 0. 0.99 08 0.07 0.06 0.05 O04 203 0.02 0.01 21 299 208 207 208 205 201 203 2.02 201 5a 5:27 824 521 518 SAH S12 5.09 5.06 5.00 5 8 & 5 6 5 & 8 <0 09 08-07 -06 -05 04 ~03 -02 —01 Lo Lat 192 1.93 1.94 195 97 198 199 410 473 476 479 482 485 488 Lol 404 497 5 5 5 5 5 5 5 8 219) (9.98-7.3) @ in 2) inst) (ale +2)—7) = amyo—aa0% Tables fora) aud (c). The lle appears to be 12 O10 0.09 R08 0.07 0.06 0.05 “O04 0.03 0.02 0.01 210 209 208 207 208 205 204 203 202 2.01 1241 1255 1249 1245 19.8 1A 12.24 12.18 12.1 12.06 120100 1.006 12.0086 2.0016 12.0004 Ye00si 12.0019 120025 12.009 12.0001 M -99 08 —0T —06 —05 04 -03 02 —01 190 131 182 193 194 19 196 197 1.98 LO 1141 MAT 11.93 1.98 11.08 1170 1.76 1182 1188 1194 048s) 0—A2) ra0i00 aoe, 206 sean 120m ar "2006 ond ad haan no aeasit—# : © ea © ine P43. Ars 3-2ar? 4 A2?—2? 2. 2) fn BS Hans aa a Ines eez yen ia? = ia. (12462-4424) = 12 18. )@)=VO > Table foc a) and (©). The init eppests tbe —f 2 ox» a0 gos cot oae ous or 008 Sor 00! tae OOS 28) Sit SMe is aot 205 20h Sat Fibed= f=) yn ete-ana-ant-atn 0-208-620-2502 posds rato 230012 — 2008 2500, — 21001 ae an nn on a0 as i ike ok ys ae on Pao ih ih 9b as us ise ase tt ae aa eee ~.2485 --2486 -.2488 ~.2489 -,2491 —.2492 ~ 2494-2495 —2497 ~ 2498 £2402) ~s(0~ 4s) aaa We we Thenem 1102, SOF) ed iy HOE = Oo = agents peht=1O1s= i > We use Theeeem 1.10.5. tim C2807 Az) 0060 cow as ano = Taso Ae ° > In(a), lt 2—fr = Ae to get the same solution as (0). - = im SBQe+AS) —sin fe Ae asf came AF = jm « > In (a), le i = Ar toget the same solution as (b). sae so CU) As) con fe fUO= Se ope ESS 26. (2) = 0 86 ‘THE DERIVATIVE AND DIFFERENTIATION mt. Je)=woni, > Were Thesrem 1.103. (a) (0) = mse B20 poll — aL) = nS = Ines flO = 1-0-=0 a ae St madeleage roa) —— » Wee Theorem 1.102. ke a one et finger ais © 1h (a), Wt e— fe = Ar to got the same solution as (b)- in OUT +A) iy cane im tim, ren, B= (nA SM teens fim In (a), et 2 }e-= A to get the same solution as (b)- Vsin(ja +A) esez—eche — eselin +Az)—ese feos Az —1 rhe anegt se sn MS = nee = Im see Seas BS tn Beco 51-36, od J) hy apg fra (3) Bepeyet Sm fm LeFRA TE. ay lg z je)=u0 > e)= fm, 2H IE 3. fle) =T243 Fe) 3. fle) =! Be » se) 3. fla)=445r—2e? > JG) timid -22) — 45224) ROSIN Fw a cL se ve tan (G24 Be 4 Az) — 22 As) (Se 20 oe «ine 940—9 tn Bets 3740, the dea a a H fi8-2) = fn, BHAA), gy Posto teaen tela? Caso aatae— Selah aap eae inte ae 1 [2rbade) + SirFBr) ([2r-48) 4 2ADHGr —2) ~@r-+31(Gr—2) 4 3421 ‘Brlste Ae) 212) tm 24Gr=2)—Br(Or 4.3} _ 1347 = Se apes ay -ae 9 — AAT SD he =13 = (STH ae ‘2.1 THE TANGENT LINE AND THE DERIVATIVE 87 oy a plait s “yg gree (82? [1 deeds J; to 2 al Dataset em al ee ashat| otgia|- iogeas sn 41-44, fad 4 > Let fle) = 3245 (o-+8)-60+8)] =v + es fey 2. 16 fs __i/s fom Ot elm oF - 1 + GaP TE, alg Tea = Bt Sah Tay @ TP (eV) eH (AR GNA) + OY Noe ile yi dined 9 0, Een. ee 1-75 » lt fe)= she Be jn f=1 LL) = tin 2-1) cr 2 vse a aa PTS TERT ae AOD Ho-sty © Let fe)= hy E Here) S00) _ 5, MFA Qe —8)[%(24 Az)—5] Fee in OS sn a IS] — [Me + Ae) athe =4 jim Fe Sie Bs olae | EOIN Ba HRS 4m papas: (eee 4B Find an equation of the tangent line to the curve y = 2s? +3 that is parallel to the line 82—y +3.= 0. > 82-9490, cey =Oe-+3, has slope. Let (2) = 242+ 3. We wi to find an 2 for which m=) = Sz, +A2)~ Fey [2 +Az)*4 3) - (22;'+3) pels fe a = ge, nee | fim, (oy #202) = ‘Therefore, we have 4z, = 8 2; +2 Soan equation ofthe tangent line at (2,11) y= 8(2—2) +11 ~ 82-5, 46. Find on equation of the tangent ine to the curve y= 32? —4 that is parallel tothe line Sr-ty = 4 Fyn Bett haste 5 tt fle) nett Wee Bd ang fon rhch mn) = im FE) HE) = ny O2*—4)- Bey mF 88 THE DERIVATIVE AND DIFFERENTIATION 4% End an egptog ofthe normal ie to he hve y= 2— fe? thai pall oe Ine == Theme sy 0; oryare ha slope Lao live perpendicular ot bas slope—1 Les fiz) 52 et We wih to find an 2, for which (ey) =—1 fey tha)-fle)_ 4, Po Hat aeyl- OF) e =a =< We have —Jay= yds en perenay 448, Pind wn equation f each normal line the curve y = 28 —2z that a pavalel o the line 22+ 18y '> We fist fad the slope ofthe tangent lie tothe given carve, Let f(z) =z" —3e. mayen SAG Hen eel tan, +2) —-Ne—n) (+e, +573) iei2=3 ‘rence the slope of the normal lve is ——}—. eps We find the slope ofthe given line by vnitng the given equation in slopeintereept form, 2241899 Bysate+% ye-petd ‘Because fi te slope of the given nd the normal linet parallel to the given Hin, we have 1 cdl eum teas atah =e? We tave fon the Zeoniiaae ofeach plat onthe given carve where she normal ie i parallel othe gh Meee muouients 02 i te equation ofthe cur, we abla y =2*~3(2) = 2, Thus, 2) ithe po ‘Shee te oral line itesecs the given eure, Berane the slope of the normal Ine iy ¥e may wte Setar aloe fos a he uation. y-2=He-2) 24-00 Similarly, if 2 = (-2)* 3-2): 42a Hata, sty tio 5) that is tangent to the he parabola y = 424 {22 Snce tangent ine ot (2,46) bas sep Bey whi Ihe eu he alt) nd (1) aoe SF we ma ve SE ante? bata tte? tay 44a ead Site the last eaten as no elution thee canbe none through (18) tangent tothe curve S50, Prove tha het sw line through th point (1,2) tha is angen tothe the prntola y= 4—2%- ‘> At any point (,7,)on the curve we have yy =4~2;" Sic a tangentine at (=y4—5,") bas lope ~2r hile the tine through the points (25542, and (2,2) thas slope “71, we mst have Piha ty tint aatay banat dest= ile? ihc the nt uatin has no seution ther canbe no ine through (1,2) tangent f the curve (x —a)g(z), find #"(a). al) (a= 0)910) — fy g(2) = aa), since gis continuous ata. 49. Prove that ther i no line through the point '> At any print (7,3h) of the curve we have yy 1 Ig oningan ata 20d J) ee ie in ae S52 A pi cosinuois a a ad /@) = @?— oP ind He) y= Pie) A=) (5 ge) =) atin a he > s(@)= bm, 2.1 THE TANGENT LINE AND THE DERIVATIVE 89 ean $8 a 4, we th ra (= in PEALE, [ale + Ac} + b(e +4291 —( Sz}= az" +b. a a ne ey = fin aeesanyrhaaorsh ()= gm, BEA SEDO) Se) = 4. f'(2)= lin Fe) hi IJ") ets, rove that (@) = im, tim £452) —fla~ ae) = fn, [OP AD=O, Led= Hom 80)] ig LOAD MO, hy Hed=fle= 88) * eso | z z “aro zor Aro Bar +4 lim, lim, Leary BO+Y'O =!"ehas deine’. Lat f be s function whow domain ie Rand (F) (a+b) ~ f(e)-4(0) for all and L. Pusthermors, suppose that (i) /(0) =1 and (ii) /"(O) exists, Prove that f"(e) exist forall x and that fe Bets ny veal wander 1(e)= fin Et SI= IO) Has Az) fla) Ha-As)~Fl0) yg) 43 Hat (Az) ~ 40) ae See OY ie (ano (a) tim Lala) — Ht ae aye #254 (2) fim LAI=*Y 402) tim KOLO yay (0) Pot the parabela y = 2? and is tangent line y, at (21) and explain what happens as you room in rea in ind(e42)=1. y= 14-2) =2-1 “Dh tangent line and the curve ecome inctinguisheble, «chareteristic ofthe tangent line. [52 Plot the perabola y ~ /F and ite target line at (11) and explain wht happen a# you 00m Bsn un Stn eg nie ii= fort (22 DIFPRRENTIADILITY AND CONTINUITY Differeniable A function is ssid to be diferentiahle al zy if f(z) exists. 2.2.1 Theorem If = fonction ix differentiable at 3, then f'is continous a2 “The theorem implies that if f(r) exists, then there must be no break in the graph of fat the point where #=2,. The converse of Theorem 2.21 is not tue. That is, a fonction ‘hat ie continuous st mj may not te diGerentiable at 2, Por example, the abealutevalue function defined by f(z)=!21 18 not diffeentible at 20 although f is continuoys there. Note that the graph of=\has « sornsr at (0,0). If function fs differentiable at a point, then the graph of the function must be smooth at that point, Furthermore, if a fonction is Aifteentinble i a print, then the tnrgnt line to Une graph of the fonction at that point fot be vertical. ‘That is ifthe tangeat line to the geaph of f a: 2; vertical, then fis not Siffeentieble at x. ‘A fanetion f defined on an open interval containing 2, is dileentiable if and only if ‘both one-sided derivatives exist and ate equal. Ouesided delvatives ace defined as follows. 22.2 Definition If the function f ie defined at xy, then the derivative from the right of J at, denoted by Sule) is defined by retin SEMI ay 90 THE DERIVATIVE AND DIFFERENTIATION ifthe limit exists 22.3 Definition Ifthe function J is defined at x, then the derivative from the left of J at =. denoted by Pfa\hdetna by Pte) atin FBIM) y Go yaty Mm Hed if the limit exits ae = A Fiscontios fom height and i") exis then te, f'@) = Flay. (2) ests then Bim, oP) HC ta contingons fot the et an tim, “) In Berens 1-2, do the falling: a) Skech the graph ofthe function J. () Deermine i jf eantinnous a 3 (6) ind fey cod fate) tng Cs) Dewey aaron : dita t BAS regent >) lim fe) = fim, (2-6) a “in, fp To Fiero On ST ie LOM. esac) 05 fim FOES 25 tim LAM) i ee zHE (@ Sine FLA) # 1-8), 4 dno ext 0 fo denial at A a tof ESE pasate. 2 © Jig. f= ly =22) =; im, f(0)= lim (82-1) ‘| Tere H/C) e t= 72) Td emi (9 S20)= im! 150)= tie £G1= $0) (@) Since $112) 4/42), 1"@) dows no exist 50 snot differentiable a 2 3 fia) =le—31 19) =0. © 0) Jim 1G) = tm [— (2-3) =05 tim, Je) = im, (2—9 (@). Thus: 7 is contimuots nt 3 Teter, Lin 1) (0 £10) = im LILO) F() = tim, LI={O) (Since $118) £419). 1") doesnot exit 0 Fe not diferentiable a 8 4 fla)=14le425 2)=-2 fen Paz) Mece 14 (232) 23-2 “tess (a) A sketen ofthe graph i shown atthe right. (b) Because / then f is continuous at ~2 (6) By Dentin 223, an a a war erg wenn A (0) sg enone of UO t= ono ort spuw o = (z)f amerea (a) saya ogy ye amon «oe ou 70 noT0Hs ¥ (8) eae Be wen re gunman 4 fo 10 = (040 = (0d = (od 28) tS =F Aint = (0) ee we eran Ose ov mongrgo Fg f0/ = 052 “al sop i" 0= Ge) Nat =(2ys “en 10= etn @) 3 208 hg igen “lO p rw xgrnenayy oo a fc ap ye sop (OS Holts # OS 6 weet get ey 09 Se ah . set ae ae oe caer oe tay fe < gizey =e we onan foo (Os =0= 2 Sor ves =O 5 a 52 =orer oars (po = 2 ye ato» 99 Mon a sw apeyenegtp You 9 J St. "HER U OP os mn VEY fsa (PY pee Het ted=2 ey ers Gar ay ee Eos nonmayed £q poe oe Get ges pee eater 2p al sop 9 THE DERIVATIVE AND DIFFERENTIATION Ry Definition 2.22, 15 (2)= tim LELO) — gy VED 25(2)= 9 LE FO) — hm tim, too oer alin (@) Because £52) =+00, {"(2) does not exist, and J is not diferentinble at 2 In the figure, note th pea {VITg Ht jayac-nten ‘ > (0) fim fle) = Hin YI=E=0; tim, ((2)= tim, =a) =0 ‘ ‘Therefore Jn f(z) = 0-= (1)."Thusf i continuous at 0. a a (0.0) in 2 ff li aie 5) = im, =f) cy, O=s=0 a amas unui bauacucy Seaman 10. 100= {2a | ay=t . Bn F(0)= Hin (1 =26)=4 4 5 M2) = 1= F1. Ths fs continues at 1 OAC tie LOAM jig Pate tn ce ~ Ns ea) a at =. nets tim SOLED. py, CBE + (@) Since P(-1) = (1) = 2, 1) = 25 0 fi diflerentable at —1 wea QRot EESt pyanat—ass. (i) im J(e) = Jin et =9)= 5, a, fe) = im, (Ge -)= 8 a Tees ag J) = 9% 2) Tho fn oon a 2 ‘ (© .1@)= iw m 22H I=8 _ yy MO-2Ke- sag Ge=1)-5_ y_, 80=D_y a ig (@) Smee $102) = 2) =8 f'2) =8: mo fis ferentnbe at 2 we se)={feniy Esa | a? Ga) A cate af te grav i shown a che o (b) Because J(3) =6(8)~18=0, and : ip JQ)

1 26. (0) 12) oc) set i ee Oh) cepa 120) = 140) ~ fglI=FO (0110)~ BqtSEIO= sy Poh £) by e184 -1_ (F,0)= jim FSET = {h) J i ot dtereninble at 2 oeause f° (-1) 4 (98 1 boease J) dove wet exit 8, The samge of fs 10,450}; f fe ifereiiable at every number acept =9, 0,9 £12) fe) F10) 1) reise 29 2. DIPFERENTIADILITY AND CONTINUITY 7 © of fie Ry f" xiate except at ~2, 0,25 f(-2) =05 4 f 1: 1,0) range of f is (~20,t00); fis differentiable at every number except 0 and 4 f(—2) £2)~ 40) set tin L2)— 1G) hott 145 aan 24 nmi re) = [22 HOSES Noord ese = Bn tim = 12) = ia! Jin 44 20)= 36 yy A= 16 = si a(t42)= ats JL in West lim 16 = 16. Since (2) = v2), ris diferentable at 2 Pa Vife <1 and 1-2 1a $1. im fe) = i (2? =1) =8= fim 9G) = Jn, my. f-tpocontinnnn 74-1 2 ‘tm Wee “= {2.57 HUSES® (oy rnd uo ss comtinoue a (0) Shah (68 f ditt a 7 FW) =P; ig Ko) = =e = ee a a 2 Be icienen tiiisjs— Onno e-nesiyers ae a ie: f is continuous at b if b sliver -2; since b> 0, only isadmissibe. = ~* P ve F.0)— in LDL hy ated = fy CED a és = 70) Saar a Ba *)=0/0= tig, B® clin, Tim, $2 =f once (3) dos ot exist nd fis no dione at . ifr 0 (b) im P(e) =O and “lin, f=) =O. (c) Sketch 2 qi Lia)=10) — ae = See e al H40)= jim, wt tin, Eg = 0 { sm [af [mS nF) = tin, [.0)]= tim [im EI] in, =0 ‘ : es} Find a and & such that fis differentiable at 1. Sketch q ity we need 4 ay SIFY 2 g2}=20) 1575 im eye dig lor+8)=204 li, 10) in, 24-1) =7 Sh ws i ison at To a0 +827 Ps LED yy BAD AT gg EDGED, ale —2) 98 THE DERIVATIVE AND DIFFERENTIATION tig, C2MAT Sg, 6 {willbe differentiable st 21/2) = /4(2)in0 a= 8. Since 20-48 tn Fxereses 37-40, treat the vasiabe representing # nonnegative integer oa iit was a nonnegative eal aumber, BTA tap is fer up to 250 shudents. For up to 190 stodents the ost i 815 per student and decreases by 30.09 per fudest foreach sadent aver 150. (9) Expres the Income as function fof the number of students (b) Prove that is eentinvous on its domaia.(e) Ef eiflerentiale at 150? 1» (2) Let f(o) dollars be the ross incom Ifzstudeats make the ip. iee2c150 _ [152 0<2< 150 18h <¥ £250 Yaa.se— 6s? ABO wae (me oce<% (600 f020 gone 18: ita << 60 (&) s(20) = 60420) = 12000; mp2) 124005 fine) = im, (02 — 1594 ‘Therefore, ti fle) = 12000 = /(20). Thae continous at Oe n 600 29) a= f fa fle)— 100). ii £400) 7 Hei=f00) ns (ae 00) 00 F400) = Since #20) #7',(20), J/(0) does not exis 40.4 cub's annual dines io 8100 per member, les $0.50 foreach member over O€0 and plus $0.80 for each ‘merce lus tian G00. (a) Find a mathematical moda expressing the club's revenue J(e) a8 function of the ‘umber «of member (b) Prove that J i contimons on ie domain,(c) I f differentiable at 6007 22 DIFFERENTIABILITY AND CONTINUITY 99 (6) f(z) = e110 — {(2—60)] =2(400~Jx)= dade}? 0< + ¢ 200 (8) and (€)Beconse fina polynomial itis continmous and diferentiabe om (0,0. Let fle) =I21 Tind (2), 220 HF 20, then since 121= Vet we havelz? = 220 = Jin, f+ 28) Asiviel_ ,, (et Ael-iaide+Az/+i2) Cl =o Ae ly Bafa tele = (e#z—2? 4 _tede (Asi tthe _ 99 lal Bale +Az]sia) arto ale SeTtie0 avto[r+Ar[eizi” Sel” = Given f(2)= fe} find Joy) i i ot a integer Prove hat £2) doesnot eit, isan integer Ife, Sau iteer, wlat can yousy sbout f” (2) and $4 (2,2 fs no an integer, then thee ison Integr Wh that N ) F(a) = 27 and Gle) = 2" s'(@)E s(0). Thereore, by Definition 1.8.1, is continuous at then (f+ 9 all real numbers then (F + G)(e) = O for 2 0. 100 THE DERIVATIVE AND DIFFERENTIATION 2.3) THE NUMERICAL DERIVATIVE ‘Symmetric Tiflronce Quotient of f at «ix (/(a+Az)—fla~Az)/(2d2). As Az—0, this approaches £"@), NDER(J(2}2) = Brerenes 23 In Raereoe 1 1. See Byercise 21.17 seomgare th GMally muuch faster than the ordinary difference quotient used in the definiiog, sd Nonetimes approaches « limit when (6) door not exist. The value of Ax is called the {erance, With aay tolerance, the result 6 exact for lnear and quadraste fonetons. We shall ned a fied folerasee of 0.001. Thus f(e+ 0.01) ~ (20.001) a TH [Note that some graphics ealeulaters can plot the tangent line to a graph at any point ad give the valur of RDER in a single operation. Hf your graphics calculator can use ‘tes of eifrentiation to get (a decimal expresion for) the exact value DER (DER? The exond derivative) of the derivative, You should use that capability (exept in Euertes 27-40), The FRAC capability may’ be usol to get the answer as. fraction: we aymmetie diferente quotient with standaed one forthe given Exerese of 1 Mj See Exercise 2.119 A. See Exorese 2.1.20 2 See Bxercses 2.1.18 1 Brercias 8 plot the geaph ofthe numerical derivative a= t2 support the wniue found inthe piven Exerc B. (a) Bxeeise 2.1.38, 6. (a) Exercise 21.38 7. (a) Exercise 21.39 8. (a) Exercise 2.1.40 (Exercise 21.35, (0) Beers 21.80 (6) Exerine 21.41 (6) Baereise 2 (c) Bxemine 21.87 (Bern 21.38 (6) Beri 2143 () Beorine 214 I [ft | Li. a a. ta Exacies 9-20, (2) ue NDER to find the slope r of te tangent Tine to ft (ey) her ite the squat of the angen line Tc) plot te tangent line ad the gph of 9. fle)=(-1he =? iQ) aie m= PQ) =2 Ue y= He 2)+ ATs ya dle-pt)—1aae to 8 10. fz) =2 422-24 2 ML. f(e)=#8-22—! (e-1) late 18 12, fle) = (2-2 45, = pois 4 D vy = fa) = 2A) 45 =. Using NDER we find m = /"(1) =A. ‘Therese the oqation ofthe tangent line fe Ae t)+ 8 yade=7 VPA a, (3) = 8.10 ie =. Tee 1.66.2 5.8. a ba M8) = 4. maf") = 08. Te y= TE) tA TOG by TIO) =O m= fi) 0A. TE y = Oale—1)+0 = 42-04 (15400001536 (using NDER) “Therefore the equation ofthe targent fine it y= 0.64(2-42)—0.2, y = 0.642 +L 2 2.0.THE NUMERICAL DERIVATIVE 101 (1) LBBB, y= LB81G(2 1) + 0.8415 = 138182 ~ 0.5103, = (2) 8.9018. y = —5.3018(2—2)— 1.6016 = /S018e $8,938) (2) -Q91T. Ty “08312 2) —0.4092 = ~088072 + 1.9591 f(s) =ten(sina): =, = tan(gin a) 0.421 m= #9) = ~1.00987279 (using DER!) = —1.009972957 (using NDER) “Therefore the equation ofthe tangent lines approximately y=~1.0100(2—3)=0.181, y= —101002 +3720 and 2. Prove that i fis « linear or quadzetic feelin then NDER(/(e),2) 8 xacly JG). Let f(e) = 42+ be + (e may be 0) NDER( Je).2)= AE 92) SE 8) _ let e)+ Wer a) te) [ale aay? +He— Ae) +e] 9 the et + tance + wae? te HBAs) —[ar?— tere aa? +82— 132) eather i By b= Fle) Beers 29-26, (a) plot f and NDER(/(2).2). For what values of ofr is (9) NDEI > 0 and (e) NDER< oF ‘what values of = does f(=) appear to be (2) inereasixg and (e) decreasing.) Compave your anavers in pants snd (@) and in pars (c) and (. (9 The answers are identical: a function increases if its derivative f" i positive ard deceases if is negative Heaay2 » (b)and (4) => 0; @)and (e) 2 <0 fea ue "The gue atthe right tow f ack anh. a (band (2): NDRR 0 and (2) neoning it <0 (6) and (): NDER <0 aod fo) decreasing itz > 0 J f()= vi-e » (b) and (a) <0; ©) and (@) 2 >0 + 7 se)= > (b)and (@) 2>2:(@) and (@) 2 <2 4 Let f(z) = 21!, Compute NDER(f\2),0) and explain why it exists even though {7(0) does not exist DER ~ (001479 001)*/9y.002~ (.14-1)/.002 ~ 100. NDER always existe since we don't divide by 0 Exercises 38 and 29, (a) show that /'(0) dees not exist: compute NDER(/(e,0)(b) by formal (4) and) by {) Explain why NDER exists. (e) Plt the graph of NDER({(=),#). () Compace ie behavit at 9 the rena of (0). Be. us (6) 0) = Beg fad Feyays oor! — (01) a (b) and (6) NDER(2!70) = = 25119, (4) NDER always exists ance we dow’ divide by 0 (6) The figure st the ight shows NDER. dock and flight. lin. oh Se)= 29 1100) = Jig 2 ge = tig t= ~c $.0) Bernie J 8 an even fonction, NDER(S (2) 0) = als Bh incominliy in Eanes ten nb iin es cry 102. THE DERIVATIVE AND DIFFERENTIATION 2.4 TABOREMS ON DIFFERENTIATION OF ALGERAIC FUNCTIONS AND HIGHER ORDER-DERIVATIVES ‘The eifrentinton formalas that ace preved inthis seen tay be wed to dlfereatate any rational fiction aud should br menorae. Algebra fondo ate dscused in Seton 2.0 241 Theorem Hee constant, Gicncant Rare bye=o ‘he derivative ofa constant i aero 2.3 Tene Ua comand ff) exis et Dife- Ha}} =e-D, fle) The devine ofa constant tie tncon the kom in We davai of de fenton he dea ont 244 Tieres, If Df) and Dag) 85 ‘Sun Dye) 2) =Dafle) + Date) ‘The derivative ofthe sam of fanetions isthe autn of tis derivatives if these derivatives exist ‘This rule holds for any nurnber of terms. 246 Theorem If D,f/(=) and Dys) " Dy{Pe)-afe)] = Se) -Dga(2) + af2)-D_fl2) = DS (2) ole) + £(2)-Dyale) "The derivative of the praduet of two fonctions is Uh firs fanetion times the derivative ofthe second Sinetion. plas the second finetion tims the derivative of the fst fumetion, f these ‘evivatives vist. Warning: In the frst form, the produet and quotient rales have their temas interchanged; in the second form the prosnet and quotient rls are in the same order Gonaral Product (Peorsion 53.) I fs) os) Me) are dillorentian Rade [j(2)a(z)h(a2)l = f(2) se) Mz) + fle)-2)- Te the fi brn, item the ot fc nthe oon fieon, es Tie rule hold for any number of tors 241 Theorem If s(z) #0 ard D, f(z) and D,o() ex, then ‘Guotient ale + fle) ale): h(a) rw differentiate the nevond (2) Fle}-Dasl2) Tote) ‘The derivative of the qutien: of two function isthe denominator limes the derivative of ‘umerator minus the numerator times the derivative of danominato, all divided by the ‘ofthe denominator, if Ue derivatives ex 24.2 Thewcun Ir is « pnitve integer oF + a negative integer and 940, then Power mle - e D, (es!) = era ‘A power in the derominstor is moved vo the mumerstor, changing tbe sign of its exponent. formula for the derivative of the power function vas proved only for the case in which ‘exponent r is an integer. In Section 2.0 ve show that the formula beds whea + is any ra fhumber, and in Section 8.5 we show that i is valld when ris aay seal number. When convenient, we replace a given product by an equivalent sum before diferent ing, because the fers fr Use desivative of a sum is ensiee te apply shan the formala for disivative of a product, a6 iustrted jn Exercine 20. Also, if posible, we replace a gi (quotient hy an equivalent sum before difleeatinting, as ilustraced in Bxercise 82. igher Order ‘The fit derivative of te function ft denoted by J; the stoud derivative of f i the fw! Derivatives derivative of {an is denoted by {¥; the third derivative of fis the fist derivative of "sae Is denoted by 7; and 0 forth, provided theve destsatives exis. In Is an integer greater thas I, the nh devivative of fis notes by 7°"), and f°) i the fine derivative of "7 Fe pig) OE (0), £8 = prey 4 = 12), and s000 (e) Ba 10), £3 = Pe) 4 = 12) ead Furthermore if y = f(2). then ‘24 THBOREMS ON DIFFERENTIATION OF ALGEARAIC FUNCTIONS AND DERIVATIVES OF HIGHER ORDER 108, Teibnis Rule (Bxercse 58) If all derivatives exis, folowing the pattern ofthe binomial theocem, Ua) = S94 289 + 19 Sal" = Sot fa +a" + fo", ote rinse Bd Exercises 1-24, diferetiate the fnction by applying the thearems of this section. f(a) =D,7z 5) = 0,72) ~D,(3) =7 =) = Dy{8 8) =0,(6)~Dxde) = 3 (2) =,{)—2e—2) = 0,0) + D,(-22) + D, whet] Dade?) +, (4e)+D,(0) = 402s!) 4404s") c0=ae+4 #2) = Dafa —$22-4 52-2) = Dete!) + D827) +0,(62) + D-2)= 327-6245 f(a) = D3" 518 1) = D,(be4) ~D,(622) +D,(0) = Its? te Fe) =Daffe? 21) = Dj") +D,(-28) = 27408 H455°—Te FE 1525-8) 4 5829-1) — 1621) = Taf et 4 et 7 Pie) = DiGet He) = DGe) +D (40) = =D,G2")-D,(2)+D,@) =" (e)=D,{o? +42. +4) =D,{z*) +D,(3z| +D,(2) een {e. ie) =, (tet a1,) = Dae) + Dye Ao Ppa t PO) =A 94 2. Ae )= y+ 8) =D) +D,6=4 B=) =0J 54) = Dyer) = Be 5) = Pyle 52) = YD, st) = Blast — 29) = 0214 5)ite—1) fexpees (3) aa sm and ne the le ia) 82a 4 Me 5, aa) =a? 62-420 TE SOLUTION: We uae the product rls. 2? +5)D,(42 1) + (32 1)D,22" +) = (224 +99) + (Ae — 1K) Ms? 42 £20 D,l(2=*~1)(S0° 4 6a)] = 22*—1)D, (52 +62) + (62° +62)D,(22*—1) = 22" 1)a9e" +0) + (92 + 62)(8e") = we" +1224 1529-6 4 402" +488" Fes) = Dyll4e? 4 9)7]= D,|(42?-4 le? 42)] = Dee? 42) - (40? 4 9) (ee? 22), (4=? 48) = 24a 43) = 609 4B 2420 + 162 ae jus" + 024 — 152% —5 104 THE DERIVATIVE AND DIFFERENTIATION 2. Gy) = Dy l(t —359)7] = Dy? 4A) By] =F — 307)DYIT— BP) +(7—3PYD,T B09) —3P)(- B02) + (1 -3y)(-994) = —1897 394) 2. FU) = (264 NP +30) We use the product rule, PG) (Bae (ee +20 + (24-4 30D (0 “a1 1) = (Ba trg NL -E)+ BF + 207-2) Beta 40+ 88—6t 43-4604 90 — 4? ~61 = 10 = 127 — 12? 8143 ALTeneat® 90107708, F F() = 2043 — 4040431 In Pxercines 25-16, compute the derivative, In Exercies 25-0, check by ploting your answer nd NDER. 3. Dita? de# 202 +1) =(e! Be $2)D,029 +1) +(229 + D2? 2242) 22-32 4.2) 62) +(229 4 1)(ae—3) = Got — 18s? 412s + det Get 4 22-3 Ox — De? 4 122? + 22-3 D.fee)- (e+ 9) —200,(e48) _we>8)—20 6. (25) ; aes G+) ray = Wye ad, 2 ea a vee) =e i $1) Oot Dita 1) bya +4) y= we fase ay +4N2)— v4) __5 i Gur? wea 2 (tte 4 1)D,(02 422 41) [29 422 4 1)D (Be : Ba 2) _ 2 —1)%(2+1)- 22+ Ne D 1e-F (+1) = (-8-2nfe-2) tee) soa e-3F a {) = 4102-202 _ 5-102 + athe aP) Grae “aaa? Ga 2. ofc pent) > We divide and we the formula forthe derivative of « sum. ptoresiet). di aet ise 4e Hde*— 1524-4) ae 38, £8). +Oar) var wire __ so? id Oa 4a) +8)" e+e re fat) ALP ses) toe? 40%)— (ads __ tate TAS a= ere} rae ae a. o2efplce—] =H), -—1)+ 62-002) 2 Ges +e fe 2S= CE tet eR DP yat a Get sWe-+s) (oe o “het s96 41542709 _ fe? 460e-+6 _ Ber + 1e +1) 2 Ke = $4835 $1942T2—9 ty GreP (Foy co ee) 2. THEOREMS ON DIFFERENTIATION OF ALGEBWAIC FUNCTIONS AND IIGHER-ORDER DERIVATIVES 105, aa +) nftatog ote +1 Fas Fas fi P+ Se)D alot AP 4 2-2) (oF oot 242 2)D (2 432) . (erie) at seo yaa 30 oy Gp tp 2G 8) _ 5 4 1 get oa 42 6 rar Fete 37 and 38 Gall the drives off Ge) = 62! 4 get 308 et be 8, He) = Pj asides ae 12, fy = Moe. Pj sti se Apa re oe ser Fon ae be 1 a Tehp)= 02) 2 +80, $4.2) = 168023 — 1202, fx) = 50802" Pe) = 10080, fz) = if nd 8 Pa) at 1 Pa (5 via tf ~ 3) DDD AD, 202? + 360-120 + 10 ig) ea 10g ee) faz" — 09 +30, 129, Fe) = 10080, (DAF) = BO} = Dee) = 108 5)) = D_D,D,(5e4+}e°*) = D,D,(002? ~22"7) = D,( 60" + Me) = 120 1125" Bxerenes 4] and 42, fd ©, Check by plating your answer and roureulstr’s seotdeivative tat 7 babe yf ate 229 yf 24 Het 8 yatta yf abet ate? Exercises 4) 46, Find an equation of the tangent fine or noual Hine, t the points Check by plotting the line the curve. y= 24, (24), tangent line» Let f{e)=24—4. Teen M2) = 82%, 20 "C2 tangent line has slope 12; 0 an equation of the tangent line is y = 12(e—2) +4, y= 122-20. y= 8/(22+ 4) (2,1), tangent lin > sma he soe ofthe gn ine the vale ofthe erative, neat eae Fintan Cifioa reef tevin sions] cote he a i = 10 normal Fine tat s¢2) = 20 e)= we gldgp GB). onal line» Let sta) = Hy Then fe) = (7M) wee ee a -) sip op tony y=t2?—Be, (1,4), normal line © Let f(z) = 42? 82, Then f'( Uojudppwlgeye ye yar T. Pind an equation of the line tangent to y ~ 327 — tw and pacallel to the line 2x —y 43 ~ 0. 4z. Thea f'(z)=@2—4. The given line has the equation y= 2243 and bas slope 2. ‘non ak Bose (8) ~ ye dal henge ne pe rongh (he) and bas ope Bsa eqain n= Hei) le y=2e 2. Find an equation of ech of te tangent Ins to ec by ploting te cre andthe Bee Be, Sede Tiere! om nd et (et elt at wba ‘is tangent to the curve, The slope of L, is given by m(z,) =v'(x,). @ @ =4)—Sor ae % 8; 40 J'(1) =8. The normal fine al 324044 that is parallel to 2r—y +3-=0. 106 TH DERIVATIVE AND DIFFERENTIATION Brcaust the slopeinterceptfeem of the equation of the given line js y= 2248, the slope of this Hine i 2 Because Ly bs parallel tothe given line, the slope of Ly isalso 2. Therefore, m(z,) =2, and by Bay (2) we have wataty th spa 000 gn? ‘then by Ba. 2) y=, Thus 1, contains 0), §) and bas slepe 2 An equation of 1 is yotes$ then y = 4(2) — (2)* +22) + = 4. Therefore an equation of the second of the requized Ines is ota Xe, pot 49, Find aa equation ofeach somal Ine toy =2"~ de that is parallel to the line 2+ 8) ~8= 0. D Le fla) a te. Tien fe) =3a? =A, At the pot (2,f(2)) the cave, the slope of the normal i FESS aL These tn nthe quo y= Je 41 an hes lage Sic th nora is Bente give tine, we must have gS =f det —A= 85 = Now f(2) =0 and J(-2) = 0 the rquinee normal Ine ae at (20) ae (~2,0) ad ea tne haw ope Renee, heir equations are y= —He—2 2 +8y—2=Oand y= —[(e +2}; 2+8¥42=9 50, Find an equation of the line tangent to y = 2*~ 6s and perpendicular tothe tne = 2946 > The given line hos entation y =x +8 sad slope $y a perpendicular has slope ~2. Let (2 Playa de? "6. Av the polit (e,/(2) on the cure, Ue slope of the tang Becave /() = 5. the required fine is y= -2(2—1) ~$ of y= =22— S51. Find on equation ofeach Ine though (33) that & tanget to the parabola y= 25° 1. Dee fle) 2st Then fe) = es Aline through the pointe P(aj2nj!~ 1) and (18) has ope CEE) 18 page tbe mg at Pit soy) NaN anys Now f(7)=97 82. Given f(z) —}e2-+28" +5245, show that {'(2) 20 forall values of & fe)= asd = (242) +12 br alls 133. Prove the general product rule for three factors.» Applsng the praduct rate twice, we have AUC): ge BC@)! = C0) aa)! -Me) + UE) 02] MC) (a) a2) + fl2)-g2)) Bla) = LF) 92) HE) a) ae) We) + He) (2): Wa) + 4(2)-9(2) Ma) In Bxercines 54-57, use the general product rule to fine the derivative. BA. [Cer 43)(22-—5) ee +2 = (ae 3) 22 — 5] +2) + (2? 49) Ee—SY(Oe-+2) +(e? +N EHS AY = dele —6)(Se+2) + (22+ 3)2){ Se + 2)4(2? +3)22 —5)8) (12 225" 20a) e (62? 4dr" + e+ 12) +02 ~ 100" 185 — 99) = Bee? 992" + 16e— 89 55. (ae 42 eF— 1 = 82 4)824 Het — YY (Se 2) (Ge +2\er 1) + Be F212 42) PI) + (Be $2)Ge +2127 —D Bde 2)62?~ 1) 4p 42)0K3? 1) + e+ (02 4 2)C2) (Gat 2)802 3-4 Set 1-4 Gah + de) = (Be +2)(120 +e —6) = 202 420084 22-3) Se (ae 44g +390? 91 (Pte Sylar aiat—5) + bet 4x Yes3)(e? 0) +98 ge He 49)62"-5) (2 = 32 -h(e4 3X22 3) 482 427) Det— 9) + Be ra He +28) (a4 24 450) —1h2?—3a"!— 95-2 + bg 9b Ae) (Be + 2 52%) + (608 + BA De + Stes? + et = 00e9— 18582 — 26? + 10a 4 a st Kae ge +1 =Dfet ges Meet HHH) (Qa? +24 Let tet Qe +241) $(2e 42+ )Or +241 Ostet) 4 Gxt +24 2s? +24 1) +2+h = (42 + 122" 42+ 1)(22* +241) +(2e +24 Nat et +a + 1) + et +24 eet Hees = Mae + 2s? +241)? 24 TIBOREMS ON DIFFERENTIATION OF ALGEBRAIC FUNCTIONS AND HIGHER ORDER DERIVATIVES 107 ‘Prove the Leibniz rue for th second deivatve. (fa) =(9Y = Ula Sol = U0) U9) = at 9) +09 + a= M9 t2T a Tot Now assume the formals i ue for DEAE) = plik + tet = (kD Le) ‘Thus the formula hold trve for n= b+ 1 and hence for any integer = Lim Pe pcre" Pat) a aNd a (terme) = mat Prove that i aud 9 ae difercniable fanctons seh tht (0) = O and 0) =0 then (Cz) cannot be 2, Suppose f(2)a(2)= s. By the product mle: (xg) + f(a)gx) =1. At: f70)-040-¥(0) =041. Sow bow tree thoren on diferentatign enable ws to difereatate ay polyoaiah aah egayt Preheat cle CegePY H eg ath oneal) + (eae eH HS Lamy eg CaP) bt (ty eet PE nee (2 eget the RECTILINEAR MOTION ‘2541 Definition If isa fonction civen by the eauation s = 2) and a article is moving long a stright ine ‘0 that sis she number of units inthe directed distance of the partele from a fixed point on the line att units of time, then the instantaneous relorly ofthe particle a unit of Gime is nitvof velocity, where =rw # 0nd UF it existe. The instantaneous site of change of the velocity ie elled the instantaneous nes 1-8, n partite is moving along a herizonal line acconding ta the given equation, where « meter ie ‘rected distance of the particle from a point O at { seconds, ind the iastantaneous velocity o(0) meters per ‘att seconds, and then Find e{,) forthe partiular value of 4 =i MO DAat 4) = 40% oo) = 4-259 =; 1) a 422-1; Ke) Ht 120842: 0G) =5 24.030 Ska aa ale slong. horizontal line according tothe given equation where s meter is the {point O at L seconds. "The postive direction iste the right. Determine the ‘SS hes the partic is moving tothe right ed when it 3 moving tthe left. Also determine when ‘e Svon. Simulate the motion on jour eraphiescalslatr, In Exercises U5 and 16, plot she motion 20g THE DERIVATIVE AND DIFFERENTIATION Pugh ates ste 34619 =P 4-3) =H +992) and t= fe2 11a _ panics - Te Sorin tote 1573 b — a hanging dieton fom right et “Jetcit ~~ moving tothe let 6 hanging diction from ot opt FLA ovine to te nent PsP — 12048 = a dea? at —12=40 -9. | 4 O'when = Land 1=2 3 AS movies to the rite d= 3 Ghangng deeetin from ight to kt 5 Sfeteas =~ movieg to te let Eo Tee? 0 hanging deton fom Beto right pet 24 + movieg tothe ght 1, 930 4 JP 204 a5 u(t) = BE = 262+ 81-2 = (64 2)2E—1} WG) = De (42 tate paride is _ roving to tbe Heh hanging deci ‘moving to he et ‘Singing rection om eft to ight roving tthe right com right to let sel e4] The instartaneous velocity is given, by ace 40 = Hota) ae aa heed a Because ot) =0 when t=1 and (= —1y the particle reverses dinection at each of these times. Table te indisten the ge ofeach fotor of v when ts in each interval. The siga of vis negative when » hus onc (odd factor, The partite is moving tothe right when «> O and is moving tothe ef. when v <0. The behav Sr'the motion i illustrated in the figure, Table 12b gives values of # and v for specie replacements of $ ‘Where wo use the orginal equation of motin to caleulate « and Ba. (1) 10 eaeulate 2 ties — a esa a felis fe eta. | | staat 25 RECTIUNEAR MOTION 100 1 ads 948-2008 aH? BF 8) oy a ae Ont yaa ase aeey he Suda (020! pale a Bok 0 + +O larga dct fin a et 2 sage 3 and 0 changing direction from ight to left = roving tothe left (P44)— C4 1)29 P2144, oa eee ra} aay +4 (442 w _ perticleis when (= <1 = vB 0 +O ctingdiretion Hom left to right + +E moving to the right 0 +O changing eiertion from sight vo left f>-14V5 = = mong to he Tak pec tcecia ern crete bean eg cia Be erences ics Be cal eine Bat aie See Perea erates Sue kn etae [A chancelics is dropped from a height of 160 8. (4) Wits Its oquatin of motion. (b) Pind the velocity at 1 Sst 1.5 sc. (e) When does it rach the ground? (dl) How fast is ping then? @ ee + 160 (2) (0 “et. vd) = Be, Thetefore, 1 see afer tie dropped ite CCocky ie “20h Jc. w( 1.8) = 48, Thus, 1.9 oar tix dropped its vdocity is ~H8 Ite. (6) It traces le ground when s = 0. Then 1624-100 =0; 2 = 10, €= 10. Thus is reaches the ground in about 32 ae. (@i eiy10) = 22/10. Hence when it reaches the greund is velocity i about ~101.2 face ‘A chaneleeis thrown éown rom height of 180 Ra 48 (4) Write its equation of motion. (2) Fiad the felcity at 1 and 15 mec (c) Whey docs it reach the run? (@) Go) ope tbs oy = 100. = 60 ae 10 1) rots velit i ~BD/e. (15) = ~96, Ths, 1.5 after it js dropped ite velty teaches the gound wher ¢=d, Then ~i6t*—48¢ +160 =0; 0= --3¢~ Feaches Ihe ground in 2 .(@)o@2)= 112 Hence when it reaches the grotnd [Aball thrown up from the ground at 32 fee. (0) Wate its equation of motion. (s) Sula ite motion sour eapiin eestor sl mtinate when and wher its Hehe point wil be (} Calculate the emul {Gh Te Se clon and () its peed at 3 see and 125 see (0 Find spend hen I reaches the ground (hay 82 yn a 18 894) ot) f= ~ 821 +82. The ball reaches phe! point when © = hate, when''=1 ood ().=16 There Haken ce al ee to each ts maxima bright of 16 (4 M8) 8. Hence, 15 or ater iia thrown ie veeity iy 8/5ec (25) =. Ths 1.25 ote after it jee (e) In oth comes ie sped is € ff (1 rates the proud when = 0 162 T= 2. HD) =2E Hence when fe aches the ground ite speed i 32 {lees the same as when twas throm. ‘velo is 112 Meee 110 THE DERIVATIVE AND DIFFERENTIATION 21. A roche is fred up from the ground at 660 f/ece (a) Write ite equation of motion. (b) Simulate its motion fn your gzaphies calculator and estimate when and whew ils highest point will be. (e) Calculate the reals. {Pius velit an (c) lia spend t 10 ex and 25 soe (0) Find its speed when it reaches the grousd. > (a) ny = 860, 4, =0. «= —160 8600 (c) ut) = de/dt= ~B21+ 560. ‘The reckel reaches its highest point Mihen? =O; dish io, when €= 178 and (17.5) — 4000, Therefore it takes the rocket 17.5 see to reach ita Taxdmern beight of 4300 ft, (d) (10) =240. Kenee, 10 see alter it is Gred ite velocity is 240 ft/sec NQ5) 2 240. Thao, 25 se aero fed ite vlad ir 240 Te/ae. (e) In eth ses, its speed is 240 fuse G1 aches the ground when 40. Then —16? + 560¢ =0; 0 = —160(¢~35); 1= 35 w(85) = 560. Hence When ft reaches the proud Si spon! i 560 ft/sec, the same az whom it was thrown, 422. Plot the path of the ballin Exercie 20 with 28, Plot the path of the rocket in Bxercse 21 with and y= and ako the velocity. ‘Land y= and abo the velocity a ao at NF |e 24, Simulate the motion ofthe particle of Be. $.0n your graphics calalator. Bxplain why this supports te results. tle Example 3 me have #2302 aad v=4¢= 61-30, For sibility, me simulate the motion of the pariicle om the Be ‘our exleulaof Sn pacemetic mode, w> let 2y(0) = 0— 120 +361 24 and GO=2. In the window [-1,4)x[—35] we let [(Min (Mac) = [0,4] and tatep = 0.05. We now press the ERREE) ey and then pees the’ key and hold itdown until the cursor is at t= 0. Notice the infoumatien at the botion of the sere: ¢ =O, 2=0, 9 =2. We pres the > Key and hold It down. The cursor represents the pprticle mosing along the lice y= 2. Note that the particle is moving to the right until ¢= 2 and == 4, when A stops and chenges dretion. ‘The particle then moves tothe left and disappears off the sereen to the left In yercses 25 and 2%, a particle is moving along a lite arcording the the equation, where + is the position at {CO see, Find the tine when the acederation is 0 and the position and velodty at that time. W sndO-Yeyael vat —atete =) @-4 2 eH PPP ERA ve OP 12, a= BE = 12E—12 =O when C= 1. (=H a1) =F In Exoscines 27 and 98, make a table giving , 1, Inchide the intervals of time whes the particle is moving ta) the lef aud right, and when the velocity or sped i increasing or decreasing. Plot with x= and y= t+ Wy. om BOF 4 St HP — 914 19) = H(t ~ 1 Kt —t), where the roots of |? — 9+ 15 = O are ye AEE =H" Val s2atena yo Yay =or TBE ee yee Ser is) =A fe 8) a= Dye = HI 18 = 62} =o OF > avis atthe origin and ir moving to the right "The velocity by decreasing. The speed is decieasing. ( key and hold it dows, ‘The cursor repeseats the particle seistak along th ine y 22, Observe that the particle i moving to the right until = 2 and 2 = 4, when it ‘Hope and changes direttion. The particle then moves ts the lft atil {= 6 end 2 = —12 when it stops and ‘eluges direction. Then it move tothe sight and disappeats of the sereen tothe Hight. 4a Brercees 31 and 22, the equstion of motion is ¢ = Jal?+ vt +9 where a the aceleration for the body. A Aon Ecppel fom a cif hts the ground in ic. (a) Watt the height ofthe ei (b) With what veleiy fon i bi the ground (c) What veloety is aod to throw it back fos original peiton? Soa the men; a 2-5-5, T =f. The eloce stats atthe origin. =—2-7O4. (0) = A tbe cif is 44 “Tap The sone fits tbe grand at 2 /see and 22 t/a isthe velocity needed to return it 32. Qn Mars, o=—1T=3 (2) The stoue stars atthe aigin 0 49 Because (3) = —4, the lif is 54 0 high, (0) w(t) = Dyp = Dy) = 126 and (3) = —26. ‘The stone Ris the ground ab 6 ftjuce and 36 f/seeis the velecity needed to retuen it 100 — (+292). Find his spoed (2) a "Te equation of motion ie #= ‘33, A eprinter ie x mets fromm she Gaish tse afer the start, where the tat and (b) atthe Fins. "Yor 38), 0) 0) == 8.25, (8) ¢=0 when + 88¢—A00= 0, ¢= f= HB VID) ty) = VBS = 12.96 SA. oft is the distance of the ball ron the starting point at ees w= 4c 100% (0) =F () oy) = 24-4234, Therefore, at ee the instantaneous veloc of th Balls (2/-+20,)/see NG Eades Be sone te; 2082 24; «= Hence takes $e forthe velocity to increase to 48 fee 135, sean is the distance ofthe ball from it initia! postion a = 2014100 ‘Tie billard ball ite the ewsion whens = 39 #0 we have . 1002 + 100: = 39, 10044 -+ 1001 ~39 = ; (10: ~3), 104+ 13) = 05 ¢= for t= ‘We reect the negative value of t, Therefore, the bilfard ball hits the cushion inf see std fj) = 160. Th the clot of the bill ball 16Qer /see when it ite the eushic, 36. Two paticle, A and B, move tothe right slong hetizonsal lite, They start at point O, s meters i ‘Grostel ioaee of the particles Bom 0 et faseonde, aad the eqvationg of motion are 14-4200 io +100; (0 s=48451 — (forparticle A) s=TR43 (for particle B) t=O at the start, for wha: values oft will the velocity of particle A exceed the velocity of pariel 8 The vdocity of Ais given by ey =D,ltl? +54) = 8145 ‘The vdoclyy of Bis gen by v= D(TA 4B) = MEHa We want to find when &, >ep, & equivalenily, when B45 > Mes ai teh ‘Thus, the velocity of A exces the velocity of B when 0 5 ¢-< 1.6 TAP DERIVATIVE AS A RATE OF CHANGE 113, VTHE DERIVATIVE AS A RATE OF CHANGE, 2.6.1 Definition IC y = f(z) the instantaneous roe of change of y per unit change in 2at 2, is f(a) er ‘equlvalenty, the derivative of y with rexpeet to 7 aby, M(t ents, The hargital cest ‘and marginal revenue are the derivatives of the cost and revenue functions ‘Rate of Change of y with rspeci to zat 2 is given by /"(z)/f(@;) = (du/42}/y evaluated at ise 2.6 ‘A-squnre of side = cm hes aren A(z) an? Find the average rte of change of A ae = cing From (a) 4 to 46; (b) 410 4.5: (c) Ato Als (€) 4 to 4.05. (c) Find the insiantencows rate of change when = = 4 AG) = 2" (a) 4/02 = (367 —17)/(4.0 4) = 80 (D) A/ae = (1947/44) = 8.9 (c) AN/Ae = (41? —#)/(41 4) = 8.1 (@) (4.05% - 2)/(4.05 — 05) = 8.05 (e) A(z) = 2e, AY A recnngle of width win and length 10-+4 in, has sea AC) i®. Pad the average rate of change of A as changes from (e} 3 to 3.2; (b) 3 to Bi (2)3 to 3.01 (3 20 3.01. («) Find the intantancons rte of change when Ao) = wte-+4), AG (b) AA/Aw = (81-7. 2/2 = 10.2 21 (9) AA/Aw = (22-79 —21)/9 (9) Ba/de = (801-101 ~21)/01 10.01 (4) 01-700 =). = 18.00 = 1800 (9 Ae) = 20.44, (a)=10 The meamre of emission io R= RT, where 'T change ax increases from 200 to 300; (b) the istantancous rate of change when T [n) AR/A7 = (#2004 — 12004) /(200 ~200) = 65,000,090 (5) RICT) = 4k TS, R200) A circular cylinder of height 10 in, and base of rin. has velume V in’. Find the average rte of change of V with respect to rat r changis from (a) 5.00 wo 5.40; (b) 500 te 5.10; (c) 5.00 to S01. () Find the instantaneous rate of change of V with expect to r when 7 3,00 The volume ofa circular esinder is given by (a) Av = arth = Wer? We simplify, the calculation by factoring 107. or at st _10x(54?— 32) =a 0. (8) V0) =2088, V1) = 1007 A creular plate of radius + ins ha area A(r) ia? and circamforonce C(e) ine Find the change of (3) A(r) and (b) C(r). (a) Am rt, Ala Ber (b) © Der, Cm Or (c) Note that APC. tasieoun fatwa A right cieola cylinder whose length is twice its rads» is capped by two hemispheres and has volume V(r) sable units, Find the instantanecus rate of change of V(r) with rqpect tor. res combine to form a,ephore of volume ter* The eyinder has volume 22h = #72(2r) Bart Thus Vr) = rs, We) = War? Let = units be the toial length ofthe solid of Bcercse 6. Vind the instantaneous rate of ebange of V(e) wich Baie = Bre, Vie Boyle's law for the expansion of a gas ts PY = C, where P alts the pressure ad ¥ units i the volume, aad Cis « consint. (a) Show that V decreases at rate proportional to the invene square ef P.(B) Find the instartaneous rote of change of V with respect to P when P=a.and V= 6. (a) Saving for V, we find V = CP~? so that V! = CP~® = -C/P? which proves the asertion, (b) When P = 4 and V'= 8 we find C ~ 4-8 = 2. ‘The instantancous rate of chaage is V' = ~32/42 = {days ofc a sickness start, the temperature ia f(t) 7, where #(@) = 98.6 + 1.240.120, 01 £10. (a) Fiad the rate of change of J. What is the tenperatare and its rate of change after (b) 3 days and (e) 8 days (@) When does the maziiuun temperature eccur and what bk? > (3) (9) =1.2 0.244 (b) {(8) = 101.12, (3) =0.48(c) (8) = 100.52, (8) = -0.72 (A) The temperatureié Increasing as long as {"() > 0, thas is 0.28¢-< 1.2, €<. The mazimum Cenperature fe at 5 days; tls 101.6. ae 114 THE DERIVATIVE AND DIFFERENTIATION 10, Find the rate of change of she volume of sphesal tumor with rspet to the adns when the radios is {a) 0.dem and (0) 1 ‘The volume ie V cm® when the rags ir em, where V enh fem (0) VL) = aah) = a8 ee, 11 Fin the eae of change ofthe volume of sere call with reset tothe radius when the rains (0) 1.5 pm an (0)2 > Th volume i V ut when the cade i rym, where V sam fm (b) V"(2) = (2)? = 16 pn. 12. Find the rate of change ofthe siriace of a spiical tumor with respect v0 the mdius when the radius fe (@) 05em and (b) Len. > Thesusace i em? when the radi Io em, where S ~4er* S') = 8 (@) $0) = 89{05) = Ar. The eotace is incesing at the rate of 4 es fem (SU) 2 se. The ance ieneasing at te rate of 8 emer, 13, Find he rato of change ofthe infer of x phreal tor with resect to the radius wes the radios is (6) 15 pm and (8) 2 em The aurucee 8 pm? when the rdive te ar, where $= Aer? Sp) = Bry. (@) S{L8) = Bx(5) ~ Te Tho ster increasing a the ake of Bx yn) {G) S10) = Ue. The surface ie increasing atthe vate of 10 sm? /m. 4d. The height of a cone twice the rains. Find the rate of change of the volume with respec to the rau when the height i (2) 4m and (6) 8m. > When the catia is rim, the neight is = 2rm and the volume is V=r*h =$472Qr) =Jer* mi. {a} Wien ha dye 2.0") Be m/m () When haa, r= 4. VA) = 82x mr /m 15: T dogo le the tomparatae thon ater mitnight:"T = 0.1(400--404-42), 0 ¢<12 (a) The average mie of charge of Twit cept te between 5 am. and 68. Gi[t00 106) + (6)} —04[400— 403) + 5) 01-2044 175) eS 7 fer, Vio) = Aer. 0) W108) = 4008)" for. Vir) = tar, (8) V(L5) = 4x(5) = 9 29 5 am. aad 6 sam. tho asta rate af change ofthe temperature with respct to time Is "40+ 24, T'S) = .l[ 40+ 265) =~. Therefore rm. few denna of 3 depres per het 16. A worker can pant y ames = hours ater starting work at §AM, and y= 964 822—29,0-<2-<4. (a) Fi {he ale al which he pang at 10 AME () Pind the umber of igs he pains between 0 and 11 AM pa) pie Sri6e—Se% AC I0 AM, #2. 2) =3+16(2)~3(2}'= 72. He is plating at the mle of Ferme per hour (0) AU IL Ant, 223. yQ)=90@) =e 8-0 9%) —(Oa ez) =H. He pal frames betwen 10 4. and 11 A 11, Vite isthe volume of water nthe pl & minutes after the draining stat, where V = 250(1500 800+ # © ahTue numer of Here per nat nthe apaage vata which the volume af wate inthe pol in handing vag he nino 0800 13) +) 260600800) OF "25H -H) oy Therefore, the average rate at which che water loaves the pool dusing the frst 5_min is 18, snlnutes (b) VG) — 280(-20 + 20, VMs) — 2501 —60-+2(5)]= 17,500. Theefore, § min aftr th farts the water i leaving he pod! at the rate of 17,500 liters per minute 18. Find the rie of change ofthe ares of circular pple when it radios (9) em and (b) Tex. > When the radius is ex the sen Azar? em? A’ =2er. A()=8z and AU(T)= lr, The ara ‘neroaing a the rat of (a) 8 em /em and (b) ee ex fm. (a dla ithe toa con of maitre # watches Cle (SC te mansina ot faction Of) =. 25 (0) {6 Tue mune diaries ge of nae (AL) —C(4a) = [1500-38-41 + (41%) - [1500 +3-40-+ (40) Thercots bees decrease of 29 degrees per hour. (b) TM) itotontancois vate of change of the temponstare at 500-490 424 Toety-et watel fe a 26 THE DERIVATIVE AS A RATE OF CHANGE 116 total revenue recived ftom the fale of = desk i R) dla, and R(x) = 2000-22. Pind (3) the “stgial revenue funci‘on; (b) the marginal revenue when == 30, (6) the actual svente fom the sale ofthe ‘Sire fist dak {e) The matginal revere fnetion in given by Re) =200-f2 ) The marginal reverse wien 2= 50 8 gen by (0) = 200-30) ‘Tse marginal revenue is $18) when + = 10. {¢) Toe actual revenue from the sle ofthe tity Set deni given by B(S1)—R(S0) = 200(s1) 491) — [2an(90) -Y0}"] = 979.67 9700 17.67 “De ntunlrevemse ftom the sale ofthe thiniy-ist desk is $179.67. (=) dolar the total reverie Gorn the mal fs aloicon ate: Re) = 6002 — he? (2) Ris the marginal veventefuretion: R=) = 600~ Se? () R20) = 600 ~ 420)? = 600 ~ 00 = 540. Therefore the margin! recone i 8540 when w= 20, ©) The number of dollars in the actual revenue from the sale of the tnenty-tiet television set i 21) —R(2N). R21) —R(20) =|o00¢25) — 025] | 000.20) —/00)+|— 596.95, lz) dollars the total cost of making + paperweights: (2) = 200 +504 22 (0) Cli the marginal ot fant: Ce) = 39-38.) C0) =~, 2H 45.00 vhen <= 1. (€)"The numberof lian in the actus! cost of making the 11th paperweight is 01) = ce) =[200 +9012) [aoe $8.4 20°] — 705 9.75 olla isthe armual gross earnings of the company ¢ year after Iaausry 1, 1994. p(t) = 0.42 4 20+ 10 PU) =O8t+ 2 (2) OF Jansary 1, 19%, = 2 wad p(2) 08) +2 = 36 Hence on January 1, 1996 the {Poss earnings ate grovring al arate of 5.6 milion dliom per yen, (0) The relative ate of growth ofthe ghssearaigs on amcry I, 1906 i p(2)/0(2) (2) = 04(2)? + 22) 4 10 = 156; p'2)/p(2) =3.6/15.6 0.231 = 22.1% {On January 1, 2000, ¢=6 ane p(6) = 8(6) + ‘wowing ata rate of 6.8 milion dlrs per yar. {6) The relative rate of growth of the gross carnings on lanuaty 1, 2000 is p'(@)/0(). (6) = 0.4(6)? +2(6) + 10= 864; 91(2)/12) = n/364 = 0.187 = 187% 2p dolas is the annual gross earnings of a company ¢ seats after April 1, 1903, where p = 50,000-+ 4.0004 eOUe. ind (.) the rateat which tio grec earnings were groving en Apel 1 1995; (1) the tative rate of giowth of the gross earings on April 1, 1095 lo Ue nearest {.1%;(¢) the rale at which the grow ‘erring should be growing on Areil 1,200 the aatcpated mative ante of gpowth of the gene camnge ‘Apel, 2003 tothe nearest 0.1% #1 = 18000 +1200 (a) On April 1, 1995, +=2 and 5/2) ~ 18,600) 1200(2) = 20,00. Hence on April y 1606 the grow earings are rowing ata rate of $2,100 per yar (0) The elative ete of goth af the prom ering on Api 1 10066 912)/(2). (2) = 50,000 18,0002) + 6002)" = 88,405 p(2)/n(2) = 20,460 /88.100 = 0.2008 23.1% (9.0m Apri 1, 2003, = 10 and p/(i0) =18,000+ 1900(10)) ~ 20,00. Hance eo April 1y 2008 the geo | earings ate growing ata rte of $30,000 pes you. {@) Tae salativa rote of growth ofthe grou etningy om Janney 1, 2008 9/:0)/p10). (10) = 5,000-+18,900(19) + 20(10)* = 290,00; 1 10}/9(10) = 30,000/220,000 . 10) the member of people a yopultion «years afte Fansary 1, 1995, where Pl 1 P(@ =801 4200. (a) On January 1, 2004, ¢~9 and P/Q) = $0-9--200 020. Therefore on January the popslatin wit be growing atthe raze of 920 jrople pee jet (@) The relative tate of growth of the population or Jansaey , 2004 ie P(9)/%(9). ¥(e) = s0(" + 2008) = 10,000 = 15 0th; P'0)/P(0) = 920/15,040 = 0.081 5.1% (6) On January 1, 2016, ¢=15 and P'\15) = 80-15-4200 = 1400, Henee on January 1, 2010 the population ‘yl be growing atthe rate of 1400 peope pes yea. (@) The relative rote of growth ofthe popation o» Jansary 3, 2010 is P(6)/P(S). (19° 200(15) + 30,000 = 22000; P“(18)/PC) = 1400/22,000 = 4.06 4.5. The margaal cost is 1034 103% 40%? + 2001+ 10000. 04 4% 16 THE DERIVATIVE AND DIFFERENTIATION 25. Lol r be the reciprocal of n. Find the instantaneous rate of change and she rélative rate of change of r with a6 1 ie clea eeee 2 the Oak 4000 «=a 1 Ha) 22, The profit ofa store is 100y dalla when 2 dellare are spent daly on advertising and y = 2600 4-262 — 0.222 > y/@) =36-— (te, (a) 060) = 36 -0.4(60) =2, ‘Therefore the rate of change of y mith respect to zis postive ‘ben 2 = 60, Thusit in profitable to incense the dsly sivertsing budget when « ~ 60. (b) ¥(ato) =36 ~ 1.4(300) = ~84. Because the rate of change of y with respect to = is negative when 2 = 300, hive peonbl ts tanmsme the Aly veining badge en 2 = 300 (e) Tie mst Vain a's bbe Stl a PAST la Seager va Die) en ThueS6—O4~0;0de= 36) Ob ‘28. The supply equation for a shirt is r= 3p? + 2p, where p dollars is the price per shirt when 1000 shirts are tuplied (x) Find the average rat of chtage the supply mhen Une pices incaed for 10 (11 (8) Fi the anginal ria change f tin epely wen the pce #0, > (a) 2(11) = x(10) = [3(11)* + 2(11)]—[3(10)* + 2(10)] = 65. The supply increases by 65,000 shirts when the: Pro Inceaee fo B10 te B11. (0) 2°= Op-+2. (10) = (0) +242. The muginal rae of anges 620 Bide pe dole 2. Find the slaps of the tangent ine atch poe tthe peg y= thease B20? tes of ttet p60 = oe? 429 -4(—1)8 + 3(—1) 6(-1) =, '(§) = 4(B)4+ 3)? — 608) 38. Find the instantaneous rate of change ofthe slope of the tangent line to y= 22! ~ 63? — 241 ab (8,-2). > aloe wits hey = 12s 12 ghQ) = 129) 124 31 Ate min +0) m the rds of a oi eit ()= 16,420 HO S!S? tg Be 2291, manshowa tat 1'(2) = 16. Find the rate at which the the radius is changing at (a) 0.4 min; (b) 2 min; (c) 3.2 min O)ite S21 setser(os) =e) 32 71) BO) Toit eo — teow 92 5. Show thut for any linea function, the average rie of change of (=) as changes fom 2,102, +b it peates fee Ua coagon ese eaaaae OL AN on $e +8) flay) _ mia, + )-4b— (ma +8) mk » Let Sle) 1 tea, ® , re) Which the inetantaneous rate of change at any pont 35. (a) The instactanecus raie of change of the area ofa czcle is D,(xr4) = 2zr, the cicamfornce ofthe circle (6) The instantaneous rate of change of the volume of aphere ix D, (fer?) = 4242, the auslace of the ophets, 429227 where the rate of change when © fae ale 1 b sme b. The average rate of ehangels 27 DERIVATIVES OF THE TRIGONOMETRIC FUNCTIONS: 24.1 Theorem D, (sin 2) = ees 2 24.2'Theorem D.cor =) 2.78 Theorem D,(tan 2) = sc? 2 2A"Theorem D,(cot 2): 228 Theorem Dyfecz)=secime 27.8 Thewem Di(ee.2) = cee col Panctins of ratinal tulip of + in the fir quadrant requiring a€ mot one radial (Ex n Before Cale taiise 3 sgunange ak fer yee eee ve nae ene k con = cin Je = V5, tan br = AV, cos f= sin for = K/5-41), sin Be = c08 r= V2, tan br ta jr = V3, tan fr = VI+1 (05.28), tan hr = 2+ V5 (028) Beers 2 (cos asin z—cose(sinzy _ (sin asin 2—cos (0s 2) ante + conte 2 Deere) =Dighy = Wane) oases 27 DERIVATIVES OF THE TRIGONOMETRIC FUNCTIONS 117 Exercises 5-18, find the derivative of the faxction, £(2)=DSsin 2) = Score Bem eiincoete & He) Dyin) Dien) = oon sin» (2) = Diftan pect 2) = secs cate Pe)=Ddwes—2ese #4) = D2 cont) =2 e084 A( sin) = Noor ttn 1) M02) =42%cos = ‘We apply the product rule, Je) = 42*D, cos 2) +ee8 2D, (42? Bo (z) = Dy{a sin x +c082)= (Isin zz eos 2)—sins = 20003 ¥(y) = Dy[S sin y—y e08 s) = Seos y—[1:cos 9+ y(-sin p)] = 2eo8 y+ ysin y W(2) = Dd ein x cor =) = Alsin a(n 2) + (cor ex sec 2 tan 24 2ese 2¢0t = Aa? sin 2 +82 cose =A sinte tcovte) = 4 conte Mz) = 2 sin 2 +22 008 2 1G) =D,{e4sin 214 DOr os 2)= 2°-Dalsin £)+ sia 2-D,(2%) +22-D, (oa 2) +009(2)-D,(22) Pees e420 sin 2 Resin ep 2 eos 2a ete eee Sz) = D {2400s x —2z sin 2 ~ 2 c08 2) = [2*(—1in 2) + 22 008 z]—(2e cose + 2sin 2) + 2sin 2 = —a%sin N(y) = Dy fy? —yPeon y+ 2y sin y +2 cos y) = 37? — [By con y+ 9m) ]+ Cin y + 2p co y)—2 iy = Sy! +y7sin y F(a) = D3 sec x tan 2) = 3ffsee 2 tan z}ian 2+ 0c x(secx)] = 3 see x(tan2x +sec*e) F(0)= sin (tan S(O = si D(a #)+an (Dyin) =sin tect tan t cost =n taekt sin t Z #(y) = Deas 9 ots) sin y eo y cos yese4y M(x) = Defect oe 2) ~ ct (cae # ot 2) +c acne Brerces 19-0, find the detvative. pjesme) = ENCES) ~Beoest ale thins (208 yYeot y+ eos y(cot y) cae seotte ete) et )= ean a (ais!) We aplythe quien role pfips) = LPibse Osi Det fetes {png lesen sess ges neaten Caen ¢ Loos 2—(eH4)in 2) _ cove ein et tsi 1) et ec + tan tnt _ | gee t=stntt (cos =) cos con = 4) (1-sin yDy(cot v)~ eoty Dylt sin») _ (1 sin (a! y) cot y(—tes 9) ray Card a waPytescp dco yeoy G=sny? fs giteny) = Tse wlcos y)~ (1 tein y)(—cor 9) _ sony sin yom y seo y bein y cory 2 ee ¢ Tain 1=sin 9} (sin y} M18 "THE DERIVATIVE AND DIFFERENTIATION = £etst)- xp x(cos 2 +1) ~(sin x~ 1-tin 2) _ costo bcos 2 sine —sin 2 _ 14 e042 —si 2 Corstih Cmte eose pty Dffe sia ae +c08 2] = (1 con ale b cor a) +(@ sin a)(3 —sin =) m. 23. D,l(s? + cos 2)(22—sin)] © Applying the product rsle, we obtain Di((e? cos 2)22~sin =) sin 2), (22 +202) (Geet 2)(-2 coc teat 1) (0 ene t= 1H (ae t+) a wD cot t= exe Loot ¢ Dace cot 1 s0e teat (eeta2? cicteott 1) (2 —sin (2 08) Diets) =~ (-p2aind t42sint cost ~5cos (+ isin ~ (+ 2sin tp ao, p (ang t) _ sety(tn y—1)— (ean y+ Deel any=1) = (aay? In Exercises 31~12, compute NDER([z),a) on yout caluletor. Then compute the exact value of Ja). BL. fe) = cons; fle) = coe 2 ain 2:0 $'(0) =r0n 0-0 sin O=1 RW. fle)=esing onde > NDER gives ~0.9900095. Now we find /"(=) $'(2)=2-Dsin 24sin 2-D,2 = coset sine Nest, we replace 2 with $e. Thus, Gx) = Gw)eou($r) + singe) = Gx{0) +(-1) =-1 3. fle) = 85 f() = RSET wo HG Hse) =M6E: fq) = EE) 3 f(e)= i ss Me) =2e tan 4 rood; co f(r) = 2s tone 4 Peers = 2e(0) 44-1)? = =? Bi fe) =seos2—sin zy a= 0 > NDER gives ~0.9990908 f(a) = #-D,sos 2008 2-D,22—D sis 2 = =2'in 2425 0 2 cont Ts, with #=0, we have F1(0) = —O%(sir 0) + 2(0}c08 0 ~ cos BH. fle) = sin x(c08 2 1) {(e) =0on (ees #1) in ef-s so f"(e) = (-1)(-2) +900) =2 BB. Jle) = (con 2+ Ne sin 1); (2) =~sin o(2 sin 2—1) +(c08 2+ Isn 24+ 2 es 2) 1p) =—Qr—1) +1 = 2—}r ~ 0.4200087, NDER =0.4202041 9. Ie) = scone pasta Sone ate gtiin at acme 20 F's) =3V2— AAR NS: 4M, f(e)=tanetscer: 1=hs DER gives 2.000002. 'i"2) = see 2+ 5002 tan Na=(B) deat AM. f(e) =teote—ese f(s) = Beas + esr ct = 50 'G) OF THE TRIGONOMETRIC FUNCTIONS 19 5. NDER = 0.5900002 W@=abo 1e)= @ 1 05 01 oo) oom 0.5 0.1 —00) —001 3° ams 2574 4850 4004 4095 8188 018 502 Sots 5006 The quotient appears to approach}. (b) jim * (a) Use 4 calelator to tabulate to font decal pac vtone of MEEHAN =P i aie, 03, 02, 001, D401 aad A i 1, ~03, 01, ~0.90. What does the quotient appoat to be approaching as speroache f(b) Find fim "+98 8 by itepeting it aa deivaive, (2) See the tale Phe quotient appears tobe approaching ~0.5 as h approaches 0 hk L 05 on on o.00t eal coe Fr pase heer Rom = 0.2574 0.4550 -0.anT C4996 h ~05, =O =t01 0.001 selene | anise — -0ons asin 0509-15004 (©) By formula (4) of Seeion 21 and Theonein 2.7.2, sig GEE) fr sim, SA eae = D082 safe Sine =} a O.1 001 0.001 0.0001 10-* -0.1 0.01 ~a.001 -.on01 10-5 Belt as sani smh ines iitinnammvioeu rome ¢ tim {6G +4)—tan Jr ia he quotient appertomproach 2) fn MEP “OEE dnl ete na bh ‘0.1 0.01 0.001 0.0001 10-* -9.1 0.01 =0.001 -.0001 = HORA Mme be Sg pore : ETT AE ome o5764 0.6576 0.6668 0.6087 05775 0.6571 06657 0.6666. 0.6667 “Goat appears to appronch 3.0) jim “SE*E DME dace ae detan ie “dhe . ee a a ecb ier abt “bot —rher water dr ore at ribet abst SESE AE arr — 4880 —a977 —avs9 -.s09s —se04 — 5129 —s023 —.5011 —s002 zor ‘Fhe ates appears toappronch —} (b) i PE cosrmeode og fpess| oe iid SPE wo i ad «is Yoder nfs Be. What doe the quotient em io be approaching as «approaches ir? 700" ¥ asa deivative, 12) THE DERIVATIVE AND DIFFERENTIATION > (a) See the table, The quotient appears to be approsching 0.5 25 = approaches Lr. oes ek lexeesa = a Soe be ae abe alae ee Quotient seems to approach & (b) ‘he quotient apes to approach 2,0) fn, SUES 51. Let f(e) =sinz, Then fe) ir 3(,0) y= Ot ace Afar oe gmt tangent ine at GrhV/3) is y~}V3=(e~ fr): 32 or Se eee eer Gone eee wis) 22h. Let f be the cotine function, That is f(: (a) Becarse 16) = cov) =0, 1G then y=0 when z= $e and the slope ofthe tangent line is the tangeat line is y-0=-(r—45), -i+he (©) Becarse SiH) sco) = 0, (jx) =-sin( He theo y =0 when = +e zo mae Sl aoe wor =n erase 0.5444 05224 0.5015 05023 0.5005, sss caer? an ar 8 0.4920 0.5796 0.6468 9.6567 0.6647 0.9116 0.7770 a.6eT2 0.6788 ar aot 0.6607 =a poor =) = Ble he in)=1 Hr apd the dope of the angen line fs Thus an eaation ofthe ange lire is Be 70, and the dope of the tangint ies'la (0) = cool — 1; thi an otto ofthe tangent Find an equation ofthe tangent line tothe graph of the cosine function at the pint where (a) 2= Jn: (i Ly the point-slope formula, an equation 2.7 DERIVATIVES OP THE TRIGONOMETRIC FUNCTIONS 121 Because IGx)= cose) = V5, Sx) =—sine)= 4 ? ‘$e and the slope of the tangent lineis —, Thus, an equation of the tangedt line is soNi=-He-k, y= dere tiv f(z) = tan 2. Then f'(2) *. , and the slope of the tangent line is “(0) = sec% i; ths an equation ofthe tangert wen) (tn) =2; thus an equation of the hs an equation f Is yt = 224 be): de 2y 24 n= Oe fe ft engin yn on = Vi satin ‘Gevaie Vaete)2 ve pt ge te b= ch) =f Dave-fcsios fhe ope of aetna Maes Ge = we tan = V than a tbs angen line at e,—V2) is y = V2(e-3x) - V2 Fos B38 price a mv agent Ht aren ode egal’ whe enh tad gE py aah poem nang tie pearing pee ing ie (OH it wy cn oh pies ys br a A sin (8) (0) = con = 4, 0 He) tence) 10) = msn r= th ox) =—4 sa} 5) = 4 cox) = 4(-8) 23: u(x) =4 eo x = 4(-1 a st iO, Boy Be, Be anal Because v(t) = D(6'cos t) = Grin ¢ and a(t) =Dy(—tstn insantancous velocity and aceleration ef the particle att see is —Bsin £cm/see and ~6 cs ¢can/see, ) The instantaneous velocity and acceleration foreach vale oft is given inthe lable below. . wodty ccreraton 0=-¢0) ° tif? =6in({s)=-60)=-3 — -Sem/ne sb 5 aS em/anet ~bemfee ° —Sem/sec oarieae VI) =3V5 84 cm/sec? ° ~0eus 7 =—0(-1) =6 Oen/see? |= 3.08 ¢ (6) The instantaneous velocity and acceleration ae given by o(®)= A= Sin f and ish W3 =3VE Ge) = 3-5/5: vig) =3- = 8-5 = Be, ox) = 3 con Ge = 9H = -$ ober) =3-] =f alGx) =3 con fe = 3 V5) =3VE Ion) =8 oe = = (1 (i Tenants cians ec Fis-kavied UEDA blge) = Jose P= aly fst =} V5 =i 122 THE DERIVATIVE AND DIFFERENTIATION tlh) = fain Sr sin jr =§- 4/5 = 3/5 (Ge) = feos f¢ = -H-4V8) = eo 0 jobs) = Feo fe = I) V3, alge) = sin B= 3-4 a(n) =haine =} otter * ‘sin D+ 089 Tin 8 +c08 6 TNR BN Roce oY Miaee OE win sina) [Fain +o 6)? sw — “Ay iviw ane pv — 1/2) v8) _ iv =]viw (F's) = Wawa vey 5 (0. A projcie shot trom a gun at a angle of clvetion Bavingcadian mhanure fo and an inital voocty of fife, If R feet is the range of the projectile, then R=(0j2/s)sin a, OSa 9. > (a) Because tp = 480 and 9 =32, ve are ives that Ra) =48% dina Diresting with rapet ta, we have (a) = 7zMeava,, ide) =1200 cone) =0 @) PG) 200 sin Hence, the sate of change of B with rerpet to ie 0 whan (D}IERY(a)>0, wehave 1200cosa>0, cona>t eos a> 0 and 0< a then O-< a < Jr. We couclude that D,kR> 0 when 0-a< x ai: -yrechh siesonl ty iibemeto pp tases in Thudlon WE cy pute Mieke rith hold fr = fooee ee a fine ifn=dt peene)= Je pyrene 1, Tata) ed cet itnaaad prove mle 1) be en When b= 64, ten RAE A ales =A, Hien Wolatarmsas art. fom (re Di(ein 2) = ~sin = D°(ain 2 ‘Thos, by taking aveceasve der Save D" (sit 2) = ~corz Ifa = 4143, Thu, by taking successive derivatives Sn fine iin= itd Dern) = re Detina)= Jas itaate® oe at rate Beeause (2) it (1) for proved forma (1) for Beene (4) 8 (2) when k= t-+1, we have proved hat if (1) holds for b= #38 leo folds for k= ¢+1. Hence (1) hols for covery positive lnsger (2. Tae formula of Ex. 61 oan be expres as D,MEn 2) = sinle-+ ar) for any postive integer nm. Wo wish prove by induction that ifm aay positive iafegr, D,"(cos 2) = eo(2+ de) (1) We prove that formula ( Kida when = J: Dg(eoe =) ——in-# and cor(s-+ 2) ~ sir. Sappos the formala is ue for sce it Fe Using this bypcthess ond the formule for coo(A-+ 8), we have gt G08 2) = DID, cos xP! Ulon +k} ees 2c kr—sinz sin A) t= cn Ny — cove tn tn eae + Joon if — haat Jon Heed +f Wie have proved that if(1) hols for n = kit hold for n= 1. Hence (1) hold for every positive inege h 2.8 THE DERIVATIVE OF A COMPOSITE FUNCTION AND THE CHAIN RULE 123, DERIVATIVE OF A COMPOSITE FUNCTION AND THE CHAIN RULE Chain Rule Ifthe fanction g is difbrentible at = and the furction f is differentiable at g(x), then the composite function fo is diferentiable at x, ane (Foav(e)=saa)al) In Leibais notation, ify isa differentiable function of u, and wis a differentiable funetion of then yaa diffentiable function of = nd yy du ona ae ‘The folowing differentiation formas aze special eases of the chaix rule. In exch formula we fevusie thet v te a, function frentable al. ta the formulen involving. the tangent, cotangent, secant, and covecant functions we atsume that the trigoaometsic function is defined at u Dy nv™'D an isany intege Dyin econ u Dy Dyce = Djjan used w Du Dots Digse we ver ta Dw Dae w= ese wot w Det ‘Wien computing devivatives by the chain rule we don't schually write the functions f and Dut we bear them in mind, If a funetion is desribed as “power of ..", ‘sine of. oF produc of «2, then we fret apply the power rule, sine ley or product rue. Tn te Sal fou of an answer, the simpler factors appear fest. Harmonie. Motion: An ebjeet moviag on & line so that the measuce of its acceleration is propartonal to the measure ofits displacement s from a fied point on the line, and the acceleration and Gisplacement are eppestely dnestod. I: is a sutm of Serms of the form A cos(éx+ a) and Bsin(ke +b) of anplitudes|Aland| BL seriod 2x/ | and freouence|bV/2s. I-12, find the derivative ofthe function = 32 +)1D,G241) =3ee+ DFR) =822+ = (10-52), (10— Se) = 4(10 ~ 50)%(-5) ~ -20(10— 26)? ) =D,2? + An—5)* = He? + de ~ 57D, (02 44e—5) = dle? 4da—3)%22-+4) = We + Ile? +e 5)? esti (7) i the ith power of 2-4 48-241, wo noe the power rule fet BOP Be YD, Qe 4 BEE 1) Bee EI) (Gr Her) =AOr(r 2904 48-2 4 )F = py(2tt— 76-4 2-1/8 20418 +4 1)D,24 142-1) 22H = 7+ 2t—1)(6° +2) D8 — 354.1) = 3s —3e? + ADS ~ 327 + 1) = 39 32? +1) BSF 6) m93(2ya*— 35241)" ) = Ila? +4)-9) = ~21e4-+4)9D, 4? 44) = 240 ES cma (2) = cos =” D,{s!) = cos = (22) = 2s cos 2” Fic) =D,(4 cos 32~ 8 sn ds} = sin 32)(3)—B(c08As)(4) = -12{sin 30 +c08 dx) |Gie) =Dafoec'e) = 2x0 2 D,(oee =) Mic) = D.(QoecP2—— enc 2) = (0020 —1)Dg {oe 2s) ~ ec? — Ir Beton 2a) an Snes tab) Seated arate sec see stan x) = sects tan # 124 ‘THE DERIVATIVE AND DIFFERENTIATION 12, f(z) = cos(8e?-+1) Applying the chan rule to the cosine rule, we have P@) = -sn(Se! + 1)Dj(s" + 1) = ~si(Se? + 1)(6) = -62an(3et +1) In Beercies 13-16, compete the derivative 18, Lede tates" 0 (oe iae + es D (las) = 2c (ee 2 an aan? seca(@tan 2 vec) Ponte tate $2 sects tan 2 = 2 vcPctan a(tan?e toes) 4. 2 sin cost" 2D a) 0054 + sn*t-D,(cos4)] = 8 sin®t-Dysin {cost + sin} c08 Dos 4) = 6 n't cos ¢— A sat cost 6. Ffcotte— coc) = 4 ete -cae*) A. tcott) =A cat terete 0" = A cot (esc4(-1) = 4 ot t sc%, Alternatively, ileot®— (ct ce?) = ff - [2 cot} = 4 cot eo Heati—acty) 6. Last 4°22? + 14 > Because the function isthe product of (427+ 7)? and (2z"+ 1) we use the product rule first, then the pow rule, In the final atep we remove the repeated factor, Note that we connot describe the whole function 02 power of something. Hast nee +14] (4n* +7)? D,(22*+1)*+ (22" + 1) Dyfde +1) (4st 67) a(258 4 1)9D (259 +1) + Re 1) 2452 +7), b22 47) 2+ 7)?-4(2e9 + 1)°(G2?) + (20% + 1)8-2(42" + 7X82) Dass? £7)82e° + 1) 4 16e( de + 7)(2e" 41)" 2(4a? + 7)(22) + 14 Sa(t2" + 7) + 2(22"+ 1) e(da + T)(25" + 1P(l6e + 22 +2) 4s Bserciae 17-24, fad the derivative of the fusction and check by plotting your anewer snd NDER. Diglesd) den gestuce= Th tent en DD Ceara F 1) 5 Pai) 9 2ta1 GANG) OF +107) _ 4 2841 ito at Fp) ONG crore (O41 BT BAHT = 24282 + 1K +-9¢— 4) GE+P sia8( SP — 1) = 2ein( 2 — Dias 1 2 sin(¢? ~ L)eos( 84 1)(64) = 6 sin(6e? 2) 20. o(z) =tan?e? > Because the function is the square of tan 2%, we apply the power rule frst. tan z*-D,(tan 2°) = 2tan 2 sects? D(a!) = As tan sects? Dattan?s — 23° = s(ten"s —24/2D,(tan’s ~ 2) = a(tan’s — 2702 tan a(see¥z) 2] tate —29)%(an zsects—2) 2. G(s) = D,[2 sin 2—3 cos 2)? = (2 sn 2-3 cor 2)*D,(2 sin 2-3 cosz) = S(Gsin 2 3 eos 2)"(2 eos 243 sin 2) 2A. P(e) = Dl eomsin35)] =~ sn(sin 22) (in 2 24, 42) = sin%cos 22) > Because f(z) is the square of a sine we use the power rule litt, then she sie rule $' (2) =2sin(cos 22 D,sin(cos 22) =2 sincos 2)os(cos 22)D, cos 2 “ sn( eos 2e\con(con 22\(—sin 22)D,(22) = 2in(cos 2x\cos(eos 22\(~2 sin 22) 2sin2e sit (2.086 22) cos Sa-sin(cin 32) ) A sin(sin 22)ees 82(3) 2.8'THE DERIVATIVE OF A COMPOSITE FUNCTION AND THE CHAIN RULE 125 sen 25 ard 26, find on equator of the tangent linea! the point, Check by plotting the eurve and lange = (2-1) a (2.9) 2) = 42" —1)D,22—1) tan 22 at (Le, 4) (2) = 42% (20) = Raven, ohn) = = 8(JH)P = 18. 9 = 10a) 44 ses 27-80, s cm is the distance of a weight from its central position at & seconds, and the positive ‘is upward. (a) Find the veloey and acceleration, (B) Show that the motion is simple harmonic, (e) Pind tude A, period p, and Gequerey J of the motion, (d) Simulate the motion on your calculator. (¢) Plot te 39 (22 — 1) 22) = Hels? 1). y!(2) = 24. y= 242-2) 495 6 sin fet (a) v= b= Ba cos Sat, @ motion is simple harmonic. (e) A paar » (a)"The graph is at the right. e248 = (8 cfr) = frm fxs (| p= bem Dyl—Jo win xt) = yp? cow Jot = —(2n)(0 coo bet) Sapo ) Becnse ~(hr)?ts a constant, then a the measure of the celerton, is | 2 ional to», the measue of the displacement. Furthrmor, because in)? ia negative, thon © and # are oppositely direcied. ‘Thus, the motion is dimple harmonic. (c) The luude i 3. ‘The perked Is 2n/fr = 12 and the Fequeaey Is zp (d) To simulate she saotion, In parametehe tet (1) =2 (0) 119), teen he [TEEEED hay, pros the key unl the euro ina 0) the press the > key to observe the motion. = cot (21~8) (9) v= HE = Brain (20-9), =16e? cos x(2t—}) (B) Bosasse a= ~(on)Pe and is negative the motion is simple harmonic. (¢) A= 4, p =25/2e=1, f= 1/1 Bain x(3E +8) (0) 0 Mf = 2Ar conr(3¢43), 0 AE —TBePain w(-44) (B) Becnone a = (On) ane Ue a am (3+)? is negative, the motion is simple harmonic. (c) A ises 31 and $2, 6 m is the distance of a particle from the origin att seconds. Find (a) the velocity and (b) ration, (c) Show that the motion is simple harmonic. = cos{tt +c) (a) 22 = —ve sin( kt +e) (b) Bz =—teXeoa(tt +e) (@) Because LF = Hs, the measure of coset +e) (a) 9 atte) (b) CR +) (0) eeanse 5 = Hy te f accdleration is always proportional to the meanice of the displacement and the accolration and ment. sre oppositely directed. Therefore, the motien is simple harmonic =A sin 2rki+B cos 2rkt, where A, Band & are constants find the velocity and then the accleation of the particle by dlfientiating wi nk cos 2xkt —2kB sin 2ekt Ar HA cn bt 1c%EB cog Debt = t=" sin rhe +B cos 2rk4) tuting, we have a =—477E%s, Because 47°" Is a constant, then, the measure of the accelerator, is sonal ta 5, the measure of the displacement. Furthermore, becatse ~4z4 is native, thea a and s are ely directed. Tin the motion is smple harmonic. cos Bet In the window [0.4] [44], we lt [Min Mc) pa t/te=} b respect tot This, on 38-26 fn the ditanes of» patil frm the orgs at feconde (8) Pind sand a: () Show that “is simple harmon. (6) Simulate the mstion on your caclato. aS in rt cont a) v= SE = Secon xt ~Se in xt, = f= ~SePin xt Sect p) Becanse a= 14s and —n? is negative, the motion is simple harmonic, ‘The amplitude > YF 4 3 = VI. 126 ‘THE DERIVATIVE AND DIFFERENTIATION 3A. = sin(6t— Jn) + sin(6t-4 3x); v= %F = 6 cos(6e — fr) +6 c00(6t +34) 36 sin(6t— 4x) ~ 36sin(6t + Jn) = ~B6lsin(64~§r) + sin(6t + x)] =—B6s Bective o= ~2, the measure ofthe accleraionalnays proportional to the measur of the dips ref te aceenglon aod displacement are oppositely diteted, Theor, the motion i simple harm amplitude is J. 35. 4 =5—10 sin?21 = 9 — 10-}{2 ~ 608 2(20)] = 5 co 46; v= 48 = -20 sin At; a— 47 = -80 cow Ae Seen ee ree ton ante pono! to eto te deen Beste iantinaund dupacement ave oppositely dicted, Herts the ation simple brroni 8, 4 a8 cos? OC cause #2 4 cos? 6¢—1) we may apply the identity cor 22 =2 cos? 2—1 with 2 replaced by OF to simpli {he given equation: Thus «= {coe 124: We diferente to find v and a. Thus, = (sin 129(12) dt = —48(cos 12812) = ~44(4 cos 120 (&) Because the acceleration i proportional io the displacement, and ¢ and s are oppositdy directed, thotion i simple haeraonic. (c) To simulate the motion, ia parametric mede lei 2y(t) = 2, y($) = 4¢os 124. The wendow [d]xeic5-5) We let [/Min,Max] = [02], ¢step= 08. Press the (ERAGE) ley, press the < ‘unt the cure is at 0, the prest the & key to observe the motion [3% (@) Mb) = 10-10 cos 9 = 10(1— cos 6) = 10-2 sin? 40 = 20 sin? Yo W(8) = 2-20 sin 8 con 39 -} = 10 sin? (b) HG) = tsa fe = 10-5 =5 6 HG) 10-}y/4 = 5y/ (€) Hib 38, K=25 sn a. (a) = 25 08 0 (a) KG) 3-4/5 = BVS 0) RIG) OKI) = tore 5 4-3 its (€or he vat of hang of Pith opt oT hn (7 preety wr00y = iy at = a Meg xmas. a = SY ==} Auinlut +4) w= Asin(ut +) 113 cos x(¢—0.5) at ¢ seconds fine, to the nearest tent, how fast the angle is changing at 31 ss. eden x(t 08), (8.1) = —U2 sin 2.08 = ~0.598 = ~0.00 rade: 42, V = fyelt00 29%, V1 = f(00 — 2) 4 = 2(100-—2)(-1)] = (100 ~29(100— 92). VIG) = #68 vas 1915 24.2, V"(34) = f66(-2) = 8.25 2-83 48, E(6) = 50 sin 120m; Bl) = 50 cos 120zt(1207) = 6000 cos 20x (a) (010) = 600 eos 24x = ODT corr ~ O82A, Tene, the inatantansous rate of change of AE) fespect tot at = O02 is. 58248 volte. (b) E02) ie © 18850. Here, Instantaneous rate of ebange of (2) wih rem Uo ak ¢ 4. A wave produced by a simple sound has the equation P(0)= 0.088 sin 1800r¢ where P(#) dynes per sau Centimeter i the efferece between the atmoyphere pressure andthe air presure af the eardrum al find the lntantnowour mie of change of Pi) with expect to a (2) $e (0) sc () Bae > The instantaneous rate of change of P(e) with respect to Ls P) ie) 0.008 cam 1800et-D,{4800rt) = 0.009 1800s)eonI8U0=t = 5.4z cos 1800zt PQ) = Sete cos{ 8000-2) = 5.dn cos 200s = 5 cos 0 = 54x = 17.0 PG) = Dr oalt8O0 f) = Ge con 125n = Sev cor r= Sess 17.0 PAG) = Scr conl820n) = 15.8 “Thus the Intantanedus tate of change of P(2) with rexpet to i 1, 28 THE DERIVATIVE OF A COMPOSITE FUNCTION AND THE CHAIN RULE 127 (8) 17.0 dynes/en? pe second when ¢ (©) ~158 dynes fem? per secend when ¢= (b) ~17.0 dynes/em? per second when ¢ = emand equation for a paricular toy is pie —£000, where + toys are demanded per mosth when 7 Js the price per toy. It is expected that in f moat, where (4 [0,6], the price of the toy will be 9 lar where 2p = #4744 100. What inthe anticipated ste of change of the demand with resect #9 tenn 5 months? Do not express 2 in terms of , but ase the chain ule. fe have 2 = S000 p"2 and p= g(t? 474+ 100) s00(2r+7) fae thechain ke B= 42.32 = —10,000p-*. (at +7)= @ eaSthen p= (5)'+76) +100] =8 ttuting for t and p in Eq, (1), we obtain 4 isin 6 monthe the demand will be decreasing ot 12-420 OS1S2 6 ppoye shat A difentible a 2 rastea)? ift>2 9) Define A’(t); find the rae at which the arca of the spill is changing at (c) 0.4 min, (4) 2 min, (e) 3.2m ) (2) = al 428 = 1 2) = 214+ 6g 1152. Th Ae dione ak 2 1220) = ee OSES" eos = (004) +20) 4140 285 nin ui ‘ne, the acca of en il opillo A(e) = 3619 m/min (e) AU(.2) = 2a{10(3.2) + 4] ~ 9571 m2/min (2) 10 Met) = 89 = et (Woe) = DUNC) = MR20) = Behr) = 5 Given (u)= 12+ 5u+5 and (2) = (2+ 1)/(=~1). Find the derivative of fey in two ways (29) () oy wing fhe chain tule : le oe +5] = Aso. (E47) +90E4 ‘2 e-1=(e4 IN fyett) 5) = PH) +s EH) +3 ap I) Fo) = 204 5 and oe) = DET) = ag Foye) = soled) =[4E41) +5] sia} ~ 2] = cone ~2)(—1) = ~con(}e— 2) = sin 2 the chain rule to prove that (a) the derivative of an even function is wn odd Function, and (8) the ‘ofan edd fonction oan ever function provided tint then derivatives exit Wa) If f is an even function, then f(-2) = fz). Dioratinting on bot sides with pert ton and wang the chan ul on the etd, we hare FemD,n2)= He) fA) = Fey SCN =“) Df wd fomcins (8) IF Fn 9 odd fesion, then J(—2) = fe) Eerentinting on both sider with peck te J we have S(-2)D,\-2)=—f(2 H(-2\(-= ek fz) =F) Therebre isan even function. {coe 2) ise the result of Beacise 50(a) vo prove that if f and y ate diffeentisble, y is an even function and Mz) =(S oe), then (0) =. fg ison even fuaction then by Ez.51(a) eis on odd function 20 o(~ 42 in the domain of g' (h(3) HC2)= S'ol—2))- 0-2) Substituting 2 =0 gives f'(0) ~a'(e) for every /z (1) and (J 9X2) =x. Prove thet if g(x) exists, By (1) fate) te) = ¥@)= 9) 228 ‘THE DERIVATIVE AND DIFFERENTIATION ssin(h/2) €2 £0 suas fine) eee (a) Prove that J i continuous at 0. (b) Compute (2). (c) Prove tat fis diontinvous at 0. oh Becaseloin 1 1.0: | cin i/5)|<@e-0 as 0. enc by te ager ore Li) axe 9 fis continous of 0. (6) 2) = Be in} + coe {—}) = 22 in con bite # Oand 0) = Hin OL) 8 0. (¢) As 2 the ister of (2) approaches 0 but the second tem colnet between 1 and 90 ig "(2) doe ot ext ‘4, Hf f” and g” exist, find (f 09)". & (fog) =D,DO(Foa) ED. aha PS DAo)s+ HD. ING +0)" 55. See Execte 27. 96, See Buercite 28 BT, Suppor tht f and gate such tha! ()gf() and Jl) stand “ (fea\2)—(Foaka) _ (Sess) containing yale) ~ gle) #0. Then OME) = Cone) _ orks Frog (2) Prove that as 2, g(2}-(2,) and hence thet (F 0 9)'(&,) = f{o(=;))a'(2,) thus simplifying the proof the chan rule uncer sit addons hypothes (1). (0) xplam why tae prot apples az, =O if /(2) = and g(z) = 2°, but not if g(z) = 9gn . Fiabe) ~ floles)) (a) beens 7pt) ens enery€> 0 te 620 sch ana |/C} = Ha whenever 0<|o(2)—a(e)|<é. Because (=) exits, of) is continuous at 2, and so dz)—y() a8 22, Fisce thre #4 &y toch tha [oe)— gin) | <8 wimerer Oot) al) 1 laff where rs any rational number, and 2) ies 1-1, find the derivative of the Fancin (2) = D2"? + 52-82) = fda M21 5918 ee D802 68 aI) a ge get 5 9 pal pp DPB (2) = DL Vi+ 427) = D,(1 +422)? =H + 4271/40) = a(t 4 407) is. vi-aF wee fence he de len by acon exon and ue the hain powe ue, Thecem 29:2 Has? Be) =Y2—22 02. be) = 8 M2) = D6 ta)??? = Hs —32)-V-3) = 215-32) (2) = D,| Vie? 1) = Dae? 1)" = Hae? — 17,40? Dy Gp = Dyes = He ata g-astpie fe apply Theorem 29.2. Thus, FG) = Yo ~2247 1! 5 209) =~ 208) aaa = feo 224 a jee De{2cos(e'/2)) = -2 singe!) QV?) = pV Aeinatl) o -sin fifi sc /F tan D(a") = 4 we /F tan /B Fs se y/#tanya/ fe D, leat JB) = Dyleer% Virl/2] — —cue%( fart) Gr U2) = — fi Beye Jaime Iniroduce a factional exponeat and thea use Theorem 29.2. 300) ~ Vain = Vite 9 (2) = VG Kin 2D in HS)” Vie = SS) PSE} Note tat a tara cos ‘a ov Vina (agai) cin shin sfc _ sey 's incorrect because y/So8 FT dors not exis. fry Joma) =D. 0-2)" = Hox (9-2) 4D + 0-2)" Yo+ 0-92) ei HO+ 9-2) M(9— 2-1 Was (vs tan V3) = D,(e¥ tan e744) = 2? tan YP tan 21D fel) Sat eed VY) 5 tan 2H) Bh et Nan 18) amet yet VE viewed figure shows a plot of NDER(Y/ ten /172). 130 ‘THE DERIVATIVE AND DIFFERENTIATION In Exercises 17-92, find dy/de by implicit diferertiation, Wt, ste y= 16:22 4298 —= aiae—isyon 2 18. 427-97? = |; e182 = 0; 19, 84 y= Sap 32+ ayy 4 8e Sh ai — fe Wot P= Tey We ditirente wth eapet to on bth ie. pet + Day? = D,(72y) ‘Then Dez? = 92. Because y is a faction of x, we use the chain rule to fad Do?, Dav = Ooige ‘To find D,(Tzy) ne ure the produ rae D,(@) = (2)Dyy +¥- D.C) ro thity Subetiating from (2) aad (8) into (1), ve ave 2, dy ya ee ae apf rel ry Ne reste alo Oy-telfl ary os Be dyate ALTERNATE SOLUTION; We divide on both sides by 22. Thus Gh) ‘Therefore where the cpt none ef the tno sf theqndatie uation ya Te {follows fram Bo, (8) that yoke ‘Thus the graph consists of two lines through the erin. Because yis nol a function of z in any neighbor “ ‘of (00), then dyide deen not exis 9 (0,0). Diffratiating both cidas of Pa. (6) with respeet to x and ash By, (5) we find ay , 2y 4320 = 5 > We diflerentiate with respect to 2, and have D,(22°u) + D,(32y?) = 0 Becaust 22°y is the product of two funetions of 2, namely 2s° and y, and because Szy? is the product of and y!, we use the product rae. Thus, /ATIVE OF THE POWER FUNCTION FOR RATIONAL EXPONENTS AND IMPLICIT DIFFERENTIATION 131 (28h 1-222) +(0=-2924 7-2) <0 (2 +90y b= -624y— 3? ay _ teva) de 34 Gay? ay, dy, dy_In—tey? zany? tay saehll cas tall Hey 1) atte ee ae cea) = ile sales Bay sale +9) ining with rexpet to 2, we have Ves 9D +9) seeds) 1 reece shah Lomi we 1-rate-t 9) = coe +n dy _1—code+y) ee en teantorrin lita oaiaisl ext on = 0; (-coctay + (y+ 29) = 0 B= E caativy, 27 =, where sone of he roof nt b= Ory ‘ifereatiate on both sides with respect to =. ia(z +y)D,{z +2) =yD,(in 2) sine Day e+ nii+d) vom etsine dd Badia) alee rayien inte +p) tein 21Gt = ycos «inl +9) oy _yeose +sin(s>y) > ain Fee Fy) ses 38-36, Sind the indicated line. check by plottirg the line and the curve tangect line to y= V4 +9 at (45), “Te” Tis ‘The norma line to y = V/1B-F=? at the origin. v “The normal line to 93° — y= 1 at (1,2). yz? ee peg neva ee saat §, Tengoat lines y = fe~4)+5= $e+8 pie 00 = 0 The ages ins onal the nonlin ==. 192. THE DERIVATIVE AND DIFFERENTIATION 38, The tangent line to the conve 1624-94 > Dilfeeniiating implicitly, we obtain se) 4 ft, = Sa? 16 ge (1a 1S oe Mma O88 = 108, a (12) 2 ovation of tangest line y=? = 22-1); 2eey—4=0 32 at the point (1,2). To plot the curve we sole for y. Then gf22—10e y= (02-1024) Se sys (ina-yiy tam Seating 0 evn y= Mey 1) y= 22 Sabing gv s2~22) ‘s2~ 1)(2—1). At the points (4) and ([,0), the tangent line is parallel Lo the 2 aris. The graph is ‘eipoe in the rst quadrant tangent 10 the y acts at (0,1) and the = zie at (1,0), 58. ind equation for the two fines through (1,3) thal are tangent to the curve 4p? —de— B+ > La &; be one of the reqitd lines and Ist (zy, be the point at which i tangent to the curv To fin thee a ye he Ee) and een phy ith wpe 2 Thay ayy ging, Ho detl oee? sae res sifl nein, Ha aBEtt = E42 encom, = Fe Because; cantats (1,3) and (2,24) by definition of slope we have inating my between Eqs. (2) snd (), weobiain r+ 4y?—4,— In, + “The eure contains (2,9) #0 this pr mint stingy Eq (1), That iy Any Ae, By, b8 F | sf 2—4P) he ing line parle to haw ase we gE [p—1); 0 = Se? de +1 eT 0 ty subtracting terms of Fa. (5) fom corresponding teres of Ba. (4, we get meek at! Sebstitating from Bag. (6) into Ba, (4), we have 169, +10=0 Sy? My +8= 0: (26m 4)= Oh y=? and vy oot? 1 ation of fis GREE A hoot en caution o 0 1, and an equation of is {mn Rawrcines 30-42: (a) Find two fanetions defined by the equation: (b) sketch the graph of each and (©) tauation, (a) Piad the derivative ofeach functios and state its domain. (e) Find dy/da by impli diflrentat tind chock with (4). (Find an equation of each tangent line at 24. ast © Gyn eave hile @ fle) = Jy dora @vtae-a nat y= file) =2V2=3, then B=}. Fhe My ffs) =-2VF=8 then B= 2 = Fe = he = fe) “Therefore, the result in. part (e} agrees with the results in part (d). FQ) nt, uid) = be Tengen ie so the graph of Sy: y=2 = (2-9 2 y=1 =O FAG) = 3 THQ) = 1 Tangent in othe waph of fry +2= —e=S sey 223, donnaine 2 2% f,{2) =-2VE=B, domain 2 > 2 2>% fe)=—t ap tonnine 22 1 DERIVATIVE OF THE POWER FUNCTION FOR RATIONAL EXPONENTS AND IMPLICIT DIFFERENTIATION 153, Bxersse 34(0) Brarcise 3) 8, the factions ace given by fj(=)= VETHIG and fy'2)=—VFFH. The Aonnain ofeach function is (~20,20) {B) A sketch ofthe graph of f, rahown in Og. aand sketch ofthe graph of fs shown ini 3. {o) Asher of he eragh fhe siven equation isshown in 6, ot 18)" Tae) = le" +10)!" ye? 160) Li@ = -Ke+0y"Pany HO Te HO=7ae ‘Tee domnin of mach derivative i (ne 40). {e) Dillerentiating the given equstion mplidtly with respect to 2, we have Geen, | Be ve Ee Baye hie) = Vere nd Z=n'e= afoul Pat Bere vd eet cd ty ase een (0) Per y = f,(2) if 4 = 8, then yy = fy(e,) = V-IFFTB = 5; min) = f'n) = FE “Thus an equation ofthe tangent line to the grant of f, at (-3,5) yo5=-He+3; set 5y-16=0 For y= fa[z), xy =—3, then py = Ja(1)=—y/(—3)* + 18 = 5; tay) = fa Sete oqulon of th tages ie to the gran of fat (8-5) a pis=Hetsy be—5y 41820 JPET, fom par (A), Beerise 40(b) Berise 40(¢) 4; y= 2 VPS, f(2)= VFB, domain: 2h Vz"=9, domain: 121>8 (oF = 97-7, domains 11> 8: fy(@) = 202-9) doe ana Bie e-¥ Mya fie) ~ aang iyo Wy = fle) = VEO then Bn = EO) 134 THE DERIVATIVE AND DIFFERENTIATION ‘Therefore, the result pat («) agree with the results in pet (4 (O49) 4 Fy-9) =p Tangent line 0 graph of fy <8, Jy) =f Tongent le to raph of Fe 0 Brerciee l(a) Exercise $1(b) Byerelse (0) a. At y' = 25, 2 © PBI 4 Jaco) JRF ail ch fd = VER domes (a) fa'(e) = -2(25 = 22/172, domain: 12) <8: fy'(@) = (25 ~; 7 domain: [2 1< 5 wetty y= tin VaRs ea ‘Therefore, the soul ip past (6) agree with Ye ois iv part (2) (0 F423, FG) = —y Tangent line to graph of fy = He) +3 fo(4) = 3, f(4) =3 Tangent line to graph of fs asd ve Bein 269 a ayaa tern Fg fy yt pay Ht 14 3-2) -— Fa ee oO 4A. Given 21? 4 71/22, show that 24 =, © We fir solve for y. Thus, 2.9 DERIVATIVE OF THE POWER FUNCTION FOR RATIONAL EXPONENTS AND IMPLICIT DIFFERENTIATION 135 dcg thine 4 42 SFB ABO =F = HUE 43) 980) = Vir TR 10 Ata ate VIER A Vass An chject is miwing alone a lise aeording to the equation of motion ‘which the measure of the velocity is (2) ; (b) 1. yore ee a? 43120 % 243, There ie no such value ot & V5+E. Find the value of f for “The velocity i given by iat =feF% Bb 0 ia never J (2) We tet The, @)Welay=i Thin ‘We eonelude thatthe inslantaneoss ve C(x) dolters isthe tial cost of producing = igre of igi C=) HA (3) 0's te ogi cmt foncbon: C2) =F O18) = Fe = ‘Thus the marginal cost when 16 Titers ave ey 1a 50 cents per lier (8) We wih tofind 2 for which C'(x) = 0.40! 2 = 040; y2= 52 =25 ‘Tharaare 25 liters are prodvced when the miaiginal cost $0.40 per liter, ‘Cfe) dollars is the tral cot of produeing z units ofa comerodity: C(e) = 40+ 32+ 9Y%e (2) C’ is the marginal cost function: C'(z) = 3+3/y/2"; C'(00) = 3.9 ‘Thus the marginal cost when 5D waits are produced is $390 per unit, (©) We wit toind efor which Ce) = 480: 2 = 450 VB = 6: == 18 ‘herfre 18 unite are produced wen te gna cost $4.50 perter 0B) = 00208, 0,10, a) =0(0 20/2090 = 29-2) = 30(300 ~ 22)-*/?{300 — 22 —, i) RG) = when ‘te daily production of « panicles factory ie f(s) wots when. the copia investment ic x Gowran of If the current capitalization is $760,000, ese the derivative to estimate the ‘Change in the dily production if she vapital investment ix increased by 81000 Le (ej and Ar =, hen Af » 2) He) = 200(22 4)? Fle) = 2002) +1)7" = BI Because th cusent capitalization $7600, hen = =760 Th, P80) =p BO 18 Fis ity edn ones by aproimaiy 1 uni if he capil ivi ican by ih oats ses he acplane was ey cer the ate the hort eetance ela aman hn foc Ine ot sght dance eines the plancand the dite. Then fram de Pribagpreat oe Pate enter ejoraney gf afoeeney en =r a1 w We witht find 4 when t=} $f y= ben" WE “heron, the lin of sigh distance betwen thy plats and the statu is changing a the rte of 2.Ten/min Soar arth plane nen diel ove the van. 136 THE DERIVATIVE AND DIFFERENTIATION 14, At & hours after 8 AM the frst ship i 24 miles north of P, the second ship is 84(t~2) mils east of P the nance teen tem sale Tagn&— (240) (020 ~2)?. Dileep, nade ~ naasan + ste —yan, = EASE 2) a a ix _ 281) +2941) (a) Avo AM, if + (82)? = 1600, « = 4, fe = EEE = i, ‘Thos st 9AM the distance between the ships ie deceasng at 1.2 Ants 7 [b) AVL AM, t= 3, = 37 + (1) =o, = VDE, f= EEE = FBS ah ~ Vas ‘Thos at TIA th distance between the ships is increasing at 34.9 knots 55, Dla) = 1,688 =P te = ag == El ante) 2 56, Find D,201) when ives, > We we the definition. 21 {e7 HES0 byzi= {Ft HZ C8 ntsi= ois £0. pay Jet tle fet 94 aye! 4-200) = EE = Be) 81. fe) = Dye? Al = fle? 971 = le? P9022) = Sa Teal Alternatively, D2? |=ago(s!—4)0,(@? 4) = 22 gn(e! 4,2 #22 88. Find the derivative af the fineton g2)= 2b > Becatse| is not diffentialeo 0 we cannot ut the product ule 350 theme Q)aawetba /(e)= a= aa Ifz <0 then iz! r(2) = -2* 90 that g/(z) tee 20 then Furthermore, #0) =i 2k sz) =a(0) Meg) 60, fle) = leP = aagn oy fe) —Balogn © = Atel Je) ~ Go gn == Be 60. Given g(2) =| f(e)|. Prove that if f'(z) and g'(z) exist, then|o'(2]|=|J"(2)|, be) = DUCA? = Hse ADU(e)7 =| se) le) ee) and 2)" $2) 2) |=|s" (=), peorided f(x) €0. Sappose fa) = 0.'Thee rm, LDL yy, [N11 ip ‘and Jim [xe] 20 and ez) ro) . Phenfore a a MASA Mee, (8) = 0.1085 fsee, (2.2) Amsin tos tos?) + O.195Th f/see, (1.8) Loa fe/see (2, Prove thet she sum ofthe « and y intercept of any tangent line tthe pagaola 2/2 4 38/2 = Reb Pg yt) =; f= 2 iffeeatiaing implicy gives "7 + ar 9y! = 05! = ag = Ee wt The tnget ab (ein) ie w= vo = —Sirale— tok a+ ore Bit tras! cai ts ayer “The am ofthe eitercept and interes! 2.9 DERIVATIVE OF ‘TIE POWBR FUNCTION FOR RATIONAL EXPONENTS AND IMPLICIT DIFFERENTIATION 137 ff 1. Diteatining inp, JY 4B VO yan a aE ees then "+9 = y= 1-4 =H tees 4 (y= ]VF and y= 3V5. eh NBA ct OS a are @y =-Wiady= vt g-Mhe as =p ‘An equsticn ofthe tangent line iy +3V3 = —VSle +): Se +94 4V5 =. ‘Suppoce that gfe) = YO= GE and ha) = Fle), whore fis dtlerntible at 3, Prove tha (0) = 6 We bave Ke)= (0-23) By the chai rate, RG) = $((9-27) 9D, 0-2" = "9 em Ho) cm tee Him PO) 08 Sion att ay hn 2 25 978) a a8) Atay (a2) a -wlplying the last two equations, we have @y oe at at Because 2y = =e VQy) © fy de 1 A) inequivalent to SHEE 14> 0 with p and q integers, and vt y fa)= 2 Let r=%, and ty Ditewntinting imply gee 18 = pl ‘Therefore f"(z) = rs". ‘The rot given in serie sot comes bea it wat sued tat fi dlienable Compute (fo 9f(0) if f(2) = L472 and g(a) = 2". Explain why the chan rule eannot be applied to parte ss compataton, i Toate) = slo) = (er!) 1p = 2 47 andso (Fo) (a) =2e +% (F090) = 20) +7 However, 7(2) = 42-2) and g() ie oot dened. Hence the chain docs aot apply a 0 RELATED RATES fz ba vatiable tha isa function of ime, which i eprcened by then the rate of change of = with respect to time is given by de/dt. The following sts abould be taken toolve problems involving the rate of change trith eapect to time fr two oF more rated variables Sect letter to represent each variable. 2 Kdentity the constant rates of change that aze given. Tammy the rte of change tht must be utd Find an equation tat expreans the elation beeen the variable. Diferentate ith respec to fon beth sides ofthe uation. Replace the given sates of change with their constant values Replace the ttiabes by thls vaum a Ine pstcuar ine of etecet ‘Salve the reslting equation forthe unkuown rate of chai. Ofer it is hepfl to draw a Sigue to find the equatios in sie 4. Be ceeful to distinguish the earahles, which represent length, area, volume, ete, rom Uhe rates of chinge of thee vasiables, Altzough # variable 7 13). THE DERIVATIVE AND DIFFERENTIATION that reproente length may appear as @ dimension in the figure, the cate of change of this variable does not appear in the figure, ‘To find the equation you may use any formulas from geometry for length, area, and volume, You may use the fact that corresponding sides of similar triangles are propevtional. Following are some ofthe formulas from geometry. 1. The Pythagorean theorera for a ight trang: +6? 2, Cireumfererce ofa circle: C= 2x° 3, Area formulas for plane figures: ‘a. Rectangle: A. , Triangle A= # 2 ltl ag fee A= VE? 4. Perallelograr: A fe Trapeacid: Aad f. Gee: A 4. Sorface arse formulae for ood 1. Right ccular cylinder: (i) Lateral area: $= 2xrh (i) Total area: $= 2erh +227? », Right exeular cone: (i) Lateral area: S = er EP i) Total areas $= xeV/FPER? ar? fe Sphere: S = dr? 5. Volume formulas for solids: ‘a. Rectangular parallelepiped: ', Cizeulareylinder: V = zr?h © Giealar cone: V = 5a 4. Sphere: V = fer fe, Prism: V-= Bh, where I is the area of the base £, Pyramids V = 2Bh, where B isthe area of the base rercse 2.10 In Exercises 1-8, andy ate funcons ofa third variable detdpniy Hest 0; de 1 2 +ay 28; Hea ott uh ady de Becanse 42 = 1 then 8 | re eee | aye meg ylE= 0548 = 29% When 2 =2, = 10. Benue Mt Ie2sin 244 cos y=3 and $Y= 3, ind $F at fr), | We differentiate with respect 10 € on both sides ofthe given equation, Because x and y are functions of ¢ we) rust apply the chain rule. Thus t asin yt vee Qewe @ Welet 4 =3, 2 = fp and y =f and ssve for az eos =a IVD) =H;—-2GVIFE-1GVDC Hao Ban reo sf +2 conan vit = 0 effin f= Seg ng eo Mi 6 24 y? = 25 and SE 4H wien y=4. Beenie the value of zis not given, we solve for VBP He tl = ER 210 RELATED RATES 159 B= 5; al gy oo, be Made by andy Vit =o 2 PP a5; ee ty A 9, oy Beane “ F at yun 2 +1) =4 and $2 =~, maa $F when 2 Became the value of y is nat given, we solve for v tan e+ 1) Differentiating with respect to fen both sides, we obtain 4 2 sec? 2 dt $= aan 241)-Peed 2 4 ay = ds We let Y= 4 and x= x and save for 4. and when #= 1, y= Alten +1)? sect 2 dE ds, de =~, Be At tae, let 2 be the bornontal distance fom the cid to the ie, 20, and let § fe the length of the sting. Ths fon te Pythagorean Tete, 5! = 404+ 2% 288 = 2 = #900; «~20. Boome 42 ~3, we nove | = Ha a8 When $= 80 we have 2800 = 1610 +x; =*~ 900 =~. Boones $F —, we haw MS] = $8. = “Therefore, when the length ofthe string released is 50 f, chestring is being paid oat athe rte of? ft/=e. AL min, Je rm be the radius ard V 2 be the volume of the spherical balloon, Weis do 5, theeeede] Beard Becamse p= t= gach THeteloe i, = Fe. 36 = TOR ance whos th balloon i Ir in dinnter, the met i incresing tte cae m/e. ‘At t min, let r f€ be the radius and V ft? be the volume of the spherical snowball. ab: Voter Because WV ng, de 8 2 therefore ttl =. Vag mtn Braue ns, $= 852, Theeived| = y= Bence when the somal i fn dace, che sai ining at th ae of a inl 8 * Acpherieal mowboll with darncler 6 & starts to mclt ot the rate of | Ri/iain, Pind the ce at which the ‘dius is changing when the radius i 21. Ae rin, lt FA be the radian and V fi be the velune of the ephesical snowball Because the snowball i melting atthe rate cf} '/min, we lave 4 = 1 We must frd 4 when r-=2, The volume ofa spree given bythe formate V = fe Difleeainting with respect oon both side, we bave rat a gh = 030 We cone thatthe radius is decreasing atthe rte of 0080 fein wen the radius i 2 Av tin, eV cic mete be the valame of the cote ile tr meters be the rads ofthe Base of the ls aud eth rere tego ie pl Bee r= ¥ =r jh? = es SE fh ate rere 14 and 14 ight hangs 16 above «horton! path and a man 6 al is walking avay a 5 Yee. ow fit i the mm’ thadov lengthening?” ‘Atoll sft be the mans distance fm th igh af the length of hs Shadow. Ten e/a similar triangles, Eph sextet 2555 gah js lengthening at the rate of 1 f/sce. Subst w ng for Mand rj we obtain —b 5. From 2p, de Bat M0 THE DERIVATIVE AND DIFFERENTIATION 15, How fs isthe tip of his shadow moving? D At eacg lees fb thediatance of the an fom the Hight and ety te the erat tance ofthe pa the ters shadow rom the bottom ofthe oa heling te ih. From sina rangls, BEE ay. ceuay sp ute, Pa 8 Dice a4 Bees = ge = fy Bem a sy = = BBE Deca GF =5, =f “Thetlore,thetp ofthe man's shadow b osing at the at ff ac 16. A man 6 f tall is walking toward a building atthe rate of 5 f/see. If thse isa light on the ground 50 ft from ‘tr builting, how fst is his siadow on the building groving shorter when he is 30 ft from the building? > ALC eq, let «fe be the distance fom the max to the light and =f the length ‘of his shadow on the building, The figure shows the masz at point M, between pint L (she light), and point B (dhe bace of the building). Because the man, . is walking atthe rte of 8 fi/sc, we are given that di/ét = 5. Because d=/dt inthe rate of change of the length of the shadow, we want to find de/d¢ when the mas is 30 fom the builing, thats, when 2= 50~30 = 20. By sinilar tslangla we have 006 Differentiating on both sides with respect tof, we obtain 4 Replacing = by 20 and $f by 5, we have ‘Therefre, the shadow i roving shorter atthe rate of oe when thr mais 308 Lom bu 11, Ast days, wm inthe sain and Vem? the volume ofthe wbercal tumor. When de ooo. v= ger’ SH | = anvil, = 4x(0.5)(0.001) = 0.001 ‘When the radi f the tures fs 05 om. its volume is increasing at the rst of 001 1,003 em? per day. 18, Ait days, wn isthe radius and V pm? i tke volume o te phesical el, When OUR nO. ated wast ephaaiel adh 001. v= go Mm tte = an015)%001) = 00r When the radius of the cell is 1.5 am, its volume is increasing at the rate of 0.00 ~ 0,028 um? 19. At t days, rem is the radius and $ cm? is the surface area of the spherical tumor, 5, f= = aart, 35 smit| = 0.004n. When r=05,ff= 0.0. San; |_| onde | = si05y0000 = o0ter ‘When the radius of the tumor ie .5-cm, ite eusface area iz increasing at the rate of 0047 =.012 cm? per day. 40. A tactile oped ia hae. Ifthe as of he alls ncnig atthe rte of 0.0 miremetes Pe dashes 1s ain the ce fice ofthe surae se of he ala at me? > tdays after the call began to grow, let rym be its radius and S pm? be its sucface area [Becaise the cell is growing athe rate of 0.01 umn/day, we are given that er day. doo We mnt find when r= 1, The aren of saphee in gven by the formula San? | Dilleentatng with reset toto bth des, weave = arede ‘tating for r and $%, we obtain 48 = Br(1.5) ye Substitingfor rand $6 we obtain 43 = Bx(1.5)DO1 =0.12e~ 0577 ‘Thus the auace ee is incresing ot teste of 0377 p3/dny en the rc a5 2 Att min, lt V cable meter be the vane of water in the anklet «mete be herd ofthe sure of ibe wale, tad let boners be the depth of the water. rom see rangle, = fer = fh “Th value of wales inthe tank an be expe in tema ofthe volume ofa Zone Ves borth = ith = adh Gh = beh = ae 4-6) 6 pean BY 6,ne are fl Shah ence, when the water is 10m dora, the water leva is lowering at the rate of 585 m/min. 2.10 RELATED RATES 141 ‘At t min, let 2 ft be the depth of the water, b ft its width and V 9 its voluine. We are given dV/dt =2. rom similar triangles b= a and s» V =}-126h-= 622. © = 12e4t, Whee 2 =1, 2= 12(0)85 = cn the ite 1 den ng he ak fein me vc viPag fn fh Bene fh =t.nd? 2) = a 3 aed. Th, when V8, the presure ip deci ot the rat of 1006 pn ‘The adiabatic law (no gain or los: of heat) for the expansion of aie is PV! =C, where P is the number of pounds per stuare unit of pressure, V is the number of cubic units of volume, and C is a constart. Ata Specific instant, Ute presute is 40 bin, and is increasing a: the rate of Slb/in* each second. IFC =, what isthe rate of change of volume at this stant? ett conds bet tne since the peste fpgan increasing. Then P and V ave functions of ¢. When P= 40 Be ce Gree has <8 cok ne mist nd Neem coer Unt in, P pounds pet equare fo is the pressure and V cubic fet isthe volume of gas. C is a constant, wwe have - 7s. Spt, y pvitad: visa Spt v Differentiating with eespeet tot, we have pa ak 5-2 and SPs we have sr wy i 1 1 Ha Hey 0-2)" = AE G9 = ay “the same i Jeesiag atthe ae of fy cable site pr seo at his inant Av tse, Jets cm be the rains and lei A cm? be the aren ofthe dstarbed region. A = xr de Because ae (4)16 = 128, Thus, when the radius is cm, the area of the disturbed region is inereming ob the te 18r em? fone AL min lr em be the ean ofthe ohm ts depth, V mits velame. We ae given $Y : ech ctadaiinegei® AV. Lent sna acini = sia ingles, £= 23: =p. V= forth = dettnth = de Vm Last oe ali th od Thus when the depth thei se, Att se after the truck leaves the intersection, lot = ft be the distance traveled by the truck, let y feet e the stance teaveled by the autamobile, and let sft be the distance betmeen the automobile and the truck ‘Then 7. From is increasing atthe rate of /min J? 190 yp Dys = PER VOPW). erase Dz = 40, Dyy =30, and when t= 2, == 80 [20-9 _ 1830+ 60(-a0) 00 Tor 4. Thus, 2 se after the (sek leaves the interseet> 60, we have Dal, = 2 aot ion the automobile and the truck te separating atthe rate of 14 f/sec. ‘A rope is aitached to a boat ot water level, and a woman of the dock i pulling oa the rope at the rate of 50 i/min. If her hands are 16 ft above the water level, how fast is the boat approaching the dock when the mount of rope out is 20°17 Let z fet be the distance between the boat and the dock snd = feet the w mount of rope out { min after the woman tegaa to pullin the boat. In the figure the boat is at point B, the dock enters the water at print D, and the 16 somaa's hands ace at point W. Because the open is pling pte sope at the rte of 50 f/min, then + is derreasiag, and 42 = —p0. Since {Fis the raje of change of the distance beiween the boat azd the dock, we wall to find $e 24, By the Pythagorean theorem we have 2 co) 42 THE DERIVATIVE AND DIFFERENTIATION auf = ade @ ‘Substituting = = 20 in Ba (1), we get ties sey 2 Sutntituting for 2, $2, and x in Ba (2) we find 2(20)(-50) = 20242, 4g = BO ‘The boat is approaching the dock atthe rte of 32./nin when 20 of rope is out 29, C dollars isthe cost of producing + uns int weeks. C= 0.082" — 2 + 102-4485 D,C= (iis? —2e-+10)D 2. Because Dye =2 and atthe present time 2 (24608-2660) +10)2) = 1020 inereasng atthe rate of $1920 per week. rows the cot i 90, x Uhotmnne hone ae dr 6,006, iE p cla isthe prion of 4 bay whe = 46 = 15,0009 792 = FEN 4 = -0.25. Demand is doerensing at the rate of 250 boxes a week Ss units are supsied per month when p dolls isthe price por unit. » = 1000/39 F 20m; dz _ 500(6p +20) dn_y z| 500(120-+ 20) _ 350 = Ab the pesent,p =20 and 4? =}, Thus, = = 25000 — 75, B= apheaap ot Atte ee a fn aS Hence, the mupply ix increning atthe ate of $75 sits per month 132 Suppose that y workers are needed to produce 2 units of certain commodity, and z= 4y® If production of the commodity this year Is 250,000 uslts azd the production is increasing at ihe rate of 18,000 units per yeae, ‘what is the current rate at which the labor force shoulé be increased? > Lett year be the time, and boti x and y aze functions ofl. When 2 = 250,000, we are ae given that p= canine eine a Door to ge stn 9 eho ra sath yal? Differentiating with respect to 1, we get a foWide Subaitaing for «and 4, we obtain $= Kesoo00-"y18,900)= 9 ‘Thus, the labor force should be increased at the rate of 9 workers per year at presen. ‘58 100s shirts axe demanded per week when p dont ie the price ofa aps+ 5p 4010 =o » = MTS _ 9.5, de = 2415 dp. P sides 4 M151 ‘This weak, p30 and Gf =. Theefre, fog — AP 3 ~ —B Because 4 = [1 ten 100 $F = 55. Hence the demand is dering athe ate o 5S shits er wos 4H. The hypovenuse p40 cm; w is sch that dae = yr rad/s. The menses f sides are 40 sn and 40 cn ‘Ac? isthe area. A= JAD sa a) (40 cos a) = 400 sia Za. When @= fy 4. = 00 con 20 $2 = 600 con Jor = 800-}-dyn = "Be, The are is increasing at about 249 emt sec. 35. Atthr, le the ditarce frm the inteneton be = km or the fs track andy km Denvoe, forthe acon tuck. See the fgue. = = VTFPE Dye ii ne po 5 y Sine Dg S heswmcigmmplle pea mae). pa — A/F. Ta, when the two rocks ate ech ‘mim from the Intersectica, they are approaching each other at the rate of ki/ lea/br. Hkand Dy) 210 RELATED RATES 143 civ 36 and 37, a horizantal trough is 16 meters log, and its ends ate isecees traps with an alte im, 9 lower ba of 4 rm and and uppet base of 6 m ‘Water is being poured int the tough at the rate of 10:°/min. How fast is the water level rising when the ater 2 deep? I the water lve is decreasing a the rate of 25 cm/min then the water is 3m deep, ab what rate is wate deing drawa from the ough? The figure llstretes one eed ofthe tough. Let ym be the dept of the water = the width ofthe aufae atthe ater, and Va volume, ¢ laut afer water began pearing into the vow Because water is being poured into the trough at the mae of 10 m/min, weaze given that dV/dt = 10. Since dy/at isthe rate at which the depth of the water is chauigag, we wat to find dy/dé when y —2 ‘The watet that isin the (rough in the shape ofa prism with altitude 16 1 and base a trepezod. Because the volume ffs prom le the ares of the base times is aliude, and the arca of @ tapeaad Is given by the formule Hoy 44)-4, we have Ya +2)-9- 16; V= Byte +4) o We express x asa function of y. By similar iriangles inthe figure, Subettuting the value for Into Bq, (1), we obtain a ye veaitss) yey, ae a ta Bnei t, we untitate = Wand y=2. Then r0= e082, $=} Thu the water evel ng the reo m/nin when the wae 2? m dep Ip Eaie 37, ne abate = Band yo8.then Y= @.3460(-1) ==22 Ts wales big ear om the ough a thera ni Lat he bs fhe ae be on the the wll dt tp yy the ground fe oe. We ae 1.5, Because the ladder is Tm long, then y= Va9—2% # Pe 2 Tg At tc since the bottom ofthe ladder started to be moved ioward the embankment, Jet the bottom of the embaskment be © [from the bottom of the ladder and y ft ‘fom the top ofthe ladder, See the igure. From the law of easines we have P+ y? — ry cos 120°= 400; 2 +9? — 2y(—3) = 400; ¥° + (z)y + (2? ~ 400) =0 + P=? = 400) Vigngsae? oye s appa of mie nae nating up aus 04 m/e ( Since 4E — 1, when 2 = de get oe Thereor, the top of the ladder is moving atthe rte of SVIT-EST wn 0.65 8/ene a the given natant If Inider of length 33 that is leaning ageinst a wall basis upper end sliding down the wall atthe ate of jft/ac, woal i the rte of change of the measure of the acute angle made by the ladder with the round when the upper end is 18 39% | y shove the ground? At tc, let the distance fm the bottorn of the wall to the bottom of the ladder be 2ff and to the top of the ladder be y ft. Let @ radians be the measure of the acute angle made by the Indcer with the ground att see a ere Sethe gue sin = Ben = fl Ho gp SE 144 THE DERIVATIVE AND DIFFERENTIATION i decreasing at the rave of & rad/see at the given instant AL. At sec after iakeo, let x tbe the horiontal distance from the airplane to the ‘point 10,000 8 directly above the observer, and let 0 be the ragian measure of the tion | airplane. e figure. 9 = r/3, 28-2 ange of clean ofthe sitplae: Sethe fig: 0= 1/3, = fy and eat il soni gp 1,00 $$, =~10000-4- 35 “The mings ign ips tates deeang, Tat ete othe ane 2 fe 12. Attn th bar akin a angle witha prpndaar othe hre nad + mom it fot 10000 Weare given f= HEE ‘The beara is moving at about 1608.5 m/min. 46. At ac afl lif the ook 230 hgh al he ear dh nae ge dh eprom, Aer 10 ‘ieee vy 50 ya os hag 50 y= 00 to = 100 oe dg .__da/dt______10_____ = 3. The dish revolves at & rade. E> Topica) HOO GOO WOOF TT] 38: THe Mie volves at ral 4A. Water is poured at the rate of §f/min into a tank in the form of a cone, ‘The cone is 20 ft deep and 10 is Giameter at the top If there is leak in the bottom and the Waser rising at the rate of I in.frnin, whes t ‘water is 16 fe deep, how fast isthe water leaking? After f mia, V cube feet isthe volume of the water, h fet is its depth, and r fet the radius of ts susface Because the water level i 1s ‘ate of|ininin when the water & 16 deep, end 1 in is jf, we given thet B= Iywten b= Wetind gV/d at this moment. Applying the formula forthe volue of «cone, we have v=: @ Beaute the aitud ofthe tak 0 0 sod the radu ofthe tank 3 by similar tare in he nr have n/a 8/0 eqivleny, = 9h Stig the vale ofr nto Ba. 0), we obtain V=Jo(ha= hi? Diferenasig with pect fot wehave Weds Hawt Weneitetshai6 and dh/ttm jp Tan Y= fect bye Because water is being poured into the tank a the rete of §83/min andthe rate of ehange cf volume of water in the tank s 4 f/min, we conclude that water is leaking out of the tan at the rate of 83x, approximately 3.81 £0/min, 45, ‘The volume of a balloon is decreasing ai a rate proportional to its surface ara. Show that the radius of balloon shrinks at 2 comsant rate. > F uit isthe radius oft balloon at time f, V eutic units i ite volume and S aqre unite fits surface Weare gven that U = a5 share be nogative constant. We mut show tha is a negalive constant. Now Y= fer? Differentiating on both ses with respect to f we have Substieting fom $= 4r? into the abore, we get Substitsing fom Bq. (2) inte Eq.@), we obtsin KS = Decautedr/dtis «negative const ky me conelude thatthe radius sinks at» constant rate. MISCELLANBOUS PXERCISES FOR CHAPTER 2 145 ‘Paccises 1-14, find the derivative ofthe function. fetta 4223 > fe) = let Me+2 (4 +32") » gs) = Waa? +212') Fedaerat 9 se)=h-8* » We apply the power rte. Thus, Dalle? 26-8) = ted 12 = Ba moe panstp ara _ tae Cony = GW =OP—ue+e-1) > CH= BPO 4y4 441-0 abt asia a 2ue 6 6F= Hoe 398 — tA Foe) = (ot —Any(ta2+ 2245) Wermuliply and use the power rule. fie) =425 +2284 524— 62" —42?— 102 Je) = e+ 10s" + the? 24s? 82-10 ~#4 iq) == Bee +1) _ ost ae =S + fe) a aaa omg p2otet 8) =e") _ au" 16y— 94 _16)—94 Fa8 8 Gre? uray? 110) = (3-26-4084 » 110) = 4288547663) P(e) = (eae? +1)" We factor and use the poner chain tle Fle) =(02? 197! = e412” Fo)= F022 Mn) fleet aya ate, Fe) =o? yeroil? — w PQ =He 2(a? = 1) a2 4) fae? 4) + (2-1) = ale? — NE —-Aa?— 19) ae) = (e427 %5- 277 > ge) = (24-2) 7D, 6-2)" + 6-2 "Det (et 2 (1) 24)-4(-29) + 6-H Ha—) A(t ays Ay ae(et 2) 105-2?) és? 002 = 527 +15 Fee 19¥90)(28— 49/2 2 a? 199% 4/402) = (ete) 5 te — 6025+ 15) = Exercises 15-20, compute the deivaive. Dee t sins 2000s} = cin 2-4 (2+ Neos s—cos 2+ zim 2 = (2+ In 242 cos2 a) Wess the poner ile Fink 2S in t-Diein 3) =2 sin 8:8 083 sn Stn aes Fed fan 4 an t-te I givin) fies = Yan yc 4 = Be £{ec003) » cord t{-sin}-—h) We se he poduet ral inthe fs erm; the shin tle nthe econ lta 2a) nec = + tan De Defoe 2) tee%(2 eee z)-D,(2se 2) Secide sec stan Be tan x ace 7 + see(2 @e-2) 2 see tan 2 146 THE DERIVATIVE AND DIFFERENTIATION In Brercses 21-24, compute the derivative and check by plotsing your answer and NDER in tke samme wixdow, af ge) ME = tele oA) Fe ata mm. ¢2)= fa -Gea Me synasiasete wrtiett fetes & I =te Bis) ay) = tas B= . "ie 20) 4072 21, tan e+ tem (coy +6? +2) = a 2a. an(e+y)tain(z—1)=1 © We eae iplcily wih rpect to on both ede cote + o3(1-+ 2) + code — 11 ~ $8) = 05 festa +9) ~ sone ~ } GE = cosl+ y) ~coele— a dy _cole=y) teosle+y) _ cos cosy GSP aamery) sea Tn Feros 29 and 30, assume that each part of the accompanying graph of, the continuous funtion f tha Sppeas tobe ane sginst fs ine segment, () Dele fplecense Gnd (0) J2(-2) (©) F,(-2) (4) 1240) () PaO). (F422). Ch} Ae what numbers i f not differentiate? tt Mest BOM e hee poi? ene errant Conte ORD 7 aseint tm 0-2) <4 (@) 0)= lig Oot © F40)= tim P= — (9 F212) = jim 2 ®) A= eee 0) fe not ientate nt 2a P4Ge48 fc lige fazez<0 BF toes anu aS +62 45)43 wees eee (e448) =2 MISCELLANEOUS EXERCISES FOR CHAPTER 2 147 OP.O)= fp Saf = lip (e42)=4 (h) fin sot dliferentable at —2 and 2 tte 1 and 32, atch tho pap of catia faction f eel on Bad having th give popes entiable except at ~2 2% f(z) > Dif'z<—2; 32 J is differentiable exept at 1,0, and 1; he Beech MQ=% saup of ip oot pia) FO =i I0=3: 2 332-1 at (2,1). Check by posting the curve and the tangent. Basen tne PO autos ah Eyatnct mga toe aera It Find an equation ofthe normal line io y= 5 (9,2). Check by eloting the curve andthe normal 1a?+3)=2(2e)_ hy =te~ He) EAN 5 98 m= play = lly = 8. Noma ies y =e Dey tet Find equation of th tantent lines to » =2z"-+4?~z having done } Check by plotting the is nd curve 3 y= 6a' F821 Yay 162 3= (Gx 1)(e43ss0F=h2=—b Ave hu=up?+agh- b= ation of tangent line: y= ates By =e) Find an equation ofthe normal line tothe extve 2 y= yEFH at the point (3) Differentiating implicitly with respect to 0n bth sides of the given equation, we get 1-Day = Hot 7204, Replacing = bs 3 andy by Laud solving for D,y, wehave 1-D,v= 3470 +P A-Dy= 14D Dt nce the slope ofthe tangent to tho curve atthe pent (31) ie and the lope ofthe norma ne share =$ Thos an equation forthe normal ine is pois Hea), Sey in equations of the tangent and normal ines oth curve 227+ 24° y= 0 tthe point (2, > 622+ oye icc ae ta pain nyt He-2) S0-44y— ‘At QL) the normal line hee elope “Hea aquation fey 1 ——(2—2) da +5y —14=0. Pauation of tangent line: 14} THE DERIVATIVE AND DIFFERENTIATION 438, Pind equations ofthe tangent ard normal to y =8 sint2> at (fy!) Check by plotting the lines and eure > $48 sin?2x cos 22. Mangent = 1(Gyt) =48 st? cos Jr = 6/5. Tangent line: y= 6Y3(e ~ Gy) +1 ama Gg = ABV Paton of ele: 9 = —fyVHe— fr) +2 ‘39. Prove thet the line tangent to y= —#'-+ 22" + at (1,2) is tangent to the curve at another point and find it, et tating) i At (1,2) the tangent line has slope 2. is eqation is y—2 = (e— 1): we "To delormine the Tatersedions of the line and the original curve we solve the fist and last Squslons siultanensy, Tey 22! 2 = 24 ly 2! —-22?+1=0; (2-1) = 0; (2-2 +1) When = iyi ~. There ac te waye to preeed Method 1: yl) <1. Hence the tangent line at (-1,0) has equation “1, which isthe tangent line a (52) Maiiod 2 Since the roots 1, -1 are repented the ine is tangent a (1,0) and (~1,0 40, Prove that the tangent lines to the carves 4y?—27y—2+ Sy =0 and 2*— dy? +52+y=0 at the origin are pei. > Wachow that he produet ofthe slopes ofthe taagent ines at (0,9) is 1. ‘The is curve is yay yO Diterentiating implicitly with rspet to x, using the product rule on the second term, we have r2yAft— 244 aey 1 +598= 0 ph Lad =0 2 Ue +1) oF equivalently, einem Day ate iis ey eh Est sth, the slope a he ange et he it are he igi. The cond eve ie AaaPaseey=0 Difierentiatng implicitly with respeet to 2, we have pit 54 8— adage * ard y=0, we have (8 = 2072-2) = (920) 9, EY = (9 229% -2) = 918-2997? toa SY ee ae -e rary ott wen 19 Ba fle) ae PHB pert MO abet tet ate t 6 se) matted ‘We wish to fird when f*(z) >, that is when 2? +4243 >0; (7 +3)(r+1) > 0. Guo is 223>0and 241> (2 >—$and 2 > -1. Hence 2>—1 Guedi23c0md 241 ct e<-$and 0 when either z <—3 or x > —I, 4AM iad the rate of change fy with respect to atthe point (9,2) #792 ~ ay S Ditmentating imply wih sespc te sing the product rle.om the second tam, we have jay agp yb? — soy = 0 é , 7 WeltseSindy=2end covet -s-ult=ag = “The rate of change of with repect 0 2 is fp tho T MISCELLANEOUS EXERCISES FOR CHAPTER 2 149 1045 ara 46, pactcle is moving along a horizontal lie according to th given equation where s meters ected distance of the pactice fren a point Oat t arconds. ‘The postive diection i to the right the intervals of time when the particle is moving to Ube right ané whea it ls moving to the left. Also shen it reverse it dcetion 624 64-12 = 41-2) = (64 2K0— 1.1) =O when C= Concusion = wis postive and the paride b moving tothe right © = is Fee price chacging deci for right let = 0 8 negstive and the particle is moving to the left Geos zero and the particle i changing drertion from lett sight + + 055 postive and the parle is moving tothe right MGW 2b 8)— (1) _ te 2t— 3) 4613) gen tt and b= Cares, aaa aasap MOT Omen teat and t= gm (v's zero and the particle is changing direction from left te right A (@) Brunton (10) iss = 162+ 00+ Because vp =-20 and 6p = 20 the uation of motion is = —16e— 2084200 (162 — 201-4 200) = ~32— 20, s(1) = ~12—20 = —54. 03) = ~12-3-20 = —116 262 fee the wocty ai se # 16 fice (it as int a dep el). f= 46/88 —). The bag hits the round after about 2.97 se. A Ms byoed it Sbost 1149 fee when ihe tbe ground = 204,42 (9 (t= -82-§v85-1)-20 $8. A balls tow upward from a Icight of 112 with a velocity of foe (a) Wite on equation of aed simulate te motion. (b) Estimate and ()ealelae ow high the tll wil go and when get thers ‘atinats and (©) nla bw Tog tthe he al foreach the ound. (9) Pind the velo at 2 veoh tex) Find the ged t 2 we at ser) Fad the velocity whe il the goin fo (ae stor sneer tia (en -28t7 96 ~O mien t= 9. 2) = 16-98 496-94 112—258 @ when 0=-16(?—6t—7)=—16(1—Tt+l), t=7 see. (= -32-2496=) Bd) —-92-4.4 96 = H38 @) The weed 2 F/oe a bth (9) of) = 92-7 486 = 128 MISCELLANEOUS EXERCISES FOR CHAPTER 2 151 ins 54-56, a particle is moving along a line where at ¢ se, ¢ em isthe directed distauce fort the origin, » in the wlocity and a cm/sec ie the aeeeraton. (4) Pied v and a (0) Show that the motion is cere () Stmulae the motion on your grapes ealeulatcr. = 5— 2 cost 5 ~ (1-4 con 2) = 4— net (a) w= delet =2 cin 2, « = de/lt — 4 con 2 (b) The motion ts “mmple harmonic becane ais proportional to the distance fom a fixed pint (s ~ 4) and appactely directed, 4 con 26-42 sin 26 (a) v= ds/dt = ~Psin 2144 cos 24, a= de/dt = —te08 21 ~ Bin 2 |b) The motion i simple harmonic because a= 49. st +f) sin ttf) ofa = Dielaherfe) + nl at b= Aeon feo +f] Didlon tJ) + cot +40) = —16inat $e) + sin(ats be] ~l6's a cnstant, then a, the mearure ofthe acteratin, is propetionl tos the measire ofthe “Geplacoment. Furthermore, because “16 sepsis, the a snd sam opposecy dicted. Thos the woen a aplebarmonie Feofiton an item in $200 if sot note than 800 are produced each week and decreases $0.20 per iter foreach fiem over 810. (8) Express the piofit aca funesion f of the number x of Hens wld. (o) Prove thet f is ‘entinuous, c) Determine if fle differentiable at 800. () 10<2<80, then fie) son <= £1600 (6) ln Fle) (900 — 28)2 = 3602204, 200) ana fim 2) = _ i (860 —2e)x = 200-800 a0 fis continues a 800 (6) pe Th 16a 00-40 0. Hence j isnot diferentisble at 800, R= CT", Gnd (4) the average rate of change of R ae T increises fom 200 to 300; (b) the fnstantancous mte ‘of change of Rwith respect te'T when ic 200, (o) Si = 20M 20" 6 00 sok () RIC) A square units is the scea ofan isosceles right tingle for which the ength of exch eg ib © uelts. Thus A= $e! A/(2) =. (a) The average rate of cange ofA with opel to = as © change fos 8 Sl 8 013-8 _ 908005 _ 5 a95 “mre "nor a (8) The instantaneous rate of change of A with respect to when x= 8 ic ANS) =. Hey= 2, fd the relative rate of change of y with respect to 2 when (a) ae dufde dy _ap-ays Pie eaive te of change of y with mpc tosis MYME voy, a3 dylds 3 2 Tu, “ee. (2) Ife =8, we bare T°; R200) = 4k 290° = 59,000,000 6 where ei a ‘The relative ale of change of y with respect to zis when = dylde 9 ]tz=cwehae Weg ‘The relative rate of change of y with respect to zis 2/e when Oty calculates are supplied when dollars i the price per calelatr.y = m?-+ (2) The average rate of change of y ith wapect to m whes th ineeans from 1610 17 is (a == VT 2108s Be i ain nasa ty Ua a acl LT, Thuy, the average sate of change of the supply when the price ierestes from SID to S17 is 39125 caleulatees per $1 incease in rie. 182. THE DERIVATIVE AND DIFFERENTIATION (8) x0) =1-+jov“¥?, The natastantous mt of change oy with respect tom when m it 16 8 wa BIaE. Therefows the inntencou tte of change of W0y when mm ~ 16 in 532125, Hence, te intantmos te of change ofthe apply with eapec tothe prer when the price S16 is ‘212.5 calelatora per €1 increase in prize 62. ‘The remainder theorem of algebra states that if P(e) a polynomial in x, and + is any real nomber, then) {se i» pynomil =) ch tat Pe) = Qfe)e =r) 4 PC), Want i, OLE? Solving the given equation for Q(z), we obtain @(=) = PED Pi ferential at every rel number Thos we maya he Dein 2.18 concie that Bay Q(2) =n PFO) _ pry 8. 1 =ph¢ 0 = An SOO - tp, St O)= a, Ee 64. Ue the defton of devivaive to find) if fe) =e? Be 41 wren fin EFSF. Me aeP— B+ Bo) 43] -GF 4) = gn teas a= S88 tn ar )-9)= 00-8 — Foe fn, LEEAD =I). yy EFERIT- VES = 2m, EEE fam Hy, ERE 3+ vie=3) ‘Bai Viet dae ~ 34 Vie—3) = tim — Se esden Sree ga Sag Are ee 66. Use the definition of a derivative to Sind (5) if f(2) = VSB FL ¥ poyenn YEE (Vie FI— ai JSeFT44) 5, Gz+1)-16 BERS NR eae tts) AE iGsetTw) 7. fla) = JT Teams = (24+ cova)"; Je) = Yo + cos 2 -sin 2} -H2-te08 2}-9/7(—sin 2)* + (2 eos 2) 0/9(-cos 2] “Ho-17 4 +0- a=} Find fe) i 2) = 3 sind A cP. Bocas we wll be taking two dervatives, it pays 19 im i Jie Tea | 8. fle) = ($1 120% £@) = 22 + 1-12 Yet r-VA) ee inet eSB) Aeon, 10, Find 7-3) if 1(2) = (212) Yor > Because ¢ieneqative, thea [21= —2, Therefore fz) = 292) = -2-3°77249; f(a) BI, 3) = 88 as MISCELLANDOUS EXERCISES FOR CHAPTER 2153 ‘71 and 72, « cn isthe dstance of » weight fiom it central pzstion at ( seconds, nd the postive i upward. (@) Plad the velodty and acceleration, (b) Show that the motion isimpie harmon. (6) Pind jude 4, pero p, and feequency f of tre motion. (8) Simulete the motion ox your calculator. (e) Plot Sein gt (a) Gyptin bet (0) Because a=~(Gyrite and —(Ser?? is ive, the motion is simple harmonic (c) A =8 p =2n/be = 18, f =U ce) > The graph isat the right, a tz sin x(t— J) 5) Fecanse (40) is constant, then 2, the measure ofthe scceleation, is onal to ¢, the measure of the displacement. Furthermore, because He) ie negative, thea 9 and + ae oppositely ditected. Thus, the motion simple hermonic. (6) The amplitude is 6, The period is 22/4 =} ard the frequency is 2. (4) To simslate motion, in parametric mode let (0) =2, a(t) =6eos=(4t—). In the window [0,4] x[-8.8}, we let i,t Max] = [0,4 tatep =.05. Pres the (TRACE) key, pros the « key untl the cursor fs at 0, the pres the ey to hearve the motion: 1 and T, » fis the directed dicance ofa patie fa the origin at # second # fee isthe Eo hfsee ts Se acelin, (a) Find the v sed a (0) Show thatthe motion Is Sep hareoni ae the mation on Jour aluntr. roe (8 a) sn fa) () = h/t = On Sts fx) +12 cat fe) a= dof = 6 ca( + Jr) ~B6 sn(2¢—Je)(B) Breanne 9 = Oe and ~9 eget, the mation i simple harmonic = 6 sist = 3-6-1 cond) 3 cos Bt (a) v= 8 apa Because u = ~O4e and ~64 is negative, tbe motion i simple harmonic. =192 cos 8¢ ~Msin Bt, 4 coe 2t cos t con 24008 v= ff = -2 sin tin ha = the sceetation i not propertional tothe displacement, the motion iz not simple harmonic particle & moving in staigh line according wo the equation s = VA BM, where A and B are postive ts, Prove thatthe measure of the acceleration ofthe particle Is Inversely proprticnal te 6? fr any je one given that @=(A-+B0)/2 a ‘iferentiate to find first the velocity and then the acceleration, BHA Dee epg = aya suey? =fp=nfeha sae sasne 9] = ie pia+ ney enn asm) GPL dane) ie (A BOB? (44 8?) =A @ stuting from Ba. (1) into Ee. (2), we obtain Sccaise wear given that A and B are coosants, then we have proved thatthe ease ofthe acderation i “every proportional. #forauy G(x) dolar isthe toil cost of manufacturing = chars and C{z) = 2? +40z +800, [a) O'ts the marginal cort fiction: Cle) = 22-440. (b) C(20) = 2(20) + 40 = 80 16) The numberof deine i the actual cost of manufacturing the twenty-first chairs E42) — CC) = [28}* + 402) +800} — (20) + 4020) +800] = 81 164 TE DERIVATIVE AND DIFFERENTIATION 78 N(x) dolar te revene fom the tl of ays and) = 1h ~ Shs s marginal event tncion | Wie) = 00 42) B'S) (6) the atl retenue fr these afte tert an a Mig) RB) = 008)“ 208) ooo) ye = oes 9. After ¢ werks there are pmy and y predators white » = pyhgt?— hye + 4D and #= 3064 -+ 375. tf ¢=10, 2= 310-104 875 = 805 and SF = M48 = Cog ~1)800 (GERRs— rb 0 = 9078 “Aer 9 wes be prodaorpopulton ie Ineasing at he at of 18 i per wee. 0. The demand eqaton fot pasta andy bar pe +x +207 = SH where 0 canty bts te de cesta wn p tata the pce per fastest pie ofthe candy 340 cent per bor end Sipe are netalng oth tec. one per week, the te of eange ne dean The rate of change of the demand is (1002) = 100088. We sols the given enation for x and diferent ‘with respect to { using the chain rule. = tt 1000 207 {oo — 45) pete , (p41)(-20) (3000-2091) dp 9090 dy a (esi) aH Gaiy a We mulily on each side ty 1000 aud let p= 49 and dp/dt = 0.2. Thus 100 0204.9) = 241. os = 100)-=9A02) = 2004 “The damod i erening by Ln 2 bat th ent ee [BL At thours after 6 7.2%, lel nautical miles be the distance of the frst ship fom tke por, let y nautical mil ‘Be tae tance ofthe oeeend silp fist the por and let = mata! miles be the distance between the ships Hy the law of cosines, Peal ayp—tey cole: dete + 29h y = VBVDe— JEDw at F— Veey)? ‘We ae given Djz=10 ané Dyy= 15. At the instant the second slip has traveled 9 natin miles (262206626) = 240 and y= 20. ence 2210) 204. 2(01)15~ /5(0)20— Y3240)5 _ yy 20}? +(0)7= V(200}00)= When the second ship has raveled 90 nau mies he ships re separting atthe ate o 105 kot £2. From the gure the hypotenuse af aight wali 10 and one Ie is 6 From {he Pythagoras these we find Ue depth of te Gough an, Det bese depth ofthe vote. From dniar tangles we find the width of the waters eface \ [gf] bed: gec0- he Te nen Ue apeldal water suefref Am where ot @-+4)] = Bs <3? andi volune ism whee Sand de/40= 05, e 4 aon 4 1mm = (40+ 20-50.1) = 100 ‘When the water i dept reskin ot at 100 mh 35, The water in the fae forms cone ad the slums of is water increasing al the tate of non tele in be the Zaan of his cone an inthe helt ofthis cone. Then V = er? AY = 8 and with to fad dB shen N=. Now ir mh Hence V= fife = fats Y= ie bP ty? Beh Dee = De dh__64 av wv i a 5 B12 = GD Beets GY =, we obtain l= Eta = gps 80.28 1 Thesurface of the water is rising at th zate of O26in se wen itis 5 in. deep. MISCELLANBOUS EXERCISHS FOR CHAPTER 2155 ‘As the loot car of 3 liz passes under 9 bridge, am aatomebile ease the ‘ridge on s rondway perpendicular to he trick asd 30ft abeve it. The tain tearing atthe rate of 80 fi/eec and the antomebile traveling atthe rate Of 40 face. How fast are the train aud the automobile epatating after 2 sec? “Refer te the figure. The roadway ie in the line AgA, andthe railway track bx fm the line TyT We ate given thet lize AoA, is perpendicular to lie TyT, “The lat car OF the train te point T, when the automobile ie at pain: Ay Afver tase the lst car of the totin is'nt print Ty and the antorobile i at point A;. The other variables are defined ax follows 2 fest is the eitance ofthe automobil from point Ay 2 9 1 fos in the distance of the Ina ear ofthe tain From pint Tat 200 {feet isthe diane from the auto tothe last car ofthe train att s0e [Becouse the train is traveling at 80 f/ec, we are given that y — #01, Becnvte he automobile ie traveling at 40 fifo, we are gives that 2 — 40, Sisce ds/dr represents the vase at wich the automobile ad the train are [mperating, wo ant to fine da/ét whon #9, Hence we must find on equation with variable y, and > Becousesrangle Ay TE; x righ trisagle, we have a Because triangle A,AjTy is ight triangle, we have a ISubsttoting from Ea, (2) into Be (1) and edving for, we obtain 22 + (0) +9? = (40? + (90)2+ (802)? 50001? +-900)1/2 Differentiating with respect tof, we get (0002 + 900)" /*(xe0001) = — S008 ante 000 = Tae Fa de _ 1600 When t ~2 we obtain ~ 1600s a8 a” Ya “After vec the tin and the autombils ar separating st the ate f about $8.2 f/ne AL t sc, let yf be the height of the man's shalow and = f his distance Gom the building. We have by ‘nile Wienges ‘When 2 = 10 and $5 = Be si cssdon o pid atonac a ia econ At days rc the radio of te burn and Aen? inthe are, A= 779 = der = a 1}(-008) =O, When the radius of the burn is 1 cm, its area is decreasiag at about 0.31 cm? /éay. Wo-{gth wice ©) fe fie) Jim, f(z) =@20—24) = 11 so Him f(a) = 11 HG) =n. Became lin, 2) = (85,7 continous 3 (9 10) = ig LO $= ig CXBY tin Cap ty C= B29 ie (4) 8 I =D gy got 28 a eee = tm S242) Bees BecaseJ18) #191) dow wo tt lene Ft ifeeniale at 3. 16 THE DERIVATIVE AND DIFFERENTIATION 8, Given He) = {EIS HES 4 (a) Sine the graph f(b) Dtermine t fis continuous at 4. (c) Devin it Ji diterentable at 4 > (a) The graph fs seen in the gure at the rg Dand lim, f(z) JA). Hence fis continuons at 4 () Becusse fin contineous ay (4) ~ Jim f'(e)~ I ‘Therefore, fn fle) =0. ; (1 .0)= Jim ig, 1-4 = : 400 = in, Hef ; Thentfore, (0) =0. ety mo. pey=eene=fHESO paya{Qr EES G- Because lim se) = 710) =0 and PC} Iet 8 ddeetabl very and Je continuo eveywhed a fe)= {ft $0 HST yy y(a)= tin (art +8) a+b, Lim, #(2)= kim, hh fe>t ans ae Because (1) ets Fin continous at Land so Jip 6) = Ji Je) anda £0) m and F5(0)= jim, A= JO) jig, WE Because f'(1) © Substitating tim, ty = tin, St (1). Therefore 20 = Wwe obtain ~}-+=1, spp T= estore ites Find the values of a, b, and ¢ so that f%(1) exists © Ite f 1, wemay differentiate f a follows ite0, (Fog) =SGe— J) +[}2~x|= fe + [Be =e +} = 26 Thus (f.9)(2) = 2 forall 2 Hence (f 09)() = 2 Therefore f 0990) Give an example of to functions f and g for which fie differentiable at 0), ge not dilferentiable nt Oy and fog is differentiable a 0. Let Fand g be defined by 1( Me=e? Hence, 9/0) docs not exist, and thus 9 is mot diferentable at 0. Moreover, because (2) Mfereatiabe forall «yan ie porticular fis difeentiable at 0. However fog is defined by (FoaXe) = s6l2)) = (2/9 = 28 Thus (fo0)()= $19, There, foe deena at land i panels fog difrentable t BiG el enieeee 2 and gfe) = 22) Then 2, then f ie 158 THE DERIVATIVE AND DIFFERENTIATION 1 Lat fe) =is1 aed g(a) = 22. Then (Fo9}) = fle) = J 0) 0 aod f(a) 1 Sina tecetinble (0 Pie) =20. Thertore, (0) = 0. Thus gi diferent {segie) = 2% Thercice fog i deretabe at. su rer={n BEER ={ 4 HEY ments sate nda ien 20 1) Fite Oi ti, MED eo 0. jin ED=SUC). tn a) af) jefe) auf) pele aa 3) 0, that if > L. (6) fs then continuoas. ah Hepes) nite)-Hteh en =n = flay) my 12,30 ase) 100. tet F and 9 be two furetions whose domains are Tie set ofall real sumberz. Furthermore, suppose that {i oe) = 2f(2) +15 (+6) = fe} -4() for a a ad i £(2)= Prove that o(e) = (2). 2 pean easel wits) 2) (yon = He) jin 22 = a) in EL at (Hypothesis =a in 02) = pte) (eypones Therefore, s(x) = 9(#) 101. A counterexample. f(e)=I24 gle) =I, and =. Here f and gare both liffientible ot 1; (fegis) le lieo jogie tot atirestnble at fe) =] tay) $pLE) PLCS, 0.9) [FONE <0 _ 1) p09 tw se) =[7= GEO) ISS a tae trajnoT Pay see eel indsction that D, "in z)= sins + Ja). if n= weave Dylon 2) eos s ~sin(a dr). There tbe frm tre for n=. We arume the formula i tre form =, thats Din 2) = sin(o+ fe). Then D_tEFHin 2) = Dg[D, Hai =] = Deluna + bin = eas +r) = ane tHe) 426] = snfe 410+ 1 ‘Therefore, the formula bold for n = E41 if hos for =& Heoee it holds for any postive ntege 104, ry =1/(1-22), prove by mathemati induction that fy _ aul oes 103, We wish to prove by mathem: sho trae form = +) ‘Then fa -ney-2) aa Decaste Ba. (1) becomes Ba. (2) fn = 1, then Ba. (1) beds when n=, Roem (i) holds when n= &. That i suppose that y= Bia th—2ey Diteretiting wht epee 3 obtain MISCELLANEOUS EXERCISES FOR CHAPTER 2.159 Feet = Hk 2-1 (2) AP Sabin 0 — 20-4 = o G4 -A= GaN, emiatnsn An 2H ei ) bay is replaced by £+1 in Eq. (1), the rsull is a, (4), Therefore we have shown that whenever Eq. (1) orn abo holds fot nk | THREE BEHAVIOR OF FUNCTIONS AND THEIR GRAPHS, EXTREME FUNCTION VALUES, AND APPROXIMATIONS 8.1 MAXIMUM AND MINIMUM PUNCTION VALUES {LLL Definition "The function Jie said to have a reletiee marimumwafee at «if there existe aa open in ontatning con which fis defined, such that f(e) > f(c) forall z in this interval. 4uL-2 Definition The function fis said to have a relate misimame valve at cif there exists an open inte containing oy 00 which fie defined, such that f(c) < f(z forall = in this interval. If the fasetion f hos either © selaixe maximum or a relative minimum value at hem {eat to have a relies entromem at 6 3.1.3 Theorem If f(z) exiss forall values of z in the open interval (a4), and if J has a zlative extremum cwhere ace <4, then if P(e) exists, /(e)=0. SA1A Definition If ¢ ism number in the domain of the funtion f, and if either #"(2)=0 or #0) dow fxs, then ¢ is called critical member of f ‘We conclude that a necemary (out sot sufiient) conditin for a foxction to have relative exttemum at is fore to bea critical number. 3.15 Definition The function f i said to have an absolute masimum value on on interoat if there Is ruber e inthe intereal such that f(c) > f(z) forall» in the interval. In such o case fe) the absolute masimum valve of J on the interval ‘3:46 Defnition The Lincten f is oaid to Save an abiolute minimum value on an interuat if there is number ¢in Che interval such that f(c)< f(z) forall = in the interval In such 2 case J(e) the aiwolute :isimum value of fon the interval “An adsolse coiremam ofa function f on an interval Tis either an absolute max value or an nbslute misimusn volte of f-an 1 (1 f has an abeclte extremum on T, then “Absolnte extrem rust ovtur ether al a tial number ef fot at an endpoint of Ti Not cloned interval ar 7 i» not eontienous on I, then f may aot have an absohte ext fon 1 To show that f has no absotte maximum on I, we show that for seme ¢ i Giine! ig f(e)= roo Him. fla) =a bat f(e)< 4 fm b where the Timits may be sed. To show that f has no absolute misimum on I, we show that for some c in 1 ei Jim J(=) = 20 or Jim fle) = but fle) >€ on 3.1 Rxtreme- If the fonetion fis consinuoes on the cloved interval [2,6], thes fas an ‘Value ‘Thontem maximum value and an absslute minimum value on [8 "The following steps can be used to Sind the ebsole exirema of f on (00) i continuous om [8] 1 Find each number ¢ in (08) such that #2) =O oF (6) does not exist 2 Fine the function value f(@) foreach number ¢ f step 1. 3 Pind the function valves f(a) ard (2). 4AThe largo of the values from steps 2 an 3 isthe absolute maximum value and smallest of the vals frm steps 2 and 3 isthe abeslote minimum valve of J on ‘loved interval 6) Include in step 1 any poists such as the break points of » defiaition in pieces, where might not exist. I is easer to evaluate f(c) than to prove that J") dose not exist 3.1 MAXIMUM AND MINIMUM FUNCTION VALVES 161 3a 18, (a) plot the function and estimate the critical numbers. (ls) Calculate the eritical numbers. 2472 — Be; f'(e) = 82? + Me —5 = 2 — Ihe +8) ‘and = —5, Thus} and —5 are the critical aumbers of f. lez +1; oz) = 62" — 4216 = 2(52* 228) = 1004.4)(2—2) 4 and >= 2, Thus ~$, 2 ate the critical numbers of 9 (2) = Bal! — a9! = Ge 45(2—2), (0) doce not exist and 0 is in the domain of Thus D and 2 are the exitical numbers of 9. Ty gA/O_ 4,109 domain of f is (~30,+00)e Sie)! 29 > A plot of fis shown at the right. ort +42~3) - Se 3e £'(0) does not exist nnd 0 is in the domain of f, by Definition Disa critical number of J. If f"(2) = 0, then O=(72-4\e +1); } oor 6 FG) =0, #'{1) =D, and because $ and ~1 arein the domain of f, by Definition 21.4, 8 and —1 are critical numbers of f. y= pL = SL ay Neier 14) noe J) dened. The domain of Fis (alee #1 = Sha Se Hd) 1 (24 1)(2e~5) _ 2? 5244~ (25435) 2? pe yg (aor +4) ies aye (oF )=0 when -s?—2249=0)27422-9=0;2=-12. 0 1+ 10 ane ~1— s/f ate the erites! numbers of f that #'(z} is undefined when 2?— 52444 =0. However, the only solutions of this equation are 1 and 4, bers which are not in the domain of f, Therefore, neither 1 nor 4 is a critieal number of f- 2¢=0, c= £$.arenpt in the domain of f. Hence they will not be in the domain of f" but will not be 2 nie, 1") = ME aM OBIE Deh eae 2 nantes (¢) = Gan ayh 7 P@)=O when 258 + 18218 924 9= 0; 2 =H9+ 95). Theretore, 19+ 3%) and (9-3/5) are the critical numbers of f. )= (2-242 + 1 G2) = se — 2 e +1) + 2-2) 4-0) = @- 2+ 1)Gr—1) when £= 2, 2=—1, and ef Thun the eitical numbers of G are 2, =1, and 3. (64+ 2P@-27 © A plot of F is shown at the right. domain of F is (~00, +00). Because (2—r)? = (2—2)¢, F(a) = (2 45)%2—2)? tinting, we have Pq) = (245)°D (2 =2)? 4 (2—2)'D,(e 45) (= +5)(2\(2—2) + (e-2)*(3)(2 +5) fe +5 (e— 2Ce + 5Y2) + (2 -243)) = (+5) =2)52 +4) ») is defined at every real number. If F*(0) =, then either z = —5, ss in in the domain of F, we conelade that —5, 2, and —4 are the critical numbers of Fs $-14, (3) Calculate the critical numbers, Check by ploting (b) f5 and (¢) NDER(f() 2) ) = +113 + 34? + 152. f"(z) = do” + 530? 46824 13 = (2-45) (2 + Mle +3). G)=Owhen 2=—5, 2= —2, and 2 = ~3. Thus ~5, -} 3 are the ertical numbers of f a 7 162. SELLAVIOR OF FUNCTIONS AND THEIR GRAPHS, EXTREME FUNCTION VALUES, AND APPROXINATIONS Me49)—(24 9] =Ae-Nle+ Nie +3) 3. Thus 1y 1-9 ace the eriial mambors of f nf) = f(t) YP — 9-20). 1-2) and J"(2) do not exist and ~2 and 2 ae in the domain J F(@ =O when t =O. Thos ~2, 2, 0 are the critical numbes of j. 12, sw) =(wh-au+ 4)! >The demain off i (~20,-+20)- Plots of f and NDER(/) ate shoven below Sw) = Ku? — 38 44)? (80? — bu) - es (A344 = ten?) wo) -2yP* way ho—as “The factored form of the denominator w"—3u?+4 vas feand by tral and error usiag synthetic divs Because fw) ie nat defined at —1 and at 2, both —1 and 2 are enitical numbers of f. Beeause f"(0) = 0, ie alio a etic umber of J. 4254, 71s detined when J i defied and fe) = 0h (@-2)? P4420; £2y2 Hence the critical numbers of f are 2+ 2/3 and 2— V3. ARE, gy BEEBE DI APES AEST ated te Fd Hite) =0 when

‘hen has an absolute masiinum valve ven rats ® Sabeiting fiom Ba, (9) ate Ba. 2) weobain yr) Oto Sa iee Bai 4 be tee= Soe A vindow of perimeter 32 ila We sont gh Ieee of he seme tude ofthe eetangle ie SLEIEE atone end ofthe 27. pede! touch the corner andthe oppost wal of the & 1 panagry. Soe the gure Ife the mere of the wnt sale betwen L the der and the pumageray, then Seic# fof the ste te ate eeicectecre secre | fe coer and the Let si be the distance of the other end ofthe girder fom the tide of he “cider, We want to id the sioleie maximum velee of Thea KO)=(at See eos = Zeon 400s et] MO) =~ sin 048 ono Saatyite Bet (0 = 0:27 20 fy sin 9 = 90 thas cont = V5, 001 @ m $V. Ohare = Ohana = POV HVE) ~ 805. Phan 10) is cootinuows on a closed interval so the sbylute maxiniom vale os i ‘The panagenay mast beat lent SYS 122 R vide. ‘Let the girder of length «fe touch the corset and two opposite walls, 2 front ‘be comer ofthe 15. corsior and y 7 tom the comer of the 10 feeder, ‘See the figure. We seck the absolute minimum value of « From slr right Single we ave 10 = 5/ey = 0 - Ading he patents meget le as) = Va" 4 205 + JRO T SO, oe) = E00 x eee FES VRE TR a gt (32250) I VEER Jonas sa) Vem , Because (2) £0 2 < YORU SYR sad oe) > 0 if e> BYR, then 6 Iw ss absolste minimum value when 2 = 59/18, Tie ean nay have lengin O/B 100+ /T5Y/TEF BS 1 8. Let V cabic ents be the specie vole ofthe cone. Let rit be the rave ofthe at, fet unite be the IRight, and A square units be the refar ofthe cone. We wort to Sod the valve of bt when A hasan abolate minimum vale Va fry he Vee Am enV FT = VS a (et VHT (0.0), DA = YP Ov HM 4% —aevFB = Bet Nat AME Lay) Because D/A <0 fr < Ge-2V3)" = 15 and DA > Oif rte, A has an abecute minimum valve when tty: Theat wv ‘241 BEMAVIOR OF FUNCTIONS AND THEIR GRAPHS, EXTREME FUNCTION VAIUS, AED APPROXIMATIONS. 28, A right-circular cone ew be inserbed is epere of given radius. Find the ‘aio ofthe allitede to the base adits ofthe cone of argo pomible wate. Referto the figure, Let uit be the radius of the cone Fr uate be the altitude ofthe cone V eabie ute the volume ofthe cone We want to find h/r when V has an abeiute maximum value, We have Veleth a 1 the radius ofthe sphere is a units, thes bythe Pythagorean theorem (he oP yr =a? PS dan— a @ ‘Substitting fr 1? inte Pa, (1), we obtain V a a funtion of Vik) = fr(-i + 2a) ‘We nite that Vs continuous on (2,20). Differentiating. we get VA) =Ja(-ah? + 4h) = —tabth— fe) Because V{h) >0 if O< h if > 45, hlhas an abmlute minimum valle when avi. 30. Let f be the equate of the stance from the point (x,y) of ling Az+ By tC =0. Dillerentatng im -A/B. Then fle) = (= 2)*+ (— 94? hea yy, = (BYAY(2~ =,). Weng the equation ofthe line = Wd, and substituting: Ale—2y) =(BH/A)(e~ 2) =~, Then! Ale—2)) +BY-¥y) = “(As +B +6) Ad =—M = ARES oe The minimum distance i Baw neo Ut = Saige Thee = Ga gir = aaa TM cane BSS pre) 24 2-4/8) -A/8) <2 2878850 w the real ont a pe a tbo minimum, 31, The cross setion of a trough has the shape of an inverted isoscles tangle the lengths ef the equal ape 15 neh, nd the de of the verex ane that wil give the maximum expaciy forthe trough IFA ssuare unite nthe aren of the cron setin, then the capacty Isa maximum when A is» macimum [As Jeb ind where 0 and baze the meanutes of th leng'he of two sides ef the triangle and 8 € (0.8) Ineasire of the angle betwee these sides Because sip hasan abeluie maimmam valve when @—2=, tn hha as abeolute raximam value wien 0 = J. Thos the vertex angle sould have radian measae 3:10 APPROXINATIONS BY NEWTON'S METHOD, THE TANGENT LINE, AND DIFFERENTIALS 245, 3.10 APPROXIMATIONS BY NEWTON'S METHOD, THE TANGENT LINE, AND DIFFERENTIALS Newton's Method Wien using Neon’: method ta solve an sation ofthe form f(=) =O, do the flleming: 1 Make a good guess fr the frst approsimation 2, A oogh seth of the graph off wil belp to obtain a reovoable hoice 2, With the value of yin frm (1), get second approximation 25. Then wie 23 in (1) to we 2 third approsimation 2 and so or untis,,, 2, t0 the requned degsee of accuracy IE {£"(2) exists in an open interval containing the foot apd (2) is ot vero on I (ef, Excite 60) then the number of carve digits i approsimatdy doubled st each iteration. Thus if 2 and Eqq1 a8 to four digits, then 2, i coret co righ digits. Sen) 1 FES a Be sure to switch into radan made ia problem involves trigonometric fanctions. ‘The vale of. 4 should be sioved fa eeinory (ana variable) atthe beginning of te pee, and again when the new value of zis in the diplay a the ead of each itertion. Bach time you need =, just pres tse recal bution. Twas the hepstrokes for cach fteration are the same. Temenier 10 se he muikiplistion Key for implied melipicadoas. Always we parenthees around the numerator ane denominator, and gn each iteration with “= STO" a8 shown tn the Slusons [Nate that une samerator shows be faction value and should progress toward 0. ‘Dire If the function is defined by y = f(z) then the diferetal oy, denoced by ds is given by dyn See and the differential of , denoted by de is pven by a where £ ia the domain ff" aad Os saa abitiaty Iucemeat of 2 Whee de i small then dy is good spprosimation to Ay mere Ay= fleas) fle). That is ye Fle +2) ~ Sle) oF Jehan fle + Me) fe) (he linear appro $2) = flea) +f'eole—%0) Wo Fe), thea whee fe) exis, yn Pee hater ot not «ie an independent vaviable, ence, the rao dy/dz isthe derivative of y with respect to =. That isi y= f(z), them Pe) =Day afk tage ‘Warning: the symbol ds/dz? for the second derivative of y i not the quotient of two aint ote et al st ean aun saa re said in the lta soo angi she emrepnding free ta Te Se cree Sook va=o ) Wale) =e ae MY a(ex) wag du) 246 WENAVION OF FUNCTIONS AND THEIR GRAPES, EXTKEME FUNCTION VALUES, AND APYROXDIATIONS VW deat) = wt “The differential formulae ace particularly weil when doing implicit dfreainion donot med to distinguish the idependent raiabie from the dependent vasiable. ‘se Newon's method uo fad the zeal yt. In I-4 to four plac in 5-10 103 pees. 2 fg) =32— be is art between 4 aud 5; take = 4. 2 425 O.lomer kya 411797 3 AilT97 0.000) Tazz 11794 2. GeS 404120 Let fe) 26 29241; fe) = 182 +9 Since f(-1) = —if and f(0) = 1, f(2) =0 has 2100: between —I and take 7, = 0 1402 wo fos decal paces, (2402, ¢1) = (182248) = STO 2 anil =008s 9.20002 3 ioe “hspet0~C 9 21867 * ta fi) eet 24 ts Fe) tet diy foe F(a), J) 2 hs ot Deine 2a “As tae T1675 foe dca paces Bernt) = STO STi “inert “160 “asa0 % fle)= abet Sela det Because f(Q)=—1 and 1(1)~1, here #9 240 off between Cand |. We take 2, = 0.6 and apply Newt tmthed as shows in the table, where thr numbers are rounded of to ive decimal places. Nole that ‘Sgro to digs and, as predicied,£, aad ager to (atore than) 4 digs To four deszoal places, the O75, DL tanta ,—1) + eet STO 05 04ers 19a sonar oss 051980 Louse oames — onease 2767 076528 © o.0LDG 2.62358 O7s1ee 0.00000 262258 {L10 APPROXIMATIONS BY NEWTON'S METHOD, THE TANGENT LINE, AND DIFPERENTIALS 247 Ba te-8 f(z) =307=4 Os a postive rot betwen 2 and Stake ry plows a — (45, (Se,f—8) = STO 2 209565 0.80516 177996265040 F Besoso oes — 17arsa — Beivas & 261944 0 585 261904 the neptive root. Let fle) = 2-26-47; fe) =te4—2 Af and f(-2) = 5, (2) has a negative roo: between “9 and ~3; take =, f= 4, = 2.258% to fear decimal placer Bi.t (g2—2,47) + Gz,t-2) =STO v2 3 10 23, 292 “erat 235019 4-22012 on sates 225895, 4 29se%6 50x05 12.2000 $ 225826 9rx10-! 13290 225895 A= 102 4520. Let fle) =24~ 108 +5 F(@) = 4210 ‘Sioce (0) =5 and f(1)=—4, f(z) =0 has a postive root between and I: take £ =0 Soy 6 = 2 = 0.900 co four delta ple ae, = (esf-102,+5) + (4e,2=10) = STO 25896 hos =I os 205 00s = as © as06ea0 3 0.5065790.00016 9.48 0306585 the lagest positive rot a1et sf e)aa—2 =1,J(2)= ly and f(z) >0 if z>2 then {has areo between 1 sad 2 and is positive if 2> 2. "Phe caletstions are sown in the sabe where the mumbers sae rowades of to ve igi trot is 1969, rounded tothe nearest thous. Fanle,ta lls, 8) = Ces =10) =870 TC Ty Tee 9555 0078.91 teal 210.0001. 55 1621 22" 424430 — 450; the nepptive root Let f(e) = 2a" 204 4 32-4 f{-2) = 42 and f(-1) = ~2, f(2) =0 has a native root beeen —2 and ~I z= i Tan 2y= 25 1252 to four dreimal places (6,3 —6e,? $242.43) SP aetetess Staseas oaorese Thosesa ra9s0r 112897 0.0N619 18469 —L12817 Tis2ei7 o.001023 182909 “19837 ‘248 BEHAVIOR OF FUNCTIONS AND THEIR GRAPHS, EXTREME FUNCTION VALAIS, AND APPROXIMATIONS. 10. Ay 232-4 Since f(1) = 6 and f0). fsa Srt 24 f(a) = 42) + 801 62— 1B. the positive rest Let f(2) 1, f(2) =D bas a postive root between Y and 2.‘Take 2p ape 1.1000 te four decimal ples, wae etae fn dete, 02 lep' 432,762, —1) =STO 118 “13 (102885 sxenigs 245 mestoer LaTies aime 28 tino “on 5 1.76030 40010"? 19.8520 In Bxercing 11-14, we Newton's method to find the valve ofthe rial te ve decal places ‘= yi by solving the muation #*—3 =0. Let f(z) =# Since f(1} = ~2 and 7() = 1, f(2)= 0 tan pov rot beimeen 1 and 2 fake ny = 2, 1.20208 fo five decizal places. (e,?-3) + (2,) = STO 12. a 4 15, 21m Doms 35 risa 3 132g 000019 Sas 1705 4 Laas 0 Ba 175205 12, V5 by esting te equation +? — 10 = 0 bajar fe)=2 We tie: ae a starting valve and caleaate as shown in the table, We conclude that 0 = 3.16228, to five decimal places a = eal) Ge) S510 i] 31-09 Ss2008 2) Stomwos “ootsess eazsme 3.16278 3| Slows o.oo. <324856 262278 18, 2= 46 by wlving the equation 29-6 =0. Le f(2)= 2-6; 2) Since (1) and £0) ‘O basa root bvtween Lane 2 take 2 "= LEITI2 io Give decimal pees a) s (t= 6) + Gq) = STO 19 2 2 stats 2 Leases 0.162099 1083181726 3 VsTi25 oooiT 9.90 L8iTEa 4 uni 0 po LSI? He 2= YF by volving the equation #97 =0. Lt (2) 29-75 fo) ~3e2. Since f(1) = 6 and f(2) = 1, f(2) =0 ba a rot between I and 2; tke =,—2. a 191208 to Gee decimal plan fae (P= 1) + Ga!) = STO 12 7 iz Tateor 2 siser cost toms L.91294 3 191g Tapio" ioeTs0 Liza 4191293 40eL0-%® Toor 91288 ‘410 AFPROXIMATIONS BY NEWTON'S METHOD, THE TANGENT LINE, AND DIFFERENTIALS M9 15-18, we Newtn's method to fn to 4 places the « eordinate of the frst quadant intersetion. 0.705 0018023 1.681) 73811 o9000T © 1.67353 0.73008 raph of he two equations re shown in Ux plot. Let fe)ale—ane f(2)=} s “the = Covi ofthe Sok ganda pot of leterecion othe raph nthe ooo f tat a betwen 0 and Bose theron appess io be pce than © 1-97, ne ake my pe or clear Sato radian the bie atte node 18955, comet tofoas decimal tam. [— [= — Ge-sin=) = Gems) S70 18 ouoT 09282988551 7 2) 150551 0.00001 star La95H8 3] 1a954s ooo .sigo2 189548 sis, At the point of inenecton, sin ==). Let fle) 0502 aod fe) = 14674, f(a)» O has a rot between fv and Jo ake =, ‘ego 2 = StH Yous decimal lees an ~ (eq! ~sinz,) + Geneon z,) = STO 1 OTs 0.0985 O86 0810500 2 OawHeo9 i491? Tish O.TE8R5 3 Os7i9s5 00m iis ONTera6 4 os7er26 6 Luss os676 con. At the point often, con 2 = Let fe) ==? con gfe) = 22 +n. Bad lla of faetny we abe oy = ri bys 0.82 to four dana places se (egh eon eg) 2 ng tvin sy) = STO 46 Bee se joss alu) “saan ‘ove 2asas2 “saan Reet 2322 “sean 19-24, for the given f (0) Gnd the Hiner apooximation Le at z= ef and kat .9, 092, 1,1.01,and 1. 1) check by plotting: =A sO=t oxo 10 1a Je)~ 22,70 fo) asso 1 10am La ue Tomg— 98 1 Lemme torr? fa=F =! = 099 110 Li fe) = 324, f= 0.6703, 1 10303 13a Taoe97 1 14032100143 oso 1101 Ler «186007 «2 2OneaeS 2ade19 20121902 24.01=201 2+. 2261 BERAVION OF FUNCTIONS AND THEIR GRAPHS, EXTREME FUNCTION VALUES, AND APPROXIMATIONS 2. fe) = 2/24, 10) : 0 0.99 1 ao Li He)=-Y'0)=-4 fe=2/2— 2.460 2006 2 10s 1.5 UG)=2-a(e-1) PHa4n24 24 m=2012 2-012 1962— eas 2 $(1) = 4880365 1'G) ost, L(2)= ust0s0~0.ti4z(~1) 09 0.98 Lo: Ld Teas tis a 0 Sr Ls) -5400-+.08815ata0 +.n08et 0.51030 5400 — 00881 5418 —0801 ‘uozea 548i Toso — 08962 414: (a) = cans, FL) = 0540. He) = 48st +080 0.98 oa 38s — A115 —.0540 34147 — 90540 088147 Sti47 4.00540, 8415 + 0540 = 07805 360 esuest = 0.8955 In Exercises 25-28, a) find dy end Ay forthe vais of = and Az. (0) Stetch the graph india oie) =2e dy = AZ = 2EH05)= 2 dy =25) > ola) =32% fy = ¥Q)\Ae—30)"(05) =6. Ay 25° yar =}-k0)= f= 08 49 = YO Boni & Me)= she dy =slae= 3 Ay =48)—064) = vb-Vi= In be Figur TA ie'he tangent st (45). TC ie parlla t the ans ite length is Ar =1.CA is pan they axis lengths dy » 0.25. The kngth of CB is Ay = 0.296. In Paercig 3 i (0) 0) (6) 9. ~32. (203) =) ~ [20° ~1(2.09] ~~ 6) = 190142 = oo09 (4~3)(0.05) = 008 (6) Aya Hate Marz and Az = 002 then (0.98) ~ (1) = [987 ~ 9-989] —((-1)*@ 3-0) = 1. (e) Ay dy = (0.0086) ~(-0.1) (0) dy =e +82)-f(e) () dy = f(@its = “has - PeL a= ol and Ae= 0.1 then 1) = (09) + 1-11 + 2-08 (a) Ay = flee As) — f(0) = f-08)~. (0) dys /e)B2 = 3A Me = A-1FO1) = 03 (6) Aya = 3.10 APPROXIMATIONS BY NEWTON'S METHOD, TH TANGEN? LINE, AND DIFFE! serch 35-42, fod ds. P= Gat 2e hI dy = 30s? 20 YA ahde de a7? dy = LEM MI) Be dle? 52) _(22-42)SH0)—SelDede) _3{2—24) de (+2) ae > @trap ga PVRT3 * da afeene 9)! ea sy Mees nee 43) eA 1a VF ton Zid Sai s2—sins) (cons cone, Paind-zoe} ay=|so}0,@)+ 2 vfan})-on} 2 feos He fess taste sees fy [2 sanz secte)secie + tan?e(2 sce sone + lan ide =2 tan x sete(ee's + tented “2 eal2e-ce 2s + cot 2o(—Peat 22 exe Ie) = Peace 2 cote cae 22 = 4 oe2e 42 ese 22 {a) let = em be the length ofan edge ofthe eabe ané V en? the vlurae ofthe cue. Then V = 2; dV = Selds. If ~ 15 and de =Az =001, then AV = aV ~ H15)(001) The approximate error in ommpating the volume 673 cn (b) Let A cn? bethe area of a face of the cube Then A= 2; Aw Be dz, I'z = Ib and de = Mz = 0.0, then AAW 4A = {150.01 The approximate error in computing the area of foe ie D3 cin on 13. A metal box in the form ofa cube is to have an inti volume of 1090 cin? ‘The sx sides ave to be made of foetal Hern thick. If the cont ofthe meta to be ied ia $0.0 per cui entineter ue eiferetias find Ue “approrimate cost of the metal to be ued in the mastfactre ofthe bow et oc tthe lth feck nse dg fhe Lat Van? bo he into ome of the cab: Beene sod we are given thatthe inter volume 1080 em, thes iowa 2 rato Becaute the thickss cf the metal is 05 em, then cach ouside eer of the cute ix lem lange than each Inside edge. Thesfore, the volume of the metal © AVem® with 210 aql Az—1. We we d lo sppronimate AV. Thus eV =3e%(B2) =3(10%1 = 300 “Tee volume of the metal i approximate 300em2. Becatz the metal qe 80.29 per eubiccentneter, and 20(500) = 60, the cont of the metal i approximately 860. (AV =119=10"= 287 and the acted cot +8.20(331)=866.21) etx meters be the rains and V mi be the volume ofthe indica tank. The altitude ofthe tank is 10 ‘hen V= Mer! eV = tse de W'r=6 and drm Arz yy then AV na =20R(0hy Pe ‘The opproxinate amount of cating material is Br {strom be the sadius nad V nie the volume of tbe cdi stem, The eg ofthe sem is 2 em, Teen Vis Berl BV — Sor dr Itt i 3, thes AV €¥ = 44)(1) = 165 The approximate nereas in volume ee 282. BERAVIOR OF FUNCTIONS AND THEIR GRAMS, EXTRENE FUNCTION VALWES, AND APPROXIMATIONS 41. A en? the ate of te ciralar buen when the radius em. ‘Then Ameri dA = Dende r= ard de = 02, then MA AA = 2e(1)(—2) 4 Theapprerimate decrease i the aea of the bur is @.49 ex oan 48, A cin Datel ol mpi io hap sch tht If komen Ine rade ad V cable mir is volume, then V= rr? Use the diferent to ind the spproimatsincrase Inthe volume of the whe the aon icreae fn 23 pnt 2.3 f» We lind the vale of ¢V whan ¢ = 22 and Ar 0.1 Thus, vee a =hne? Ar =4e(22)(0.) 608 “Theineete in vale approxmalely 6 49. V ca she some of the spherical ror when the rads i em “Phen V= rd a mterien f= Ls and dr = then AV dV = Af: 8)}02) ‘The appreciate income i the volute of the tanto Pr ex S50. Cseands the prod ofa pendulum when the length is fet, Then 4n2€ f Dividing the dierent age BME Me ot og el ute he the ven, ne ga Df = BE Ea mtg = py = aha “The peadsen sul be lengthen rif = oe S51. Let Re the soar of the elec resistance ofa wire, #be the measure of is diameter, and. be ‘ease of slag Lis estan or some const 3 I 2, AR = = de ~ 0g. ee 2% = 000 hen] S| 4-0 0 =4%. Hens the sponte wot athe este 4% 52 A ceitacor gest pat on tothe 000 cru sig cach of rao $m. pon menting i Sher tht te rao ofeach sgn lem ao eget oad the appeoninate Ieee a pn iat wil be ese > lsc mtb he ai of ech sins A sue tr te total pe hat sat be pint eshte e190 cst aed on ah ey te foul fr he en oa ie me Shae cat A ncn the pit eid od he ob te ts, is he perenne in nt ended By Ea, (1) ne ave Gn Satyr ar tp 22H) Br 2ae ig = Matar Se wath aw 8 and Ar = 0.01, sien |e = 0.01 m, wget 94 20.0) — 90967 = 0.60% Because AA'~ dA, we conclude (ETH i the apprinimate perrent icreas of punt needed. The result sane for any auunber of sigas. S10 VR tesla of gs and P b/s the pee tha, By iy en hat fr comtant GPG a Goat ¥ Toe ith = 200, Vo egg 10, The ain conti ba voles e102 HA Theadibaele PV 2? = P= G- y 55, Boph's law ie PU = Pom ev, To find the scat ponale: ¥ we take aP Divide the diferent by the given to get othe iva bythe given to et P= SAF w= 10 APPROXIMATIONS BY NEWTON'S METHOD, THE TANGENT LINE, AND DIFFERENTIALS 283 A tghty-wound flexible ape of length L feet, fastened atthe top of aa incline that makes an angle # withthe Sorinoutl, allowed fo unwind down the nein. IFT seconde ithe time fer the tape o completely unwind, : ar shen T= hace, Show ta =p, We bre tect @ Taking the tril nec side we wt Wat = Bese 8 cond a8 @ Dividing aqenion (2) by equation () giver TEgr = =Sl/ pe Bo 808 "Glare Simplifying each sie, we get the eeguited rst om 3 an 3, eqns th em san 2 a2 Rania moun wine cy op eee Mra sod oy andor decal nt fina seao La ychetanstn {) ade = tae 42 (he eat Gc ae. Eis tems shah of tie gute ot yrume and ae tt ay arden, me Gee) SL or prt fp) and tae the ons a geting tot ch cpt. To Bi Be pane —fe+ a: Newton's methe wil fl ita sec ate Com Tend ay. tahes, = 2—fe peas (e; mint e 34). Then ty 45, = 20088 Tofino take x = 2¢— fr feo A5 (ei be 51) The 2, = #509 = 4910. at = (tanz, +29) + (tan'e, +2) = STO 2 1.99180 -0.2492 7.0807 202629 5 200629 001515 61212875 4 20875 0.0006 | 6.1161 202876 sey (tsey bey) 2 (tantey 2) = S10 3 491285 o.0e7i E2218 a48isi8 4 40218 0 geno 49nni8 fa =~2, find 0, and 9g to four decimal places [We wih Uo Gnd le frat two positive rots ofan £22 =0. If we sketch the graph of y = tan z and y= 2 lo the same ages, we see that there ate points of fatersetion with positive x coordioatesdlightly less than BIST aud few ALTl. However, cate the tageat function i usboused ear these rot, values of 2 Jeite dose to the root do 20 lead tothe root, and those whieh do lad tothe rot require many Heraons: tie tan 2 sin 2/eon 2, we multiply the given equation by ex to got the equivalent eqeation ing 2pm r=0 rakiaceie ence er Sets Li f(@)=1ns~2e core TO a saan We put ‘out calesator ito radian mole. To fad ay, we le! 2y=14 and eondude from Table 2 that 111656 t four decimal places. To find a, we lt = =4.7 snd se frome Table b that = 4.0042 to four eal places. Tables ‘ive iy heat conduction problans. The potive rots a] _=, mz, —25, m5) + Gasin sons) = STO 7] 1 Tsuis 2.5800 1200879 2) \aumi2 ——a.6se20 Lt 1168000 3] Lisa oom T5al6 11esse3 4] ress61 0.000008 74800 1168861 254, RERAVION OF FUNCTIONS AND THEIR GRAPAS, EXTREME FUNCTIOX VALUES, AND APPROXIMA Tabeb a] aga (ene, We, core) © (Oh Fy = ee Ry) = STO ay « ~omaTe 08689 ase 2) dessa 0.015078 © 905825 seni a] sewz7 0.000006 8.01665 seni In Beers 58 and 60, approximate + to fv pave by using Newon's method to solve the equation, fue -Ln fe) tn f@)=o Ten) = mubcspnarrid ejeepee sans in onl Peete an 1a ois sim PEG ue 60, cos 24120. Le f(a) seen th: f"(2) = sin =. We take 3.1 38. starting value and calculite as shown ia table a Because f'(e) alo hes a 20 omergence i Very sw and it wl tae very many eres We start again with /(2) =a 2, f"(@) ‘ind calle as sow in table b, We onclade that = = £.14159265,corect to eight decimal places Taben TF Geng t) sinty 257 ie TOOK 14n0 8075 Soros 2.5010 “atueia —S.106z68 oss G2exi0-t axssi7 zane samme Lsexi0* “orresa 3.32169 ‘Tabe b 2 aan ees, =510 Ts TalelO —280085 gOS 2 [aaaasiose a4 19-4 90008055 3.14150065 23 [saaisaass 2.88% 10"% 1.900000 2.18150265 Macelloneoes Beers or Chapter $ [Brees 110, orth given inter ) seh the graph and (D) End any abs acne 1 Sle) = SETHE [btw Se) = oe “Thee ace no etc aumbecs of f. J'(2).> Boral zn (840). “Thefore fi ereading 0 [-6.40): Beate f(-8) He abvolee minimum vale of {3 U occurring when # = 5 Sore ‘That is 0 abosute maximum value. Set (2) = 0: 20. The eitcal number of Fe. Because (2) > 0if e0, (0) is a abselute maximum value, Tinnot aeleeed interval and f does net have A abeslue minimum value on Ms ELLANKOUS EXERCISES FOR CHAPTER 8 255, 3 f(a) =|9-24 ee (2a) =k: fe) =sanls— K-20), 2 4 33 $(0,=0, and £"(-8) and J"(8) do not exist. The ei Aumbers of f ace 0 and 3. fis cotinuoas oo 1 so absolute fsema cecur at aa end pelat of erica! uatber. $2) =5, $0) =9. (8) =0, $14) = ‘The absoste mnimam valve i £3) theabwolste maximum vale 0) O ane Me) nl9- Flee t-1 5] 21 Me) sane Aaah ag 49 0) = 04nd (8) aad (9) do ot ext. The eiieal ‘numbers aff are @ and 3. / continnon on Iso abeoite fextnema cecur at an end pent or ciel samber. HN) =8, $(0) =9, f(3) = 0, (5) = 16 “The abeohte minim alii 0) =0 a54 ‘Reabecne matin vale bf) 18 fel==2y 104) Because inf) Ean nS sot sehen ont Beese ig. fle) =o) doce no: have an absolut ininsm vaio L Soak re)= Because imp fis) =a oF {doesnot have an absolate maximum valveon 1 Because [(2) >0,f(e) i increasing on J, Ths the abslute frivienern value one at the left endpeits (1) =, fia) =2en ts, 26 feb Set (2) =0: ew 32 = 05 ssunbers ae —Jr and J. fs costae on tw ansite exten ocur aon eid poit or a eral name, FH) 9, 04) = 2, 10) 8 1 “The abolete minimus vale f(s the sbolte maximum vale i fx) f(e)~ tows 2 € Je] =h fe) = $1) = Bein de, Set #2) ‘Fiscontinuouson Io abslate extrem occu at end pains or 2 crieal mumbe 40)=4, 109) = 0. Hs) <4, Fe) 0 1a @fonal (2) =Beon 32 he sbnolte minimum valor is f 286 BEHAVIOR OF FUNCTIONS AND TIER GRAPHS, EXTREME FUNCTION VALUES, AND APPROXIMATIONS Pett f-282<1, oon pyal? Hoeees! fe isis hea re eS Se) = fim (2r+9)=5: fa f(2)= lin (+4) =8 and so fi costinucas on. (2) is never gad ["(1) may not oxi J-2) 2-2) 43H LO) =5, FO) = eee Le save smcupene ud qe epi wpa ftrcs HEE re (BRIG) = me 2)=05 i, 0) = Yin (eH 8 nor if tez<3 5 egeres and so fs entioueus on. f"(0)=8 and /"(8) may wot exit. fl-9)=9-(-8)? =0, £10) =3-0= 8, £3)=0, £6) =56)— = 10 ‘Tae absolute minimum valve is f(~8) = 18) 6) Ue abate masimen val te Pan 14 nd gaily ag sll be alg xsan ofthe nc on atl I Pee Fal er evar se fey noeeeo.s2 3 Ye fo omen oo inf {souls gana prota pl x0 (a) 1=[-2,3}. The eritical number is 6. f(~2) = 4, £6] = 0, £08) = 9 Se) ‘the abeolute maximum value is f(3) = 9. ber i F(A) ~ 100, 4/6) = 0, £(2) = 4 4/8) = 0 and the alnolute maximum value is f(~4) = 100. if — He = (2-38). Set 16) Skellie: cetera ee oe econ 12 fie) = 9-945 1 1s coutnuons on any tai 12 The eel aumber ‘The able minimum velue 5 (50) Te lBih Thee bro ciel number f(-2) =-68. fa) = 8 ‘he abu inimam alee {-2) = 63 and the abet masimum vale i 1B. ffs) sin 2 bcos t= [-Iut f(a) coe ean = Set J) =O tans = 2 =e 79 Fie continous om {n almolteettep capa a ed pot ora steal nanber in(-1) tenk—1)~ =O) aE Ym LA, (1) an cost = 198 a and fe abe MA. fle) = Deon 4251 =[-1.3]. Pla) = 1 Bain a, Set fe) =a 2 =}, [oe ed meaning i Nortel) = O81, fs) = Vat ie 2.286, fr) bbe m B86, f(3) =2 000343 5910 FR GOS STN tahoe malas ao fe) = Vat ta Eni 5a 16 oth Spt of Rl’ hwo ink he one, Chee ying Peta 4, E(B 2) = 38 22-4 ap = (a2)? mal-9) 44 = 05 J) = 0)? C0) +: eo (Pe piece nb wanton os esto Boies they baw ee scim(oa rh FO)" Se vig Soares Cah, VID ceo rsaximm value i fe) = Vi jrw2e ins 20 the yt of Roll thea pas vB) 16. fin)=2ein ai oda) ‘value of is pavald to the secant line that contains the endpoints. (0) J i » polyaonsial function, i i continanus and difereatble everywhere, in particular on [2d] snd (Gb) Focther "a polynomial fnction for whieh the above js also true, Beeate f(a) totes old, and so there ie a number e in (2b) such teat fe) =0. Then Ja) =s"(e) ‘number in (ae) asd a mamibre a (sock that F(C) =1'0)~0. Mence, by Roles theorem, there £8) =0.an6 [M6] = OM) 20h Se) atm 7) 1 a £9 = 7) =F") MOG A= Se Fatnin pt () where 2 le’ theres dow not. apply to f(e)=|22—A1~6 on [= lcaaee ete che 29 and 24 nd why the mean vale them fi atch the paph aod the mena Hine fw=(ie tes F-0) nig 29) 22 nd 0) cnt, Tas nt aeenatte on (0,357 258 BEHAVIOR OF FUNCTIONS AND THEIR GRAPHS, EXTREME FUNCTION VALAES, AND APPROXBIATIONS In Bxercbes 25-32, (a) plt the graph; dete (b) the slave exteema off, () the value of «at whi feces the intavele‘on whieh. i (2) tneeasing and (@) ctereing. a Extiies 37-1, locate the po {ction and rhere he grah inconeveprard tnd downwan. Cote anally Band $1 f(e)= P4324 p fa)= ae 462 3ele=2h $0) Sec fla) =O: 2 =O, 2= 2 Set fa) =O: Hie) | Feil f'4e)| Fis sap fess a =F =] = | ereaing ——— concave downed = fa fo ‘ative maximum coneave downward tered * | = | decreasing concave dowoward zal a | 0 | étereaing point of infection tesco = | + | decreasing concave upmard eo | 4 | 0 | 3 | slative minimam concave upward dee 4 | > |inereaing concave upward and 58 flo) P42 42-5 + Powae saes l= Gr Diet Ma Get Set P(e) = 6: “Sa fC) = tee = ia aT Le ales Se iste Tor SE Samer soe pee = | oe ie ay @ [eee om etintcten set ie aco | |S eican ak oo ict Ss 2 and 38. f(2) = (2-3) 4 1 fe) = He 3)", Hla) Ke -7? St Pe) =O: 2=3. Je) ee 0.78) doen, fe) | £2) Fed f infra _rcaph infbas a ae Se ae ae aera [els acy pinches ie See, «| See 28 and 40. f(2) = (2 +2) > Thar av no arapols. The graph i ymmtric about he Ive = == 2. Fe) =o ee) fen Because f(2) <0 if 2<—2and [(z) >0if > =2 then ft decreasing (Can 2h neseasing on [-2.r00) and bos aleve minimum of f(-2) = 0 | Bt £50, Beene #¥(2)> 0if = 7 ~2, the graph isconcave upward fr all = Sd sere ae a0 poids of inflection. The plot appears atthe right. er Mand A. fe) = 2m tenn 2 (fos) ec in fe) Yom ie) = 20 he ine weer Splat Bouin fino ad etn the raph i mmente Tialmopet to oe oon tea a= = mated stan Decne J) UT € (Jefe) and sf Orth fm eros 09) Pilihow we ac ecapvettetn Boome M9031 fr cee Oand PUNTaia See das he gephisewsne spear ide cae, ce wenard ft e Dfe)0itz>q Ma eOite ce fel e0ie>- [Mel tite ce Ta)coitere Maycoiece sp) > 0s > 1) Se) =G HE) =O. FE) <0 2 Vif aces Me) e. (©) 2%) dow not ena: Ma) >0 it <0, He) > O12 > (6) Me) G Jia) ike e (@) in Pe) 5% lim, Me) = +00 Me) > Cite ces Me) e Reena 46,8) een 46(5) Brerce 4640) Esecioe (6(8) ‘2 BENAVIOR OF FUNCTIONS AND THEIR GRAPHS, EXTREME FUNCTION VALUES, AND APPROXIMATIONS a Exercises 47 and 48, sketch past of the graph of the continuews function f 00 an inter toch cea eating the conditions Ako dram a segmen: ofthe tangent live at cach point i there is ong, Wh (a) Meats PO) = He Pe) does not ait; Fld) =0; fz)

0 bez ca Le) OW) P@=6 (e)=0 =H =0: Se) aczck Me) c0tber ce f"e)>0if 2 >d. xercie 47(a) eres 4700) Exercise (a) ec 48(0) $8 (9) 0) = FO) =U FO) = Fe) = 0 MC) = 05 LA) mAs H(A) =O 0) <0 if bs MG) Wscece Ma) 0itz>a Ge) Po dow wot exits PO) =, Se) = HC) =O MO) WO fle) 20H 2 ca: Je) >0 ihece: Pacoitere In Exercises 49-82, determine fom the figure, the graph ofthe derivative ofa fanetion J continvous oo Re following iniormasion and incorporate i ato» table asin §.6; the intervals on which fi increasing evtdnive exirenar incrvab of encave upward and downward and abscsias of pots of iafletion, fpaph off i tbe only zeros ave thow sated. IG) dows not exes (2) <0 ‘9 for Faerie 50 for Bxeseee 31 fox Beoise 2 9. me EPC tao, 1 7 a 1G) Pe) fishes a flan zea 8 18 ee Se = lathe main conve dowmnvaed = Gereming Som aerered 0 Geereing tof ition + Semteg os 7 seainary oat of intetea i, aoe dart P(e) fiefaae » F —deerasing 3} feative misimum concave woward + concave opward 0 poitt of infection =a Concave doward [lative maximum concave downward Soerasing sonrave downward 0 decreasing pois of inflection MISCELLANEOUS BXERCISES FOR CHAPTER 3 263, ered ~ + decresing 2 Fentress 7 24 + 4 ineresing I. Teron ef fate 0 and 3. Le) Fe) Lie in toate tonne ra cen daw Gas lal Haeent ine pot actin teren Coucave dowawacd 7 . a2 dine, dae ristnemusiman —poyeintaflntcion 22 on oe eee ‘outave spare fr Paso off 1. £" for Fxercige 51 Lee) fe) fifo feats r= = eran eoncave upwar pect... Giant mumregias CuUUEEg, “tence Grerening concave downward a= dine dne ‘ative minimum verti anpntive eneif ae 2 Seas concave desraward . eel 0 = relative maimum — Goneave dowawned ot 7 ecco fconeave downward versace 83-56, the graph 1 for Buerese 53 1" for Eencine 58 Lisfhas Siationay increasing increasing lncrening reative maximum decreasing LG) £0) F sad sega of the ‘the figure. Make a table asin the previous xorcies SMe) "e) finftns » raph iojhas a | Fe + ~ —tnereatne tonesve daiwa Fnfetionaltangeats sppeat and sketch pone grap of concave dowvacd concave uprard pola of infection concave donmant fonenve dovaward concave dowawant point of infection foneate dowaerard ‘ements domiward ‘oneave downward Y for Buctlse 54 2H. BRKAVIOR OF FUNCTIONS AND THEIR GRAPHS, EXTREME FUNCTION VALUES, AND APPROXIMATIONS relative maximum ol aa infection point Aeereaing concave uprard z Teltive minimum concave apeard Increasing, fonenve upeard fl fox Bxerise 55 for Rrerese 55 Tor aencse 36 os. Le) fie) Liafhne raph ifhon ceo ee ae © ative minimum Goneave upmard Sier ft be? 448236 viel 0.29 Vid = 0.14 ita St} — a Taian coecepran ive raximam concave dowavard oo ears pnt tition eet inn. eaeccinined Ps ineasing conane opal 8. $0) = 8YE— a fle) = 22 — 1; F(a) = FP. f=) is od graph ie gmmetial about the ofign, Set f"(2) Be ee te TL mane ore PO) oat 0) dot it Hert fa) f hikes enh hans FW — [oe | ceasing concave wpa Fi SBME |avs]e | $. | Siti thatmen Socec poe j Gao! "| | [Bees camera } teeeiva| | |- | meeeing Sowadererad poy oo ee 4y2 |o = | relative maximum concave downward 2>2yt - - decreasing. concave downward mm. se) = 221”? $4 2h Pela HPP ae Me) spe) = 1.0) a8d J") dasa ei Lie) fe) p/h _ gu ftas a Gd a a a |e: [paki aan sneer Hale |S | eeaing ” Sone wand 0 Jase | dar. | vera tnget put often ere ¢ IRemaing " Eoeave downward ST h/t ‘neraing pinto nection 4 at + [ft [ES Sear 1) = Asin enh Lt et ae of nisin ha issn int i AS eon = VATSBTGa ken tesa) ‘hen fe) = VATE lake oh = VAT+B sin(ke+ 6). Bratt the abolte anit vale of the sn function i 1, the abatate smeximum value of se) = VAPEBE 7. Ife) =129 sb, derne asad 40 atte ph of f ll lave apo of eto a (210) Pat rey Sar be: P(e) = Gaz 2b ‘Because the graph of f has point ofinflecion at the pat where > ~ 2, then F@}=0; | Wee D=0 @ xf ‘Because the graph contains the point (2,16), then {G)=1G Seed — 10 @ vet ‘Solving gs. (1) and (2) simultaeousy, we obtain ¢=—t and =6. Thus tS fa)= 614 12= 622) Because f"(2) changes sigs at = ‘hen (216) is a point of inflection. MISCELLANEOUS EXERCISES FOR CHAPTER 3 267 Ma) = 02 + bit 40x; S'2)= See? 4202-46; He) = bas +h ‘The graph of fis to Lave a poit of infection at (1,1), Thus £0) atbpen-t 62425 [Because th slope of the inflectional tangent at (2, 3a42042=-3 Solving equations (1) (2), and (5) simultaneously, me obtain «= 2, 11 Je) = 54 prove that the gap ff hae and 0) =o ne of ifthe eintae: Chak hy petting ote = Dat 41)? (a2 — 224 19-2127 4 108) (P41) (es Mee) 2? ae gt PO ae eae TO ear HiSe Det = (CoP Hh) tet se) ae tet a1) apa Sob ["(a) =O: 44041 Vin 2am em YEn Re 35> 1 eat get Vier ivi Dad ati Vint tev ayy ars CR a sh mmge MOO=Wav) WS tts vi _Ye- vi) a iat) Ti=3) 7 =v Beem dap fhe ent are qs te pnts oan ae cons eee, eee poeta neee senna Ue Stre= Hsere={2" 58 10) = babe 0) S6akd (m0 2 10) Sige es fn bens Phan arsatve nds vale eP Fea Ben yee uprote ht nbn iter gent th I. Then 819 ad thu (0) ex, Meee he gah off Ile re es ea Totnamey tote nok ce eo peak eae Integer, then 2""?.<@ if 2 <0. Therefore f[z) <0 if <0 and f"x) >0 if z> 0 and so the graph of f has Set ofntrion a te ig fos ees lit 0 om rm beg: Dense op. piv tr f Seti Eutbemin ogo md'n nin) S0 teats nl b's ot pe en @eeO ne Thee Phe0es Ci and fous tandee [hea rnivemiinum vend r=. nz i ten fa) saath ap of fia sits Be Thentar th wa a fhm wp inten 1 thong and ohn edand nce f bas ove tee a OF boven seca 7 acon by ahr he ae oho npr) Maclean aes at Cope (©) eee 10a frm gone fh ag a’ 4 Te se sd the dg uu ope Caer Vey at sere Us (lacoste doje ctebeeut gain Oech Vice me oer at (Ge? ~322 +63). Set V2) =0: 2 = K1GL Yar). Only $16 - V6?) 52.600 € [0.7] Wo) 0, Vensan)= 220. VG) =e. Thea twas nme a ia on 2m {0 Beri tot Anon tx avng sg hue eo hane alm of 190 The ree rn fhe bon it we Se) = 2+ 2809, 0,9) = 20H = 00) Becanse {2} < if 0< 2 <20 and S'2) > Dif 220, then S ha an absolve nr Yale whe 70 {88-EHAVIOR OF FUNCTIONS AND THEIR GRAPHS, EXTREME FUNCTION VALUES, AND APPROXIMATIONS Th (a) Been 16, A sign with margin of at the op andl bottom and 2m a the side iso conta 80 of print. The ftal cen of the ign ie A(z)? hen the with of the printed gion in whee No) = 1248542 230.) =8-20 = HEB) pease (a) e080 <5 and Ae) 20 25, then Ant an absolute minimum vale whea’x = 5. Te smallest sigh 9 m wide and 18m Jog. () Exerse 108 Tee growth sate 7 Sah i ot proportional to the sumer of Bh ad the mom 10.000 of capaci fle) = AeUOMO—2) = KLOWEE =), 0 2 <10 000. Je) = 1000022, $(@) > Oi x € 8000 and (2) <0 if x > 50 then f bas an absolitemeximi Yalie when = = 8000. 7. Profit onan itm i 3200 if not more than $00 ae produced ech wok ad decreas 8.20 pr item fe each ive ove 800, When = items are old he pres f(z) das, where J(2) = {200F_ |, HOSE 0i0.¢ 2 <900 and fe) < iF > O00 then J hae fa abeclte maximum vals when # = 900 19, Find the dimeatons of an open box, bang ata bse so» vlan of in tha ean be constr the ent ammount of tateral > Let tig be the length of side ofthe tae no tron Fn! ‘Tan the it of the ex i An AG) i i the amout of water needed to consuct he bor, Ae)=2 sare P88 0cn aa) =e HP Because Ai(2)<0 if 0-<2-< YI and A)>0.f => YOE then A hasan sbolte minimum value = URE Thevtore, the dimensions ofthe maviel box are Vk n. by Yak ie. by Hk, 41, Lat bethe point cn the bank sates A. Soe he Siu. ff) km is Soca of piping hen the porepog sation srk from ©, chan : (3) =|P|+\PHl= Vez I+ y/0— 3+ WF x (0,2 O-TAE Tmcatat serene PEE VO, 210 c-aPat Lola gle 7 (e-snd 20— are positive); 300 - 190 = 10; 900 = 54; = 32 is Ue crial number, {F iscontiauous on To alslate extrema oceur at an endpoint ora eitiea! number. 4 = 16-4 10/5 37.26, (12) = 96D 4 Tot = SefAD + 2 AL me VAL 32.02, (20) = 38 ‘The least amoent of piping i reguited whea the pumping station is 12 ken fom C. {8 IFC) isthe total cost of x chats, then C(2) (Sou 2)z = sone — 2, 2 > 300 ce) 08,900 the abo ‘macirmur value of T. The largest posible wansaction ¢$108200 ‘8. units are demanded when p dollars is the pice pec unt and x*-4p = $205 2tz dollars is the cost Drofucing 2 units. The profit is P(e) dollars, wher P(2) = 2p~202 = 2(20—r4) 202 =3002— 2 £20. P(e) = AN) — Se = $100 24), Becanse Pz) >0 if =< IN and P(z) >0 if => 10 then P(IO)= 2000 ie sbiolate maximum salve of, (LA coved tn oan Maving = volun of 7 iin ob i the form os righ crear clindan I tbe cicstar fan botum are cut ftom squat pecs of tis, find the radius and leight of the can if the lest amoust of 1a tobe wed in itr eoamaactre, Include the tn Ut ie wasted when oblaning the top and bottom. > Letrin. and Ain, be the mdius and height ofthe can, Because the volume of he ean is 2Tin? thea ) 4 @ ‘The squates sede to me, the top and bntoen of the ca ate 2r in Jou gu each side. Theteor, the tot ‘ares of oth squares Sin? "The lateral ea ofthe canis 2evh ie I'S in’ isthe total aren, then MISCELLANEOUS EXERCISES POR CHAPTER 3 269 S24 20h @ Subntiina fom Ba, (1 ato Ba. (2), we bin Sir) = 842m (2) = sro s6r? « sie P=) (= tera = OD Because Sr) <0 if 00 if +>}, thea § has an abeolule minimum valve when =} Br sinning r=} io Ba (1) obnin b= ted Bini then a ttl a CO tha cn Wen plan te ie wt 1 wa dead ad +? tp = VER R(e) = (1002p — 1002V96— 7, x €[,6} «1. R(2) = 100/96 — 7 + 1902 - == sie Se Rs) = 0: 3000-2002 = 6 =? = 1%, 2= 32 ‘Yi is contison on Toe solu cou ocr Or an ado or acral nan. "R(0) = 0, RBA] = 1800, ROG) = 6 ‘Tha Rhus abisute maimem value wien z= 3V2. The maximum total revente is $1800, 12. Therfore, the cals is $n. and the ‘The grorth nie fe jlly propatiosl tothe number of infociver and the number 11,060 —= of ‘susceptibles; {(x) = k2(11,000 - =) =2(11,002-2*), 0-< r¢ 11,000. f(z) = 11,000-22. Because f(z) >0 {f2-2 Sao aa /(@) C012 > So then | nos ta seat Sanu ale when #3500 “The growth rte fis jely proportional othe number = ef inhabitants and the moun 3,00)~ 2 of rom He e002) Hanis, 0 0 ec 100 ad Pie) celts i000 hen Fas a aula imum sue when 2 = 300, Find the sore dance frm the mnt (4) oa pinto th are 3? teewrs dander ie Fe Fe inanca'» a wn oun sat, Qle,y) 00 the curve 2? — 7? = 16 ie gwen by ToB oF a Bienes inn the coe, theo? = 16,47, Sitting ina Ba (1), ov ot Bee a + (ya) = ty? fy +32 oF) eaate AQ)cO My 0 My >2, hn 42) > 24 aay mini ve Al, wen y=2, P= 1642 = 20, B fs. Hence, the shortest distance is \/24 = 2/6 and the points on the curve Bh lome to Pare (L203) y= 16, wad find the point on 8 the aguare ofthe dstare fram the pot P(0,4) tothe point (2) dallas tthe profs when = rade are old where le) = The ~(e? + 25e + 100) = —z? + 502 — 100, 2 > GMa) = ~22 4.9 = 225-2) Pils)» Oi 226 P(e) <0 fe > 25. eae P() haven abmalute mannan value f= ‘2B radios shoud be produced day. PQS) = (25) + 50(25)~ 100 = 525. The profit i $525 ‘Afler t sec the panicle are at (20) and (0p) where == t2—% and y—C—2, and lt F(Q) em be the ince between them. FO = E208 + FW 10.) = ad A441 92a) maha —4P 4)" peauee P(Q'S0 i O0 if t>H F hae an aeclity minimum vane wien TPB Sai, far tdaty dhe atlas ‘he felocy ofthe veral partie fs 2) —3. Hencl the datance tween te partis fant sRcr Jace, and the vloy ofthe bent particle cafe td the wlecy eth ete parle co Set Exercise 9 with A= and w = fe L = ((B)*°-+ 67) = (14-4) = (8)? =HB m. 270. mexavion oF FUNCTIONS AND THER GRAPHS, EXTREME FUNCTION VALUES, AND APPROXIMATIONS {2 Aladder isto reach over» fence h meters High toa wall w meters behind the fesee. Find the length of he shortest Iadde That may be mse. Ler L peters be the length of the ladder, Sethe figure at the igh. Prom sar lagen we got f= Brew BE Then [erent lop erm emo hw) a) Genin) (es eg) ey oat dg) 4 HW, Las on shee minim vale Po AB When t =H 7%2, te minut leaps U2) — eof OPIS = PROMEEC EE es 49, Lit the radio of he ender bern and he Bight Vi the volume, From sna igh agen BERS near V—erth wrt 20) = 80? 24), rc 6 CA) = wr ~ 67) = br Became Vi(q)>0 if O0 if r>§ then VQ) ==-GO-29)= HF & the onli wae oY. 4. A tet isto bein the shape of ane, Find the aslo of the mean ofthe rad te the mente of ahitude for a et of gv oleme to repue the lest aia tp We want to sinks the Intel sae area of the cone. Let r unite be the ada of he cone, A sia be Dd of com gage is helt sr at of oe, We ae te ema hemeee Dea A? Then Poa + 8) tense A lari nae © i minima, we wat te iad 1/8 wien has a minima. FY cbc iste olan oft come then Veprh Pall Subestting rom 8, (2 ito B.()s merge = fa 20) = AML A) avreten) v0 Because 2!(h) <0 if 0-0 if h > YOV]s, thea + and A have an absolute mini ‘value when & = {/6V/e. Prom Eq. (2) we have Vie i= (SS Ae the nina, 1 = OV) and :. a> Vreviay dys The ent regi the Ie eer f he aio ofthe adit te altitude }V2 4. Let Rem be the caus ad Hes be the lita ofthe ight-ialar cone. From similar ingen = HE. If V(ft) em? is the voleme of the cone, then ae ieee ee SHAH)? 20H —b) _», 2 #38) ee itp > ve) = fo EEA) = ecae Vl) <0 He 23h and VQ) > {FH dh, Vas an abel imam valve when H = us) MISCELLANBOUS EXERCISES FOR CHAPTER 3 771 (Ove of the seite angles cf a tangles mse fx and the opposite side has length 10 in. Prove that the fra eat when the Giang is owelo. i aal bbe the cartes ofthe ther Un sides. By che aw of ese, a4 19? 229 conde = 100 Vind 0) Diterenating ply wi repel o 8 on bth sides af (2), we get Iasi = Vibe b4 Tam (Jib 20) wee ® ‘We ido mas the meane often where 4G) favan je = » Diferentistng (9) with rapeet to 6 sad eubtitating fom (2), we pet to cHonay yo oVB#2 20-2) Ma = Yo49) “I-28 = iar se A(o)>Oifa-cband A> 0 > hen Ahsan ate maxim vale won = ha atl ofadge ieptohe raha “Tio cos sn FO) ~ ality Pe lnel=h re = Re Se PC) Usk cost = in fan = Latha sind = pt os b= eb P continuous on I abeolte extrema our at an endpoint or ail number FQ) = 1004, Fag ena = HAH, Fgs) = 200 esi Pease 0<1-< 1, seat Mie 0 = La. z in. be the length of two opposite sides of the tretang (@) he aco is 81 in? The the length of the other Seo sisi 8I/xIFP in. i the perimeter ofthe tangle then Pla) = = B1/z, = >0. Pi Fea) Besse P(e) <0 0 <2.<9 and Pla)>0 if 2 ¢9 then A hae ax absoite sninimim vake when = = (8) The perimeter 36, Then the length of the other two sides is (18~ za, If A(z) in? isthe ara ofthe fecangl thee A(z) = (18-2): ~ I8e=24 0 2-< Ih AG) = 1822 Beaune A/(2)>Ditz< and A(z)> ifs >9 then A hasan absolute maximum value when = 9 Lat zm be the length af one piece of wie, Then (20~ 2) em i the length of the other pce of AG) emt is tbe combined aren ef the to squares thes AG) =e)" GG0~ a= pe fet 25, 0c oe 2 Ae) (2-10) Because A'(e) <0 if 2< 10 ad A(s)>0 if 2> 0, A has an absolute misimum value when z= 10. ‘Therefor, the total area of the two auases will be leet if each pice of wine has length 10 em; that ithe wre ahold be cut is bal ‘A piece of wise 80a long is bat to form a rectangle Find the dimensions ofthe rectangle so ati axes is ‘sarge powible Tet sem be the length of the rectangle andy em be the width ofthe rectangle. ICA cr? is the area ofthe rectangle, thee A-~ sp, and we want to find 70d 9 = that the value of A isan absolute maxenu, Becasse there i 80 em of wits, then 2ee2y= Ble ym abe 0 Subatiating fiom Bg, (2) into A= ay, we get A aa fection of x. Thus ‘Ma)=2(4-2)=40e—25 Az) =40—22 = 420-2) Beene AY(z)>0 i ¢<20 and A(z) 0, then A(20)= 400 jn an absolute maxisnam value. IF 12220, then » = 20. Hea if the ares ofthe rectangle isa large posible, i isa square 20 em by 20 xn, 272 BRHAVIOR OF FUNCTIONS AND THER GRAPHS, EXTREME FUNCTION VALUES, AND APPROXIMATIONS 101, Solving the demand equation for p, and replacing p by P(2), we ebtain (8s! 2 18-10% 6-10", > 0 W224 18:10"Ps — C102 and Cle) = 2Qla) = 20-1072 2s LL IO? ‘IE5(e) dollars the weekly profit then Hie) Cle) = We ze 110s 9-10 Sfp) = 224-10 8218-10 = (1 ~10-93}22 + 18-90-25) = 101080 —2)022+ 18-109) Mecause (2) >0: WO-<2< 100 and Sz) <0 Mf £> 1000, 5 hus am absolute maximum value 000, P(1000)= 11 1000 unite ace produced a 811 eaeh- 02. te ~329 420-5 =0. Let fiz) = te4 Be? 422 —& fe) = ee +2 Besa {t= 2 ed 12) 39, a) <0 ha putin ot bene od 2a % 2 Lome oggsise meres LaT16 5 iris aoser22 153951630 4 116801 .oocsa a2 LT 103.26! 29 +3627 +2280. fle) = Ie! — Ae! 4 3c? +208 (=) 12s 12 a Te? ‘Beenune (2) ix comtinwons, fl=)= 33 and {(0)= 8, then f has a zero between ~1 ani 0. We ‘= "2.5. From the we conclude that th root is ~OA82 te three deinal places. O] a= Oe, =4e,74 90,7 Fe, 8) © (ide, = Hae, See, +2) =5TO 3] 5 ‘o5n “385 raz 2] -oasaia oourt 36890 —tasits 3] -v- 0, Thus by Definition 4.5.4 the .of f has no points of inflecti Suppose p<}. Set Me)= 0:2 = (I-tyiet P ap esas he grap is oneave op, fhe graph has a point of inflection the graph is concave down he graph has a point of infection _ + the graph is concave up “Thon the graph of [has two pointe of infetion, eth ifzeo a igeso e<0 NO. @) fe) —Brlede—ta 244 Tozer) HOcr 3 fr alle i 1). <0, then /1Q) , tha f(s) 20 90 f iin (48) Hc 6) >) fal in (4s), Th J). 9 fal 0 $i Tet feo ses Ht treo zea 0 ‘exause gi contevous on [dead these ate 20 cel mambo 1: the sbroat minimum value of [Od othe smaller of (0) abd f() =4 Henn (2) 25 for all: in 1) Wz <0, then 92) a8) = fr a =n (0) Hee» 1. then g) > 0 mop i inert fe [lyso) Hence pe) > g(t) —4> 3 fr all zm (Ippon) Thas gfe) 2 for al 4 00 3 8 the aba a nermie+s eco B seco pane een (0 fle\=aeltte—tl= dees feces ranferk ioeret feat eas eel farebs—b ifz>1 Fecaute hie contisuous on {sl} and there are 20 extial numbers in [1], the abvolute minimum value of om [i] is the smaller of A Gal this umber m-‘Heoce A(z) > m for al 2B [Ol 1250, then 4(2) <0 ao his deereming ox (—20,0. Hence hz) > h(Q)= b> m forall xin (~20,0) 1 => ise We) >0 ao 8 ie iterating om [le4o0). Hence H(z) >A(1) =a 2am forall = tn (15400). Thus Als) > {or all: 40 ie the absclute rinimaz value 2 <0 then (=z) and (1 — =) are both positive and decreasing, and so f(z) is decreasing Hy Sects Gelade 259 ules Peingenag adept Ege my etree 10 ve [nabee bse shea Now of(a8) <1 Because #(2)>0 if 0<2< o/(a+8) and /'(2) <0 if a/(a+8) <=<), then J bas ae # & MISCELLANEOUS EXERCISES FOR CHAPTER 3 275 relative maximun valve at 2 = /{0-+0) of Keb)=(l'-) “Gent =a se as 9 a5 tn eater in ww get Form CeO le ‘clique asymptote y =2 + (76-4). ete anette G72 canton tenrnten, lip 06) =21= a, (oats) tn, AERSY es] vepevtt Haft =e We ‘toh +b) = {fesse Hla tee tba gL + PUL YH eat FOUR THE DEFINITE INTEGRAL AND INTEGRATION 41 ANTIDIFFERENTIATION 41 Definition A function F is ealed an antiervutice of function f on an lateral Lif P'(2)= f(e) ‘every value of xin I If x dncootinuoun, we require the enesded derivatives of F 10 the cnesided limits of 7 Se Exereise 36 4.1.3 Theomen IF F i partelanantidvvative of f on an interval then every antidervatve of fon ‘given by F(2) +6, whee C is an arbitmey enustart, aad all atidervatives of f can ‘obtained ftom Pla)-+C by ascing particslar values to C- "The syd [ devotes the eprration of naidiferentaton, That [Pes [ erey= reso 1, (1) states that when we antdiferentate the deivative of a function, we obtain fsnetion plus an arbitrary constant, We have the following formas for antiiferentaton, setae foseva “LOT i fs and fe dened cathe ae intra then Juntos stance = | te) de+ f rsterae ALT Mon If fy Su fied othe ae neva, yo fy a ona, Jlanesete)esenf(eilde=e] Ae) denen | hfe) dette.) 6) ‘When applying Theorems 41.6 and 1.1.7, we use only one arbitrary constant. ‘To find the antideivative of a product cx quotient we may replace the given func by a equivalent sum and then uae Theotem 4.1.7 aillntated fn Exercise 28. allowing “seme atticffeentnton orm season 41,8 Theoeem If # -1 fs aratlonal number, fraatiec pac st Soh gb tio ie i mapa hy ral 1a tienen fine den mores “Lin There [erode min 46 110 Then facede sn e2 112 them fated -enete 112 Thee focronsienoe sc 110th [ee est te= nee ‘Graphic Sapport Assign a value to the constant C and plot its numerical derivative in the same window at ‘rgieal Tencon to sey Une ape Haen Ezerivs bt In Brercses 1-90, perform the antdificentiaton. n Burcises 1-8 and 24-28, check by finding the Gavative your anne, In Exercnee 0-12, 28 and 80, aupport your answer graphically 1 fartarnafetecnierac 2, freee =2.ye sles 41 ANTIDIFPERENTIATION 277 We apply Thewern 4.15 sod ‘heorens 4.1.6 [eaca[rtacstereasec i Rg =g+e=-}rt+c The Pi)=-Roar#=3 BecosseF() the given neti, we ave shows tha Fi) an antiervative ofthe given fonction Sel da 5-204 6 JuvFeen [ean vp soneeMse gees [arterer pts cnse sc co We apply These 41.8 aad Theorem 41.8 witha = =. net ayn afr iPay=: +o tyie epniletantipareornes ay Let Di6/5+0)=D,6y'? +) wee (ot -atyinna| tesa ot na sef-2ttec apt bt way? ayty= fav? ay hey = Pte e—2yue Wemmltply ard thes apiy Theorems 4.7 and 1.8. q [ete-24e= (Ss*—2")de=5-42° be" s0= 2°. ‘De gue sows «ot ofy = 29 ~ 2") and y=NDER(2*~ 1"). ‘i (tot 44a — 62? ta s)de a feb 4 ot 2e 298 45 4 (0432 —aP de = 2243-Jo 8x4 40m 2 P25 6 file +Dide= [iP seas =e Bee vi =y0hec = [oP at =o ae 40 (e-4+d)ee: +0=3+}¢ Babee acm (celal ade nde eA J ane atl wal 4 + ‘278 THE DEFINITE INTEGRAL AND INTEGRATION om, [ost We seduce adil with factional expats then divide and apply Theorem 4.1, i a Ce mB Hl 98 esnee a. [(Yex gehen [ce rete ntl at? 2c ma [pha = feet —near= dafand}s ert) gear gan a [osce-teona 0, [Gem x—tsin aie We apply Theorem 4.7 and then Theor 41.0 and 110 [lereee-Asinske5 oo zde—t[sinzde—Sanetseas+C ws [stg aes [bg hydra [unsecrtemaeeee mm [ome de= [ote gh den [oot race de= cw +6 ar [(ececnertednde= toe rata 28, [9 o2Ft=5 we ttan tt ‘> We apply Theorem 41.7 and then Thorens 41.12 and 413 face Senet tan pée=3 feuds ats [anc thanrdt— teat to B.e 14+ C (este ssc Nas 29, | eeen—8 tanto} = (oe =1) = e000 — 1) EH2eot 8-3 8+1+C so, faiaesectemt ay af ing ata ing an 3a ned tA fen 9 4b = 3100 —Aan + a Beesley 81 30, the graph of = fonstion in geo, Sketch auerivative Fy continues svrrywheee hi the given values aL is Fons » 0 4.1 ANTIDIEERENTIATION 279, or Fo)=0 2) gives? Bae y= |e —s)de; y= 2-30 +6, Sabatcing (oy) Thos anequnon ofthe ave hy = = Se 42 Bea sya ys [3 aen3.fe +0.226940. sutettuting (0) (94) ies 422-09 | C=544C, C=-50, Tho a ogiton ofthe curves y =25"!*—50, Spa Ymr tery! = [a tepde Pate? Gy y= [te 2P +G deny = 2B eC 4G Stititing (0) = (0.2 gies 2= 04 Cy Cy =2, coy = 2? po + Gyr +2Subwleting (e,9)= (-1,9) vee 8 = (1 H-1P +C-1) 425, =}. hn, an ogeton ofthe corveie y= a? J 4 fe 42 ‘Ag equation ofthe tangeat lin toa curve at the pois: (1,8) is y=2+2. If at any point (z.y) on the eure, ay fde) = 62, find an equation of the curve Udy/dz y/ then 28y/de! = ey ds. Thertore, we are given that % Anion vbw y= [evienae vc Sena y= 49 an conf he es ie oc he i (13), espe SESS a Re Seca T=340 280 ‘THE DEFINITE INTEGRAL AND INTEGRATION in Ba, @). Tham, (a —2thde; f= 232+ C). At (ily he spe of the tangent line i Jee-Je-bes Sting =i wher #2 ive 12 +6, 6 =} Tha ped pate Cy, Sobsitting (9) = (1,1) gies T= FCs An equation of the curve ny = je? je" fo + ao pratistn [aaentes, ene (3 ait fiton 10) =0=246,-6 Yiare-2. y= | (te—2 22 224C, ll) =-2= 14 Cy Gp =—byl = PB Jee-te-thee Jo-2?-24Cy yl): $46, 0, 24 =f -P 249 MY O'(e) = 407 +82 +4, and O10) ofa) = [sat +82 +e P+ 4e 442+ K. Became C(O) =I K-26, Threore Ca) =a! +42 + Ar 6. 4M. A company has determined thatthe margsal cot faction for the production of particular commodity ie rent OG)~ 12s despot where CC) dllare ie she tatl at of prvacng + unit of the commodity If tbe overhead cot i $256, what isthe cot of producing 15 eta? fp Tue coo fonction is the anficeivativn of the marginal est that is che fuser pane fos a onl ested conch ate eB OT a ee (18) 1258) +22) + yh +20 = 394 Te cont olin 15 a 5 br de = 3:7 + K. Because C12) =20, then 20= 3(27 +K; K=8. 32748 (b) C{0) = 8. Thus the overhead cos is $800. ae, Re)= [02-Sede~ te —}-+C RO any, C=B Re) = et a {A Rijs) = 15 —te, RE2) = [19 —4ayas = 15-2? 4K. Mecawse RIO) ~ 0, OK. (=) R(=)= 152 —22 (b) Because R(2)= ps then 9x = 1bz—2s" so that 9 = 15~ 25 48. "The afency of a factory worker is expresed as a percent, Fer instance, if the worker's eficieney at a peiclar tee i glven as 7D pererty tian thy wenter ix pecfring at 7D pereent of her full potential pout that E peeent isa facry workers efceney 1 hours afer begning work, snd the rate at which peetine (30-80) percent pot hour the workers offeieney is 81 pereont afer werking 3 br, find dienes afer working (a) #hrand (0) 8 » Webave eK Be Because B 35-88 Jos-spares-a?ve 1 when t=, then 41 ANTIDIPFERENTIATION 281, Substituting for Cin Ey (1), we have Bl) =S5¢— ar 12 4) =39(0)- 4 or = 98 (8) 8%) + 12= 96 “The ettciony ie) Wh perce after dhe and (4) 26 pvent after 8h. “The volume of water i a tank in Vin? when th depth a the water ink meer BL uit in 4.9 ¥ = [ 4+ i 9h, = AP 48849) 4C, Brae then C= 0,10 V= x(t? + 62° +58). When B= 3, ¥ = L170 the volume is L173 Bases 100450, v= [e+ 10s soe = 267 +52 50e+0, Vio) =1000 2°77 45 50¢+1000, V4) = 2-47? 45-42 50-44 1000 = 144, The punting is worth S124 foainnna Gh Uispeine(ee UEst (0) = lim, ()=0 then P“0). = el Because #(0) = ig (2) =0 and Tle Fanaa ff (-20 42): tet Uis)=[9 £42. Show that U dis ot have an aatderative on (-20, 420) O(-1)0 Ae C1) 21 ands tenes Oa tt U dn ate he vate fn (~The nermeste Vac tcon fi deivaiver (Beaoe 14.86) imps tht eae U dos aot ety the lates Proven, ten I banca acre say faction nt iy O aceon sac le) =1 forall x9 (1, Terre fe) =O forall xin Hah ae) ofr HES (2) =0 if 2 is in (1,1) and 240, g{(0) does not exist because ¢ is discontinuous at 0, Hence g is not Stevaati on (si) sod or Thvrs 41 or co ply fie a itas0. wey mteim{c* $289.) Deane PM) He) fr al Fis ota antarain f SOME TPCHNIQUES OF ANTIDIFFERENTIATION Ye pgm Ly bt lec net ean ob ana Spe ani zit Bagh a ncn dettd col an that Fis onesie foe Tee fe) Jraensterateriten so of Subetten To apply Theres 42.1 wena et = ls} 1 filo hats = ge) de [ateits(e de]= { 40) dua 0) +0= F(a) +6 shee Bay sai Wf. yc th eh ry be aren Jristenis'e aa} frteyete)=Ftaten +6 loc Argument In pra, fe) = x + then dum ed and0 [rezone [ g0)-hou= fre) ee= An impatant sil eae when fe) 422 Theor If gis direstible function sed n 1 eran sub, Joorve ay +c mer te [Note the use of bracs{) t signal an implicit substitution. 282 THE DEFINITE INTEGRAL AND INTEGRATION xen Ut in Bares I= id the sda 1, Check your answer o suport raph 1 [ yim = Aa) (-tay) = Ah aoc = fata 2. [vamie for atPiaan af Ne-oiPac-far-o%se 2 [=F 8ar =| feet 2ede) =1te 9814 0= Bet Ot 4C 4 [rot sihae > We apply Theorem 4.2.2 with n= Jresenter={ferrittarey =| 2M sco pers itac 8 fete = nae 4fP—1PEetae) = shhte? 140 = aye? EC 6 [seven de. Let w= 4-29, Then ay Beene di22? +1) =e da then dzjede= Ju evi den) [dna fea A PRC 1. [gti A c-atrteP an leo at] +e =f ia b Let u=3e+ 1. Then du 66 de, 04 atta Se a forse sahaen [te 27Paen [ee-2PPae— Hee o,f var =Bde = Bf se 91 Mrsette) = 4 Fs —57"* +0 = Ra 5? 46 un, [evi Bde, ta wae 42. Then dade aad 2 = 02. Thos aVEFEde = | (w—2Vidu = [ (2? -261hds = 27 -$8!* wos Td han 08 The Itig= [aR Ee fogs sf te fomanafurihaandeh aioe Fiera? ooo He+2f Yor aac 8 A dm ek carr eave fot fo recat ar ™ oP sekla ewesitivte don sere te Je-nercdene Jew e pram fe yptton gee ene 442 SOME TECHNIQUES OF ANTIDIFFERENTIATION 283 bf VS=R ade, bet ua tte Then 2 = [G—1) and de =f Th Vint a de Jur ae-0es Bf Get 62+ we = Hw Bul a8) 4 0 = 5 20) 249 26)°7* B20)" HO b feta eas Because (2.4/4 at de = (243)! dh we Wt P23 mem 3be dm Then Jems sere fier +a Pyeraey= [ulu—ang an = neo) fe ecm geet 9P! farsa +e co 9 8 = 3 con (Aa) = Jin + inf de = fin ees) ~ 3000 fe) 40 ~ Bon fe + 62 in? dem 2 [an 2430 ae) = tease? [0 Gat wy Hoa de Het a, Then dus 841,501 4t = fu and Jieeteuan= actin de = hf secbe(6éo) = fan be +C oct 19 e8 =} fea 2248) = feos 8+ yee By cov sy? dy =f fen yet By dy} = hase y+ set Bde I t ' E ' : ee ne nulbaibs ifn edez aes Ca faut ve atin Pane [ulten dha fora ore Asin = aca eon a) Af sine de) HO temas eee pate Geese} \iedSureisia PEG =-a[ earn wh rom ir gyree wes eben ft Then d= dyad Jasin = ivwer te be Josmae-eonattae rn 2evi=emte te =i fem 20)H9(0 sim te de) =] C2 — con 2n}V2 4.6 = K2 ~ con 224 Aen te pide date 24 ‘THE DEPINITE INTEGRAL AND INTEGRATION own tdt tat scan = odin ta, [etn ata [ten He 40= fore 4 m, feat oesoa + Leto sin 0, Then dum coe 0 dP, and Jo? em eae Joseptrcntuntose 2%, foun aesenantie~ [andes seated [leche =i) 2 Die 2 fice setae mua te—fe2e4€ oda ae ni a IM a tet w= BE Th du ae and $= fe Ts etl a= Jucugen % [HE enter etaentee stn eee = afer spl csiVeaea eC Bf He? $1VER IIIA de Let w 24-2224 Then d= (42 40 da = —Aefe + Ide, Ths =eenet Caen ovire ef 2elde | vie? yee} = 3] tee = 145202 Yao? 226 a, [OEE tat wa dy. Then y=3— wand dy = ae A SMy _ (6 WH) _ Fyn _gyat 1 sgul8402 40-9)! nD (oi (=i) = fut oe2ya = lil 4.62 4-9! 188-9)! 8 [ VEFE (6 +1) dom [flo -3)41F Quan) 2 fot nia den 2/ (vate aety ae =U fie pec=foenh fare Herre CO a eg nde fettn, (OEM os futdan fer ee a ye. 9 [OSE Lael [ERB as fomnprecanereatec wo. (4) Baha sit wate den(ind)ar -Bplar Jey Sptan [or menrse Sega (208 caveevematan-ymanmen |g 6 | ign [Zep Meet tin tat tan ctede [Pig SP hdu a aa 48014 (tsa aa Are ne VP go cide = Jet Jee PP 2c 2 2 4, [in esinlcorz)de. Tet w=core; dus mn ee, a eslenate=— [ined sien sC=rndens) 46 42 SOME TECHNIQUES OF ANTIDIFFERENTIATION 285 foce tan coon 2)ds Eis cratinannaslnisdhe Jovstecmsie seen [ovedeosirec sfoeveriterateeeavton eee 2) Cle) = (52+ 4)? and C(O) Beaune O(0) = 10,10=42) 4K K=P. Thas Cle) = fYae TT (2) =3/IEFT and C0) =0. Here Ce) =2[ te + ae =3-}jie +4 += eM ak Decause (0) = 0, then 0= 4°/?+K s0 K = —8 and the total cont function is C2) = (22+ 4)?—8,, Re) 44 2 +5)"% Therdore Re) = [4+ Lole+5)~Mee~ te We, Besse R(0)= 0, 0= 0-FB-4C; Cm2, Hence Ris) = de ~ 0g Its une are demande when p lle pri per ni then p~ R(c). Thos pom dn M42 pele 45) =4024 20610420 $10; pon +5) = Ue? +28 He 49 ‘Toe marginal revenue funtion fora partislas article is given by R'(z)= ols + 8)~*—e. Find (a) the total efenue function Rand (®) an equation involieg p aad = (dhe demand equation) whete 2 unit are cemanted orien pdollam isthe price per amit (Rte) = fjebee+ 0)? a b=, Hence seen fnction Is R=) (0 Beato Re) = yaa 0 aif6)"" and 0 ‘coulomb isthe charg feet a cea Breaseq = 0 when tf, then 0 == dpa 30e = =Jp+ C=. Tosh gan eonons hs d+ dyad Bence te gratet charg in calonds, ‘qcvalombe ithe charge of electicity at tse and = Hf = 4 cos 104. a= [coe 1204 dt= sin 12H +C. Because q= Owhes {= 6, then 0 =. The @ =Jysin 20¢< $y eae the peatest hares fy coulmbs, V allam is the value of the machine fe yer, and $= ~BK¢+ 1)" vm J -s0oe + 1)-ae a 00(44 19"! +. ecmse ¥ = 700 when ¢= 0, shen 700 = G00-+ Ci C= 20% Tass Vie) = 500(1+ 1)°74200, and V(S) = 500(8)~! +200 = 325, Hace ater 8 yeats the val of the machivary 16 $025. Sn 60: dt =~ fy os UC. | “The volume of water in a tan iV cube pte when the depth of the eater is ters the cate of change of V with respect to his gives by A=a(2h-13)4 nd the volome of water in the tank when the eat 3. Bae sahs vay= [rea-rayen Beaute d2h-+3) =2 0, then Vei)= bef kPa) = fot +0 Because V = 0 whet A=, then o=}:0)'+C fev and ths V(i)= [Oh fer Ved) foto?) fem atte ‘The volume of ate inthe tank when the depths 3m i IT wade 286. ‘THE DEFINITE INTEGRAL 53. ICV pan? i the vole ofthe ll ays va [(@—4-2a1- (3-97 Dee Vad when #53. then 3 B10 GC= 19. ‘Thus V= (1271424. On December 8 t=7, V=5"4 29 Sl, and the volume. pn se venta he ven ofthe talon a seconds thn SL = TFT V= [CH ge = HU+ 1442 +6 secause V=39 wen ¢= 9, then 39 = 9H Cem Bh (eV =Ye any SP HEA 0) VO) = F997 44-48 4 64, The volume eter? 5. fire syPae (a) [caesar fee srnete er de FAP 4a 424 C, Gola antestidentée, [ter Pde =H[ dte= ht +c.=[te4i 46 Beet +2034 2k? ar EC = Bele EHEC {@ The anewes to pats (0) asd (b) are the sare with C= 9+ Ge ce ae [oes ay sete) Ex ( 2-42) and math re (0) et v 1242 (6) Faplain the teeare in appearance ef the answes obtained in (2) and (0). -@ [ote entac= facet eset ante [lat ete rsnnien ett 42st ch (Q) If waa? 42 then de (2 +2}(242) JeQen=W rear earees Math Gat 12s? +6) 4 Cy a fot eat eae rd ( Tue anowers to parts (a) asd (b) ate the sare with C= $+ Cy a1. (9 [EGE ane [en2el™ a neritden [eons Veen fetes elt oc, (e=ut (pees yemnden fie, [OE cen a[ bund Cyn VEA 9G aye se 8elt=1) 4 C= RP nett 4 (6) The anoezs in (a) aad (0) ate esate with C, = —$4+Cy 8. (If de and 2) stm [VERT des [ yes uPann [eaten = foemamerewmaccten stem ahem yore nian WP ate Me (ites YeTT then 2a vs Land dem am a0 [YFAT Paes fan wor eatsnryen te ete Bec he nef i oe HP ve 1, then 240, 4c, a [asnscorece (jin aman situnimete [tsnscentent [ot wate -1 4G ehuaineinara-aecte [tuesomtan mca] esee Aityn teats (6) [2sin com ede Jone de = — Joos 30-+C,——H1 = 2sinte) + Cy aint —]+Cy (a) The ansere in (0), (b) std (0) are the same with C, =O, +1 and Cy =C; +} Go, Evataate [eae cot 2 de by two methods: (a) Let w= cat s; () lt » = Ge 2. (6) Pxplain the diference fppeaiance of the szswers brained in (8) a4) (a) am eat 2 then a (0) tenet don ots drt [aeeoc ott (0) The tecece beeen the soutons is Has ~ cts) +6, Cy 4.3 DIFFERENTIAL EQUATIONS AND RECTILINEAR MOTION 287 DIFFERENTIAL EQUATIONS AND RECTILINEAR MOTION ya dfs. A Sitferntaleqiation ‘The following ope ars wee o Gnd o ction of fnionder diferetial equation I we a ate the variables 1, Seperate he variables ‘Tat is, wit the difcental equation inthe form Ho) dy = Saha 2, Antdiferentiate on both side of the deere equ oy)= Fete wae Gy) =s6y), F'@)=f(2), and ¢ Is an atbteary consant. ‘This equation alld the complete Toletion a the fiferatal eatin. 4. intial conditions are sven (Uhl iy replaceient for & andy are given aubiete the given valu ie the complete luton, wae Fors and subuttate this value fr ¢ inte compe seluton. This rte fa {he pertilerwlstion of the diffrent equation. eb, the equation shold be solve fry. I the diferetisl equation bas the simole form dyfdz = f(z). step is ot necessary. The steps for Gadieg the complete solution are ilustated fs Hsercies 4 and 8, aad the the for fading the particu slsion sz iotrated in Exercise 18 "A seconborder diferential equation in = and y 8 an equation involving =,» dy/ts, atd ds/dy? ‘The This resi in an equation ofthe frm ition af =. There wil be two "The step for fading the solution are ilestmted i Exercise 12, and the steps (or Finding the pocticule solution are strated i cn. Ta partie moves in stnight ine and ite dimectd distance from the origin is units of ditance at of time, the the ita of west © and the mite of aeceliestion © ar gives 0 ft ® daed ° ‘We use Fyuatins (1) and (2) when we ate sven the aceleraion and with to find the tie, distance, and a ilustated in Exercises 24 gad 28. We tse Bq. (8) when ve are nol iatereed in ime, ae illstated in 432. Ie the inal velocity yp we wolee Bq (9) in Exetee 48 to fod tht. tar Fao? If the only force acing ob a patile le the force of gravity, then the socrration i eonstant, and is insely 32 fat 1-14 find te comple sletion of the ditfecetal sunt = one eu norco aan Fit, wesepaate the variables, Note that for any value of, Eq, (1) elds sC79)-SS a4 “Tie Be. (i) the complete ection of he intl eustion 8 fi tet fesean [yt eal eae pode 6 BS rena cones ot —nrrm [capa 8? = P= PP ato Vie yittpcman- [eterna aafron ase” Wee atic We aepraethe vsable and hen entire om oth ie, Pay Pade [tar fevemee Because d(z?-3} = 32" dz, then praf[e-n'oe en age 274, aKa? 430, Replacing 3Cy by C sce Cs any constant, we Base Faye-Fac y= Ge acp? sich is the genent sation. 9, Ba 2S ps tantydy scot Joneses = [nce tn yy stan 46 10. $2296 Resin ede scones [sin ae = fee 20 ds hoon de = pan tere Ba one nay tarde [as [ee-tn y= Ba fe res ce for [ageeereuinensetrlescr4 {1.3 DIFFERENTIAL FQUATIONS AND RECTILINEAR MOTION 289 Sa lEHa Te y= dy/ite, Ten yea? = o dy faz and we bave “ woes [Arie Became (22 ~8)=2 de, then Jee-3y"+c, ‘We ansditeentinie asin get. g=ffce-aP are, [ee ea ayeoe2c Spf =a rete! Heo arpenseacge cn eset; f= Joiner 30 (Jems sin a+ 6; je= (eos H+ $ an St+C, Jats 9 =f sin St —f eos t+ t+ Cy y= ff mun even’ [an yeotvas] shuns c= joc, whee, =6-) t= [queers ees fuanee gee Execs 15-20, Gnd the particaas sation ofthe difentileqtion erring by the Batent [dye [taae-idnye |e A kere Because y =-6 when 223, -6= 9-9-1246; C= 6, Therefor, y fo —2? te 46 Ba e+ tet 2h y= F when & conditions, ya [tsteaneretettense © Pi eh yi acer 2 wy 1) Sa feoP ea 1-926, cig Cy 8 By 1, mba th prt wen tite Sas Jay ar= fonteen tem rece Pipa ca json fom it = tin i+. Deas 62 when tap th S22en fe +0214 C;C=2 sin jt =2. Join Se Because y = J when Heo [eony = fain e+ Fpeonty mar 4997 m0 ry 25 oma OE Ge Thom AOA EPH EG, “Beane! 2 when #1, —2=4(-) +0 C, =2. Theor, fea nd 5 2 4442; a= [0084 el stv ede eset sa 42082040, ‘Because w= -1 wien S-442-2405 ©, = 0, Thereire, w= Sot 404204 2¥. 200 THE DEFINITE INTEGRAL AND EVTEGRATION a Shae y= fant oot vs > y/= dof, thmn dds = @y/ta?, mad we hove and yefetategt tn Brecses 21-34, a patel is moving on aloe; at secands «fet ie Gnected distance fom the a (Rend ite wlocey, and a feet per second por second is its aceeration. In Fxerise 29-32, Tele wands nn va dhn VIFE c=} [240872 doh a = att 09? =. acouee 4 = O when (= 0, 0= HA? +6, Caf Thereonw, «= fre at -§ mm vattad een [a Aidt =4t-H2 +0. Bomiin twins tego ner feat Toerefece # = 4t={ -6 Ban b=2h; v= Band e=O when E=°. & das aude = 6-208 Jes-2nee “Theron, -P4Cy, Beans v=2 when (= 0,2= Cy Jose +2 =f 40 42+ 0 when 16 we bia Cy =O. Their «=2¢4 48 3° when = 0. Expres ands in ter of we ae given Jrreeese, seca = on = hn 8a ha 0 wien ¢=0, then e~0, and ye te Fgyntions (2) and (2); w and «ae xproed in terme of Because 43 DIFFERENTIAL BQUATIONS AND RECTILINEAR MOTIOX 291 a= 02; 8= when f= and 4-3 when t=? Sr tere en [Ces 20Us Hayes e + nies [PsP +c) tt +P +e +c, Decoane 4 = 1 whom #0, C1 Denne 9-2 when f= 2, -2-= $4 9420) 415, = =A Therein, o= Ait 440 —4t stand o =] 44 shea gaa 8 es [ar ata JP —30 + Gy Heeatse oJ when $= 4 FF |H Oy =H [2 Seat = 18 — het 4p. Beaase = nben t= 11 dads C= iV/2 cou 2t~ Jn}iv = Band #=1 when tyBcoetts— Jr» ~-2V8 fete Jeet) = -2VFein(ae—- J) 1-6, 1 22 -2Vfbsinl-Jp) 4G 2 = WHI + C4 p= aE sine — fn) = ~V3 | in Jo)(2dd) = VE aslte- He) + Cy YE co) + C51 = W842) + yi C= 0. Ths = WFconet— Ie) 4 when £6 Expeene wand ain tens of dod, we ae given Thus, ¢, ‘Because v= ded, then se 3 ut = -24in 34.6 Benue #4 when £0, then Cy 4. Thus sis exprnsed in toms of by atin ead um fp= eds deena [rde = $00 [aes 3? 400) ~ 8006: © ~ "600. Therefore = 09 — = 5m = 44, wd sn; [sae = 800d? = 06 +c. Boas 24, Theor fo? = 005 ~ 1000 400 (orthisd? -Siame}o,toatswee +840; C212 Theos, ? Find 22 equation involving vand & 08+. Betause += 20 when 1008 = 241200, when 3 200) = 25004, ee ) 202 THE DEFINITE INTOGRAL AND INTEGRATION Jreo= [eeetures piaFeec Brean 92 whan 4 ham pteraiic Hones C= and oe VTE In Baers 35-43, che only foreeconsfered is tha: du vo the acceleration of ravty, which we tke ns 32 ff or 08 tafe? ia the dowayrd diction, See 2.7 for exact wales of tho wine nd coin = feat) () 08) = 40 - emo) =f VB) 03824; 0) 428) = wef) (o) a4 Bla com ee fy) = G16 ~ V5) = 0.6508; 4) (72) = Hd ~conz22 Deon fot: «= [Deon rt = Sein ot 4G, Becouse «= 0 when #= 0, 0=C. Hence = fin rt (a) 6) 4 singe —20-+ V5) = 1.001 (6) «(25) = dai fe = 2/9 —n008 (6) (48) ~Sin(ae = 3p) = Lyi DV ~ 12109 (4) o(1.2) =$.intte — jx) = 38. At (one tf be the divest distance ofthe al fi the growed set v fn hte ‘loc. The postive direction is tpward: We have «table of boundary conditions where Ts Shiau vajoerem the imesad wlocy hes the ball ler in grand and Tene and Sf te the time and distance when the tone reachm it highest point. The acceleration of te ball that due to groity. Therefore 2 —32. Thar Gant, [ov=-s0feev-—m04.6, tease = 0 whe set J, we obtain Cy = 20, Hesee 10 4200+ Cy Sace #= 0 when t= 0, C3 = 6 Thue er 4201 Prom (0) wh 2420: T= o Te bat wil be gol uprazd ef, Prom (2) with , we hai $= 16-29. () Tel wl ec lh of From 2) wth += and t= (0) Tent i wi tke me rte bat ke he rot rm (1) wth ¢= fad e= 8 we bave v= -226) 420-22 (0 Therefore be bal late the ground ih pr 20 /e 136. Att sc let s tbe the directed distance af the ball om the grouad, ad let vf/se be ite ‘locity, The postive divecton is upward. We have table of boundary conditions where € ste tad aad » fore ae the time and veloly when the ball stkes the greand and Tsec and S ft [Se the time and distance whea the stone sachets highewe point. The acelraton of ue ball [that due wo gravity. Therefive a= ~82 Thus 2, [do = 22 fats w= 244 C;, Beene v= 5 ven t= 0, we cbiain Cy we have ~ 1624 207 = 0) —A4(H—5) = OFF = fa-s0e4 5: [eem [eam sone 162 4514.0, Since s =O whee = Cy 0. Thus aioe 45 From (1) with and {=T, webave )= 327 +5: TS, (a) The ball will be going upwacd fr fc. 4.3 DIFFERENTIAL BQUATIONS AND HECT From (2) with ¢= jp and 9S, we obtain = 10> By+ 20-3 =H (The ball wil reach a height of MF ram (2) with (Therefore it wil take sc forthe ballto strike te round From (1) with t= {and » = wehaven = 0268) +5 =. (@) Teteore te ball will ste she ground with spend of 3 /, At tse let aft be he dicted dinase of the bl fos the tp ofthe Washington Monument and tf, we have = 1622-451 <0 106? — ‘tnd let + fer be is velocity. The pute direcion i upward. We havea table ofboundary fs © ‘conditions where fs and 8 f/oe are the time and velocity when the ball strikes the ground. @ 0.0 1-558 ‘Toe sceratin ofthe Ul hl dint pavity. Tse =m; [or=—mfave 2120 Branee¥=Owhen = Othen C= 0, Here enna: Mt $35 Dee = Owen 1 = 555; =: above the grouad sod i ling tw tse te 1Gt = ro sg= 10 BIE 89. = When (P+ Hl 8) = 10 9Y— Th fd (= “Tae ball :akes(a) I scond to ach te Erne aed (Bi he rena pew 9 see AC sce let be the dicted datatceof the bieais fom the round anklet vfYsee, tg ‘pita voit. The pote deetion is upwards We have a table ef boundatyconions where 0 150 Tine and fie re the ine end veloc when the binoas sie the gow The no 23 dpa; [ae = 24: Breniee y= 1D when =D, Cy = 10. Therefore & I y= 180 ‘Thus #=—162 + 10¢+ 19, Wah #= 0 and {= 7 ne have ~ 167 + 101+ 190 6:2 = fil = VF 34 (a) I wit tae {14+ VHF) 4 sector he Binwelas to aie the ows. Ta (1) le t= §{1-+ VA) and += 0, We have v= —1001 + VF) +10-= -10V9F 98.5 (0) Ths the sped ofthe biocalason imac is IOV AT 3 85 Pe. Fa coin wy ja i big bi 00 abode pnd wie a TD ao. anyone (a) How Dong wl i ke the von (ofeach ite gett highs (0) wha he geton belgh? {O) Hew tong wil ia the sone te pete top ofthe hone en ha way de, ond (@) wha ev Shatin? (2) How lag wl i cake the stone te srk the ground and (9 nth what sly doe she the gon?” ‘seconds after the tone vas thowa, let tet te te vane oft see shove he gonad ‘et per second beth veloc of he sone Borer the prio divin eon ai pnd shannon‘ terete ef the 294 ‘THE DEFINITE INTEGRAL AND INTEGRATION de, fen te fata meg Becatoe = 40 whan t=, that c, 40 and (920+ 40) de—= Because the top of the howe is #0 f above te ground, then += 60 when t= 0. Thus ¢, = 60 and +10 C0 = 261 °-f+ Gy + 60+25 ~ —26(t- f+ 8 “Thexefore(a the stone taker Jee to each its rst eight and (0) tne Neh 16 9 (Qiks sone pases the top of he Reuse on my dawn shen 960. Tes oo= 14? +004 00 tayo “The only postive slution i itsmay 2own (@) Replacing t by $in Be, (1), me have -s()+10= 40 “Thos, th velocity ofthe sone ie ~H0 f/f he stant Ht pati the top ofthe Bouse a a may own, speed is the same as when it was thrown (@) Tae stone strikes the ground whee (2-162 00460 =o ee ee ef _ 52 VB. asy eae the soe tl abel 3.5 st site tbe pound. Pam He (1), 3S) +40 =-8y5 = -738 (0 The seocity of the tone is sbout ~T5.8 ae wheat ste te groan, 4 abowe yo head andi ling at» se We B24 tp 88, #16 gt Ag = 1624290, «=O when ~I8e +200 = 0; =H t= HVE GV) = ~80V3, (0) Yor have about 3.34 scons to actor (h) the boser sks at about 118.4 sre 4 Act aeelet ft be he dnd itace ofthe balls the tring pont and lt + foe bits velocity. The positive dition ie upard. We hove a tble of boundary conditions o ere /se isthe velit of the hall when 36 above te ground (ot equivalently 16M above tr staring pint). The celeraton ofthe bln that duet gravy, ence anv dem a2; [vae= a2 [én = -s0040, (@) Became» = 40 when «= 0, me ge C; ~800. There f= 600. Tat 6=16 ane v=). Beet the bal is ring F> O. We have =? = —64(16) +1600 (0) the veloc of te ll i 24 fae whee it 30 fom the ground nd rng 168 s4084C, $ we eanclude that it tas the stone see to pase the top of the house 2. From Ba (2) we obtain AB. Att ae the boulder ba 43 DIFFERENTIAL EQUATIONS AND RECTI = -B.8L4 tp = O84 180 (a) 8 = 407 bgt ag = 497 + 1900-42 (0) a2) oN 2.4 highs 4.07 16004 9 —500, 492-1501 408 0; fy = ATS UTS 4 TOS). (ee wl be 900 fk ap ater about 3.73 sec going sp ad une + eyt4 4g = 10 se coming dows. 1190)? 980 m 90) 1 -+Cy, Berse 9=0 when 0 when 1. Became » = 1209 tan 2, ons? 6 hen t= 8,2 =, The sh Ji-oe+20at =a 4200+, Beene 90 when £200 Cy = -SE +20 The bal ope wher 1B. then s= ~3¢9)*+ 20€8) = MP. Tae bal wil oll 1 fer bon the 3s) gph ook. 000 Hie — Ho fe Sede = Fs 100 P= 10D A she = Goi the satng pl atte pion af he stomebde when te yey HA one ving ae 0 cn 35 age wine [otarenteecy tee 20 in t= 61-220, MaCee= et [At tae, lets meters be ite tance from the staring point, atd let v fare beite velocity. 0 IE atthe paitive direction be the discon tn which the axtomobile i tenvaing. We bate » oe table of beunday conditions. Let A mee! be the ronsant celeron. 20, we have 0 19 22 when 6 = 20, we have 2004. The automobile should maintain «comsant acceleration of 1.62 m/s, Whot comlant ongative acieleraton wll enable driver to eres Uhe speed frm 190 /he to 60k for while traveling a distance o 100 meter? atthe divert treveling casts Lt s mare he iatance sath ofthe eign aconda air becking begin, land let vanter/sec be the velocity at thi tne. sy ma/ste i is stating velocity and » msc! is the constant (eesertion, Asin Esercive 32, we have anvfe isan fusca [ow te nan hs nw Ths, 196 ‘THE! DEFINITE INTEGRAL AND INTEGRATION ‘Because 60 km/hr = $2mee. we ate given that » = $2 when 24200) = (FFF? ~ (PP = 25? ~ 4 The constant accleation is ~3 m/e, 148. Consider the stating poi tthe pston ofthe car when its vlc 100 BB = 32 (see Bxercie 47). Al t sec Jet simetes be th distance of te car from the staring point, ‘avd lel m/sec be ite velocity, Let the postive direton be the diction in whieh the ca is eavelng, isthe tao of Boundary colons Tse ste te asd » ter in te distoace ofthe ca fret the starting point when Uhecar comes toa stop, The accleration x ~ 8 m/sec [een tec, Reames =2P a 2, he G3 Theo, ie dea ae BB [arm [i-se Meg = APH P+ Cy Sees = whee t= 6 C= 0 Tae sonar 22 ncn ie We bave 02-8849 t= waa (0) Thenfore it wil ake 247 we fo he ct to ne a ep tn (2) tet "Thon s = 425)? +22 = RE 3 48.22 (0) Hee the ear wil tne! 4822 m before sopping. He oa fats eg tg = 2H 4t= -2 2+ Fl SL. From 2es = 2 — uy} we bave 2(-8)25 = 02 042, vg? = 400, rp = 20. Because ne = 20 Be 260025 LE ~ rH, he ret di 20 oe, 2 n/n 62. A Mook of ex sider dows hate with conan acelertion of 3/2 ‘The ebue i 36m ong and ‘NEPis hee to tech te bottom (0) What the nia selocy athe te? (9) Wat fs be sed ff fetter hastened tm) How og dort tab he st othe 2 > hes tn, stn ee date te ck cides down the cho an et mine be the ee of the ‘ect he scerton 3 fs, hen ies to foaeneg, “ Haxic, 298. The ball Hane $i Jae). Jecrepsepasouse, Breause 6=0 wien t=, thea Cy =0, Furthermore, when (= 4 then # =: 36, Subotituting thee values Bg, (2h we get wameecy — C=3 we replace the values of Cand Cin Equations (1) and (2), we have vaaee3 safe toe (3) Because » = 3-vhen (=O the initial wloct is 3 m/see (To tnd the time it tae fer the ice to travel 12, we let ¢=12 in Ba, (4) and solve for ¢, Thus, aeyesae d= Faas al 2Ne+4) ‘The only postive root is2, cit takes the ce 2 sec to go 12m. (G) Substituting #= 2 in Bq (0), we get v= 9,20 the sped ofthe ce i 9 m/sec after St has traveled 12m 443 DIFFERENTIAL EQUATIONS AND RECTILINEAR NOTION 297 Exencies 88 and 3, nd the othgonal trajectory the family of cue, hats another fan of curves seh SL any pent (62) thve nour ofeach amy thcogh it sad the the tangent Test he to curve st poi ae perpadiai- “Te amy of parabons 2° = any, 2 ‘an ais, the slope of the tangent line tothe parabola ofthe given family through P is 2F w shat the dope of the tangent thr cre of he fay wens through Prot fe Thus an ftv dy =a de; pel ve “* 246; 40y? 6, a family of lige ‘Thefamily 2 +9? 4 dept ody fv $a gh 5M 2% wence a ay pin 2) 0 om 2 Pan fa Hetero the tn inc f= Be jac fytin= [Pn de Giy—2= Cry, a family of hyperbole, AREA Dano fm ad ce integes wih m $m a Fs. fonction dtd forall ingen hen Erp 86m) + Hemet) HH 42) 40-4 Fle N+) “Tae gical proper sven bythe allowing heen should emoritd 446 Theoren — SP) F-1)]=Fln)-FO) a AAT Theorem If is 2 positive integer hen fb (ormua acetiae+) es opie (Gorman) nin + 1)(Q0-+ 1)n? 430 = sot Niet (oven 9 Kote, EG -mra orgies tle paras genrananra Base ‘Defaition Suppose thet the fastionf ie caninoas an the cloed interval (4) with f[=) > forall =o [a and that Rls the region bounded bythe curve y = (2) te £ ax sod the line 2 =a Sed £8, Disde the interval a. into subintervale rachof lengths Az = ()—a)/n. ad lt bea number im the ih subinterval lz; 7, Then the mesure of the area of the region Te siren by 3 wi Sos where f(¢) nthe aide of the ith rectangle Inthe mathod of sserbed rectangles, f(e) isthe “taolue iinimam fonction value on the ith subinteval, andj the method of arewmscribed ‘restangs f(c) ie the abvolte maximum fonction value’ oz the ith stbinteval- elu, the Tint the ante no matter whie cis the Hh terval 4m Becreues 1-20, fod the um, Use Theorems 442-047 when approprte. LE G@-a-3 8 i $1361 62-e-ten € Eoronsfe PU pre mogteur a £ e+, bee dae Eee A replace sucmsivey with 2, Tand a Thos Eesot csr ye orwrecernrsorrrerire cen? 9436-4594 66= 293 8 Eanore Senne be 6 Beene Be fw so eas Srboiedotet ebaty o Wit ce rcosive wih 3,45, 8 and al Ths Esta-atatata' ata taeo bb eek E aS sre PedePe@alelersotiss <9 we Boge be hed Bee Bios BME 820 28-0, ue Boutin Bor sey naka sok - 20) 40.22 - 109.00 as B tity cor 8 yecing Thom 146 wih FO) we Zest -10}) > We amiy Theorem 446 with F(3)= Sav — 10) = 10-10 st Elbe rhal=-Schr-Hl= fbr H-dM ine roe wt ra) Thaw, AAAREA 20 Biers ps2 Zee pasty sine) Esia+?) = Leer Fhe snertn en . 7 . Seu-a= Eur-ay=1Fe-0 F Herth g Meaney = Jon +1)an(n +1) ~ tn + 1) = fla + DRA S0~4) = nt In? fa Eyot-sty-att ry Breas (8-34? Suerte sao rence 1-20, es te method of hinseton tof the are, A sguare ants ofthe repens oe inside cided rcteagles an insted, For cach exereae draw fiat showing the gion and the Rh eta. Ta ‘2-28 ee Jon) a the neauce ofthe abtade of te ih eetatle, mhee m =H, +a) i he inf the th subir Erevan 2,22 and te egion bounded by 9 = 24. the x as and the ne Tmsiigd cectangles Let f(2) == Toe ced ater (0,2 vided inom pbinerae cach of lng Bends The ss sibnteral (ys); where f= 1.2, Boose sinceaing on (0,2) the sbolc ‘aati vale off 00 (2-yo2) Mya Slee) ~ (ei)? = OE NBA = = ae? = ae ‘we may apply Theorem 4.46 with P(E) = (3*— 3-47. Thos, An tin, E senar= lin £, + Thhearen of the repoa i$ aquare waits. pe mangle eto) = 2 Telok nied tat nitrate ‘Th Bntae Jin increasing o [0,2 the abyoite Sle) = (2)? =(a2? =H a2)*=4 A MANOA an Grados + There of thereon ef ase it ep fo) he atin. an ith r= m= Heit) = A= tm Exempae fg Heyes gn aft2toso=§ 300. THE DEFINITE INTEGRAL AND INTEGRATION tn Bxeste 29, 2 and 35, the eon above the = axis ad othe ight the ine x= 1 bounded by the Geter ent; tel Ge curyy ane? Lak (GQ) 242° Th ced incl [1,2 side nto msubierva of leapt ‘Bow dThe th ebinteral a gga where 29, inscibed rectangles Bese & dazeasing ow the bole minis value AF om etd He). fis) = F088) = A(t am tip, £ sevare vin, £6 wut a bd, = ino-(+D)-Ke ‘The aren ofthe region is square units. 2, cicansrbednectagie, Let f(z) =4~- 2. Beete J is dereasng 00 18 ard Sop Sea= 1046 asda Aste) = ~(4é A= ln Eirepae alin, EG atk S01) on [BA xa ian 9) yes sipds—(C+.)2-Kered) ed] “Therfre he aen of therein i uate wit 35. midpiee m= He, +4,,) =$0-+4 Aa) (14 (—1)82)]=hle+ (2) =e BEL fim) =4—(4 2) tig Go Astin Emacs tin So oa 1,31] = 3-Day dy MIAN 3 MOD] ~(1edjed-Yirdfe-b)-¥l+4)-s, ote) b- feeder deed) Er eeatedea cick Oo ke vanunnr winners Sree terns 1%, circumscribed rectangles. Let (2) =4—2. We wish to find the area of the region bounded by the Perrier ae nt Pe a es at Iida lial rg ier Sone terre gore soem rcest renee n meee erga cs lea LES Soper bets : lg recone de etme erie 44 AREA 301 ~ pig [IG-MOEDg, MOEA) i foi 8) sft Be Dee at Ay aunt wits be the are of the region unde bythe curve y = J(2). the axis they ais, snd the line #1U1 The aed intra 1] ve ito m subintervals cach of length Ae =f The th subatenal wie) en ‘ecae Jie decowing on (03), tb aboute maxinum vate of J on Heea)= 10+ 6 Has) —4-[a-vhf tesa tm, [CSP YA ue = in] Yate ne 4 Hod) ATA, 2A,= 141} <9. Thas the requ ave ie 9 square units Inscibed rectangles Because fle) = 12° is not momtonic on the intaval [-2,1} we divide the region into two paris Le Ay square unite be the aren che eg Ry fo the eft of the y is bounded ly the eaves the acy and the yas, The cowed interval |-2,0] is divided into subiniervaly, staring from the origin. Thus As, Decase fs increasing on [-2,0} the absolve minimum value of fon sical He) where flag =4~ (8 4 dal? a t-(-8) AE Ami, Speoten tn, SA ste lb Es-8 F4] = ti [p-.-g.meeuee lim [5-0 +8)2+8]]= lat Ay muare units be the area ofthe rpon bounded bythe cure, the = ax they ax andthe line # = 1, ‘The clowed interval (0,1) is divided into n subintervala Thus Az = i/mand 2, ide. Because fis decreasing 01} the anaes vale of fa («8 Fe) wore Hep ‘The number of squire units inthe are ofthe region is A, +A, = $84 N= 9. ‘midpoint. La f(2) 4-22. ‘The closed Interval [-2,1] fs divided into m subintervals cach of leapt ee Bn, The ith eubisterval i [5-+2,]) where fm 1,23,0m Let the altitude of the th rectangle be fir) where m; = 2;—]ae=-2+ 81-3 fim) =4—(-2-B)aBif 24-24 3) +048) 1S So (t+ ahirclananyeaie As tip, E simgas= im, E(-8- Se4(+ Bhi S83 = Ha(C4-3) ae pint - By S = ua | - 8-2 +Be e+) efor] = 180-0 ‘Therfee, the aren ofthe region is 9 square units, 302 THE DEFINITE INTEGRAL AND INTEGRATION In Bxercins 27 and 2f, the region is hoonded by y= 2 the = axa, andthe lines = 1 and 2 =2- {Br inmoribed rectangles, Let fe) = 2 Hocane f(2) ronpositive on [10] art - Sonueativeon [02] we conser the two separate intervals (~1,0] and (0,2) Eat A, aqure ite he the aren of the region bounded by the cuve 9 = f(=), the Iie v= and thes aki: The close! interval [-1.0 fubintervae cach of length = f The th subintreal i PTGS The foetion f ie fnreasing on [-1,(, and in order tohave inserted vectnnlen we take ~ f(z) a8 the ntsnde ofthe th rectangle (ni? wide iia = = sla) = HA +18) lim, BS, -s0a2 A = ip WB, o-)= tp eee Lat Ay wun art the en of the aon Boal by the ere y=) te ln = = and the = Wee SS oY SUAS i eaters of kngth z= The th einral sere ee athe neon fs ncening on [0] abd in ne eh acted estar we Fa) mealies teth etn $e) = 10+ (1932) ip, Bt ~ sep lee) | 25 Then the reid aon nt snare wits 28, circumscribed rectangle. D Let fla) 2°, See the Sigurt. Because [(2) i noapostive on [2,0] and nonnegative ow [2 we these invervals separately: Let Ay aquate units be the azesof the een BR, bounded by the ee rls and the ssi, The'coved interval [1,0] i divided inte wbinterale, carting from the ‘Pour Ae fm and == aSs. ffs docreming on [1,0] and in onder to hove creumseribedrectanges take =f) ab the at < Let Ay sqaate unite be the are ofthe spion Rg handed by the curve y = 2% te Tine = = 2 and the = ‘The daecs inves [0,2] 8 divided intern subinicrvoln, Thus Az = 2/n ont 2, — las. fis increasing on [0 ‘and Sn order to lnve crcanueribed rectangles we take f(z) a» the altitude of the ith rectangle, where (ash =) 85 In Bxercies 29 and 30, the negon is bounded by ‘Greamacited eetanglee Let (2) = 2° +. Deeaase f(s noaponitive lind onnegatve on [0.1] we consider the to separate intervals [~2, and [0 ‘See te figze, Let Ay square unis be tb aren of the zen bousded by the curve jon 221 and the zante The cowed interval [-2,0] i dived "The ih vubintreal [ejay where "a i2.8,_ The faction fis increasing on [20], arin der have ‘Sreannesibcdsetaugle we take ~ f(y) athe altitude of the ith rectangle fle a)= fea li ids) = (24608) (208) AA AREA $03 (tis) in. (8 (nsw + E (o41-v)= ow (FF O44 F 3) Pos? 4, 4226 Let Ay squaze units be the dre of the region bouaded By the curve y= f(z), the line z= 1 and the = axis ‘Te chee interval [0-1] divided inte'm subinterenls ath of length Ar =h, The ith subintervals (2, 82 swhere F123. The function fie increasing on {0} and in erder wo have czeumscribed rectangles me ake f(z) the stiade of the Hh rectangle. Heh= 104330) = ok Ag = O+E Ths the require area iE squace unit, = 4m Beemae 0) ie nonpritive on [-2,0} and sonnegative on (0.11 we teva [-20) and [Set the ge above Let Ay square unio bythe ata of the eon bounded ly the cave p= Y(a), the Ine 21 and the = aus, The cowed interval (-20] {Geico iso » intervals cach of engin AB The th sublateral is =, or] where f= 1.2.3.7 The fanction f is increasing on |~2.0), and ia order to have inscribed rectanghs we take ~f(#;) as the altitude of teat neange 7 = se) =-4-2 +100) = (248) (4 te ‘The thec inert (i Avid inte Wainer (2), the line 2 =1 and the = axis oflength ie = hehe dh mabiterval by 353 wher F= 1,231 The fonction fis inerssing on (01) are in erdr t have iseribed rectangles we take etet “f(aj1) asthe abide ofthe sth setae. fs, Ao tin, © steuadda= in, 5, * Pat 1,3 M024) De ae (ved) of iad PE po AED (08) hq = 643. Thus the reqited aren is 27 squate units, Be nsec oreo pres el vie uma Seen epee ne tiren Seen earn cenre| the ah rian FH, —84h- A= tip S]o)—2e,sy)se= ip Sikes hgh.2025)] | Wb, +h) 306 THE DEFINITE INTEGRAL AND INTEGRATION 32. "The graph of y= Ale] and the = ass from scctin to Sed to as of this Glnagle » Asketch of the regions shown at he right, We utile synmety Let Ay gaze wnita be the avea of the part f the eegion to the lft ofthe x axe ‘The dosed intereal [0] is divided into subintervals of lengta Az= 4/ starting from Om thatja,|=-4i/m. The bight of th ith seit rectangle in =[si=4—% s-tala4$ Let Ay square units be the area ofthe putt ofthe region tothe right of the = divided into n subintervals of lengh == 4/s starting form 0 so tha agin | invcrbed retange fe4[ey|=4- fant a0 Ap=Ay=8. Thetfore A= A, © Ay vad) where 1=1,2decn. Let the alade of the ith rectangle bem) Weta oP=ahis Rev) = tai = 2 IA syare uitsis the area of the reios bout by the cave y Am tip, EE flmpdem tim, SS Iy(t ais 1h tie, ( fue —a4) a) te ais and theline 23, 2 nn 192041) a7 nln) a2 = eae ate)? 1 The aes ofthe region 9 aquare unite, In Bvercies 37-42, a function f and mumbessn, a, and bare given. Approximate the area, A square units, of ‘egn bounded by the cue y = (2), the 2 axis and the ing 2 =# and # = Phy deing the following: Dive Interval c,5] nto m subintervals of eqaal lesgth Az unles and use « caleclator vo compute to four decimal pl ‘he rm ofthe ares of m iesctibed or ceeunseribed (as indicated) eetanges each having a width of Az unit 37 se) =. The interval [89] divided inte 10 abner of length AGL = 0.2. We wnh te vein i decremsingin [18 the abeoate minimum vate of on 12) AD [UL2)+ M10) + 14.8)+ 18) + 129) J2)+ 124) + $28) + 128) + 7]02 = (het thst detest dts tale tive = Lao sare)= Jp 0-1 subiaterval of length jh Beene J is deteasing on [1,2], the absolute maximum value off on fa 2]i8 Hed 1-449) = (Wf) cf +(Q) +) + CR) +40 +BY +) + (80 + + GRY + 2) + sna ches hatha hae cht ght ghee ghee ghe th} = sma =2, 222, dreunseibed rectangle, The cles terval [1,2] i divided inte 44 AREA 908 In Brecises 39-2, pu your ealedator ito radian mode f{e) = sins. Theinteral gr] divided into 8 aubitervaa of lmgth 33m -f Ihe We wi ue Grcumacribed rectangles. In [Jx,41], sin is increasing so is absolute maximum valve is at the right end of -sabinterval: in [Jr $x], sin zis deceasing oo its absilute maximum value isat the left end of a subinterval A fin jen pein prin fersin tain he +i rnin ye = 18500 fielmeons, a0, bale, a6, lseribed retanels, ‘The clad terval 0.x} it divided ino 6 -rbintcrvals of length yr. Because f is decreasing on (0, |r}, the absolute minizmam value of f on ey..#4) 8 fe) = covs, = cod fr) (> ewe ye con ir Ho fet nie te ie tee - 98004 $e) = sin =. The itera fede] divided ito 8 suicterva of lngth fe — fe] = inverted rectangles. fs [Jz fel, an i increasing oo ie absolute minimum value i at the let end of a bine in ef] in +n decesing 0 Satter vie at terigt end ofa sutaera A> (in Be tin sin or in a in d tain an rain Bebe x ns see) bade ‘ebinterval of length dye. Because f is decreasing cn [0,4r], the absclute maximum value of f on fg F(a.) = 08 2. =e) : A. <[eoe 0c fyr co fr coe je se fy sco ye] = 1280 Wench toprove that S ire) reua= 2 a+ ¥ c(9. sow Emo s00a1= 0) 90) +04 60 +) +60) +--+) +60 By bylying the succstive ad coounutative las of ation tothe ight side of the above equation we get E Fe +600) = 18a) +10) +800) +--+ F(m} +60) + G(2) 46¢0) #1 +6(m) E roroo= ¥r0+ £ oo Prove Theorem 44.5 By Definition 4-41 ELFE-D = Mle te) 94 -F H@FA-D =F) + FPO = ERO, ireamscibedrectarges. The dowd interval (,5e] is divided into 6 F ris =Mle-dede-+hb-o+ed= Ree FO= Lew) "Thus both parts of che theorem are proved. Eiste2eet (ea ta ant Siento-n4 4241 ‘Rddieg the above equations, we obtain (osama geod (net sins arin) (m4) £06 THe DetINrTe INERGRAE AND WWreoRATION ey f ane bien) & Wewithiopme $5 a Met Nen ty) == 17941 —$ Uieg Those 4A46 (elecopig Pon teat erm and Forms ton the sca tr we E60 e Sing Bye MOE pe tne? sage ay = |nle(et—1)4.n+ 1) 4. werthtopmre Beat doe + 12(n—1) £8) = Palm ean) Beawse (¢1)¢ 42462441 [¢-- 146 —aint] Using Theorem 4 sees is elie] r= (1) 227-28 41g, ‘he fat sm, and ormulas 2 and ton suze tere we have Beeb Somer Baa Ber Sip hs Plat P-2 bain em 1s dolos 1) = bist) +n)? hola Nes etnies) dela 16ln— 192 41) + Helm 1) 10020 +1) 415) = pln + (60+ 90? +01) = han 1) + 1Y8n? «3a 1) 4STHE DEFINITE INTEGRAL. 4 Riemann sem fer the function fon the coed intereal [.] is any sue ofthe form Dae The ost A feqsprtpeatg} Is alld 8 partion of the Interval (6.8). The largest of sambes 44 Age,» Age called the norm of te partion and i epreented by 1A ‘he subintertis Bev equal length the pectiion i rela: The sit that appeaesin Det 44.8 is Riemann sum fora gular partion, Riemann sxe afproximate better when the sre chosen toe the midpoint rather than the to ight eons of the Hnervele 45.1 Definition Let fe a function whose domain includes the closed interval [eH Then fi seid to. ‘tegroe on [6,0] i Uae isa number Lsaisving the condition thay for every ¢> suis 42 Oni hat or evry partion fer wheh I< ad fc ay he Taverval fej peyh = 2 [Epteoae-tec gb Eftedde=L 45 TILE DEFINITE INTEGRAL 307 452 Definition If fs «fenction defined os the closed interval!) then the definite ategral of f(itegtend) rom a (lower lit) t9 8 (upper limit), [24(2) ds, given by [ite jin, Eres A eit Fe pion rp, have [Cresson tim, Eseae 45.3 Theorem Ifa fasction is continuous on the closed interval (8, then its iteprabe on fa ‘Thefoloring extension of Theorem 469 ia omesnes wel Theorem if non sed sod boned be Conc trv oly then ota on [8 ‘sod ony it actos tere exp for aot pits whch, fr any¢ > 0 estan in Soe ire tera of te ey of ng the ASA There Lt the fnction be conn om [8 ad f()>0 for a =i a8. Let R be the recon Sune by the curve y= fs) he satan the ines ® eae 28. Then he mate ofthe arn the ein Fes pes by Aa tin, Sviwlaz o> die, Shea Xie condn (2)20 nso sind, me eae fe) mh 7) Definition a> 4 tben [s) de Defaition If (0) exits then ["s(=) de = 0. 4.5.9 Theorem If kis any constant, then [fete = 40-0) 1 Thre If te fon f andy se inepble oo a ee 93 negate om an [ister +ateriae = [rede [ated ae lid ng i sh ae Free [ners ital oil ind nr recs 4-8, Gd the Rien su for the fnetion om the inrval, sing che patton Aad de ven rt ty Shekh ie graph and show the eiengey se tssre of whan tet ate toe ten of bens Jie) =2, 05253. 3 Soper = Ho)dret He Soe+ HoOae+ flee CN 5-0) + L255) + 10.9)228-128)+ 2819-229) Fab eI 4 181 BCT Fo), 0< 7 £8. B Hope Hm\aye HeadBaes Moder + Moe e4 Seer 5 SP (18—0) 417 (1.25 ~ 18) £1.75 (2— 1.25) 42.259 (279 —2) 4 275708 = 275) F=hi sess E epae= Mw) sede Snare Medde (1.61 =1)4 (225-167) + gh (287-295) thy x20) = 1.1 619 {THE DEFINITE INTEGRAL AND INTEGRATION phy -isesti ey =~. Hy #0, Hy = 035, = © Breas = son) ye + Sea) + Hes)aae + Mug det + ea)ge + Teer + Serine + Fone phage (025 +1) phagr(0 025) + ggg 5-04 phy U28 05) + glean (225-2) + peg x (218-228) tg px(8 235) = Loss in OSaee F flue SwpOye+ foyldat aM upbase + Seder + Ma)ase sin jx (Gr 0) sn feeb Jo) tan fre |e) rain Geax Ge —Fo tate fale J) 237 © fe) Beane, cree £ fle)de= Heart feedBae + Nels + ede + Hse say 20125, afeiteonbesie oon —Be) x(x +2) + col fa) x (hr tbe) + os Dx(de the) 008 FG =13.6008, In Exercises 7-10, approximate the value ofthe definite integral two ways (a) tie a calenlatr to compate to ‘decimal paces the eonespording Riemann sur witha repslar partion of n subintervals and w, as the ‘Sapo f each subaterval (0) tr the NINT capaity of your calla 1 Ke) =p m Arabs? =}, (0) Ji Herter 41) 41+ 10 + 108) 4 10+ 108) 4 199 41 “a LBs Bet e+ hat y+ oe + Be tah 0.267% (6) 0.000, 8, [{E4s, m= 10,05 te eft endpoint of) ara Sgtaos ['Lden[bedrtaly eats ts tet drt dat talon) = om (b) Using NINT, we Hl the vale o be 0.287 EG 8 Babe) [2% seen fee) tafe ate che ect rae sie r= ta 2 ta, f(a) tan dep = 6 ute rg endplae (a) dx = EGE [recs tem tae a csc tee ten ete He + we Bebe ~ 0m 0 0.767 tp Facts 11-26, determine the xa: va ofthe ate atgtal by interpting i the meat of te st plane rein: Cock ning NINE a 1a, [e-em ac 089) — HO —2 ein the Ha end 9. (a) n=8, mete le endpint. Ae 2 Pere > aiden en ce ut hil anti 3-2) = Suerte [itetame TS 13, [2 Vi F ae = retin ondrat pact of eee centered at (0,0) fav 2) = fe C2F 16, [1 /i6=7 ae = arcafunpr half ie ered (0.0) of aud) = f(a)? = 8 445 THE DEFINITE INTEGRAL 309 = de = araltiangle (0,0), (4.0) (44) ane rianle (0.0), (20) Nas to2= 2. Tren (0) =3, 1(3)=0 and $4) the integral ithe monrae ofthe aren ofthe rangle (0) (0:2), (2.0) above the = ane mitus Un meanre ofthe area ofthe tangle (3,0), (4.0), (4,1) bw the = ati, Therefore o—2) =4ays-Jay=3-]=4 (520) aatapezié (1.0 * (an 5)as = asea(teapesoid (1,0) ( 1. Hence the valee of Ta (s2)(80)) =H #192 (= 2,9) 2.0) = HS +942 (1) 2)) + areatriansle (0,0), (40), (449) = HOye-+ Ka = he = area trangl (0,0), (2,0), Piiaalae at f(@) =11~ 2h Then f(-3)= 4, {(8) =0 ane #(2)=2. Hence the vat ofthe tegral nthe measure ofthe aves of tangle (1.0), (-3,0), (~8.4) plus the ttmase ofthe aren of tingle (1.0), (0) (32) Taereore [yimzke=Hon+ 402 =10 [fte-nae (triangle (2,0), (5,0), (5,8)) ~arealiriangie (2,0), (0,9), (0,-2)) = Ha) —42)2 =} 412) f()=5-% * Ga —21-8)de = seeo(sinngle (8.0), (10), (1)2))—area(tiangle (5,0), (2.~3). (~1.0)) ane) Ye Lat fle) =6-[x~2) Then (9) 6-24, #2) =6-0=6, (8) =6-6=0. (te — 2am atearapenid (0.0), (4s (6) (2,0) +aatinee (2.6), (0) (09) =A +02) Hoe = P astesanie Let fle) = S4le al M8) m3 +1 4, A= 340 B4)=34401 Hence te vue ote atepal te mame oft are of bape (5,0), (558) (243) (210) sheet mesie ofthe arn af merida), (4 (0.7), (0,0). Therefore slot siie= y+ 3K0 ree Because y? = 22 2? = 1~(2— 2 et View weaspaleh ocelot age) =a Because 9? = 5442-27 = 9 (2? — 444) = 9-(2—2)4 (2-2) +9? * JEETE—T de = naleumicins centred st (2,0) rave 3) = Jo(3)? = fe 50 esis Sa ng Mth wos W Gab toa abe Be Paine ae Becaoae each regi above thes axis ie congruent to one below, the integral. 300 THE DEFINITE INTEGRAL AND INTEGRATION In Bxerises 2 and 30, aply ‘Theorem 4.59 to determine th exact vale ofthe itera mofiecenen ew [irenetaew [eat eay 38 (a) [ede me(1-sy 26 0) |? Voom Viie-Camtvs (9 [tem 1-2) =0 {In Braries 31-12, ue th rete bow 1020 fd the vl of theft. Check by NINT. Mises, [rdea} [ eeae=a, fewsae=o, [ase |p au [Peetaessyena[’ sternal” edess|" cen 2a) —aese-(-a)=6-6418 218 Ey {Oe a 0 [eee [scenes odes [ade (Phcorem 4 (Theorem: (rcotem 4.59 and giver a [*(o-se+pteee |” {Get -4e—1e e417 = -408)- 4-1-0 a. * Oat e- Dae Bo fede = [see eo | (Theorem (Theorem 4 (gives vals and a1a)-129)1=9 4pfige- nf Sffaseses [sex (2) 490) le—O ae vo [tease na[ewtetent[ (te dense [Sate te]ote ble 0 [eos v4tte= [ete rteees ites [nates Bones [fen [Laetetalcwzdenitsp-a-frss-onie {5TH DEFINITE INTEGRAL St £-2f'de =. [fists sas etoden ff ints axa" yenedes [ade] 40) 4408) =8 fe Exec 49-4, wee Theorem 45.12 prow [*j(a}ée= ['sie)de+ [lake (1 fer the sve ordering bce-ce. Pn Theorem 45.12 4 cob Prom Thewem 45.12 [oem tee |e [freee [reer fone opin De ie Apolying Deion 455; “Poroies [row free rose [ieee with is oqunaet to (1). ‘ahh ie equivallat (1 be.ca. Fram Theorem 45.12 46 c ‘Because f is continsous on [—1.2), then f is: rable on {1,2 Thus, [reas [tres [neers reyes | aac (tire 4511) reece [reper (tienen 451 Hadae (Theorem 45.11) (Desnition 4.5.6) {2 "THE DEFINITE INTEGRAL AND INTEGRATION 18. eause fis contianous on [= then fm Teste 4.53, fi nterable on (3 {EIT no the Definition 458. [Preies Prieee+ |" states [Maen (pears [[rtee) (J Psenes [Trere) = fi nodes [2 rote [tenes [re A. From Theorem 543, 5 kftwlaye =h 3: s(w)Ae Therefore Flyp FE feoae=t, BBE Medae Cb ). ‘by Definition 4.56, ji des) {tabby Dean 488 and Thane 652, [Has == [Fede = -Me-¥) =H) We FT pe oo (eas Rem, ewe [sles [sete=tan la t= Ete tay Heme sxttien $= feeb imme vt [fede = ti $+ (Stee Jeane Then $= Be)he=T ado = IT = (9 Sune f aod ten Thm $= fla) rT dsl + ig (7478 {6 THE MEAN-VALUS THEOREM FOR INTEGRALS A MPANawen irae ncn fan pare inp othe cna terval ol andi J) > ff 2 ila) en [jreraee [laayar 1 noi, fad fe tie a ot eta na. : oroe> row 4162 Thecem Soppore That the fan fis costnsoas on the dod Interval a Mm ont M swerve the abc cima and imal maxima fanon valu of fon thm im 2? 121 @ be the requited syzabal. Phe fllowlag inequalities are equivalent: VO—# © VE=H, 6-2 @ 2—2, $0 2,4 @ = Bees {Con the interval [45], then @ i fall = f= Finegan |) Bene = seit} then 1 ¢ fle) Breas 1—(-1) =2, by Theorem 46.2, 2-1)=-2¢ |" hyde <2-h 1, tt (0) =2E$ 14 hy Beene J deceatng om [0.1 then ‘he abate minima Sod maimem values of fo [1 ease as [3 Io. i fe) ~AeaePe Won, thn 1) = 1B cate sn + Osha 2= I2sn oer?) 2 00 fad M= JQ) Oare ie aude minim ad msm vals fom (fc Be by Peron 442 webave old) = fs [7c she 0 =0= 1, by Theorem 4.62 wehave an, [aide de > Bice si: fs an od function nd he Snteval [fees ymmtrial the vate of the itera i ‘Theorem 43.0. In Faercies 21-90, find to the sear onehundedth the vale of essing he sene-vetue theorem cg: ae MNT wo proxi vl of hse fo gis a E {#8 de-=5) By the mean-value theorem for integrals thee exits a umber ¢ in [0.2] such that [j Pereeen ga ore 2 4VE JV5 = 1.1547 = 115 isin [0,2 22, [sue =f. By the meant hore fr Intel hei amber in [24 oh at af faite mp MB eg. ah = BV SV AT #2008 806 i a 2. By the mean-alue theorem for integuals there exists a nuinter« [2] mech thot Ym 1.458 51.38 AG THE MEAN-VALUE THEOREM FOR INTEGRALS 315 fe-ve | Wea (0) = P= and fb] =|86) We wa oad munber¢ with 0g 6 < ath tha | fpe-nee=s000) [o} Beau the value ofthe deft integra in a. (1) 6 8,2. (1 i equivaleat to 2-0 Men ea §Yias ({tvseeste= 0 By eter hee rea he aera (ih at [ietesentyen ere r aye 08 8 JT; 2+ JIT ~ 25 in in [1,4 heer fringes there xine» mer in [04] sek tha (P+ 4c +5) A Hee P+ DI P= 0: pears Since 1.2 enti he interval (-2.), th ony tale alan af = [fig des o4ter By the mens-value theorem fr integra hee exists «number I ais £2,657; 2.457 inin (24) BS aval an ae 0.2027 : gs t= (mn tre = a 74; 588 = 086 setae ‘Because the value ofthe given integral is ~0.5493, we wish to find a number ¢ with 3 <.e°< , such that 0540 = Footer tine =~ gfigy = 8.719 estan"\(-8.719) 445 1744 74 ee 3240, te the mon salue there for integra to prove the inequaty Dy he eal thn er neg tre cia ume in 0. i [° Bat gh [0.2]. Therefore L pigeshick ‘By the mean-value theovein for integrals there exists « number ¢ in (0,2) suck that f. pig ayo Bat ghz St beans 20. There | bdr < a murmter ei (fe) such hat lcm see se ni oe aera because By the mean value theovm for integra ther a Jo, cones = (od pat cnet 316 (IKE: DEFINITE INTEGRAL AND INTEGRATION x [fieyeuss © en eth nt wer yh mane a rea he iia ran sin 2 a = (6a fOr esuse Sin 1< 1 fr al fen sin $1. Theron, [ins vearee 1. By the eansave theorem frites there exits a umber cin [25] ch hat sudsabi sayy 5 {8 By the meanvralue theorem for integias there isa muruber ei [5,0] such that Sigea* =z pa os hg s gate bal Tans [ightes? 18, By the neamralue theorem foriabepala there exists a number ¢ i [,2}such shat itis in 2.5}. Thereore 0S J she Sb [famyeete= tit postin tate utah tte [on brede st og [Marcas pcan cod 2/2 cntinunon om the cloned interval. by the mene-vaae oem fortes hee amber in (-}3), such that [if coer ao tok ‘Becmuce resin the interval (-}x,fz), then 0 O18 (02) cost Ye/A = 048 tn the figure at the right we show the gap ofthe fnction J, The average vl of th fonon the J A ae a fs much thatthe area ofthe rcangle the same the ars ofthe rion Sete oe at fea e he faction fhe value at whe the average value out in thes enonnae point at wich the rectangle interact the cure 6 THE MEAN-YALUE THEOREM FOR INTEGRALS 317 J he pmitive diresion i dowaward, $F 325 [av = 12 dy w= 32¢4C 3 Became w =O when t=O, then C= 0. Ths v ad8u tet HQ =. La AN. be the average value of fo 002) Thee Av afffsteaetelt a “Obcerve from the figure that Jf ais the arn therein eclsd by the triangle. ‘Thus jee =n E2zses He) = VAG. The graph of fs the par in te firs quatant the ctcle of radius centered at 0. Therefowe [sds = eT} = 2x, the measure ofthe area of the uarercinde Mase th avenge leo 08. 7]8 phy | Vda de = bo ~ Fe Fe) = VIS=H, Te graph off 8 te upper Sec of alin and enter the igi Hence [stots isthe measure ofthe arena the cemirle. Therefore [Vie F tea ee ‘Ti te average valu of f on [ns zh [* Vig=F ae] Sopra fie on 7, eae et fk aa 4 ‘By Definition 1.6.4 we are given that Ma) d= 22. Thos, fh, fle) dz: siren find [24 f(z) de. : Xia tene> 2 Mawatha me et Tra on Hs 484 ‘eg fa rs [isex [fae Pewcfize Pe te gy fine : 2) We apply Theorem 45.2". Beeatne fix integrahin ite cat af dieantimitice i contained stall set, ocaave the dicontinnites of] fare a subeet ofS, then [fs itegrable. To prove (0) eres} a ans, omg [tes [tees qe i folowsftoen the mean-value tore san pen tat [ fede =0 Bose no Senegal iat he ea mnrcneencht [! Fes oh-ab0= flea Hc Prove the seeoad mean-valve theorem for integrals IEJ ie constant, the snl i val mune not constant. Because f Inconseuous on (a) Une ext rutabere fy atd 24 i [ad] such that Horm) 0) fey) Malipying by o2)>0 we obtain Heagike) Hebe) feyoke) Therefore by Theor 452 Hon) teite< [isentee < sew) [tei Dividing by [ace > 0 vis Sten) < [‘Hemtadte | [ewe sey) | Byte inermetitealve here, thei sumer en (,) sus tha Hom [pedtorte | [Patane rete [orowtnas =| ene | s) =, hemo oben "ede 10 | det hae 450, [én mw There continent o a8 those nub ein (8 uch tat [Ye dem eK 48 "THE DEFINITE INTEGRAL AND ITEGRATION In Beets 4-88, use te scend mance Ure to prove he neat 4, Let $62) = play and 2) ~ # Thin wurter ein (4) me hat [sS-sh[i-« Beenie 0< 6 ten lg <3. Tenens [5 < [Pade 8: ta fe) = ofa ae) = 9. Te i er (1) eh i Pee-vdel Beoine eh <1 el [oI en (ss [Lee |; inde s [ade » Lets and g be the fuesios defied ty Y{a)=sinz and ole) “Thea J and gate continuous on [0x] and gz) >0 forall «in (0x). Tos there isa murber ¢ het on) [[emecensoe|ede east in) ee I [fssazer< [eae Bt Je) eae) = Ten een mein eh a aa con ede ote "cor de pine <1 frale The |" ater [nee hr te i ben 1) ch a (igspesata [ee ‘Because isin (0,1), ther cor ¢< 1, and therefore SPE -< opt < E, Thue figtte< Joee 47TH FUNDAMENTAL ‘THEOREMS OF THE CALCULUS ‘Az Fink tot the fusclon J be iateptable on the dened interal [a] ant lt x be any camber Getzed by then I(x) lacontianouron [eb]. IEF is continuous at z, then Pi differentiable at = and Pee fe) ( (ar 2.0, the derivative in (1) may be a detivative frm the ight, and if = Mvarve i (1) may bea derivative fom the ft.) Ea. (1) ean be wetten ms Ef oes in Gon 142) ak) ie = + [10 Sr Bain 2 72 Second. Ln te fnton J be tins ea tht doe intra [8] a et be » union Fundamental Theorem that “Othe Callan a(z)= fe) fossa fal The [Pivaano-a0 (22 de (2) map 9 Siva form he i ad at ie foes iretyten tatoo [fie toes =e * (aetSe—1) de 4 Jf veesarden [cee ager age f [eweriaalfviemeanabieent [eve te = 3 f° 4 te I 47 THE FUNDAMENTAL THEOREMS OF THE CALCULUS 319 devvave in (2) may bea derive fom hela.) Soles gsny be ay finlon st ian sntldetvadin off. Unuly we chow the andesite hone const tr er. We dete ite) star) by [asf or se) Furthermore if and are coatanie then [e Heal+ a xe = fre) sa] +0) sl Note the ditinetin been the indefinite inegral f(z) de, whichis defined to be a tinction p such that J(e)— (2, and he definite cara, [2J(3) ee, wich isa samber defined to be the mit of Rirmann wn. The ween fandamentaltheoren of the leas sta the! we may ue the indefinite ftexral co calelate the number repressed Any the defini nigral If we change vale in nce Integral, we mist Alo chase Ae limita, Tis, f(a) = 4, ga) =A and a8) =B, the o(s) do = de aad [frocenete) ee | soya Execses I-34, evaluate the definite integra. In Bveeses 1-6 nnd 28-04, support your maswer with NINE Pecramssatats -P 1a] =t60-940 = (72-30) ~(9-9) = 36 Jo-vsfe--3+9- Vieni ae =] Moen. 42) mf Ge] = Hea 5) = =e?) -4-v PO], 40) THE DEFINITE INTEGRAL AND INTEGRATION as [ev aap feayMaran =} e+] sera) 1G n ae os voseacearaneise a aa fie seaw= [fone nats wl =jas-044-D 3 i taten entails Whe a=0s= peed jo are = mo fo tyetde o tue VETE Then Paz 2a 4 6 de = Dede When 24, 422 when 2 Sane [fot rset 260) adr aah = i a [tjenstes [eos [fe a-a¥] sye-2"] = fo-wey-0-> a fh vareen |" dar ides | yismaees | yareaee | yemeee Val? aici) + | 06e= vicar =3vF vieiee > Bovause /F+e] is an even function, then aa]? ySrirae af? Vor E awn es oA $03") w/a ay) ~ tev} 2. teu EFI Ton [ieraverta bet = E-GeD =F me ten VFA ae dentate [0 (ery Fide= [2 oP -aptoedey=2[ (neu ate-jeee}d-deiet 447 THE FUNDAMENTAL THEOREMS OF THE CALCULUS 221 Hin a I aare- if" (+e dem “Vie Seve fee (AA tt Oar Be 16+ 192) G-243)— 9 ap, ye z ar ire Gi dem |) a+ = [ct eel Geu= 3h 407 = $0") i [ie sen re de= fein 20] = ph(o-0) =9 =| (in 20+ 30) de + -fesete + ina] 1-0 +Ja-9 [yO nae see) te coe Jo—eon0) = Hein Join 0) [th sete tem fon [vet het tan fete = 2 [an ftone? fat frat) = 2 tan? fer = J 0 =} afar et i471 ie ee ot ei fi ein 455. The af virta=—Z{" Vist a= vive Bf Vita 4-[" varia)= Vane fata afr a = spt [at 4" ese Beat cone 1) 4a fini Po it a [eo net Beate) = Lp[Prrinag [2 OTT aed = YEU) 30 YP TT ‘ad woe the chain cue: Than, ial “ages a letsatn nf [Mt hate ff tele be = ich =| ne ante

2 Tyandso f(s) <1 on 1, and (2) =4, ten ithe ste maxim value off on 1. There nd sbsolice minim of fom Lbecase lin f(2)= out fie) <}on | pznandv aint ieame i Ise cel number of in. absolute aimam value of f on beolte maximam value of f 08 1 9) The grap the funtion ix prema the sight 4 ) 2) = ~Scon x. f"Gr)~ and fr isin x fore fre the eitical number of Gin Heine = $y2e 202, Sboolite minimum value of fon Tia “2 and f(s) = -3. zl absolite maximum value of fon Tis Oand f(0) =O WIFE I= F840) S@)=55h 5 Gand fe) Oost 2V85= 3 the absolate minimum value of fon fs 0 4 ‘Because im, f(z) =, ther is no absolate maximam valu off on eo fiz)S2 en and f(0) =2, ther the absoltte maximum _( \ ‘alos of f on Tis 2. Theres no absolite minimum value of Fon I beemuse lien f(2)=0 but f(e) > Von L i 164 GENAVIOR OF FUNCTIONS AND THEIR GRAPHS, EXTREME FUNCTION VALUES, AND APPROXINATIONS 5 He) == Bay “fos: Theefore hha no sbsol:e maximum value o (a) is never 0 4 thers are no ertical aurnbes of 20 the absdte minimum value ef boa Tis 3,2) > (@) Tie em 2,8 orn ha 3g e2) —Se(~2e) 3042") ot - Bam endpeint off and im (2) = a= "Thereore g has no absolute minimum value on (a) never Uae there aze ao exical numbers of 9 and g(2) < $a L. Therefore g has 29 PIU) doce not exit and 4 Sin P(e) is never 0, 10 4 isthe critical number. 70) =8, Pa) =1, FO) = 3 P has an abolute minimam value of ton T and F(t “lin, Fe) = bot Fe) <9 for al ete FickceF has no abeoate maximum valve on 1. 3. fa) =14—27h 1= (20, +00) Wedsie 12? 201-2 S28, and 4—z*<0if2 <2 ore >2, then at isc te Wee? s@efice ieee ie) {# iracece St ile>? te tap? Because f(-2) = —A and f*5(@2) = 4, then f'(=2) isnt defined Beene §1(2)= Band 7 (2) isnot defined, Also, {"(0) =0. Thus, the ‘teal numbers of fate ~2, 0, and 2, The abnolnte minimura value of jis 0 and is occurs at » = 2 and at = = ~2. Beemuse the values of f(2) can be arbizaily Isrge there is no abwolate maximum valve of f. Thee i relative saxtmum ‘ale of 4 which geurs at the extieal number = =0. 3. $0) = VIFF l= 09): de) =e SH docs nol exe bak fig not 1G) ene 0. $Theahealte minimum vale of gon Tis 2 ad 8) Hecate ig (e) = 8 and 2) 8 forall zn hip oo aah cin ai Cat +i) tet, x. {3 feeaaleal “lim, 7(2)=1l=0 ® 5, then f not continuous at = 1. Therefore f'(-1) i aot ‘toed and thus “tie cleal me of f. The graph of fon Lin shown at the cght, The able maximum value of fon I's and it occu at 2 = —L Because 4. (4) Wolds and /z) >0 on, there no abeoite minimum vale of fon 3.1 MAXIMUM AND MINIMU3 FUNCTION VALUES 165, ego tee t= 88) . “Bim. fle) = =o there ie 1 « volte minimum valve of fon pee tunic marion niued Fen t2and f@)=57 i te)" {t HESS 4 » I re) eres: (05 2<1)+(231)-(@21)=Os2<1) + Fr bad Lasts fotontie onary ee cli ps wh bane, leven ts Be csc ee rae [ pblenn nied) Gna erey pot 0) —[z} T= (1,3) FU cow recrerearonn a oicouren wauro mt Beg Je) = or Denn Tig F10) Se) <1 eral eink eaaeeh ¢)= 22+ [22-1 1 = (1,2) Bie gap oh aan the fe ies} feta ase Gt ittee amet dan fa 2 lewhere. Tha th ete! Gikint band =. Deus line Aa) > 3}, th i abate mini value hin 1 atsolie matimun value af hor is and 42) 27 =m 8s, I= [-jrfk)~ one tan =~ O and Oita ‘es O eh ea sure, i, 9(2) = 400, 9 bas noabeolte pce bedate minimum vale oon is Land Me) ) =tan 25;1= [-Jfe] = r J He)= taal Fp, whic not dined, f 4 Beet jp Peter if) =i an Se don Ble i se miata ob. Bee eke secure anes) Se omee Haka roto daned d 2 de, feb notin the domain off. Than has ia mute Pua, J) 2 fide) = tn = V3. fc concnie tn the sheche usin value df "V5. 31-46, te abit exten ofthe fncin on the cased intr and ces by Ho fa)= 2* 82° 4 16 (0) I= [-40); (0) 1 2 fats nee Se een i cotinon on cater, fhe an sbcue mariner alae aod abit ita vile parson coke Ie) The cal outers of on (b) The ciel numberof fon{-3,] are 2 and ‘Fi-4) = 144, f(-2) = 0, (0) = 16 S(-8) = B, f(-2) =0, f@) = 6, £0) Be tose ncn van Bee sce ncn vale 10, theabece arimum vi fo 26 166 BEHAVIOR OF FUNCTIONS AND THEIR GRAPHS, EXTREME FUNCFION VALUES, AND APPROXIMATIONS 40. fe) = 24867416 (o) 1= [0/48 (W) I= (1.4) = © Fe) td Noe delat) tale be) i ‘al Becase fis continuos on each interval, f hap at abslite maxim value a and on sbcluto minum slaw on each '(2)=0 whee ==0, 2=—2, 2=2 a {@) The erieal rumba of fon (b) The eral sarabes of fon al 19,3] are Oana 2 sod? “| F(0)=16, F@2)=0, 1) (2) =0, 4) = 1k The sinlae maini valve fe Tn able nim valve oO: Ae Fp the ebcolutemnasimur value 25. the aksolute maximums value is 144, A. (0) =28nt, 1 Fon FG Because fis continuous on I, f has an absolute maximum value and an abeolute miniimem value ox I. Fp ta p= tes ted won Ct fr and fa 1) = The absolute minimus Yalu is ~2: the absolute maxinum values 2 49, #(2) = hee 2, Beemise lim, f'2)= so 0d im. ft 43. (u) =p th Becatse y » continuows on lg hes an beolate masimei velue and an absolute minimem valve ont Because y/(u) exists ererynhere on Tand g(x) is never 0, f has no critical numbers. absoluts extreme value 991. s(-1)= band g(2) =} «The absolte miaimum talue is ~1; the absolute maximum value is 44. fe) 2223 1-1-5] A plots shown af the right Spot mfets)-1 Ga Because f'(2) is defined and f‘(z) # 0 for all 2 in 1, there are no critical snumberso fin [and oo the absolute extrema off it oest atthe exd- point of [-5,2), Beene f(-8) =O a8d JG) = ~7. weconeude that = 0 inthe ataoate maxims valor of Jon Land isthe absolute minimum valee of 08 L 45. fiz) = (2+ 1), [21k Pe) =e ‘Because fis continonts on f bau an abso axiom value ad an abenute sini vue oa S'(-1) does not exist and -1 is in the domain of f: f(x) is never 0, Thas —1 is the only critical number. =I The sbclute minimus value; theabslste maximum vale is 46. g(z)=1 (2-399, 1 = [54h le) = He 3 8 ‘Becose js continuo ony fbn an cbt eit vale ada beste rabimam value 0 #'(3) does not exist and 3 the domain of f; {"(z) is never 0, Thus $ is the only critical number L(-3) = 8 £8) =$ fA) = 0 1 The sbelute minimum value is ~5; the absolute maximum value is tn Bxercies 47-52 lotto estniate the absolut, extrema ofthe feta onthe dosed interval, then us eal AL f(a) =#4524,12[-3-Ih fa) = 3845 icauce fis eoutinuoas on ,f bas an aoise maxim value and an absiute maim value 2 Pa) exists everywhere and (es never 0. Therefore, f has no etic numbers f{-3) = 18s) = 10 we alslate maim values 10. fe) The abeolute minimum waz is ~46; 3:1 MAXIMUM AND MIND{UM FUNCTION VALUES 167 ef] A plot ia shown atthe right ‘1s continvows on 7, 9 has an absolute maximem value ard solve minimum value on S)=3s? Gr 9 = 8c? 2-3) exist everorhere. (2) =O when tie! mime of Tare 8 ad 1. ‘evaluate atthe critical number and al each endpoint of I ol) =20, 8) = 5. (8) = 76 absolle minimura valve is =5; the absolute maxicaun values 76 secs = (bred f(t) ne Ita ) Seat 6, The ertieal sumber of yon tis 0. sec($r) = 4¥3, 9(0)= 2,90 ‘absolute minimum value is 2 the absolute maximem value is 2/2. cos 2; = ede] FC 2 = ne tha iif )= Sooo =—8 f(fn) = doonhs = 9 SGe) is § which occu at jr, andthe absolute inimem value of fis ~3 whieh cecursat Ju (2-1)! 44,1 = (0,2): (2) = He - 177 (1) is not defined and f/(z) is never 9. “The eritieal number of f on 1 is 1. 40) =3, 0 1 (2) brolte minimum values 3; the absolute maximam value is. y= eth 1= 10,9 © A plot shown s the igh. ‘oe t) e+ aa = i ee ee Jute minimum value on I, Because f'(z) exists everywhere on I and fa} wee 0, howe cite nabs fO = (0) =-2 *) Be eae uasentemacnariawes I-46) ee he gto ibe ied ar 8) oserinite otiactnc l Min, dpseitecanre=(egecone f2 Choose <= JL in the definition of limit, The there exist « 6> 0 such that if mumber in 1=[f,2| such thatthe sum of the number and is ey Bethe mmr. Wecesk she extreme of fie) =2-+hs ss) = 1 al that doesnot contain 0. (=)=0 ven t=, 28=1, 241, bot only 2 = 2a 8, si) +} } (a) 2 = 1 minimioes the sum (b) x=} reaximizes [-1.1] such thatthe nurnber minut sare is () maxim 2 be the number, We sek the exrerm of j(z) =2 2% '(e) = tte f moon sey itera. fe) = when 2e =, = Pel fel (6) 2= 1 minimizes the éifersee. a solv the indicaed exerci of Beercises 1. 1B. A felé of length x mx encloned with 10 mw of fence. Ta wi fs of) = 120 2)o = 1902— 24 0-2 <1 alle) = 1002 “Tho arn is grentet when = = 00 end the fl is 60 m2 60 14. A garden of length = fie gecloned with 100 fb of fence. Its width is 41009) = 50-2 and ite B Me) = (00-2 = B02), 025 £50. ae) =W— =O when S22, Aoyed, aa) ees, = 0. The area greatest when = 25 and the garden 25 8x25 8 procal i (a) minimums (6) maximum. Jp Ji continuous aad 6) exits on (0) minimum, sand fe) int JQ) =4 1(1)=0 (a) 2 =F mnasitmizes the 1, A Geld of length 2 m parallel to veri enced with 240m of feces width in 4240 —2) mand Web 0226040. oe) = 120-200 when 22120. ado o, ‘The ares is greatest wien 2= 120 and the fd i 120 ms 60 on, 36. A gerd of inglh 1 paral to a house is enclosed with 106 ft of fence is width ta aren isos) =X100~2)r = 502-2? O< 2 < 100, wx) = 1850, (100) =0, The awa i greatest whin x= 50 and the pandaa oO W420 137, When sin. squares are cut fom the comtrs of 8 in by 18 in, sheet and the ides re turned up, the 4 V(s) in® where V(z) = uh = (8~22)(35~2a)e —4z9—462"4 1000, er 0-6/1) = J - ‘on {0,7} and F(0) 0) < 506 41(-1) = A +3 = bon (O,x] and (2) = bis the abvolute minimum value. ‘the area of the largest rectangle with two vertices on the z-axis and two on the parabola y = 9— 2? gus recree centage cise fie aS a iO). ‘The two on the parabola are (+2,9- 27). We wish to maximize the area A(z) = 22(9~2*) Se — 22%; Al(x) = 18-62". A’(z) = 0 when 62? =18, V3G land 2 = ~J3 €1. ACY3) = 2V(9— 3) = 12,73, Aa) = 0 j= fs"(k — 2) = ea? 18, 0 < rch; J"(2) = be Be = a(k-B Be amici ope cate Creston ake }) =0, #'Gk) = 0 and #2) exists on [0,4]. Hence 3 is the only critical number. £0) =0, 1G) = ZR, FQ) =0 J has an absoluse maximum value at 36, greatest detrease in bleod pressure occuts mben Zt ing of the drug is taken lon tn css ee ws ate « comes thes =n 4 ty lle ts ilies en oe ot ei oe op ort (ocak ey eeepc ame is continuous on {R,R] so f has an absolute maximum value on (JR, RI. [UIE ence eee VG@R) = A(R JR? 4) = Je, VER) = &(R-$R?— AR) = Aen, V(R) =0 > 4 V hasan absolute maximum value at $8. Boer an ls tne af igen en mai ne. oi mia i EH Ba sy ates cavemen (=) = ke(144? — 23) = k(14d?s - 2°), 0 < 2< 144 S'(2) = b{144? — 32| = 34(9-487— 2) ioe cc be tessa b tal a etae es nian hia (z)=0 when 2 = 448/3 and S'(z) exists on [0,144], so 48/4 is the critical nuraber. (0) =0, S(48/5) = 06,57 k, $148) 1S hasan abcolute mesimurs valve when 2 = 48) and Vit—2 = 44/6, ‘dimensions of the strongest beam are 48/8 = 83.14 em hy 48/8 ~ 117.58 em. Six) be the stiffs when its breadth is = em. Because 9 diagonal of the bean isa diareter aE the log, te {ta} cm, und for coms postive constant ky S(e) = ke( ct 2)879,0 leach side of the equilateral triagle 2 longs then (1032) is lek forthe suare. Hence each side the square tas length 11032). ‘The number of square fet ia th ares of the ecuilateal tangle 3 VGz! and the number of square eet in the aea ofthe equae is [400 ~ 2) Henee, i Ais the coal ‘tea ofthe triangle and the square then (a) = jie? + 00-327 Because 230 and 10-3220, we find the absolte maximum and minimam values of A on the Inverval 0, if. Dierenating, we ave fvie+4a0-32)(-3) ‘We note that A") i defined for all = FAG) =, we have = hVE2—(a0~32)-3) t= 4fiz+(10~32)-8) t=4ySe-30492 = VE 4 oe = 30 9-43 _ 3010 - 4/3) “Tay oi ‘We evaiunte A atthe entical number 1.88 and at he endpoints 0 ard 4 Alo) =#$ = 6.25, (Ls) = 2.72 A) = 385 ‘the absolute minimum and maximum ofA ae 2-72 and 6.25 reopcively. Ths the combed ate ofthe figures a9 unl os posible if the sie of to triangle is exactly 440 ~ 4/3) long. Hone, the wie 3 Decat co thot the piece that is beat ito an equlatral Giangle is FAY —4/3) & 3.69 T long. And to ‘he combines area as lange as possible, al ofthe wie shosld be used forthe square 4) Hoa) = 1.88 2 ANPLIGNTIONS WVOLVING AN ABSOLUTE EXTREMUMON a CLOSED INTERWAL 0 1580 9 <4 0 and C'(z)=0 when == cb4. For what ue of F wil the asolute minum value of C occur ata umber in the open interval (0,8)? ) More generally, et J(2) = Ve" F210), 2 € (Ot), u>9>0. the absolute minimum value of / surat a nurmber in the open interval (0), show tha: av < by/ad— {) Ith atuolote minimum value of C oceee i the intenval 0,2) the the rita mamber & mut be a that terra, that ia E> 4. 2) To locate the critical number, we Sind the detvatve, sett tozero snd alive fe = Hen 0 when BE me ue eV SET RY at a ay Pat ahd aA BO- Tio aa (a? 4 22) : tat PP xa ier 0) then the ei! munber must be int, that “the absolute minimum value of f occurs inthe ntercal 4s; the postive root muse fess tad Mocha Jin nt defied at 3. ‘Thue Jie dbeontnuows 00 (8) m2, f(a) ==} a= 1,b=2. fis net defined a § Te ft decontious on (1,2) 2. j{z) =3(2-47!), 93-455 F@)=%e-47" S04) = ig, SEA te) ‘Therefore, J ip net difeeruiable at 0 fis not differentiable on (~4,5). th seo=Gsth, Gosek (2 iteed s Fe=Le tesa By Theorem 2:34, (3) =2 and J',(0)=—2. Thus {0} does not exist nd se ffs not diferaniale on the pon interval (1.3). Hanes, condition {Gof the hypottesia of the mean-vlue theorem i not satis. The graph of Pemtctng]m shown et the right Not that there eno pot on the gra of at which the tangent lin to the graph is paral to, the ine trough the serpin ofthe grep 25, f(e)=at~2e9 42242; fle) = Ao 624s} Besnue fis a polynomial tuncéon, jis cmtinuous and diferentiable everywhere; 4(0) ‘hus follestherem Holds on [0,1 Hene, tere is some number c€ (01) seb tht #6) lent one eal oor ofthe equation 4° —Ga"-+4e—I'= Oi the open interval (01), an, (2) =29422 +4; fe) =302 42 Seeause fis polynomial fonction, [8 continous and differentiable evaywhee. Suppose f(@) 70) 20 Pau Rolle theo nl on ol), Heney thre Scone number e (a) uch that 6) = 0, i312 0, Bacau this is impose, j cannot have to real ook. B, f(a) =408 +30 30-2: Je) = 2a 4 90 49 1) 2 and f(1) ~8. Beeause J is continuous on Pt] and 0 is betweet ~2 and & f(a) = ¢ for Shomer x between Oand © Hence bs coot ef the equation. Now suppos that the oqstion has toot cal Bis rot rs Then f(r) =0. Ht -< ey colder the terval [fr] ard ifr 2, consider the inter TEuak In ether case fir) = f(z) =0 aud Ralle’'s theorem helds. Thteore eve is a ntinber between Sach that Pie) 20. But 72) i always greater thaa ef ental to 8, Hence ovr assumption ends ta ‘Galradiction, There, the euston cannot have another wt. 33 ROLLE'S THEOREM AND THE MEAN-VALUE THEOREM 18% -mean-vale theoren to prove that if = >0, then cos > 1— Bs? Dove s—(1— fe) Then /'(2) = 2—sins, Become f is contioious ond dfferentibl thearem Rolls on [0,z}. Hence there isa number ¢€ (0,2) such tat Te) HO) =(2— OF) feos - (1-4) -' €>0, then sin ¢ 0, Therefere coon (fet) 0 cons 1}? con = ~(1~ Je) isan oven function the above inequality i alse tue i = <0 he mean-valiue theorem to prove that it > 0, then sin x > =~ s* f{2)=sin =~ (212), Thee f"(2)=con=— (1-42), Because fis continuons and difereniable here, the mean-value theorem holds en 0,e), Hence there fs a number e € (6.2) sach that, flz)~ [(0)=(2~0)f"e} fein 2 (2— $29] 0= eleose~(1-}-")] > 0, by Exar 28 ox ¢~(1 Je!) >0. Beeausez > 6 sinz—(2-He5)>0 peor shen Se (ovine) ins>e—{o sin 2 ~(2 19) inan odd fanston, the sboveinogoality i reversed f 2 <0. ‘ie ean-valie theotem to prove Rezaoul oltre. Inoqualty: > 0 and +> 1, where Fi rational, chen tre). Then s'(2)= (12) r= r[llb2y— =I], Deeauwe F is continuoos and the mean-valus theerem olds on [0,2]. Hence there sa number © € (0,2) such that Fe)" = (Fe) (042) (i4r9)—daerth +091} £0, then 1 +e > 1; because r > 1 then r—1> 0. ‘Therefore (1 +e)"~? > 1, Hence (ireyndgra}oe (s'> Lars H1<. 20, then -1.¢e< 0, 0146-1 and (1-40) < Lo that arf +7"! —1] >0 and the above ality i til ve. The inequality rain tae S70, The inequality ie evened if er the momvalveUmoren to pove hat if ¢ At) where iy whee 9 ad on weeny toto he aa I a fanetion is ether increasing on an interval or decreasing on an interral then iti aid to bbe menatonicon the interval Theorem Let. the function f be continuous on the elord interval (at) and differentiable on the open interval (05) it $(e)> 0 forte in (a8), then f te increasing on [0,8] (3) 7") <0 oral = in (a) ther J i deeeasing on (ext) Note that the hypottesis of ‘Poeorem’ 9.43 unay be sauisied even when f'(a)=0 and £1) =0 or whes { is not diferentiable at a or L. By letting points where /(2) aeto ot ndefined be code of subintervals, me haves (if £2) > Oat all but Bnitely mery pois of an interval I, then Fis ineveasing on f (i) 7) <0. al but Gately mary poists ofan interval I then f is decreasing on 1 184 BERAVIOR OF FUNCTIONS AND THEIR GRAPHS, EXTREND FONCTION VALUBS, AND APPROXIMATIONS cad Tint Lt th incon fbe continous a al pine ofthe open intel (a) canting te ma fe eat for cand uppooe tt Jot tal pointe of (0,6) ep poly a (i) (2) > 0 forall values of» in some open interval having ¢ as ite right endpoint, if /"@) <0 for all vahes of x n some open interval having ¢as is left endpoint, {f tas a relative maxinum ralue at (iy F(S) <0 for all values of e in some open interval having ¢ ass right endyoim, a if fe) > 0 for all values of ¢ in some open interval having ¢ as its left endpoint, {fara relative minimum salue 2b ¢ ‘The flowing steps mate se of the Frat derivative test to skeich the graph oft function J. 1. Find cach mumber at which J) is eer aero of undefined often by factoring J “Arenage the numbers in tneasing ode. 2, Use (hese numbers to partition the nurse line into open intervals. 5, For each interval (aumbes) detenine whether (2) i podtive or negative (eer ‘undetned). (2) will be pstive on an interval if the number ef negative factor ven, and aogative ifthe suber of nopaive factors ino 4. For each interval state whetaer 7 i ineasing or decreasing. 8: For each aunts, stove whether J han a maximo, = minimam or aciher, evaluate f(z), Determine if the graph of f hae a horisontal or vertical tangent line conne, ofa verueal rymptote, In step 5 we may need to we Definition $.1-() which states that 2 =e is vertical tam to the groph of f at P=(of(6) if both J" (c) and f"y(c) ae infinite. If m some of interval containing ¢, the curve lis on one side of the neemal line at P, then the grap i fo have a casp at P, Figures 5.4.34 and $4.2 ilustrateverbea! tangents in the later Bacup. Tatermedinte-Value Theorem for Derivatives, If f'(z) existe at euch point of closed inthe closed interval [ and kis a rumber steely beeen J'(e) and (6) then there exists a number ¢ in (6) that fle) =f. We do rat nssame that fis entinunus. See Esrcive 5. Exercises 3. Ip Brereions 1-18, (a) plot the graph, and determine () the relative extrema of fc) the values of x at which telatve extrema ones, (8) the intervals on which ft Snetesing, and (e) the inirvals on which fs der Contre by eglealus. Tf(a)= ata Me) = ted Set fe) =0 ane obttn the xa number 2 fe) | He Conelusion za = Ts decensing om (205) er | o {be a telative minimum valve tee + ia incoasng oP 4-3) 2 fle) = ta? S262 fe) =6e-2 Set f(s) =0 ane obtan the critical number}. te) | re) __| eoaaon =F =| Fis dewesig on = i o f has a relative minimum value hee + Fis inceasiog on +3) $2) 2827-201 (2) =0 Ger ifenl) =o) 25-4 5.4 INCREASING AND DECREASING FUNCTIONS AND THE FIRST- DERIVATIVE TEST 185, (2)= 299244 152—5 Slt off neha the sight, We ind the derivative of and factor Ma) ~ 328184 15 = S(e— TV 5) {fle} = 0 wien o— 1 or #5, the crteal mombare of f a1 '. We comider the thre intervals #<1, 1<2-c5, and 2 >5 Ut duarmined by the eieal sume Uhest at shows Tn the fst row he table, The facor +1 ie O for = =1, negative for smaller value 2 sod postive fr lege vals of thn i own in the second row [the tate. The third tow shows Ue sxe ofthe feior 2~5. nthe throw, the sign of Js) i the tod ofthe sgn in rows 2 and 3 ‘sing Theorem 3.43 and the conelonion fr the etal number is loated a the critkal number and atone pat ofeach inte branch, ier | 0) | concion | + 7 is increasing on (25, —) + t & 0 ‘has relative maximum value 4 ena Sy ° i atv etre + | Fsteremingo 49) ‘The eonelusion far the intervals ‘ound using Theorem 2.4. (=) ® Ppa f(a P— me +e $42) = 0: ste— I)le—2) <0: 2 =O 2 = 1, 22 £2) Fig) | Concsion = T is decreasing (= Tr ris is decreasing on [1,2] Thasa relative minimum value Fis increasing ot (2,400) e+ Ns? 241) + ° ° £1) | Conelasion 7 Tis decreasing 8 (= —T 3 ° Phas. relative minienaen value + | pia tenening 0 [-1, +00) Fel eslitesest=b aD =D = c + + + = = “ = ° + F@) + 7 = 7 z fs fima fh fhee fi ee on(-2e,]] _maximira on{,8)_misimom on [5 co) has a relative minimum value 186 BEHAVIOR OF FUNCTIONS AND THEIR GRAPHS, EXTREME: FUNCTION VALUES, AND APPROXIMATIONS Set fe) =0: 2 eos be= Oe = da ot fe = ess = | 3 fe) | $2) nftas =. SHES Teresi OF [25 =F] 7 “4 relate misiwum vale /: ebee fncreasig 00 =] é sr 4 lative mation valve reese deceasag on (r2 8 f(a) =2 con3e, 2 €[-24. 24h fs) =~6 sin te —— Set f(z) =0: —6 sin 2e nes, oo a foc opierneei AMAA Roe pee WY YY pahredin | lative minimum value Ferien se corte Bade 2 Geter ccna die = |aeceasng on +38, Be patel increasing on (bx +36z,3r + en] =e 7 5 tative macimum valve il 1. The domain of fis (0,400) and f'(2) exists for all = in domain of f. ~L is notin the domain so theve are | merical numbers and no lative tions, ar 1) roy end 4 fis increasis 4 0. fe) =254 neh etn a pasa ferrets [7 3 ecaus:{'(2) > 0a8 every number in ite domain, Lis inrensing on (~2e,~2) and (2,420). ‘There are no relative extems, x Sia) =(1~ 2) +a) se) = 2 )(1 42) +31 +242? 4 Ama) +2}4)-2(14 2) +301 e)) = (1-2) +2241) a ae yil=wene et : Te} _| Concaion | K | {Liiesingon so | LI cetnerie ae [~ fs decreasing on 1] ‘fas aeative mim vale Fis'ncesing [1400) 8.4 INCREASING AND DECREASING FUNCTIONS AND THE EIRST- fe) = (2422-0? “A plot of the graph is shown at the ight SG) =e +2) e— 1+ 22 + 2)(e— 1) 22 +2Me— 1) +1le—1) +(e +2) Se 42Ke-1}@e+ 1) Tie critical nu a PRIVATIVE TEST 187 “Tie table ie filled in one row ata ime, P'(2) docs wot exist when 2 =(. The eitiel numbers of fare 1,0, snd 1 ret TRere-} e=jeret zr wea) v + + = + et = + baal 2 = = Q + + FE) z + ¢ = + fis fimo fis, fla fis Shan fin usin] decreasing telmive increming’ elaive decreasing lative increasing fon (20,~2) minimum on[-2)-f] maximum on (3 ]__minimum on [h-+00) fey | IC © fcpa4 Henow 70) oie) Ba) = 2-90", 72) =1 - 97 Su fa) =O: Pa, Pay Poa; e= 2 Conlin Ts ereasig 68 (=55 =I hae a telative masta vale t 0if => 0. J Increasing on (0,22), 51810380 (2) = fet Be = fee) 0 and (0) doesnot exist. The graph has a cusp at the origin. rte Fle) oven 0 + ‘Tfhas a inceasing ME decreasing TONE, incesing te) ° eA a Perse 19-32, compu () the elativeextema off, (0) the vals of zat which the nlative exten ce (6) the intervals on whic fi lsteang, aod (2) the intervals or whieh decreasing. Check by plotting, 19. fe) = 268 Oe JG) = 6s! — IB Bet fa) =0: bole a3 ie) Conclusion eat ‘Phu atelative maxim value a0 ocacd 4 is decreasing on 0,3) 2-3 Phas» relative minimum value Bee {is increasing on [8, +22) hows at the right. We fad the dvivative off and fate it ie 629 =Ke+ IYI) © when r= -1 or x = 9, the critical numbers of f are saad &. We conser the thre inurvals'r <1, lercd and £05 Are determined by the crtcal suber; tse am shown ie tac fey ofthe tae. The sign of each faclor ix show in the next two rows, In Mourth row, the sign of (2) ia the product uf the signe th tove 2 and Toe ceaclusion for the intervals is found vsing Theorem 4:43 and the conslisoa Tor “ Efe) in evaluated atthe eeical numbers aud at one pene increasing Teative decreasing altivenescang sees] asin aft 8] minimum en {6 =20) 0 iS increasing on [=5o, — 3 {has a relative maximum value 1s decreasisg on 2, ~ 1] ‘Phas arelative minimum value Sis tneeasing on (1,1) ‘has acelative maximum value Fis deceasing on (1,9 ‘1 has arelative misimun valve 98 increasing on [52 +e1s4 515, Ma? 0 = H(e 42)(2-2)¢2241) use 3+ 1 >0, the eitical numbers are ~2 and 2. The table is fled ia one row at a tine. 2 Pare? increasing dative —daweadlng on (5-9) rene Telatve _bereasing ‘maximum on [2,2] minimum on [2-406) w 5 7@.NS—1 ‘feat infinite brane, {100- BEHAVIOR OF FUNCTIONS AND THEIR GRAPHS, EXTREME FUNCTION VALI, AND APPROXIMATIONS ta farthest se) 1-27 Sas") =0 Bgteo.- 8 "The domaia of ft («#6 0}, and f(a) exist for all x i its domain. ‘Ti ony crt nembris YP 8 126, 70 ocrevi - Fie decreasing o0 (003) avi |ivixis] 0 {kas lative minmorn vale vice + 4 sinereasing on [V3,428) w. fo)= d+ Alo: Bahown at he it. Note ht 0) nt Me) = 2d 251 GeengeD a a ‘Tae critical sumbers are and } The table file in one row aba tine. | red eecoe=0 Oee<} es} =>8 Bai = + + + + 7 get = = = = - o + poet + + + dueenterst_— + + + FO) + q = dem exist a + Fi/tas a) inerasing —_seatve Terkel Geaeating—ebotve — focganag on (coe-i)_ maximum asymptote on (Gf) minimum on 439) Fe) (fais 2 2 sQ)=4 te TR HW ‘Set f'(2) 6-32 ie ie Conclusion _ Fs increasing on 97] Yinas arative maximum valie ceasing on 2,3] Becaase 5— 2? 2 0 if: 1< V5, the demain af f % (-V8, V5). fe) = 2 4 yb 2728) =(6= 2-2) = = SE he crtieal numbers ate +e, "The table is filled in one row at te Values atthe endpoints re one-sided its, wos eer ene Teal Geereasingicloihe — Gocisacing lati decroning verti Tangent en fv/-e]_ minima on {-ee]_maximum on fevv}__ tangent Te) 0 2 @ 84 INCREASING AND DEOREASING FUNCTIONS AND THE FIRST-DERIVATIVE TEST 191 fa) =2—2(5— 29°; (2) = —212 — 41-49. (5) 20 or any value of 2. G4) doesnot exist and 4 isin the domain off. Therfore teu cetcal wanbe Conclusion is Tnereasing on (20) corsa’ exist] f has a telative mazimum value = | Fisdecrmasing on [ft0) Ba ae = Homa Fixe cd vncpncnls a Fiamsiesa ery ' Bek caer ae Jin #"(@)=-v0 ‘gro ine tothe graph of £5 vertical a the polut (1.2). Othe points on Ove gaph are (0) and (2). plot ofthe graph is shown a the rght. ) = fee te, [fe dk "e) = dove te tan te J) =0 ton te 26 4g <0 ot te =r, 2-0, 2= 1 "he secant function has pevio 2n, {has period Se. Thus we add Jim, where is —1 or 0. se dr tan te nce chet fer detitr | not aeiont Jeidivc cies + [parame eohalis 4 0 | # bas ntve masimmum vahue etliece chet hin 7 Ff indecreasing emdetHr | notdetind | not defined +Hece cde this - Fie doeresing 6 a2 S}rorte=feratnenk. Because the coweantfuncion bas peiod 2, {has peiod . Thos we add kx, wher ie any inte [re Me) | conta = ot defined [not dined | 7 has a vera nymeoe brezclpthe ~ | iedecresing sheets 3 0 [kos a rdative minimum value dethr + ' ° = fe) = doen't ext «+ doesntexiss 0 * ts vertical vertical relative ‘s Finfras a deceasing XT! inereming YSTESL sncrasing UNE decreasing 1B fle) =(04 9202)". J) = Het MeO + He e+ = Yat 1 P(e—2) nto = 2) + (2+ D) = e+ I @= 272-1) Set f'@) Ste 21 oH) and F2) donor eit ond and 2 ase inthe domain of J. The eit mumber of fate 1,1, and 2 A fle) is) | Conclusion y pent + | faowsingon Cal t Hdownot | {Jas relative waxizum va 0 [fama | arse wleret = F's deceasing on HI.) ° {Yes a relative minimum vale leze2 + lise ou not | {J inresing on [1-40 ° fai| {Seneca ss ves + In Exerciens 99-8, compute (a) the tative extrema of F, (6) the values of at which the rdative extrema (6) the intervals on which f is increasing, atd (@) the intervals on which fis decreasing, () Use (a}-(A) to se pet) tes-2 nega w= (BT lace ter senna oa fd wa oben Kiss, ((0) =0. Wince ~2 and O ave critical mbes of f £0) He) | Conclusion 7 ay oo ST 2 | 5 (docmvesta| 7 tararcatte mesma mine Se ee ree 1 0 | Phusarsaits mina rae tee ‘| Fi nceatg on Bs) te Eeesptajet® West msed= (Bog FESETO WG" E53 Fis LA Tae aru: a ai di ro ad ss Jey aca esa Fiafrne» deceasing, tte en ‘S.INCREASING AND DECREASING FUNCTIONS AND ‘THE FIRST-DERIVATIVE TEST 193 -1) and f'(2) do not exist, and /'(0) mar lt Se) - exon ext T isineressng on (6 —T eso rats ener elie 4 isdecreasing an 1,0) {has relative minimum value 7 i neresing om (0,2) soe a esative maximum value J is decreasing on (2.420) pen be fineen fi dfn’ on (0 1). Benue [i-}==P=8 and f,(-0)=549-8 Fs eontioe at 3. bene H=s41=6 ant 1,C1)= Via F=6 in contour at =I, We conde Taf continent on (7). Kow 2243) Meena ro-{a f-teeent WotcecT =oire artbcmors by Theor 3) = 20) ond f.(-8)=-1 10 J(-8) doesnot eis and the grap hata comer at = Sinilass, because (an1 and fy = 6/Vw=F 24/3 JC) oes not exist and the graph has a corner at == 1. The table ie filled Sn one row at atime that at z= 3 and z=—1 we compare values rales than siuiply signs, The information in Ube last rons is used to Sketch the graph shown at te ight Be-S s§ Sere bred Jere lee l lere? wi Tere? + @ = n + 0 eeeanterit — doeertait $0 we Ta [hase Ti Tissa finn [bass Pode fbwa lcrasng lative decrasingon[-f,~1) tative ereadng. relative cessing. vertical (-so-3) masiimuen go relative extremum —msnimun [-yf} maim [7,17] tangent yon = é © c FEE oon indie mcr Cg FERTIL aARENORAATNS aut test ee) ect a. He)=}-fimerDF t-rce cas fe) Vogt ten e-2-7itoxe J(1) and f' do not exist. Also, f'(—§ ‘The critical numbers of f are ocr f has a rdative minimum value 7 is increasing on [-9,—7) {hava rdalve wtnne vat Fi tecrning on f has a relative mir A is increasing on [—4,0] [hase ative ssi vale Ff is decreasing on [0,2] {bona slate minimum vals J is increasing on [2,+00) [4-497 Steck yy freed) tect oe roe (Cr to /O>Gern ws F(A) does not exist. Also f"(-8) =0, #"(-1) =0. seid menbarat Fass 2<-5 SEs -424D o ey 12) + 1 —— dossn'texist + 0 = ‘SF s/oasa increasing TUstive decreasing relative increasing AMV decreasing In Exercises 39-4, the figure shows the graph of the derivative of & funeton J coutinuows on R. Deven ‘he citi mambers of f,the intervals. on which J 8 (H) ineeasng,(e) dereesng, and (d) any teatve ext 90. (a) s(2)= 08-3, 1,8 Sa) =0a0 4, -2,0,3 0.(5" doesn't exist), 2 7" (b) 7(2)3 0 on (20,8), [2,8] Hie) 2 hen 4-3} (Os) Poston Coot ro) (6) 1(e) <0 6n {8,1} [8,+00) 0 on (=20)~4} [-2,0} [yto0) f"(=) S 0.0m [0,2] (@) selative maxims at 3. 3 ive maxima at ~2, 3 telative maximum at 0 relative minimutn at 1 relative minima at —4, 0 felative minimum at 2 {A4 INCREASING AND DECREASING FUNCTIONS AND THE FIRST-DERIVATIVE TEST 105 {F doran'v eis), “i, L (J =0) LJ" dosnt ehis -2,9,2 (f= 0) 0 (7 don’t exit), 2,8,5 (f"=0) Zi) 2002 (0,1) £(2)20 on [-2,0), [1,0) LG) 2 00n [-2,3], (5,420) G)So0n (coo) (te) fe on (355-2 [OE 22) $0 (~20)-2), (3.5) ‘elative maximum at 1 dative maximum at 0 relative maximum at 0 ‘lative linus at 0 ative minima at 2, Zelatve minimam at 2 [ican iletin point) (2s an infledion point) (0+ inflection point, vertical angest) Given that f is contiauoue on R, {(0) =0, f(4)=2, f(8)=0, f(2) >0 if e <4, and F(a) <0 # 2>4, etch a groph of f under the following additional hypothesis: FP iscontimvous aid (8) ig 1G) = 1, tm 2 Foe rouazs “Givin that J Is exntimnas oF R,J(0) “Blowing addition) hypothesis {fseotinuous a3 Lketeh @yraph of funder the a lim Sol # Mayita gi Be) s+? 4m PO) Because f has a rdative extremum at ( $02) = 6 10) 4 (2) = 05 de = 19, “Because (2,3) ison the raph af f F2)=% PeaQPebas su Lners 196 SKHLAVIOR OF FUNCTIONS AND THEIR GRAPUI, EXTREME FUNCTION VALUES, AND APPROXIMATIONS ibe + will have a elative maximum value 2). 48. Find ob, and o nich thatthe function defined by f(x Tat and the groph of y= J(2) will go through the pot ( bf) = darth is defined fr all 2. Therefore, « relative extremum of J must occur at point wl 2) =0. Because we are given that a lative masimume value oceus at x= 1, thee Wy=0 2a4d=0 ‘Becante 7 is the value of the fnetion a 1, chen f=7 abbtent “Because the geapk of y = (2) consains the point (2,2), then 1Q)=-2 feiBte=—2 Subtracting Ea. (2) from Bq (8) we obtain Bapb=o end evbiracting Ea, (2) ftom, Ha. (4) we get Cad. Therfore f(a) =-9e' + Se—2 and f'(2| = 8241 We) 1, then f has a vlative maximum at 2= 1 49. j(2)= 08° + bs? bez +d; fe) =30z?+ 2be +e Becaute f bas relative extrema at (1,2) and (2,8) and (1) and f"@2) ext, chew 9, Sobatitating in Ee, (1) and Fa. (2) we get b= 18 IS(L—=). Beeasse fe) >0 if 2 0 for 2>| Alo, or 2 alfcenly ner 8 (12) > Dand p— (p+ g)e> 0, Thu [changes gn fom 7" 9? 4 at derivative tert, hon slate minimum value at al number and ¢—1 & odd, This (1—2)?—!> 0 for #<1 and (1-2) and p—(9-+aje <0. Thus f° change sign fom ec the tera yf hat ae minima va a fe) Braue p and qe pit, fy ia eral number ané pj For # in (a) 2 (a2) ate postive For 2 <52o, p~(ptq)e> 0 and for 2> 52 p—(p-+q)e <0 Sof” changes 47 to 7H" at eee Hence by the ftv derivative tet, fas a rsative maximum vale ftom * Fey Hence by the fi a a are Sif f'(e) <0 forall x in (@b) then D[-f(2)] >0 for all = in (ab). Hy the fiat part of the proof, —F Increasing on [a 20 tha 1 decreasing on (a 152. Prove Thescem 3.44) © It fe) 20 then ~f'(e) > if (2) >0 then ~f"(2) <0. Thercbore =f satisfies the hypothesis of part THenee —f bas a ilative masimuma valve at ¢ and so f bas a relative minimum valve ate. 152, f(z) = 24, where ki an od poskive integer, f'(2) = ke**, Since k > 1 and k—1is an even integer, J" fever negative. Because f(z) ever changes sign, has no telatve extreme, 5A. Prove that if is increasing on [a8], 9 increasing on [f(2),f(8], then go fis inereasing on [a,b]. > Let a2, < 29 <6. Becaus: f ivinereasing on [0] thex flo) < f(z) < f(a) < $0) Breausr 9 is ine on [¥(@)./0)] then o((a}) gz) for 2; <3, in L. Laue gia doereaig on ) Bor any 2, and 2, in T such that 2 Czy then flay) < f(a). Beeause f(2) a positive on T we may divide HEY Te) aed pervect fin ney. (18 a 1) epi Ts ys Te ay Mile ean to 7boy 0,the graph of Fis concave upward at (ef(e): G) #7) <0, the anh of J concave demnwaed at (6,/()) ‘the following theerem may be sed iff") is zero or does not exis. 4.8.3 Theorem Seppose (c) exists and I san ope interval containing Uy Fa) > Ofor oll valu af 2 Fel, the graph of fis concave upward a (ce). Gi) FG) eo for all values of 2-2 in 1, the graph of fix concave downward i$) 3544 Definition The point (c,f{e)) iss point of infectios ofthe graph ofthe function f ifthe graph bas! angen! line there and ff the exis an open interral I containing ¢ such chat If = & I then either (i) 2) <0 ike < cand se) > 0 itz > 08 (il) "@) > its c, ‘ana iyi 72) oF 1/F(2) bas ielative axis oF minimum ate The infleth agent econes the enave and ir wen cose to i 125.5 Theorem Ifthe function f i difbtentiable on sore open inierval containing ¢, and if (cf(e) is int of indlection ofthe graph off, then ff") exis, fe) = 0 However, if f"(¢) =, ve cannot conclude tha: the graph off has a po ‘The folowing theretns ave unfal n shetshing = grape ‘Theorem If f(2) is covcave upward on the interval (9,8), then on (a8) the graph of f ies below chord joining (o,f) and (8 £0)) IT fle) is eaucave dowamard on the interval (6), then on (a) the graph of f ies the cherd joing (a /(0) and (0) ‘Theorem The grop of «subic polynomial i symmtrie with mepect to its point of inflection. of inflection at "The first of these theorems sugges thot we lightly draw the straight Tine comecting ts points of « curve and tien ws concavity to delermize Whether the eve bows under ot the chord The flowing steps may be ase to locete the points of iftection ofthe graph off. 1, Find #2) and 2). 2. Find all numbers efor which $"(e)=0 or fC) i pt defined. 2.1 the graph of J has a tangent lie, posibly verti, atthe point where = = and U[t(e) changes sign ast inerases through the value then the geaph of has ‘of infection nth point whare 2 = c. B51 Seeand- Let c be a citical numberof s fanecion f at which "(c) =, ard let "exis for all values Desivative Rest for zn seme ope inverval contaiaing ev Th i (c)exintn ‘slive Extra (8 /%2) <0, then f asa restive maximum value atc Gi) #77) 0, then J has a eeative minimum value ae ‘Suppor J'(@) — 0-1 the Fit noneero derivative ab « i of edd order, then « fea paint ection. Hr the fist nonsizo derivative at ¢ is of even order, then f his & lati lasimura ot tninimnm at aceonding ae that derivative f negative o” postive. 135 CONCAVITY, POINTS OF INFLECTION, AND THE SECOND DERIVATIVE TEST 199 e035 joes 1-8, find any points of infection ofthe graph ofthe function. Delermize where the graph is concave ‘and where ie coneave downvtard, Check by petting the grap of J ard the inflectional tangents, S32 = I2e= 1; f(s) = Oe? +822, (a +3. Set aie Conesion Twaph is concave dowaward pint of infletion raph is concave upward 32? 122 = te(2—4); "x)= 6212 = (2-2). ‘oicave pointe! camcave dowward —inflecion st 2z(24). Set g(2) =O Conclaion rea ‘Eph is concave dowaward point of infetion ‘raph i concave upward 225 1), i2e(2 1). 23 Oat, fle) = 120? — 1 “The ctieal numbers for f" az ( and 1. A pt isshewn atthe right eel e=0 Oge 0, and the wep contave upward Bjn0 sf 2>0, the gi inceasing on the interval forte} ‘hee 9) =0 ke a eelative minimam value of 3, and because 1/0), the graph of g has a horizontal tangent line at tbe poist (0,0). Te graph is shown at the (2 Histol ne 6s rat icoOtres MINME)>Or fla) > 0ifz ><; P(2)>0ife>q fllz)>Oifz>e ‘£"2)<0ifz>e MM2)<02>c (b) FG) = 05 26. (a) He) 1) FO =O Pr>0ife>e (8) Fe) dee ext; 28. (a) I") (0) Fe) doesnt: Peyovisea Meo=k Leste Fa)svitzse — fepsitece — Ma)>0ite>e Seleouese Me) = 4x: ln, Pe) = 04 SMe) 05 tin, SH) = 201 P(e) 058 220 Me) S06 204 SEMAYIOR OF FUNCTIONS AND THEIR GRAPHS, EXTREME FUNCTION VALUES, AND APPROXIMATIONS. In Eserese 91-28, find the relative extema using the steond derivative test ard ehvich. Check by pleting 4g 4 Se The, f'(@) = 12 +02 +18 = 8 ~ Het) Lhasa selative maxin ‘elative mininum value oat 427 Ge! — 8s = Bee -9) ‘The eal numbers of g are and 3. We) eee — 122 -D) ‘Because g'() =O and g"(0) = —18-<0, by the secoud-derivative test g(0) = 27 Because (8) = 0 and g'(S) = 18 >0, by the scond-derivative test ot ina reative mininum value. ‘The geaph of gis shows at the right. Se —fa le) = A 248 se) = 1968 253. Sat gfe) = ate) ae) | sfle) | phase (da + B(2— Ys —& ° + ‘elative minimum value ° ° = lative maximum value 5 4 ° + ‘lative minitnum value +5 hohe fo 20; 0) =24-2et = Get; Me) 24H sa fe) =n 2=0, £V9 1 | fe) fhasa -v2 | Rv? ° ~ ‘elotve mamas value ° ° a Se belo vi | -avi | o + relative minimum value $e) = 12274. $0) 4-7 000 5 an inflection point. ss mele Bh se) = —teaste , de. : fe) re) Fhasa mea 7 = save asia alae ek | - ° + ssltive minimum value 4.5 CONCAVITY, POITS OF INFLECTION, AND THE SECOND DERIVATIVE TEST 205, 2m te € (0,4) se) =Beote; ge) = Ws te : = Jr Weapnly the sconddenvaive et A =32-< 0, then o(fe)=2 ea relative maximum salu of f eG) = 32> 0, then ge) =~2 va reativeminimam value of, fa] 3h appears a the right. 4 A Weyer 2 eae grt Hg 2) (0) doesnot exist but 0 isnot in the domain ofA. i] W0(1) ~2 > 0, then Ad) = 8 ina rative minimum value of b : aT $e) = (043) +] 43)? =e 4 Ge 48) 2) =—Ke ty gers) +249 alee Feet) $2) =0: 2 a slative minimum vale of f ie grag of « funcion f for whlch /(2), (e)yand /%(x) exist end axe [positive for all 2 () negative for al f(z) > 0; the gray les above the = axin. Because f(2) < the graph ie below the z axa $2) >0, the fnction & always inceasiag. Because (2) <0, the funeton is always decreasing. J") > 0, the grog in concave upwacd, Tecaane Jc) <0, the grag is concave downwar gap has horizontal asyraptote, "The graph haw a horizontal anymptote the cosine function, fired (a) the relative extrema hy the secondlesivtive tat) the pints of incon; the Skpes of the infetional targenta (d) On_ sn interval of leagth 2 and containing the point of tion having the smallest positive abscia, plot the cosine and the infletional tangent, 2. (3) (2) = sin 2: f"(2) = ~eon 2. f (ex) =O where kis cay CB is an even integer then {"(kx) = —1,00 f basa relative um vale, If Eis ato fateger then f* (Es) = 1 of has a relative am value F(t §)=|=0 where ie any integer. Each such = gives a pint of f Because") change sien ) The inflectional engen has slope (E+ })2)= ~in(&+ )) whic i “Lith ineven and #1 It vo ‘206: BEIAVIOR OF FUNCTIONS AND THEIR GRAPHS, EXTRENE FUNCTION VALUES, AND APPROKIMATIONS Pe = ante +, J"(2) =2san 2sect2=2 tan 20 an*e $1) Tobers kis any integer each such = gives a point of infbction because /"(z) changes i (0, (b) The slog ofthe inectiosal tangent i fer) = 42. fa) — cot 25 f"(a) ~ cnet $62) = Desert (a) See f"(e) =: cots =O s= Je +t, where & is any integer, Ench such = gives a point of i Became J"G) changes sen. (bn +r) =0 [B) The slope ofthe indectional tangent is f"(j-+ bx) =—ese¥ +x) = eas f'(2) = eve wot 5 (a) = ene (2 ae 2) (Ge + Ex) —0 where & is any integer. rote then /*(2) = Ie f hae « eative minimam valve Ite inan odd inter and 2 = 2} Fr then f"(2) =—1'0 f asa reative maxinum value “44 Find the rlative extrema of the scant function by applying th second- derivative tet. b Let fle) ssee 2. Then fie) see ston = If (2) £0, then tan z= 0,50 2 = kr, where kis any inseger. file) see 2 tan esece ec a(2 tan! s+), Pinjancten{} fhemess tee ws cee baciasin ah wi OIE EN cama tn rice es Fug er aati Ht Ck se tr 7 See In Exercises 45-50, sketch che graph of the continuous function f through (¢,f(c}). (4.f(d)) (fle) ¢0ile< ds (z) itz co; f(z) < 0if Mxie (2) >0ifr>d e0ifz>d oF) =) igo gs Laon Gara iy 0ite>e itdczcqha)o0itere — Pa0% Me) >0ifzen sa <0 Jha) 01 ES (Ma) <0 ihe od Rececdpu)>0hesd — deeca fMa)ebihese ata wn, JC) doom exits) ME]=HME)=0 FC) se) =0; ay Fe) a Hla) =a MOO 1: $e) =0: Fis) <0 don't exist f"(e) = 0 C8) <0 Gi0fecsed S>owtr>a (e)rowz>e fis >dteoa fara fase es = ae + ba f(a) = 3ee* 4 Ybay f(x) = Baz +2 {is polynomial, 2) ant J"2) exit everywhere. If has poittofiatlection at (1,2) then (1) 2) =0, Gat is ‘and 62-4200. Solving thee two equations simultaneously, we gel (0) = ot +122-+ ce, dearmive 5, and c, 0 thal the graph off will hare pinto infetion at (1.2) to thatthe slope af the nfleconal tangent there wil be 2. Soppot your snawer graphical ie)= Sst $e eg fo) = oar. Fis polynomial. /"(@) and #"(2) exist everrwher, d (12) is poist on the graph, tnen fil) =P andso2=a+h-+e (2) raise (12) ia infetion potat, then Jl) =Oand 6 0=62428 (2) | 3} the slope ofthe tazuent at (1,2) is -2, thea f'(I) = -2 and so H a2 be @}) | 6 (2), (2) and (3 smaltancously. we fad a =4, =O. f(z) changes sige at 2=1,,then the kraph of f has 2 point of [ —s on when, jo) = 120! Oe i shown atthe sight TA plet of az! be! ter +d; (x) =Saz!4 2be +6; f"(e)= ae +28 {i polynomial, J/(2) and J"(2) exe eversuhere. Because J has a eelative extremum at (0,3), Beceum, the graph of J hen a point af inflection al (1,0, “Land f"(0) =0, Therefore a+b 4=0 and Ga +25=0, Solving theee equations ana bn jen 0, dnd, f(a) = a2 +e 4 12 + de + 6 determine the values of a, 6 and e so the graph off will havea paint Infection at (I,~1), have the etigin on it, and be spremetti with respect to the 7 axis ‘the graph conaigs the oigin, then /(0) =. Thus, ¢= 0, and fia) nas 462" ea" saz raph symmetric with reapel tg they ax and ae f(2) at et cz dz = ac! bx) + cx" — a, Dh} dae 208 BEHAVIOR OF FUNCTIONS AND THEIR GRAPHS, EXTREME FUNCTION VALUES, AND AP®ROXIMATIONS ‘This isan ideatity for all zit and only if} = O and d Wel=attes's f(a) =tar +205; Je) Because the graph contains the poat (1-1), then f( ater-l Because f is « pelynonial, (2) inileetin. Thus, °C) =0, and Teb2e=0 Seseating ies. (0) am ee) we 2 Therefore, Se) = $e and f"(0) = Bet B= Bee? -1) Because j"() ches sgn at 2 = 1, the the poph tJ tas a pont of nfetion when = 55. The numbers }/8 and -}/ ae etical auaber off and f*(e) = als? 4 1 > Because fo? +12 1 Hhet+ 112 Aux =-4V5,f"(e) 0 and f has a relative minimum valve, 56. Prove pati) ofthe soeond-derivative tet for lative extrem '>. We must prove that if/(e)=0 and /"()>0, then f hasa relative minimam value ate Brense fle) =0 and —frle)-<. t ollows from pat (}) that —J as a tdative maximum value at ‘Terese J bas» selative miizmom value atc St. The groph ofa function f has a pont of inflection at (c,f(e). Bocaue we requte a tangent line then J ma ‘be continuous at «The tangent may be vertical, 0 f¥ and J” need not exist at c, and Bence need nat centinwous ae. 98, J" (2) exists forall valuxs of 21m Igand at a mumber © int, f"e) =0 and "(exists and in not 0. 4") exo, then (c) exists 0 the graph hasa tangent line atc, Thus, 1. Thus, at cach point of inflation. We ate given thet (1,1) is a point 2, a=] nnd sbbeiuting int a, (1) given 6 end (=) haa the some signa =. Suppose ("(¢) <0, Thes ‘Suppose {*(c) > 0, Then Treo = jy HIE eg Pe = fin LO=LO, Teg some opm intra cong Hc ar we pe incre otanng © LoL) LHe—I)» gE) g muwer cdeote ose Weieinty and e eee concave downvard + increasing concave upwaed 0 aera pint af nfosion = Increasing Concave downward = lative marimar coneave downward = decreasing foneave downrard {for Exercse 21° for Exercise 21 "for Exerdse 22" for Exereae 22 ‘36 SKETCHING GRAPHS OF FUNCTIONS AND "THEIR DERIVATIVES 215 Lisfhas a ph ifs a Lee) Snereasing ‘oncave dowiward 1 0 eativeraximum concave downward 0 if £0 and 0) Erb off seaacave upset eye by Thee 333- 28. Given f(2) =o? —r2-+ hy where ris aratonal number prove that (9) 10- 1, f basa teletive minimum value at 1 se fe sro- ES We note that 7 F@)=re~ fn) A wf (0) Ons es Men) co edae Ha) 0 and PI) 20, She esate tak relative ment value a {ifr cO ort > 1 tea (1) >C Beease (1) = and f*(1) > 0, bythe sxond derivative test relaive minimem value at 2. f(e)= 24302456) = 824 Se I) = be Ge (ey a ats 30> er all eit > 0, Torefoe f ie ncrsin: (F< the 116/00 J inh 0a>0 a that fae lative minum vale (To Fie) = 6+ 0. Thon f Salat minimum va Because J has xo other eral umber, J bas no otter relative extrema. ‘31, Sketch the graph of the astroid 29/9 + y* ‘The portion in the first quadrant is the graph of a func Chinn hi potion snd thn coil the graph by eset rope. > Theprapt i oymmetie wi cape tothe andy eso the ine Wont ote pany aes een ee er <7 Difleatintng wt rapes 0 # we obtain V3 2A Get) =a P 0 ‘hus yi negative ands the graph decrees fo the pit (1) tthe plat (10, Pesthermore spose co approaches 0 fom the ight and 0 thopeaph several angst (0 by apne as hszontaltngent (0), Difoatiting E,() with expe to = me na PPE 2) fe) een (asst) and har ne ative ats Phe y ie Jeg 202) 4 2) Because 7” in postive F< 2 <1 the graph in onenve upward in the fret quadrant 3:7 LIMITS AVINEINITY 217 Ar wveniry Delton Let f be» function that is dette af every number in sme interred (4s). The ini of 10), a's nereatesvithoat bonis, oan Ai if for any €>0, hovever smal, thee exits « number N'>0 auch thet If #3; then Wre@)-1 0 and I f(s) = thes hin Plea) ie (8) ie) > 0 and ig fe} =e the fi leh) (6) i, e) <0 i 2) = 40 she i 1(2)42) (i) igo) <0 asdf 1 (0) imo) = a i, J) = 450 the i) 9) (0) ig oe) =U aig 6}= ve he i Le) (0 ig 06) = 406 the on TE. Soppoe tat fin fantion defn by 13=fs ‘here se) and (2) are polyoma ‘To fad she limit of f a8 z~ 400 ot a8 2s, ft vide both i) end a) bythe lighes power af = that cppacs i the fononinaion ine 34 ty uve Lies Totes 18 together with Ue ater lint tnoten tes fb Ha of {ican be sown that if an hae both polynomial fants tes () Wake age oy sen than he dep oh ten tin 9) N then fe) #8 band for some numb: N, if then f=) me GG) Hen, (2) (one +5)] = 0, ad for some number Noi 2 To apply Linit ‘Theorem 13 we divide the numerator and denominator by = s-$ 5-40) im, O~ lt ** tts wert Weddle noneater ane denominator by 2, Ts etm timo im basa ay Dividing the muresator snd denominator by 2, the highest power of = which appears, we have ed 3-3 ip 4 tin 0-200) 5(0) T+ ia 5+ tia THF be 22 $B 22 1 ah tim #2 =, 2 sr e42 i S47 LIMITS AT INFINITY 221 se the numeravor has the higher degee, we fst remove &fetor of the numerator. ‘Thes we divide the numerstor and denominator by 22 im +=$- lin, 2=—co (Thee B)) Bie cvide the numerator and denominator by 2 Grannies ne + =o, hen # CO and at = = or equivalently, = VF. (+4 _ Va 4h 3 Yen _ JE Bq. (1) wehave ¥* rs 3 0) vr z 8 Vd yo are AG ae ae ss aa [veri Frits Lia (WeFe—2) ee both em ave aint, we nna of VFE Perez 7 lin, (VFR ~2)= ip, VERE J ie ( Pasay VE tree 4 i ig ms pe ie TEE ie Decne 2 tn, tan #0 and 2 = VF ao tht [Fezas Ves, Sie = Spee Eg VO Vind e By Eavatiog (1) and (2) we have lim J = te “eae oni eT oa dVieF 12 VE-9 = 1209, abHIAYIOR OF FUNCTIONS AND THEIR CRAPIS, EXTREME FUNCTION VALTES, AND APPROXIMATIONS a ee, faa, valine de et eer) tm Exercises 89-46 ind she he Yorzontal aud vexiesl sryrtoten and stetch the graph. Check by plowing 36. f(a) = BAY are > Became lin, Bes ses tip td = 2 or lim, 1) = The gosh Sonate Tl Map epi (22) oe = Bis a vertical aeymptote. «horizontal agympiote 36. ale) = 143, > Bocause A(x) —Mz), the graph is symmetric with resect to they axis. We sketch the graph for 2>0 tee srmmety (o compete the akerch, Base wo (3) na) = 14-244 sai ssa pant ntti ei positive numbers as 2 approaches 0, and 22+ 1 approaches 1 a8 # approaches 0, then Jim (2) im = ed ‘sa vetieal anyaptote y= Siva hoizontal asymp AT LIMITS ATT INFINITY 223 ot =2 ica vertical asymptote 0,7= 0 herzontal asymptote. wm Gunction, the gph ic symmetric with rapa to ce) =o), the panto gst wth exp othe ya, We ach the paph br 20 Be cine cs wat Bema sz) $1 if z>2, we conclude (hat the line y =—1 isa horiaonlal asymptote. Because 4—2* the vertical esympiote. (And by ssmmeley z= 2) Sine 4 inthe downward direiion. Since (~=¥ approaches 0 through postive nurnbers as 2-2, then lis APO onli that the curve sppreaches the asympote fom the leftin the pmard diction. Beans (0) =0, Bos estar ie eign, We ue the soppy, seh, at apne Ue dene’ anh dew ip Oe) = tip by A Nadas ict. satin 80) = BR mage? G{G) san eves Fimetion, the graphic aymmetric with respect to they ais, or im, G(s) = tay 3. y= Bes horizontal asymptote y= 2's a horizontal asymptote Tae F(z) isan odd function, the graph isspmmetic with respec to the org. {204 HAVIOR OF FUNCTIONS AND THEIR GRAPIS, EXTREME FUNCTION VALUES, AND AFPROXIMATIONS 2a 2s $8. Ke) = Gar fiamig Gee) Because fim. Az) =-4o00r lim _ Az) =~oo2 = fis a vertical asymptote an _ ene) faces) =e fa vial i —_. lim Me) is a horizantal as te eens ie Me)= an yeaa Oe alte ovzntal yet WMDs > Because h-1) = -h(2) the graph of hi symmetric with respect to the cxgin, Beeaase 27-9 > 0 if 21> then the domala af fis (os) U(8, +0) 1 tin tim, te Tera he TE sp ad Ns) # Lif #> 9. Renee; the fine y ~ La horizontal asymptote. By symssety, the ine evizonal scytapiote: Because 2? —0 if.z= 23, we test the lie 2=3 as a posible vetial asymp {hia by syemciy 2 =) Because «/7?=8 appreackesD through positive numbers as = — 3° then lim, 75 = + ‘Thereoe, the tne the right in the upwatd ditecsion. Because hz) 1s m0 deed 1 Jig the curve dom not approach laymplote fom the lef. We use the domain, symmetry, and the agymptoies to skerch the graph below ee Bxecise Exess 48 Exereis 4! 45, f(z) = iE Domain: (el121> V3) Vis a vertical agptow. Bacar eg fe) = oo. pee fe) tase = VF re aoe hem tim, AE ae 2a ip we VSP VED hg Fel tem (ip gto linet st) line IS ey Vero Jim, on im, aes D end eo y = Ae ia an ablique acympt Magarge PTE te Josue Pa) Deca J(cz) 2S). the raph is eile st the ya Hence y = ein ae am oe see 16.10) = pop hme = era esy oaie (9-8) UL 6 Fe) = eh EPA TT Home Coo Utd) f(z) =~20, 2 = 8 ib a vertical asrmptcte. co, = 2 ina vertical asrmptte or ,tim_f(e)=0, ‘Phe graph i ayamitsc wile tepect tothe Tine ‘a horizontal asymptote 27 LIMES AT INERATTY 295 47-4, ind the agmptte of the wah off. Check by pbting the graph and the asymptois Sqpartitghyaet iste) 20 oF bevaune in, f(e) = oo #1 ls a veri amps Y= 241s an oblique asvmotcte. )=tooand tim _f(2) ijmpote, Because the degree ofthe numerator is Loc chan the ‘ofthe denominator, therein an obligee sxnptote, Dividing. se an 1)= 2-742, a) line y= 2-7 an obtgue spite Te exe ls above the ayn 90/248) > thal e's > At, and bow thenepmpte dfs eA. plot shown above Pahartseclyaststale) Tin, fle) = 450 ot bese ig 1) TRB 12) <0 or bcate tin (2) =0, 9 = +3} an obligne asymptote. 20 the line is a vertial asymptote ja Eafaetar lyn et tts) lim, fle)=-to0 er Beene im Je) 19) = ‘a vetical asynptote oF beenute 4 2is an ebliqe asymptote, im (2)= toon, 1) PRS fe) mt tree i a yee? «i Lu, f(=) = 0 ane im $2 isa vertical ptdte und the euve aporoaches the asyaptote in the pad diretien ftom the right an fom thelef, Because Patel set) zimtetl the degree ofthe numerator i one more than the deer of the den rae eT tie a vertical asymptote ~ 68 on oblique asymptote 2454 » the line =2+5 is at oblgue asymptote. ‘The curve lice above this asymptote if ita > jand blow Wit ec fA plot shown above, sett deo ttiale) “490 ot becanwe lm_ f(c) = noo, 2 = 0's a vertical aeymptote st y= 42 san clique asymptote ~ Sects) co or becaise lim Je) = #20) 0 a vertical asymptote, (or bremnse Tim, o(=) =O, y = 2 ie an oblige acymptote 226 BENAVIOR OF FUNCTIONS AND THEIR GRAPHS, EXTREME FUNCTIOX VALUES, AND APPROXIMATIONS I Bene ad, vate sac init Gm te gah fhe ection Jin the sete >) lin fle)=1(b) Jim fle)=0 © (2) lim J(@) = 400 © lim, Fee) @ inte (© “tim 2 (@) fife) =1 im © tigre otye- (0 info =3 () slim f@ @ linge (0) lim, =8 Alot he ran of fnaon J satisng he gen proper and wha domain Iim_ fla Bi fle) = +0 lim Jl) “im, =~ ie bare he) ~ 0g 10) = B88) — 01 9)~ 04 H2 60, tas f(a) =SEE4 Given c> 0 we sk a2 8-<0 sh that WeN then 2 N then een then [pty] <« eeitaaN then et >) sec then 2127 era>N then 2>441 otecy tim 2 Och that if > N thes f(2)< © Lia Je) (0) Fine Ae) oo: given any number N'>0 thee is « number M < such taat if 2c! given any mumber Nc thee iva number Mc sch that if 2 < M then f(z) 0 weselean NO ich that ifs When |p cease . Brease > ite cN thentst> (QJM hi holds itwe tke =)" 87 LIMITS AT INFINITY 227 (at lim (eA) = 400 by showing that any NU there exists a > O such that C= > Mf then =*—4> ® at (1) equivaleat to fa > Mathes 2794-49) M0 then 2 > and we have itz >M then 2> Vion “This hold i we take M= 4 +N. Prove tat tin ((—2—27)= Mieing 6160) Given wun N&O we wed an M> Orch thet We>M then §—2-26-N Foe above holds if ve tale M= 6K, OP SKETCHING GRAPUS OF FUNCTIONS 1 thetch ofthe graph of» fonction J, proved a follows “4. Determine the domain of f BS vind any = d's intercepts. Vou need te uppoctimate in-rotaf ft) "5, Test or symmetry, The graph is symmetric with respect to the y avi iff 8 an even fanetion and with ‘expec: to the origin iff $8 an odd function. The gaph of wcabic polynomial i symmetric with respect to its point of infetion, “4 Check for agrmptates, I f(z) = a(e)/M2): fa r= alisa vertical asymptote if ha) =0 and a2) £0, by = bis torizontal anymptote lim (2) =8 ot ia f(2) =. ey =me-+06 an oblique aeympice If y and hate polpuomlals, the deyte of ¢ 3s ove mote than the ‘ogre of fh, and long division gives a quotient of iz-+ 5. Compute fz) and "(c).Factoring s wualy help 1: Detersine the crkial auenbes of f. Thee are the value of sin the doin off for which her J's) doesnot esto f(s) -0. 7 Tae the numbers of step 6, a8 well at mumbo at which f i at defined, ad the fit deivative to beate Inuervals where f ip metowing (P(2)>0) and decresing (f(s) <0) and relative maxima (whee Ghanges fom intasing to decreasing) and sini (wae changes om decreasing t ines). A table may be bepful elude ade nite at == «which dif from f(a. Soe xerce 16 A Determine thecal mtnbew of J, tha the vals of for which /"e) dor ot eit of") 9 Use the numbers of stop 6, as well as numbers at which fs no: defined, and the second denivative to ncate Intervals where the graph is concave upward (/"(2)> 0) and dowaward (J"(2) <0) and pointe inflection (were concavity change, provided the graph has atangeat line there}. A table may be Helpful 10, Bvaluae f(e) at each number determined jn steps 6 and 8 aed at one number in each interval where the ‘raph is unbounded. Join consecutive points where tls possbe (odo so without leaving the demain off. Bow the curve bow che chord in an interval where the graph i concave upward, and above the chord ‘shere it ie concave downoad [BEHAVIOR OF FUNCTIONS AND THER GRAPHS, EXTREME FUNCTION VALUES, AND APPROXDIATIONS “Jp Bxecises 1-24, ketch the graph of j by frst nding the flowing: the relative extrema of J; the posta ‘afection ofthe gaph ef f; the inervalt on which f is inereming and those on which f is decreasing, where [gaph i concave epwari; where its concave cownvard the slope of any aleetonal ‘angent; and the horiten ‘retical and oblique asymptot,if there are say. Ineoxporate this infermation into a table HOSP ESET 1") = te 922 465; Me) = a" —182 46 = He Yle-1) Se Je) = 2= 4 Fie) | 1) | Fe) Fisfeasn gre ifs a Tecreasing ——toncave upward relive minimim covenve upward increasing concave upward inerasing plat of nection increasing concave downward increasing point of inflection increasing concave upward a) Ste) =e? be dele —2) 0,2 ‘Jisjhesa graph isjhas 2 Tecrensing coneave apa Mationary point of section Geereasing concave downvard Geerensing ‘poi of inflection ~ ecrasing concave upward telative minimum concave upwasd inerasg concave opment 2 Qe mae I)(e—2) fMe)= 9222-2 se s"(2) 1a 055, 2252 Marat fe) | fC) | £0) | Soften ppep i o EP aceressing —eorenve upward ose |a | 4 | setaive minimam concave upwonl 3 |4 | inewaing coreave upward ore [060 |e | inerocing point of nection + |= | increasing Coneave downvard 1 | 0 |= | celts msiintiseoncave downward Sh = |= | deerasing concave downvard wo. | Tas |e | decrnsing point af nfetion I | | asemasing Concave upwasd csr |o |_| selaive winimum coreave upwart + [4 | ineasing coteave oprard 4 @)=8 4 +e . Pa)= tet 1+ Baa? +4) ‘hen by the factor honrem £-+1 «factor of f'(2) and (e+ 1Ne—2? “The ertial numbers of fate ~1 and 2. Se Table a f(a) = 122" — 242 = Bale ~2) ‘Phe critical numbers of fate 6 and 2, See Table b. Because [isa polyomial fanction, there are ro asymptoles, ‘There is no symmetry. A sketch sat the right. Fe) (gaphb/ | coscave point concaw poate eonenve ih aed inl dewamand inf pat Teh 1") os 1650 5 as! $242), <1) Set f(a) — 0s == 0. Sat Me) = 02 Ste) | Pe) | Condusion erasing, consnve upeaed — relative minimum concave upward ineressing conenve upward increasing: sot point of infection Ineessing concave upard He)=324 442 40045 pe Becanse =! 42+1= (243? 4]> 0, ¢= 018 the only critical number off. Hf =cO then fe) <0 andif x $0 then f'(2)>0 Thu fie decreasing on (20. inereasing on (0,420) and f has mative minmom value at O of 4. P(e) = Se + 2 412 = 16629 +e +f) =ee+ H+} 0 ‘Thus the raph off is concave upward everywhere and as ne point of infection Thee ace a0 asymptis or symmetry. f(-1) =I and f2) =9 Poi-(, est re-{E Wshro=f tpt i oe ios! r Set f(2) =0: 2=0. "(0 does uot exist because f"_(0) =2 and f*,(0) =4 ‘ (2) | fe) | se) | Fisshasa graph isfhas a q Fe ——[e] aeereasing tones upward 4 2=0 [0 [0 fame. | lative minimumeoncave upward F Oce + |£ | incteasing concave upward i 48 SUMMARY OF SKETCHING GRAPHS OF FUNCTIONS 229 280 BEHAVIOR OF FUNCTIONS AND THER GRAPHS, EXTREME FUNCTION VALUES, AXD APPROXIMATIONS fect i231 (2-0F fect We—a fe>1 Because f'_(2) = 4 (1) =0, then (1) =0. = | isthe only enticl aumber for f. See Table 3. maja fen, f2<1 rei(aemap eas. Berane f* (1) = f04(0 is the only ritieal mumiee ben 0) 20. for See Taleb, Ther are no asreipotes or smatetry. Te araph is at the right. ‘Table a (mexa zt oF > 7 4-1 6 + fe) + 7 + Tl Tnereaing on (->0, $0) h so.seiive sx at L Ste) a i) ‘Table b Fel zr Re) = ° 1260 ° + Se) ° + as a | deweead —infstion __upmasd__| to, s@a{e Kestrey fishes (F tashro-fartis 122? ifoce £ Se: /(@) = 30. Seb f"(e)= 06 2=0. The paphisoymmenic about the ova. HTP. LLL | fifi gagh afi 5 7<0 {eo ee era J reo fo ft [a | statonay paint otinfecton Sf ig + [+ | itsendny Sea par A wso-{e feshsre-[3" ech ro=fee feed Pv Se. '() = 0,226.80 f"(2)= 0 2=0.The mavhisermmetic abot the yaso| S(eh_| Fz) | Fe) | F isfhas ‘graph is/has a 5} +=0 ee ieentag ——taerapard ; 0 |e [a | Sass sic pit ot tnnton A tee + | ‘ eneave upwaed a 3.8 SUMMARY OF SKETCHING GRAPUS OF FUNCTIONS 211 (se afeca, Hay af0-D fea o-2) tac S62 peer [ten HELE {1G ener 0) dos ne xe scaph e/a 2 oneare up stationar? Dein of inetion eetesing foncare dovnwar! +e J 8 an old fasetion the graph offi symmetric with expec he Be ao spp. HG) = Wee + 18s" = 182%" +1) SIC) het $302 = ahe(2=? 41) (2) = 0, then 2 = 0, and thus 0s he only ertical sunber of f. Bacawwe E )>0 its 0, then f in creasing on (0,400) and F Bae ne relative Because f(@)< 0 ifsc 0 and f"2)>0 ifs > 0 then the graph of J is downward if <0, concave upward ia > fy snd (00) potato The tangent line a that point isthe» ais because (0) = G22? = 84 1)'e~ 2? 420-2104 P(e 4 Me -2)8(e-2) 42041) y= Aet Ge) Lae) Fe 1) 00e— 14 (2+ Ie —2)52 4) (e+ Ile Me +8) +9214 )) Het 1Mils?—16e +1) s-svi., ssavi., BV 0.065, 2 2198 $Me) | Fiefhas a erapha/has Increasing cones downward ; stationny plat of infltion dl ineeasing Concave upvard x inerasne point of nBetion inceesig eave dovaward , relative maxim coneave downed eaeasing concave downward savaring point finftction ptt Me dsaeasng Concave upard feltive minima concave vba ingeasicg concave uprard i) =e 8 Lo) = Bele 49 4 Mo Pm ae 4 4) 30) = He)= He + A)ter +e 18), zaot-fo. sheet eazy aft YO) a2, 2x 2 =H VB) 182 Le) | $4) | $a) | Fafa raph infin + P= inerenaing toncave domnward © fo fo | stationary ein of infetion + | + | inerasing ‘neave upward tat |+ fo | inceasing point of infection of | eat ‘oncave dowawaré sa fo |- ive maximum concave doweiware STE demi exere domme 9 |= [0 | deceasing point of infection FT = |S |eceasing ——Comeave upmard 0 fo | | seative minim concave upward + [4 | increasing ‘omeave upward 282 SBHAVIOR OF FUNCTIONS AND THEIR GRAPHS, EXTREME FUNCTION VALUES, AND APPROXIMATIONS, 15 Jo-(e gy eeoes nya fett CECI pas tocecte io eee theres l" rere Fs aiscontnsous st Be Because tim f{e)= Lend lim | ty HO eto) ae , ST re | fimo anh nyhas zt" oceake [= [imeetng tase nad an ipertt iet 3 Jane Jane | Httiv mini, eo pint of fetion ye |B | ices" Satie domieara coos anced fee, herencd 1 teo-(ciaw tices le tees fe={ae fockes SL O)= focecr” Beaute Lip 2) = adn, (2) =I continuo a (0) oes not exbt x)= 0 and 1°) = ey Set fz) =0: "Theres ne value of z for which J"(2) = 0, Breas Ti f(a) =f = ITs verti weymptot, Besos Li 9 “GiE's4-1 in an eblique ayaptote The graph is symmetic with repel to (1,2) Si) | fe) | se) | fsfhasn __seapinfbas s__ = Siecaaing ——toneare dowd ° ‘ative maximum concave dowawart es Searing, coneave downward dae [ane | ane. ‘erties asymptote we | eccasing concave upward 4 |o | | a vlatve minimunoneare upward + | |fececasing concave upward fte)_| fe) | $e) | finfhasa __ sraph shana St | f= [incense concave upward oF fa fimcessitg Goin of inetion . imcessing concave dawneard 1 [dae | ene. | sehtive maxinum m0 point of ifecton 1 jump = freak + |e | eeaing concave wpa o ([F ineeasing point of Snetion + incessing Concave downward 38 SUMMARY OF SKETCHING GRAPHS OF FUNCTIONS 223 43400 f@)=1-— hy r= sede fe)=1- He rel= + VI0~6.16. f"(=) never 0. se=Ztha=ess4 gy se ((@)=0: Deer d,s y=2+3is an oblique asymptote... The graph is eymmetric with respect to (3,6). fe) | Fe) | FG) | fiaftas graph nfs zy increasing concave dewanand zor, [032 [0 | — | relative maximum concave downward acres = |= | decteaiag concave downward z=3 [dae |dae dae vertical asymptote Sesen | cecceasing concave upward eazy) | 1232 |0 | + | relative minimum concave upward net + | 4 |tocreaing | caueave upwerd ofa? = 1)—2e(e +) ge t f@) pe) = a= DI =) Set MG) =0: Tim, (2) “lag f(e)— ee = cod e=1 a verted asap Beste js even, the rh erotic wit epect othe ye Sle) | Fle) | $7) | Sinha sraphis/tass ee ad ane fase: [ane wool empile = | ielative maximum concave downward = | decreming — comeare downoad das. ertical asymptcte S| cecreming concave upward the axis a horioatal aymptote Hy the quotient rae, ai 202) ates toe ee Becaase f'2) <0 ¢ # 22, then fis decreasing on the intervals (-20,-2) (C43), and 420). Now Se) =—( 44)? fle) =e +4242 99+ -97 att 2 1414-0] Bafa? 439) ere ‘The only critical number of infletional tangent s (0) sespec to the rs. is 0, See the tale. The crgin is = point of inflecion and the slope of the 3 Because f is an odé function, the graph, shown above, is srmmerrie with 2244 BEHAVIOR OF FUNCTIONS AND THIR GRAPHS, EXTREME FUNGTION VALUES, AND APPROXINATIONS aed ered esd tered > = > far) + + yeszt | — dosin + + + aye-2p | 7 —tomwernt Te) ae . = teaser wah] | comave _verial —camave pat of —oneave vetlel coeave | doeamart _anmpinteupmaoi—_infestion Pe ne eee a RAE = aa Tea eye SISA eG 2249 eo 29) up ‘ba hodzontal asymptote Ten od fanetion, the graph is symmetric with respect tothe xg He) _{(e) | fc) | tease ere inns a . =| decesing ——_—coveave downward , Wi 0 Gtenasing ait of ation 5 “| decreasing toneve sand + | eatve minimum coneave apart S| inerasing coseave spwacd SI © increasing point of ection = to 2 = ereing coveave dowavard © Gcerasing pot of alecton |% | decreasing cave dowamand =eedgreye +26) _ at aset _ s(st-3) =etolek f(z) = 1 Gap ea pte =e tetas erg Se J) $02 =0,4 VE Se ["(e) = 0:2 =O Beease im _f(2) = 400 and im, fle) = 490 sad 2 = 1 ae verinlanptotes, Because tim fe) = or Because, li (2) ‘xymptote, Because J is an odd function, the graph is symmetric with raped! tothe orig 1 | == jean oblique fe) [0) |) | Siefemes ach inn a incrasing ___coneave downward Telauve maximum coneave owarard decreasing concave dow enicl seytptate decreasing coneave spward Gecasing Coren downward vavteal myrniots decreasing coveave epwant telative minimum concave spwanl Inewasing—coveave pward 35 SUMMARY OF SKETCHING GRAPHS OP FUNCTIONS 235 ar ea) Mes Me con Mes a%e— 1) ai aren eah Mea, (nae Np dles Khe ty Mer bee ble dere 1. f(z) is never 0. f(=1), f"(=1), £'Q), £2) do not exist ibe sinew nn pt he tin ae “Lip {Fe} 2} ~ inl 1/40 = 2/0)! 3] > iy OVA =/ foro", sagas 0-294 4 oP YRmK1 2079] =3-F=0 Listes ap fase Thea ose upward Palit nisin rea tena Seeckag awe upd lie tin covcwe epee imeeakg conte upmard ein nngrat pnt of neton iiceaiag "cave dowsward Because fia even fnetinthe graph i syumette with respec to they alae) (e*=aNG2)___-oe isd CaF Fae The ely citleal number is © See Tables. Now Tables ze ered eres “ie % $ = = = Mesa? , + + * * yes) + + + docabedit + Te @ = donee fi Gecieaing doe aot decreaeng 40) ik f= Se) = 1022 =9)-? Fhe) = 1-260? 926) (9A = -10(02 9) 7-42 6 2-04} = 20608 9) 43) = 3 ‘Thos /” has no ctial numas so there are 2 pola of iaection. See Table b, The graph is a the igh. 236 BEHAVIOR OF FUNCTIONS AND THEI GRAPIS EEXTRENIE FUNCTION YALUBS, AND APPROXIMATIONS Tabe feed were f+ + yersy | - —— docart est ye-3se | - - Te = deem oat raph if | concave vertical | mica” | upward _asmpioie _domanard _asmpioie ugar | points of fafiection of the raph off the there the graph is concave upraré: were itis coueave downward. (b) Ceafirm analytically and incorporate tl Information into a table. Then sketch the raph off 25. J) mob 42031202 ie 42 fe) = He +60? 26 = 208 AYP Pie) = 122+ 122 =28 Set “penn -$VB—}x—tus, 222, =|VB-}e219 Se fs VA +3) x -2.05, 1 = 25 = UVF -3) = 1.05. The graph appear to be fpmmetrie with respect tothe ine 2 =~}. Sabetating = uJ we get (2) =u! —3e? + HE we eve He)_| el | se) |_fiafhovn yeni a Fey decreasing concave upward zon | ‘lative etinimam coneave upward cree Inceasing—coueave uprard ran inceasing point ofnfetion ere} Increasing eoneave downrard a relative naximumeoneave downward decreasing coveave downrare ecreasing point of nleton at | Fert Teseeer yereripedes ‘creasing, concave uprard 2 relative minima eoneave upward Incensing eoveave upward 26, f(a) =204—15e 4 2c 12e=16 fe) = 89 =e? + G42 12 = (2-298 -286+8) $f (2) = 2a" 0464 Sot (6) =O 22, == 2) R09 VHB) 0.2, 22, = 9+ VHD) A 45 — VB) w0.8, tS+ Vi) ~ 220 Soe) | fe) | se) | fefrase grok nfnoe a esas eoneave ape coro [a ‘inimam eoseave spor + [+ | increasing coneave upward aso} [a | inceacing point ofnfeton + [= | ineeasing Concave downrard 0 [0 |= | ative maximumeoneave downward = [= | decremig coseave downeard wore |= || decreasing point ofnfetion = | + | decreasing Concave upreard aaras}o | 4 | selative minimam coseave pad + | | ieeeatng concave upward 33 SUMMARY OF SKETCHING GRAPHS OF FUNCTIONS 257, fet-25 ife<-5 ifz<-5 f2<-5 nayeiosnclafg 3 H Sets: sobaflee tiltesmonfe Whctes aes tas aa58 FB. PCD). £18), and 19) donot exist. . Bree jeer, te phn sumac th roy the 9 creasing increasing exneave downward » sebtive naninurn concave downened p Gecreasing concave downward Teltive minioum to infection OE oF ingesing concave upwaed fe) =32°7—2 ‘he domais of fs (~00+20)- Deans fis a 04 function the graph some with spect to he orga Thee are no apo, “ as? atea-2"") yaaa gee a “The itil mumbare fr fate —1, 0, and 1s So the able T= y ‘The citial umber for fis 0, Beaute /"(2)>Oifz <0, f"(2) <0ifz <0 a snd the gro off as a vercal tangens at =, then the graph of fis 4 concave upward f x <0, concave cownvatd if > 0, and the ongin ie point of| Infleeion, The graph isa the ight zea z=) derei T=5t THR) = 7 - + + + + wee] og + + + + ° eS yer | + is + te 4 + + to |= a + ee + ° = fis) | decreasing relative ‘erasing on [-1, ] reative decreasing inva | on(Coontl_minimem vere ungeat at $=0 __susimnum onl ts) fe) [Acwva=0 = ° 2 feva=0 a 48, pe) mf af fea gy sede fea eee) FO) dows aot exist. $"2) =2, (0) dow not exist, . ‘eneave spars Inceasing eaneave upward verealtangea’_ point of nection inceasing ——cancaveupward 288 suiLAVIOR OF FUNCTIONS AND THEIR GRAPHS, EXTREME FUNCTION VAIATS, AND AFPROXDIATIONS 30. s(2) = 24 VITE The domain of [is (004, $2) = Delt — 2) path 2)? = aay) =A) Ge 52) P(e) ~ He 2) P62 52) 434-706 108) eae) 9Pti6s ~52?-+ 204 216 — We] =H yas — 962 +128) set fle) === 0,82 Set fle) 024 =! 8 a og ~ SBE 5 notin the domino | sz) Sia) | fiafras 6 graph in/has a Fa Fes crening ——eonent tprard ceo fo | fttive minimome eoscave upwaed dares, FL Eteoaig fneave upeacd ‘eos, | | 520 © |inceasing point of ateton eres? S| increasing aca donna S52" Jo Feative maximum concave downward beret TZ Seremtag acer downward ° BL f@)zsnztas, Suse) =G2= ar yt (2) ~ e082 sin z fe) = sin 2c ig) =0e2= in [ie _| se) | P@ | fisfhas a pahisfhas a - decreasing coneare upward vil o atv nimam eoneare upward + sreaing concave spar ve increasing point of infection + Hereasng concave downward vile ‘elative maximoumconcare downward - Gecreaing, concave downward o | eecreasing point of inflection 2 ‘ecreming, ——eoneave upeard Bsn 2e-S cons, 2€|- © Sin price wth peiod wand amplitule VISE = VAL (P(g) =6 cose 4 10sin 2 Sa f(a) = 08 Gow2z410sn2==0 Geerening ——_polutof infect (ereaing _coneave uprard ‘lai minknaen sone uperard fnereasng concave upmact Increasing pala of nection increasing concave downward ative maximum soneave downaid Gecreaing concave domnmard Tiverarer Terereriy 39 ADDITIONAL APPLICATIONS OF ABSOLUTE EXTREMA 259 | ADDITIONAL APPLICATIONS OF ABSOLUTE EXTREMA ‘Taeores Let the fncslon J be conavous om the interval I containing the sunt. If fe) ie mative (huernute of fon T tad cs the only nicaer ip Mor which has veative extremum, thn f(@) Ian almiuce extremum of on ls Fathers (@) if fle) ie relative masimnem value of fox GD it fle) wea seatve minim vale of 7 on 1 Ji ifeentiabie on 1 we may apply the fllowing rest: forall =), (9 F@)roiks ceand f(2)< Ui +> thea f(e) ban absolute maximo value of fon K GD Pa) 012 > then f(c) ban abolute miximum value of f 9 29 "Exerdacs 14, us clea to confirm the result of put (@) of te ndiatedexcrce of Deere 1. TEx 21) + inthe sadiun 60/97? ines the height, 129/r nt is the lavenl are, 20r* in? the end ffs? tad $24/a? inthe cost ofthe sides and ends. The ttl cont C(t) = MUQO/e Fer, 9 >. Ce) = (1207+ fer) trbe“XP—15/e} Bree O(0) <0 0-r the ‘Chas an abslte misimun value whoa r=. “he coot i least wher the base radius is $/T57¥ in. 1.48 in. and the height is 60// 250 in. 673 in. (Gx 22) rin. the mdi, 6/n7 ins the eight, 120/r if theater en, 2° 0? Us the eo, i? ad G2hfin® ie the cost of the wides and end. The total cost in Clr) = M120/r+ 297"), 7 >0. Cr) = H120¢78 tar) = ther HO =20/). Beene (0) < 018 07 < YSRTR =r and Co} > 0 ry then Chas an abot minim vale whear =7y “The cost Test wen the se rai’ YSH7> in m2. in nad he gh 50/9/0007 i £24 in (Ex 23) A pare wth margins of 1.5 in atthe wp and botom and Lin at he sides Lt contin 24? ps. ‘The Iegth of the printed rplon is 24 In. The wes x A a? where A(2)=(242KHl+9) = 30430-+88, 2>0, Ae) =3—teP= FEMS 19). Hecate Ae) DDH thee A hea sels iim ale when “Thesmalet pages 8 0. wide and 9 on (6424) Alot with alloys 22 wide a the fat and Best and 15 fat thesis iso contain » 18200 8° ring. The length the bulge 18200 9, rae en ie A.A? ware A(e) = (2+ 30(1200+ 44) 14520 + 42 +296000/z, = >0. A(z) H4z"*(2?- 9008). Because A%(z)< 0 if Then J(c) sam absolve maim value of Jon I then fle) an bea aiimom value of fa I Berea 0 << VFO and A’(2) >0 if «> 9000 then A has an absolute minizum value when 2 = W000 = 94.87 ‘The smalest lot has area 2.8684 {The Feld is 12437 ft by 183.14 8 0 sein th nga ofthe othe wishin 220, dalla the ot la) = 249) + 288)0) = 20) abe + EO, «0, (x)= 06 inte" = ex“? 202) Decne Cs) <0 ifs <8 and Ce)> Qifx >t ths Chas an bole isin vale when ==, ‘he dessa the dae 405 by =O. ‘x meters is the width of the base and 128 m is the height, C(z) dollars is the cost. (fa) = 2a? +412) HBR = 24a? + BD, 2> 0, Ce) = 48x ~c0018"? = 48=%(2? 125) Beaune O(a) <0 if <5 and OMe) >0ife > ‘The dnsions ofthe box are 5m sqeare by hen € has absute minim salve when = ‘Sebi. 40 penAvioNoF FUNOTIONE AX TAIT GRAPHS, EXTREME FUNCTION VAUWES, AND APPRONIMATIONS te bein 7 wd tox esol 288°, were a a ect ving =n TAGS J its chic econ to evimate te ginescs of he bs core the least enon esa (0) Contin soar estimate using cles, aca SoA, Tin. eth wide the ae ft bo. The sh nth of te be dein, and cee co ffl 06? emt min trite 278 se f=) foe Bt HR a 0) Te BA A see ee ence and ite) 0 4 he haan abit iia re when = eS bee mein yaaa fl Coenen nin tthe ihe tu of th ot Th eg fhe tn 5 A carer hx f= Bf Mit eeu of ne btn 227 sien ey (te) ef) = BB 9 8 Hla) = 0-187 SENET De wie) <0if 2 <4 Zand Wiz) > 0 ifs > 4/2, then M has an abstute minima value when M = 499 in. by 3:4Y/3 ~1T9tin, oy 96/(49/2)'in = 2.88 in ‘Te denon of the box ae 4/3188 9. Cape te eh ie ove min CC) lias he Pa wow i Because O'(2) <) wa" ia og) <0 lela then Ce) 284 spe 427-4 => 0.010) = 3197 ro and Ge) > 01 => 90 hen Chae an abate win S00 mh DZ UAE a bate naam hen teen 98 A Ce eset lL et) nln pez mile, then C(v): gar? +400e77. C(e) = 080" — 40007 6er2(o4 252, Because C'(e) <0 PS BOUTS ~ 9.04 =v, and (Ce) > OE V> 8 Teen € hes an abeoluteminimim value when ¥ =f ee oe sich ate inna ven Gaye av 9.4 bois ree ok he inten the ruck hae sraveed 4! (he cr bas waved 900 FT Al tae San oboe mis een 1, Tf econ afer the 1 Oe stance bewean them cS. We wih lo find the sale ad the dna rine stv of tal makes San aberhe TET Tae wane d= a0ge #8. 2) = 4400140) 2020 2 = 7300 On and fC) 20 (> 1442s an sete mime vale Fhe? ee O Secs es clowot 144 sx arth rack eve he iiensstin 12, Tro atplanes A and B ae Syingbossoney tthe sme lite, ar thed 0 lenses to the wet and 20 Monet ge et 3 B Llonetes per minut, (a) a how many eae Le doves, and () what wl be thir cows dstanse RTS SRt Sas Ze me ple A apd de sort of lane Thee Beer oe rane ing toward pnt C. {mites tr they sat It | 4 an be the direc diane from point C south to plane ws 2 eae distance Deween plane A ard plate ecosseplase A i 20 km east of C whe 6 pe a aso km soho when (= Dank iing aint Sm yn 20H @ Deca nage ABC i Hight wane Fatt: 3) nd yin Ea (3) ae Now 1= dis niko whe 2iinimie, Sobatsating for 2, “de 0160? 20-0 pisfetising wi epee tf we tan ty en 0x3) + 2020-4) = 06-21) {49 ADDITIONAL APPLICATIONS OF AUSOLUTE EXTREMA 241 Because ot) < 0:1 < hand 60) > Of > ten and z have an absolute minima vale wen Th aptamer fin Sulit at Pai (1) ad), ota #32 and y= ~2t Thus, plane 4 is 32 km east af post C Asest, Subetuting fr and yinta Bq (3), we obtain ‘and (b) the closet ditance in 4k. yo2°—3274 52; m(z) =y/@) = Se*— (+5, = (-ao,tce). We wish to find the value of = that makes (e) sn aboslute minimum: ava) = Ge -6~ (a 1), Romane l(a) <0 whon eel and me) r0ile> ty 5 has an absolute minimum value when = and mI) =2. Therefore, tbe leas slope for tangent lie 40 the glen cove i? when the point of tangeney x (1,3) ‘Ap equation of the tangent lie is y~3=2(2 1); 2 y+ When F obs and A ohms ae the internal ang external rexiance, P watts i the power where P{W) = BR(C ERY? PIR) = EAC er) FIR RY) = Ee RYACe +B) ~2R) — Ele + RYE Decatse PAR) > DIF O< Rc ad Pie) < OER >r, = bas an absolute masimum valve when R= 4 months after the stact ofan epidemic, P percept of che popalaton is infected, whgre “Za km north of C when thes are He) =rilt elem Pe) Oeste Hae” fel) ar easel DAS 0it0 <2 ¢ Land P(e) <0 its Sit then P bas an abolleacinum value when # = 13 mpath the et people wil be inated, and 75% of he popolaion wil be ined 4 cardboard poste containing 32 in? of pine regis ito have a apn 2a tthe tap adatom and ia. the sce. Fad the deals af the ealet pine of ordhonrd that con te urd to re ie poner, eo the igure Wee ths printed gues havo wih Pin ad Legh yh Beau the gia a ak is sd, ihe erodes idl (ois. Sxaso the rncrgin 9 the tp and boon the MiuMt of the cadboar ier 4) in: Bacave the Se of he pte tegen 32a then aay 8 ® I WA in? is the area of the caréboard, then Nel= (e+ Shri= (e+ fIB + )= 30 Be te A) dis for 20. Ditstig, inn aG)= Bh ve ten 2-8 seh Because Ai(e) <0 If 0-< 2 < V5 mad Ae) 20 if > $5, then AGVI)= 79.6 i an abyolute minimum salve. Substituting 2 = 3/4 into Eq (1) we fing y = 4/2 ic dimcasions of the amallet piece of cardbourd are {3 + V/) ia: 1.1m by 4(1- VB) i= 1029 Ine fin th cadboerd ad the pated elon ae sar tego Xs) dar the otal event when = uns asl: Be) = 2002 {Xe lars the talento producing untupr day C() = e442 + 1400 BB) dlc he po for 0, Mesce He) Ine an abate = The tm sould predue ut dai. (2) las the otal event whic 2 wits a2: Rs) = Abe iz) declines is the total cost of producing # units per day: C(z) = 24 +802 +6000 (z)~ Of2) = ~22? 4.5202 ~ 6000, 2 >0 (Gh~a) Pla)>0d vc¥l ind Pla) <0 i r> 9. Mens Pe) hs an abate 1 Tie company shoul poder 8 Se aly. EMG) Cle) at 4 508 +200 U2) Cle) = —22? + 1202-300, 2 >0 wie 120= 4(30—2). D(a) 0.1 g.230 and PCa) 0 if => 39, Meee P(s) has an abanute 2{30)8 + 120(00) 100 = 1500, « The maximum profit $1500. 22 BEHAVION OF FUNCTIONS AND THEIR GRAPHS, EXTREME FUNCTION VALUES, AND APPROKDEATIONS, 1 and Sind the poiat 28 Find the sartet date fom the pitt (0) w a pat on the cave ? fhe cave coeet op Wiiiasel Tees nn ite aqua la least andthe squase of the dstanee fom pitt (20) the Gade tan pre pl bee a eee 2, Saba Be (web came Q on te care the i= 142% Subang to By. (1), we ave wre esas eases fa)=teateale=) Becwe HE) LUE sc iiged Ye) >0 if > the of) =3 fas abwolte minum value, Alo, The stories distante J = VS and the points onthe exrve that are lost ae (1, + V2). Bi. Let « be the eqeare of the number of mite in the distance fom the rign to the point (23) on the 0:3 — 8 + 96, = € (~2p.+09}: M2) = 202-36 = 2062-8) Beceuse 2(c) 0 t= > thee 29) ~ Hf an sbolte minim. [A cee afgthat maker an abecinte inimam mates te distance an absolute miizaum, Whar 22}, y20~Se= fren the clonot pt to the ongin is P(Q) ata distance of VT units. ‘The oe tx aabe’-3. Th ine feng P to the origin Bas slope = Hence the two lines ate perpendicular, 12. Find the stortest stance from the poat (2,3) tea point on the parabola y = 2%, and find the point By 2 the prrubola clomst to A. Then show that A bot on the nermal fine ofthe parabal A dheaner i leat when its mares Jest andthe aeare of th distance fom point A to pint (sy) en iter ngiven by on Gea) Dt = le 2P oP) = at det Ha) = Aa ad el Became #(e) 20 i €l and o(@) >0i=>1, thn sz) has an abolute minimum value when £= 1 SEP he sortase slstace Is 4/5. On the parabola of = 2 and v0, when += the slope is 2 The ‘Seecement (Iy1) @-})is (P= 1)/CE—1) = —p. Thus A Tir onthe normal Hie ofthe paola at B- tn Exerc 23 and 24, a Norman window consis ofa rctangle trove by 8 comic. Ube permet, Noman windne to be 32 determine what should be the raion of the wemiele and the Bight of fectange so tht he window wii adit the most ight BE Ln sh be the madioe of the verre so that 2rft the width ef the rectangle. ‘Then the height of [Sicogies Hote st 21). The window will adi the most Tight hen He aes ts geet. IFA) (i {Gila of te window then een wean a de ab ee[ gy] ba) =22—(e +e AMC) ‘Set A'(r) = 0: 32 = (2-44)r; 7 = 92] is the only critical number. The radius of the semicinde is 27 heh ofthe rectangle S2-£2) 2 wn. Se aS teave maximum ara, and by Theoron 21 they sve the since acimum aze, Beane is Tino ue esveresne thcen ele son 2, Aamne that there tas oly bal a much igh pec ge fot of rn a he rectangle tae aa fe sacrl nde he te eight of tera ‘Stee gue The wth fhe retange ef Then andor permet of ticide + printer of cane pt ‘Rare ¢ (ther) nate fern @ It Lunt theamout of ight tad, we have pee ccendes fom et) (oryhet Har?) = Beh sda? 2 Subsitatng on Ea (1) ot Ea (2) we Up) = arbre) Dientitng te et UG) <0, these dimasions give ‘39 ADDITIONAL APPLICATIONS OF ABSOLUTE EXTREMA 248 " st Be Because Ur) > 01 re BR = gE and Mr)

Вам также может понравиться